You are on page 1of 256

,

NGUYN HUU IN

, ,

PHUONG PHAP

QUY NAP
. TOAN HOC
.
,
(Tai
ban l`n thu, hai)

,
BAN GIAO
` XUT
DUC
NHA
.

,
,
`, ban in, cac
c
nguyn gc tu
ban
Ebook
1.0 cua cun sach
. tham
,
,
,
ban. Moi
khao, cho y kin sai sot va` lo` i khuyn tai
lin
h.
.
,
,
,
Hu
u in
gia: Nguyn
Tac
in
. thoai:
. 0989061951
Email: huudien@vnu.edu.vn
Web: http://nhdien.wordpress.com

,
Chiu
nhim
. trach
. xut ban:
d

Giam
c Ng Tr`n ai
,
,
,
Tng bin tp
. Vu Duong Thuy
.
Bin tp
. ni
. dung:
Ng Long Hu
.
,
Bin tp
t
ai

b
an:
.
,,
`
Truong Cng Thanh
Tr`nh bay
` b`a:
Ta. Trong
Tr
.
,
Ch ban:
,
Hu
,u in
Ngun

51
05/796-00
GD 00

Ma s: 8H663M0

,
`
` I NOI
U
LO
,,
,
rt manh
hoc
`
Mt
trong toan
nghin cu u
. phuong phap
.
. dung
,
,
gia thuyt la` nguyn ly quy nap
hoc.
va` chu ng minh cac
. ,toan
. Co
,,
,
,

v du. trong cac


mn hoc
`
v s cac
ph thng dung
. o ,chuong tr`nh
,
,
, ,
,

` d
v`
nguyn ly nay
din giai va` m ta. Nhung d
hiu thu dao
,
,
dung
tao
du
` to,i.
ky thut
trong hoc
o. c ban
. ap
.
. tp,
. sang
. rt t sach
,
,
` cung

noi ring v` vn d
` liu
da co mt
s sach
Tai
`
. nuo c ngoai
, .
,
,
,
` theo ti cung

nay,
chua d
`y du va` rt nhi`u nguo` i, kho tip xuc
, , , `
` Ti manh
nguyn
du
dan
o. c voi tai liu
. nay.
.
. thu thp
. va` khao sat
`

ly quy nap
to
an
h
oc
theo
m
oi
kh

a
c
anh
v
a
minh
h
oa
cac
.
.
. ,
.
. bang
,
,
` tp
cung cp
bai
., trong chuong tr`nh ph thng. y la` lo,ai
. sach
,
,
,
ng phuong phap
hoc
` tp

va` thao lun


. nhu
. tp
. va` giai bai
. cho cac
,`,
,
`

ban
yu
th

ch
to
an
h
oc,
c
ac
th
y
c
gi
ao,
sinh
vin
c
ac
tru
o
ng
su
.
,
,, .,
ban
` tai
` liu

pham
., va` cac
. o lo p hoc
. sinh gioi lam
. tip tuc
. phat
,
,,
`
kha canh
trin. Chuong d
cua nguyn ly quy nap
u xem xet cac
.
.
, ,
,

hoc.
chung

toan
ti da khng chung
. Do khun kh cua cu, n sach
,,
hoc
. ke su., tu,o,ng du
minh can
ong cua nguyn ly quy nap
. toan
. va`
,
,
,
,
,
,
,
,
dang
tin d
nguyn ly quy nap
` thu tu. ; su. tuong du
ong cua cac
.
,
,.
,
,
hoc..v.v.
cac
kha canh
toan
Chuong hai khao sat
ky thut
cua
.
.
.
,
chu,o,ng dung
`, chu,o,ng ba mi
` Tu
`
cac
bai
`
nguyn ly nay.
khao sat
,
,
,,
`
`

tp
v
m
t
lo
ai
ch
u
d
ch

ap
d
ung
phu
o
ng
ph
ap
quy
n
ap
to
an

.
.
.
.
.
, ,
,
,
s, H`nh hoc,
hoc
. nhu: S hoc,
. Day
. a thu c, T ho. p, Lin phn
s ...
,
c`on nhi`u bai
` liu

`
Tai
ti tham khao co han
. chung
. va` ,chac
,,
,
,
,
. co sai sot trong th hin
tp
. hay chua noi toi,, hoac
. y ,tuong mong
,
,
,
,
,
ban
i va` b sung. Moi
. ch:
. doc
. cho,y kin sua d
. lin h. gui v` dia
,
duc,
Nha` xut ban Giao
. 81 Tr`n Hung ao,
. Ha` Ni.
.

Ha` Ni,
th
ang
5
n
am
2000
. ,
gia
Tac
3

, ,
CHUONG

QUY NAP
HOC
NGUYN LY
. TOAN
.
va
` quy nap
1.1. Suy din
. ...................................

hoc
1.2. Nguyn l
y quy nap
. toan
. .......................... 6
,
` gia thit quy nap
1.3. Giai do
an
. quy nap
. va
. .............. 8
,
,,
hoc
c cua nguyn ly quy nap
1.4. Hai buo
. toan
. . . . . . . . . . 14
,,
` d`
quy nap
1.5. Khi nao
ung phuong phap
.
.
.
.
. . . . . . . . . . . . 19
.
`
1.6. Bai tp.
. . . . . . . . . . . . . . . . . . . . . . . . . . . . . . . . . . . . . . . . . . . . . . . . . 22

va
` quy nap
1.1. Suy din
.

,
nim
. trong thu.,c t la` suy
minh hoa
. hai khai
. rt hay gap
va` quy nap,

din
bit:
. ta ly cu ca dao Vit
. Nam ai cung
S c co me. co cha
Me. c dan
` ba` cha c dan
` ng
,

`
S c co vo. co chng
,
Sinh con d

gai
th` trai.
`u l`ong chang
,
,
cua ng th`y boi, ta da bit th`y boi ch
y la` cu do
an
m`o thi, nhu,ng ng th`y boi trong cu ca dao nay
` rt khn
do
an
,
`

la` dung
mt
dinh
lun lun dung
ai cung

co me,
.
. khang
. co cha.
,
,
,
,
` d
dung
` cung

Tu
cho nguo` i d
dung
lun,
o du` ap
n boi cu. th nao
, .
,
,
,
,
` khang

ngha la` khang


dinh
ring cung
dung.
Buo c suy lun
tu
.
.
,
,ng khang
dung
dinh
dinh
cho nhu
. chung ap
. ring bit
.
. goi
. la` phep
,
,
,
Phep suy din
o v du. trn la` lun dung

suy din.
vo i hai cu d
`u,

va` quy nap


1.1. Suy din
.

,
,,
,
hoc
`
nhung co th sai o hai cu sau. Trong toan
rt hay dung
phep
,
, .

da cho

suy din, chang


han,
goc trong cua mt
. nu hai
. tam giac
,
,
,
0
0
`

la` 30 va` 70 , th` di


dinh
sau dung:
Goc thu ba cua
u khang
.
,
,,
da cho la` 800 . Mnh
` Tng cac
goc
tam giac
d
` chung o dy
la:
.
,
0
la` 180 .
trong cua mt
. tam giac
,,
,
By gio` ta doc
. lai
. chuyn
. cuo` i dn gian Vit
. nam:
,
,
,
Bn ng th`y boi ru nhau di
xem voi. To i ch voi du ng, bn
` so`, tn
` V` to,i cho.,,
th`y boi chen vao,
. tay, xem con voi no th nao.
bn th`y hop
. nhau b`nh phm.
,,
,
Th`y so` du
v`oi voi noi:
o. c cai
,
,
, ,
, ,,
, `

te ra ch ging con da cu. c lo n. Ti so` vao


- Tuong voi la. lam,
,,
no un cong nguo` i lai.
.
,
chn, vi

Th`y m phai cai


. cai:
,
,
`,a tay cai
` vu
ct
- Voi ch ht
ct
. nhu cai
. nha` thi. Ti m vao
.

cai.
,
phai cai
tai voi, ch:
Th`y nam
,
,
,
bac
ch noi m`o. Con voi tht
- Cac
quat
. ra tu. a nhu cai
. to
, ,
tuong.
,
,
,,
phai cai
dui
Th`y tum
voi, cuo` i khy:

,
noi sai ca. Ti da tum
no trong tay, th` dung

- Ba bac
la` mt
.
, ,
chi x dai.
cai
.
nhau `n ao
` mt
Khng ai chiu
. ai, bn th`y to ting cai
. goc
,
cho. ...
,
,
,
ng th`y boi d
`

Mi
khang
dinh
ring cua m`nh d
`u dung

.
,
,
,
,,
`, khang
phat
biu khang

do
dinh
chung. Buo c suy lun
tu
dinh
an,
.
.
.
,
,
,
,
,
biu khang
`

ring tin to i phat


dinh
chung
du
o
c
g
oi
l
a
ph
e
p
quy

.
.
.

,,
hoc
Chuong 1. Nguyn ly quy nap
. toan
.

,
,,
,

nap.
dinh
chung o dy
d
la` con vt
o la` con voi. Nhu vy
.
.
.
. Khang
,
,

`
`
biu khang

co mt
4 ng thy boi d
dinh
chung sai. Chac
u phat
.
.
,
,

` d

quy
th` se dung.

ng nao
mat
Ta thy rang
phuong phap
o sang
, ,
,
,,

quy nap
nap
a d
n kt qua nhn
. sai. Phuong phap
. co th du
. dinh
.
,, `
,

hoc.
rt hay du
o. c dung trong nghin cuu khoa hoc,
. nht la` toan
.
,
,
,
,
,

quy nap
` dy
Nhu vy
ta phai hiu phuong phap
th nao

. chung
, .
,
,
,

`
dung
` d

va` ap
th nao
d
dinh
dung.
nhn
o. c mnh
khang
.
.
. du
.

hoc
1.2. Nguyn l
y quy nap
. toan
.

,
,
,,
gon
hoc

ngan
dinh
toan
.
. ta ky hiu
. mt
. khang
. la` P( x ), o
,,
,,
,
dy
mnh
d
x la` mt
a v` dang
`
. bin s. Nguo` i ta thuo` ng du
.
.
,
` d
nay
`
Voi moi
o), P( x ). Trong cun sach
. x (trong mt
. tp
. S nao
,
,
,
1
ng s tu. nhin , S la` tp
s tu. nhin (bao
ta ly x = n la` nhu
. cac
,,
,
,
` b. cac
s nguyn duong). Ta su dung
g`m toan
mt
.
. tnh cht rt
,
,
,
,
,
,
,
quan trong
cua tp
.
. s tu. nhin, thuo` ng nguo` i ta cng nhn
. nhu
,,
, ,
mt
` (du
o. c goi
` thu tu. ).
. tin d
. la` tin d
,
,
tp
cua
Tin d
rng
nhu ,ng s tu. , nhin co
`: Trong mi
. ho. p khac
,
,,
mt
. ph`n tu nho nht.
,
s tu., nhin n u,ng vo,i mt

Cho mi
khang
dinh
P(n). V du,
.
.
,
,.
,,
,
,
,

voi 1 ta cho tuong u ng vo i khang


dinh
P(1): s 1 la` mt
.
. s le,
,,
,
,
`

s 2 cho tuong tu ng vo i P(2): s 2 la` mt


... Bang
. s ,chan;
,,
,
nhu vy

khang

dinh
ring
phuong phap
ta tao
.
. chung
. ra day
hoc
P(1), P(2), . . . , P(n), . . . . Nguyn ly quy nap
. toan
. cho ta mt
.
,
,
,,
,

kim tra khang

. sai voi moi


phuong phap
dinh
P(n) dung
hoac
.
. n.
,
,
,
hoc
Nguyn ly quy nap
l sau:
o. c th hin
.
. toan
. du
. qua dinh
1 Trong

0.

,
,
,
nay
` khi noi d
s
sach
n s tu. nhin, ta hiu d
o la` s tu. nhin khac

hoc
1.2. Nguyn ly quy nap
. toan
.

,,
inh
ly 1.1. Cho n0 la` mt
d
`
. s nguyn duong va` P(n) la` mnh
.
.
,
,
co ngha vo i moi
. s tu. nhin n n0 . Nu
A) P(n0 ) la` dung

va`
s tu. , nhin
B) Nu P(k ) dung,

th` P(k + 1) cung

dung

vo,i mi
k n0 ,
,
khi d
d

vo,i moi
o mnh
` P(n) dung
.
. s tu. nhin n n0 .
,
,
,
,
` phan chu,ng. Gia su,, ngu,o.,c
Chu ng minh. Ta se chu ng minh bang
,
,

lai,
d
dinh
P(n) trong inh
l 1.1 khng dung
vo i
` khang
.
.
. mnh
.
,
,
` d
mt
o. Ngha la` t`n tai
. s tu. nhin n n0 nao
. mt
. s tu. ,nhin
,

m n0 , ma` P(m) khng dung.


Ta ly s tu. nhin m nho nht
,,
,

` thu.,c hin
ma` P(m) khng dung
(di
du
`u nay
o. c do tin d
` thu
.
,
,
,
`, d

tu. ). Theo di
ta co bt dang
thu c m > n0 , tu
`u kin
o suy ra
. A),
,
,
,
,
,

` bt dang
` a lp
chon

m 1 n0 . Tu
thu c vu
. va` cach
. s tu. nhin
,
,,

m suy ra P(m 1) la` dung,


nhung no khng keo theo du
o. c P(m)
,

` trai
vo,i gia thit
dung
cho s tip theo m = (m 1) + 1. i`u nay
B).
,
,,
,
,
`, mnh
tu

Xut phat
d
dinh
vo i cac
truo` ng ho. p ring,
` khang
.
.
,
,
,
,
,
. 3 co th ny sinh gia thit mnh

chang
han
s 1, 2, hoac
. nhu cac
.,
,
,
,
,
,
`

d
d
ung
v
o
i
m
oi
s
t
u
nhin.
Sau
d
o
d
ch
u
ng
minh
gi
a
thi
t
cua


.
.
,
,
,
,
` a xy du. ng nguo` i ta ly lun

ta vu
. theo nguyn ly quy nap
. toan
,,
,
,
,
,
chu ng minh nhu vy
hoc.
quy
. Phuong phap
. goi
. la` phuong phap
,,
nay
` g`m hai bu,o,c,
nap
hoc.
dinh
. ,l trn phuong phap
. toan
. Theo
,
,
,

thu nht ta kim tra khang


dinh
mt
.
. tnh cht vo i n = n0 , goi
.
,
,
,
`
k n0 , P ( k )
la` Bu,o,c co, so,; sau d
nu vo i mi
o chu ng minh rang
,
tnh cht da bit, th` suy ra P(k + 1) cung

thoa man
co tnh cht
,
,
,
y, goi
. la` Buo c quy nap.
. la` P(n) co tnh cht da cho vo i
. Kt lun
,

hoc

moi
chu ng minh theo quy nap
. n n0 . Cach
. toan
. la` tranh

,,
hoc
Chuong 1. Nguyn ly quy nap
. toan
.

,
,
,
,
,,

d
cho ta phai kim tra v han
khang
dinh
cua mnh
`.
.
. buo c cac
.

,
`
1.3. Giai do
an
quy
n
ap
v
a
gi
a
thit quy nap
.
.
.
,,
,,
quy nap
hoc
Phuong phap
dung
trong
o. c ap
. toan
. rt hay du
.
,

hoc,
nganh
`

nghin cuu va` t`m t`oi trong toan


khoa hoc
khac.
. cac
, ,
,.
,
,
`
ap
dung
quy nap
hiu cach
phuong phap
y du, ta xem
.
. cho d
,

xet mt
nhu mt
. s v du. sau dy
. phep suy lun
. co ly ma` G.
`
Polya da d
cp.
.
,
,
V du. 1.1. Cho tru,o,c mt
t`m tng cac
s tu. ,
. s tu. nhin n. Hay
nhin 1, 2, . . . , n.
,
,
,
,
Lo` i giai. Ta ky hiu
. Sn la` tng phai t`m, ngha la`
Sn = 1 + 2 + + n.
(1.1)
,
,
,
gon
Ta hy vong
la` t`m ra cng thu c ngan
tnh tng trn, cng
.
. d
,
,
,
,,
,
ta tnh nhanh, gon
thu c d
hon la` phai thu. c hin
o giup
.
. l`n luo. t
,
phep cng

cac
trong tng. Ta cung
bit dy
nu
la` cp s cng,
.
.
,
,
` th` ta co th co ngay cng thu c tnh
ban
., doc
. da bit v` cp s nay,
, ,,
dung
tng. Nhung o dy
nguyn
ta mun minh hoa
. qua tr`nh ap
.
,
,
ng di
hoc
ly quy nap
ta bo
`u da bit v` cp s cng
. toan
. nn nhu
.
,
,
qua, coi nhu chua bit.
,
,
,
,
`, dang
` s tu., nhin

Ta tnh tng Sn tu
thu c (1.1) vo i mt
vai
.
,
,
d
`, 1. Nhu
,ng kt qua tnh toan
cac

lin tip, chang


han
`u tu
. bat
,
,`,
,

` bang
truong ho. p ring ta xp vao
n
Sn

1
1

2
3

3
6

4
10

5
15

6
21

,
,
,,

Muc
chung (khang
dinh chung),
ch cua ta la` t`m du
o. c quy lut
. d
.
, .
,
,
,,
, ,
,
s tu. nhin o hang
`
voi bang trn, mi
trn trong bang cho tuong

,
1.3. Giai do
an
. quy nap
. va` gia thit quy nap
.

,
,,
,
,,
,
`
` toan
phu.
u ng vo i cac
s o hang
duo i. T`m ra quy lut
. bai
. cua mt
,

` rt nhiu yu t: su. kheo leo trong quan sat;


su., nhay
thuc
vao
.
,
,
,
, .
,
,
` cac
kinh nghim
cam du. do
an va` kim tra cua ta; tu
. da trai qua
,
,
,
,
,
`
cac
bai
` toan
tuong tu. , tu kha nang
`

trong tnh toan


lin h. bai
,
,
,
,
,
`

tuong tu. voi di


toan
u kin
. moi, v.v...
,
,
Trn bang trn ta d thy quy lut:
. Tch cua hai s lin tip
,,
,,
,,
,
,,
`
`
`
o hang
trn bang
2 l`n s d
duo i. Tht
`u tin tuong u ng o hang
.
,
vy,
1.2=2.1,
2.3=2.3,
3.4=2.6,
4.5=2.10,
5.6=2.15.
Nhu
v
y
giai
.
.
,
,

`
do
quy
n
ap
c
ua
ch
ung
ta
d
a
th
anh
cng:
T`

m
ra
quy
lu
t
an

. vo i
.
.
tru,o`,ng ho.,p ring n = 1, 2, 3, 4, 5, 6.
cac
,
,
tu., nhin la` mo, rng
Tip tuc
quy lut
trn cho bang
. mt
. cach
.
.
,
,
,
,
,
,
` Ta du
s vo i cac
s tu. nhin bt ky.
a ra gia thit thch ho. p vo i
,,
`,a t`m du
.
quy lut
o. c. at
. vu
n ( n + 1)
.
(1.2)
1+2++n =
2
,
,
,,
` nhu, vy
Mt
o. c goi
. gia thit ,ta da lam
. du
. la` gia thit quy nap.
.
,
,
,
,

Nhung cu hoi dat


. n =
. ra la dang thuc (1.2) co dung voi moi
,
,

1, 2, . . . hay khng? Ro rang nu (1.2) dung voi moi


s tu. nhin
, .
,
`
`
thay n bang

th` bang
cach
n + 1 chung
ta se co dang
thu c

(n + 1)(n + 2)
1 + 2 + + n + ( n + 1) =
.
(1.3)
2
,
,
lai,

Trai
vo i moi
. gia thit (1.2) la` dung
. n = 1, 2, . . ., nu 1) no
,
,
s k suy ra cung

dung
vo i n = 1 va` 2) no dung
vo i mi
dung
,
, ,
`
` khng co cach

` khac
la` phai ap
dung
voi ca k + 1. iu nay
nao
,
, .
hoc.

nguyn ly quy nap


ta phai kim tra
. toan
., Ngha la` chung
,
`

nhung di
l 1.1.
u kin
.
. A) va` B) cua dinh
,
,, ,
,
,
,
,

Buo c co so: vo i n = 1, cng thu c (1.2) dung


(no c`on dung
cho ca
n = 2, 3, 4, 5, 6).

,,
hoc
Chuong 1. Nguyn ly quy nap
. toan
.

10

,
,
,

By gio` chung
Bu,o,c quy nap:
ta chu ng minh cng thu c (1.2) dung
.
,
,
,
,
`

cho ca di
u kin
ch d
o ta gia thit cng thu c (1.2)
. B). Voi muc
. d
,
,
,
,

` d

dung
vo i mt
thu c
o va` se chu ng minh dang
. s n = k 1 nao
,
,

(1.2) dung
vo i n = k + 1. Ta bin d
i
1 + 2 + + k + ( k + 1) =

(k + 1)(k + 2)
k ( k + 1)
+ ( k + 1) =
.
2
2

,
,

Kt qua la` (1.2) dung


vo i n = k + 1. Theo nguyn ly quy nap
. toan
,
,

hoc
vo i moi
. cng thu c (1.2) dung
. n = 1, 2, . . .
,
,
bu,o,c qua tr`nh
Tom lai,
n gian trn ta thy cac
. qua v du. do
,
hoc.
t`m t`oi va` chu ng minh nguyn ly quy nap
. toan
.
,
V du. 1.2. Tnh tng

Sn =

1
1
1
+
++
a( a + 1) ( a + 1)( a + 2)
( a + (n 1))( a + n)

vo,i a 6= 0, 1, 2, . . . ; n = 1, 2, . . .
,
,
,
,
,,
,
Lo` i giai. Vic
truo c tin ta phai t`m ra cng thu c gia thit quy
.
,
nap
. cho tng trn. Ta tnh
S1 =

1
,
a ( a + 1)

1
1
1
2
=
+
=
,
( a + 1)( a + 2)
a( a + 1) ( a + 1)( a + 2)
a ( a + 2)
3
1
=
,
S3 = S2 +
( a + 2)( a + 3)
a ( a + 3)
1
4
S4 = S3 +
=
.
( a + 3)( a + 4)
a ( a + 4)
, ,
,
`

Chung
ta co th du
a ra gia thit rang
n
Sn =
.
(1.4)
a( a + n)
S2 = S1 +

,
1.3. Giai do
11
an
. quy nap
. va` gia thit quy nap
.
,
,,
,
,
Bu,o,c co, so,: Nhu da kim tra o trn.
,
,
,

` d
vo i s tu. nhin n = k nao
Bu,o,c quy nap:
o.
. Gia thit (1.4) dung
Khi d
o
1
k
1
S k +1 = S k +
=
+
( a + k)( a + k + 1)
a( a + k ) ( a + k )( a + k + 1)
2
1 k + ( a + 1) k + a
.
.
=
a+k
a ( a + k + 1)
,
Nhung k2 + ( a + 1)k + a = ( a + k )(k + 1), suy ra
1 ( a + k )(k + 1)
k+1
.
=
.
a + k a ( a + k + 1)
a ( a + k + 1)
, ,
,
,
`, kt qua vu
` a tnh va` bu,o,c co, so, suy ra gia thit quy nap
Tu
. (1.4)
,
,

la` dung
vo i moi
. s tu. nhin n 1.
,
V du. 1.3. Tnh tng
S k +1 =

2
2
4
2n
+
+
+

+
n
1 a2 1 + a2 1 + a4
1 + a2
,
vo i n = 1, 2, . . . ; | a| 6= 1.
Sn =

,
,
,
,
,,
Lo` i giai. Ta phn tch: S luo. ng s hang
cua tng la` n + 1;
.
`, s hang
s hang
d
tru
d
khac
`u tin, c`on lai
`u co dang
.
. cac
.
.
k
2
(k = 1, 2, . . . , n). Ta tnh
k
1 + a2
2
2
4
S1 =
+
=
,
1 a2 1 + a2
1 a4
4
4
4
8
S2 = S1 +
=
+
=
,
4
4
4
1 a8
1+a
1a
1+a
8
8
16
8
=
+
=
.
S3 = S2 +
8
8
8
1+a
1a
1+a
1 a16
,
,
`, cac
biu thu,c cua S1 , S2 va` S3 co
Do 4 = 22 , 8 = 23 va` 16 = 24 tu
, ,
,
th du
a ra gia thit:

,,
hoc
Chuong 1. Nguyn ly quy nap
. toan
.

12

2n +1
(n = 1, 2, . . .).
(1.5)
n +1 ,
1 a2
,
,,
,
,
,
,

Bu,o,c co, so,: Vo i n = 1, cng thu c (1.5) dung


nhu da kim tra o
trn.
, ,,
,
,

` d
vo i s tu. nhin n = k nao
Bu,o,c quy nap:
o.
. Gia su (1.5) dung
Khi d
o
2
2k
2k +1
2
4
S k +1 =
+

+
+
+
+
k
k +1
1 a2 1 + a2 1 + a4
1 + a2
1 + a2
2k +1
2k +1
2k +2
+
=
.
=
k +1
k +1
k +2
1 a2
1 + a2
1 a2
,
,
,
,
`, nguyn ly

ang
thu c (1.5) cung
dung
vo i n = k + 1. Nhu vy,
. tu
,
,
,
hoc

quy nap
thu c (1.5) dung
vo i moi
. toan
. dang
. n 1.
,
,
,
V du. 1.4. Tnh tng cua n s le tu. , nhin d
`u tin.
Sn =

,
,
,
,
Lo` i giai. Ta ky hiu
. tng phai t`m la` Sn :
Sn = 1 + 3 + 5 + + (2n 1).
,
, ,,
,
,
hoc
xy du. ng gia thit quy nap
toan
ta tnh tng o mt
.
.
. s gia
,
,,
tri. du
o. c lit
. k trong bang sau:
n
Sn

1
1

2
4

3
9

4
16

5
25

6
36

,
,
` su., quan sat
cua ta va` kinh nghim
By gio` phu. thuc
vao
.
. trn
,
, ,
,

kt qua ring d
d
du. do
an mnh
tng quat chung. D thy
.
,,
,,
s o hang
`
cac
Sn d
`u la` s chnh phuong: S1 = 12 , S2 = 22 , S3 =
, ,
,
,
32 , S4 = 42 , S5 = 52 , S6 = 62 . Nhu vy
a ra gia thit
. ta co th du
chung la`
Sn = n2 .
(1.6)

,
1.3. Giai do
an
. quy nap
. va` gia thit quy nap
.

13

,
,
,

Ta se chu ng minh (1.6) dung


vo i moi
. s tu. nhin n.
,
,
,
,
,
,
Sn = 1; biu thu c
Bu,o,c co, so,: Vo i n = 1, tng ch co mt
. s hang
.
,
,

n2 = 1 vo i n = 1, nhu vy
. (1.6) dung.
,
,
,
,

vo i n = k, (Sk = k2 ). ta se
Bu,o,c quy nap:
. Gia su (1.6) dung
,
,

chu ng minh (1.6) dung


vo i n = k + 1: Sk+1 = (k + 1)2 . Tht
. vy,
.
2
2
Sk+1 = Sk + (2k + 1) = k + (2k + 1) = (k + 1) .
,
,a theo cach
lam
` cua G. Polya.
Ta xet thm mt
. v du. nu
,
,
V du. 1.5. Tnh tng b`nh phu,o,ng cua n s tu. , nhin d
`u tin.
,
,
,
,
`
.
Lo` i giai. Ta tin hanh
t`m cng thu c cho gia thit quy nap.
. at

Tn = 12 + 22 + + n2 .
,
,
t`m mt
Ta hay
. s gia tri. cua tng khi cho n = 1, 2, . . . , 6.
n
Tn

1
1

2
5

3
14

4
30

5
55

6
91

,
,
` bang trn ta kho co th t`m ra quy lut
Nh`n vao
chung cho Tn .
,
, .
,
,
,
,
Voi thng tin t oi nhu vy
. khng cho kt qu,a g`, nhung vo i kinh
,
,
,ng

nghim
v du. da giai va` so sanh
nhu
. ta co th lin h. vo i cac
s trong v du. 1.1 va` ch`a khoa t`m ra quy lut
day
. chung trong
,
bang sau:
n
Tn
Sn
Tn
Sn

1
1
1
1
1

2
5
3
5
3

3
14
6
14
6

4
30
10
30
10

5
55
15
55
15

6
91
21
91
21

,
,
1
3 5 14
7
`
D`ong cui cung
trong bang ta co th vit lai:
. 1 = 3, 3, 6 = 3,
, ,
,
30
9 55
11 91
13
,
= ,
=
,
=
. By gio` ta co th du
a ra gia thit
10
3 15
3 21
3

14

,,
hoc
Chuong 1. Nguyn ly quy nap
. toan
.

,
Tn
2n + 1
`, kt qua v du. 1.1 ta co
=
. Tu
Sn
3
2n + 1 n(n + 1)
. la`
Tn =
.
hoac
3
2
n(n + 1)(2n + 1)
12 + 22 + + n2 =
.
(1.7)
6
,
,
`
hoc
Ta chu ng minh bang
quy nap
. toan
. cho cng thu c (1.7)
,
,

dung
vo i moi
. s tu. nhin n.
,
, `
,
,
,

cach
xy du. ng trn, dang
thu c (1.7) dung
vo i
Bu,o,c co, so,: Bang
n = 1.
, ,,
,
,

` d
vo i s tu. nhin n = k nao
Bu,o,c quy nap:
o. Ta
. Gia su (1.7) dung
,
,
`

se chu ng minh rang


no cung
dung
vo i n = k + 1, ngha la`
(k + 1)(k + 2)(2k + 3)
12 + 22 + + k 2 + ( k + 1 ) 2 =
.
6
Tht
. vy,
.
k (k + 1)(2k + 1)
Tk+1 = Tk + (k + 1)2 =
+ ( k + 1)2
6
(k + 1)(k + 2)(2k + 3)
k(2k + 1) + 6(k + 1)
=
.
= ( k + 1)
6
6
,
,
` toan
da giai xong.
Nhu vy
. bai
`
rang

,
,,
hoc
c cua nguyn ly quy nap
1.4. Hai buo
. toan
.
,
hoc
Nhu ta da bit nguyn ly quy nap
. toan
. g`m hai ph`n,
,
,
,
,
dung

vic
nguyn ly.
o. c tn trong
. kim, tra ca hai c`n du
. ,khi ap
.

`
Nu ta bo di
mt
u kin
o, th` ta se nhn
. trong hai di
. kim tra d
.
,
,,
,

ng kt lun
v du. sau d
du
o. c nhu
minh hoa
. sai. Thng qua cac
.
,
,
`
` hon.
va` hiu di
u nay
,
V du. 1.6. Chu,ng minh rang
`
moi
`
s tu. ,
`u bang
. s tu. nhin d
nhin li`n sau.

,, ,
hoc
1.4. Hai buo c cua nguyn ly quy nap
15
. toan
.
,
,
,
,,
quy nap
hoc.
Lo` i giai. Ta chu ng minh theo phuong phap
. toan
.
,
,
,
,
`

Gia thit rang


mnh
d
dinh
dung
vo i s tu. nhin n = k
khang
.
.
` d
nao
o, ngha la`
k, = (k + 1).
(1.8)
,
,

Chung ta se chung minh dang thuc sau dung

( k + 1) = ( k + 2).
(1.9)
,
,
, ,

Tht
hai v dang
thu c vo i
. vy,
. Theo gia thit quy nap
. (1.8) cng
.
,,
1, ta nhn
o. c
. du
k + 1 = (k + 1) + 1 = k + 2.
,
,
,
,

Nhu vy,
dinh
dung
vo i n = k th` no dung
vo i n = k + 1,
.
. khang
,
` toan
dung

do d
d
vo i moi
o mnh
` bai
.
. n.
, ,
,
`
` toan
nay
` la` tt ca cac
s tu., nhin d
H. qua cua bai
`u bang
,,
,
` tht
vy

nhau. i`u nay


chu ng minh sai o du?
D

. v ly,
. cach
,
`
dung
dang
thy ngay trong chu ng minh ap
nguyn ly quy nap
.
.
,
,
,
,
,
,
`
hoc
toan
nhu
ng
b
o
qua
ki
m
tra
tru
o
ng
h
o
p
n
=
1.
.
.
. bit:
i`u kin
l 1.1 co mt
.
. A) va` B) trong inh
. y ngha dac
.
,, ,
,
,
`
iu kin
thu. c hin
. A) tao
. ra co so d
. quy nap.
.
,
,
,
,
cho vic
i`u kin
tu. dng
v
a ra nguyn tac
. B) du
. mo rng
.
.
,
,
,
,
, ,, `
di
` truo` ng ho. p ring
han
u kin
tu
. A); nguyn tac
. trn co so di
,
,
,
,
` k d
` sang truo` ng ho. p ring khac;
tu
nay
n k + 1.
,,
,
,
O v du. .1.6 ta khng kim tra di
l 1.1,
`u kin
.
. A) cua inh
,, ,
,
,
nn khng tao
thu. c hin
. khng co
. ra co so d
. quy nap,
. v`, vy
,
,
`
ngha g` khi thu. c hin
ki
m
tra
di
u
ki
n
B)
c
ua
inh
l 1.1,

.
.
.
, ,,
,
,
thu. c cht la` khng co g` d
ca. Ta xet thm v du:
mo rng
.
.
,
,
,
,
V du. 1.7. Chu ng minh rang
` vo i moi

thu,c
. s tu. nhin n bt dang
sau dung

2n > 2n + 1.
(1.10)

,,
hoc
Chuong 1. Nguyn ly quy nap
. toan
.

16

,
,
,
,
,
,
,

Lo` i giai. Gia thit bt dang


thu c (1.10) dung
vo i n = k, vo i k la`
,
` d
mt
o, ngha la` ta co
. s tu. nhin nao
2k > 2k + 1.
(1.11)
,
,
,
,

Ta se chu ng minh bt dang


thu c (1.10) dung
vo i n = k + 1
2k+1 > 2(k + 1) + 1.
(1.12)
,
,
,
,
k

Tht
nho hon 2 vo i moi
khac
. vy,
. 2 la` mt
. s khng
. s tu. ,nhin
,
,
,
k

`
khng. Ta cng
. v trai cua (1.11) voi 2 va cng
. v phai cua (1.11)
,,
,

voi 2. Ta nhn
o. c
. du
2k + 2k > 2k + 1 + 2.
Ngha la`
2
,
` toan
da giai xong.
Bai

k +1

> 2(k + 1) + 1.

sai l`m nhu, v du. tru,o,c khng


` cung

Tt nhin v du. nay


mac
,
,
,
,
chu,ng minh trn la` bt
kim tra Bu,o,c co, so,. Thu. c cht cua cach
,
,
,
,

dang
thu c (1.10) dung
vo i n = k + 1, nu no dung
vo i n = k. i`u
,
,
,
,
` khng suy ra bt dang

nay
thu c dung
vo i t nht mt
. gia tri. cua
,
,
,
, ,
n, chu chua noi to i vo i moi
. s tu. nhin n.
,
, ,, ,
,
. n = 2 bt dang
thu,c (1.10)
Nhung ta co th thu vo i n = 1 hoac
,
,
,
,
,

sai. Vo i n 3 bt dang
thu c (1.10) dung.
Gia tri. s tu. nhin nho
,
,
,

thu c (1.10) dung


(di
A) vo i n0 = 3 va`
nht n = 3 bt dang
`u kin
.
,
,
,
`, gia thit (1.10) dung
chu,ng minh o, trn tu

. lai
lap
vo i n = k
. cach
,

suy ra no dung
vo i n = k + 1 (di
V` vy
`u kin
. B).
. theo nguyn ly
,
,
,

hoc

quy nap
thuc (1.10) dung
vo i
. toan
. ta co kt lun:
. Bt ,dang
,
,
,
,
,
moi
. s tu. nhin n 3 (chu khng phai vo i moi
. s tu. nhin nhu
` ra).
d
` bai
,
,,
dung
quy nap
hoc
Trong vic
phuong phap
toan
ma` ch
. ap
.
.
.
,
, ,
,
,,
,
chu ng minh di
l 1.1 th` mo i ch du
`u kin
a ra du
o. c
.
. A) cua inh

,, ,
hoc
1.4. Hai buo c cua nguyn ly quy nap
17
. toan
.
, ,,
,
, ,, ,
, ,,
nao
` d
co so d
co so
mo rng
quy nap
. chu khng co nguyn tac
.
,
,
d
l lo n Fermat). Ta xet mt
o (nhu dinh
.
. s v du:
.
,
V du. 1.8. Chu,ng minh rang
`
nhu ,ng gia tri. cua ham
` s f (n) =
,
,
2
n n + 41 vo i n = 0, 1, . . . la` nhu ng s nguyn t.
,
,
Lo` i giai. Ta tnh f (0) = 1, f (1) = 41, f (2) = 43, f (3) = 47,
f (4) = 53, f (5) = 61, f (6) = 71, f (7) = 83, f (8) = 97, f (9) = 113.
,
,
,
,
tip tuc
Ta co th tnh toan
. gia tri. cua f (n) cho to i n = 40, tt ca
,
,
` d
gia tri. nay
`u la` s nguyn t. Nhung vo i n = 41 ta co f (41) =
,
,
412 41 + 41 = 412 . Kt qua f (41) khng phai la` s nguyn t,
,
` toan
la` khng dung.

nn kt lun
. cua bai
,
,
,,
,
,

Nhu vy
d
vo i 40 truo` ng ho. p
` co th dung
. ta thy mt
. mnh
.
,
,,
,
,

ring, nhung khng dung


vo i moi
. truo` ng ho. p noi chung.

V du. 1.9. a thu,c x n 1, vo,i n la` s tu. , nhin du,o,ng. a thu,c


,,
nay
` lin quan d
` toan
h`nh hoc
n bai
o` ng tr`on ra n ph`n
. chia du
, ` du,o,c rt nhi`u lnh vu,c toan
bang
`
nhau, nn da
hoc
thu c nay
.
.
.
,
`

nghin cu u va` d
. bit
nha` toan
hoc
cp
n. ac
. d
. cac
. quan tm
,
,
,
,

to i vn d
` ra cac
thu` a s la` cac
da
phn tch da
thu c nay
thu c
vo,i h. s nguyn 1, liu

vo,i moi
`u d
o c`on dung
. n?
. di
,
,
,
`

tru,o`,ng ho.,p ring, cac


nha`
Lo` i giai. Bang
cach
khai trin cac
,
,
,,
`

` a s du
hoc
h. s trong cac
thu

toan
tt ca cac
o. c
. , nhn
. thy rang
,

khai trin co gia tri. tuyt


han,
i khng qua 1. Chang
. d
.
x 1 = x 1,
x2 1 = ( x 1)( x + 1),
x3 1 = ( x 1)( x2 + x + 1),
x4 1 = ( x 1)( x + 1)( x2 + 1),

18

,,
hoc
Chuong 1. Nguyn ly quy nap
. toan
.
x5 1 = ( x 1)( x4 + x3 + x2 + x + 1),
x6 1 = ( x 1)( x + 1)( x2 + x + 1)( x2 x + 1).

,
,
,
,

,ng c gang
Nhu
chu ng minh di
vo i moi
`u nghi ngo` dung
. n cua
,
nha` toan
hoc
`

cac
cng. Mt
. khng thanh
. tho` i gian sau, nha` toan
,
,
,
n
`
1941) ch ra rang

hoc
(nam
vo i da
thu c x 1,
. Nga V. Ivanov
,
, ,
,
,
,
,
,

truo` ng ho. p nho hon 105. Nhu,ng


di
vo i cac
`u nghi ngo` ch dung
,
,
`,a s cua x105 1 la`
vo i n = 105, mt
. thu
x48 + x47 + x46 x43 x42 2x41 x40 x39 + x36 +

+ x35 + x34 + x33 + x32 + x31 x28 x26 x24 x22 x20 + x17
+ x16 + x15 + x14 + x13 + x12 x9 x8 2x7 x6 + x5 + x2 + x + 1.
,
,
`,a s nay
` khng co tnh cht cua cac
da
nha` toan

Thu
thu c ma` cac

hoc
. mun.

V du. 1.10. Chu,ng minh rang


`
vo,i moi
d
` sau dy

. s n mnh
.
,
,
,
,

dung:

Nu a va` b la` nhung s tu. nhin duong, ma` max( a, b) = n,


th` a = b.
,
,
,
,
,
n ky hiu
`
Lo` i giai. Bu,o,c co, so,: Vo i mi
d
` cua bai
. An la` mnh
.
da cho. Ro rang
`

toan
A1 la` dung,
v` nu max( a, b) = 1, th` hai s
,
,
`
`

a va` b phai trung


nhau va` bang
1 (do a va` b la` s tu. nhin).
, ,,
,ng s tu., nhin

Bu,o,c quy nap:


Nu a va` b la` nhu
. Gia su Ak la` dung.
sao cho max( a, b) = k + 1. Ta xet hai s a1 = a 1 va` b1 = b 1,
,
`, d
khi d
o max( a1 , b1 ) = k, tu
o suy ra a1 = b1 , v` gia thit Ak la`

dung,
do d
dung.
Theo nguyn ly quy
o a = b, ngha la` Ak+1 cung
,
,

hoc

nap
voi moi
. toan
. An dung
. s tu. nhin n.
,
,
,,
` Ta tnh du
H. qua: Cho a va` b la` hai s tu. nhin bt ky.
o. c
,

max( a, b) = k, ma` k la` mt


. s tu. nhin. Theo v du. trn An dung

,,
` dung
`
quy nap
1.5. Khi nao
phuong phap
.

19

,
,
`, d

vo i moi
n, th` no cung
dung
vo i Ak . Tu
o suy ra a = b. Ngha la`
.
,
,
`

`
s tu. nhin d

tt ca cac
nhau. Tht
u bang
. v ly!
,,
,
chu ng minh sai o du?
`
Trong v du. trn cach
Ta xem lai

. toan
,
,
,

hoc.
b. cach
chu ng minh va` nguyn ly quy nap
. toan
. Buo c quy
,
,,
,
to i di
nap
`u k 1, khi buo c quy
. trong,chu ng minh khng nhac
`, Ak sang Ak+1 . Thu.,c t trong tnh toan
chu,ng
nap
. chuyn ti,p tu
,
minh khng dam bao k 1.

,,
` d`
quy nap
1.5. Khi nao
ung phuong phap
.
,,
quy nap
hoc
dung
Phuong phap
toan
trong nghin
. rt co tac
,.
, .
,
,
,
,

kt qua va` chu ng minh kim nghim


cu u, du. do
an
. kt qua.
,
,,
quy nap
hoc
` vic
Nhung nhi`u khi chnh phuong phap
. toan
. lam
.
,
,
,

`
` d`ong, bin d

chu ng minh dai


i phu c tap
., gy rt nhiu kho khan
,
,
,
`
`
` toan
giai bang
quy

trong chu ng minh. Nhiu bai


phuong phap
, ,
,
,
`
khac.
Chnh G. Polya co

nap
mt
. co th giai bang
. phuong phap
,
,
`
`
` toan
chu ng minh bang
hoc

noi: Nhiu bai


quy nap
. toan
. co th
,
` trong chnh cach
`
khac,
cach
khac
d

chu ng minh bang


cach
o nam
,
,
hoc
chu ng minh quy nap
. toan
. khi ta phn tch ky ni
. dung chu ng
minh.
,
,,
hoc
`
Trong toan
nguo` i ta hay dung
ky hiu
tng.
la` mt
.
.
.
,
,,
,ng s
Thuo` ng tng co dang
A + A+1 + + A ( va` la` nhu
.

,
,
,
,

`, d
nguyn)va` du
o. c vit Ak (doc
n
. la` tng cua Ak , k chay
. tu
k=
,
). Nhu vy
.

A + A +1 + + A =

Ak

k=

,
,
,
k goi
la` ch s cua tng, c`on va` la` gia tri. d
`u va` gia tri. cui
.
,
,
,
,
,
,
s hang
cua dang

cua ch s k. Mi
bn trai
thu c la` dung
vo i mt
.
.

,,
hoc
Chuong 1. Nguyn ly quy nap
. toan
.

20

gia tri. k (k = , + 1, . . . , ). V du.


n

k 2 = 12 + 22 + + n 2 , ( n 1 ) ,

k =1
n +1

102k = 102 + 100 + 102 + + 102(n+1) , (n 2).

k =1

,
,ng tnh cht sau: Nu cho a va` b la` nhu
,ng
Phep ly tng co nhu
,
,
dang

s, ta co cac
thu c

k=

k=

aAk = a Ak ,

(aAk + bBk ) = a

Ak + b Bk .
k=
k=
k=
,
,
,

`
`
Ky hiu
t
ng
khng
ph
u
thu
c
v
ao
ch

s
,
nhu
ng phu. thuc
.
.
.
. vao
`
gia tri. ban d
`u va` gia tri. cui cung

k=

Ak =

Ai =

i =

A +i

i =0

,,
,
,ng v du. o, ph`n tru,o,c, trong qua tr`nh tnh toan

Tro lai
. nhu
,
quy nap
. tnh tng
12 + 22 + + n 2 =

k2

k =1

,
,
, ,
`
ap
dung
Bang
cach
tnh cht cua ky hiu
.
. tng va` cng thu c tng
n
n ( n + 1)
s tu., nhin k =
cac
, (n 1). Tht
. vy,
. d thy
2
k =1
n

k =0

k =0

( k + 1)3 k 3 = ( n + 1)3 .

,,
` dung
`
quy nap
1.5. Khi nao
phuong phap
.

21

,
,
, ,
,
,
cua dang

V trai
thu c trn co th bin d
i
n

k =0

k =0

k =0
n

(k + 1)3 k3 = [(k + 1)3 k3 ] = (3k2 + 3k + 1)


n

= 3 k2 + 3 k +
k =1

k =1

,
,
,
`, cac
dang
ra

Nhu vy
thu c trn rut
. tu
n

( n + 1)3 = 3 k 2 + 3
k =1

k =0
n

1.

k =0

n ( n + 1)
+ ( n + 1),
2

,
ta co
Chuyn v va` tnh toan
n

k2 = 3 [(n + 1)3 3

k =1

n ( n + 1)
1
(n + 1)] = n(n + 1)(2n + 1).
2
6

,
` . 1.2)
Tnh tng sau dy
(bai
n

(a + k 1)(a + k) ,

n = 1, 2, ..; a 6= 0, 1, 2, . . .

k =1

,
,,
,

Ta su dung
dang
thu c sau
.
1
1
1
=

.
( a + k 1)( a + k)
a+k1 a+k
1
,
. bk =
at
, nhu vy
.
a+k
n
n
1
(a + k 1)(a + k) = (bk1 bk ) = b0 bn
k =1
k =1

1
1
n

=
.
a a+n
a( a + n)

,,
`
Cui cung
ta nhn
o. c
. du
n

n
.
a( a + n)
k =1
,
,,
,,
` ta c`on d
Vn d
` cua ph`n nay
` cp
. tip o Chuong 3.
1

(a + k 1)(a + k) =

,,
hoc
Chuong 1. Nguyn ly quy nap
. toan
.

22

` tp
1.6. Bai
.

,
,
`
xy du.,ng gia thit va` chu,ng minh
. 1.11. Tnh tng bang
cach
,
`
hoc
tng sau:
bang
quy nap
. toan
. cac
a) Sn = 12 22 + + (1)n1 n2 ;
b) Sn = 13 + 23 + + n3 ;
c) Sn = 1.1! + 2.2! + + n.n!.
`
cac
cng thu,c sau:
. 1.12. Chu,ng minh t nht bang
hai cach
1
a) 12 + 32 + + (2n 1)2 = n(2n 1)(2n + 1), n = 1, 2, . . .
3
1
b) 1.2.3 + 2.3.4 + + n(n + 1)(n + 2) = n(n + 1)(n + 2)(n +
4
3), n = 1, 2, . . .
1
1
n
1
+
++
=
, n = 1, 2, . . .
c)
1.2 2.3
n ( n + 1)
n+1
1
1
. x0 = ; xk =
. 1.13. Cho n > 1 la` s tu., nhin. Ta dat
( x0 +
n
n

k
,
tnh tng x0 + x1 + +
x1 + + xk1 ), k = 1, 2, . . . , n 1. Hay
x n 1 .

, ,
CHUONG

, ,

KY THUT
. DUNG PHUONG PHAP
HOC
QUY NAP
. TOAN
.
hoc
2.1. Mt
nguyn l
y quy nap
. s dang
.
. toan
. ...........
hoc
2.2. Mnh
d
` trong nguyn ly quy nap
.
. toan
. ........
,,
,,
,
c quy nap
2.3. Buo
du
o
c
xy
d
u
ng
trn
P
(
k
)
.
.
...........

.
.
.
,,
,,
,
c quy nap
2.4. Buo
du
o
c
xy
d
u
ng
trn
P
(
k
+
1
) .........

.
.
.
`

2.5. Quy nap


. toan hoc
. va ph, ep truy hi . . . . . . . . . . . . . . . . .

` tng quat
hoa
................
2.6. Quy nap
to
an
h
oc
.
. va
` tp.
2.7. Bai
.
.
.
.
.
.
.
.
.
.
.
.
.
.
.
.
.
.
.
.
.
.
.
.
.
.
.
.
.
.
..................
.

23
31
36
40
43
51
55

hoc
2.1. Mt
nguyn l
y quy nap
. s dang
.
. toan
.

, ,, ,,
d
i`u kin
bat
`u
. , l 1.1 cho ta co so mo rng
. A) trong inh
.
,
,
` gia tri. n0 . i`u kin

tu
l 1.1 cho ta mnh
d
` khang
.
. B) cua inh
.
,
,

dinh
P(n) dung
vo i n0 + 1, n0 + 2, . . .. Thu. c t nhi`u khi trong
.
,
,
,
,
,
buo c quy nap
oi hoi hai gia tri. n = k 1 va` n = k cua
., phai d`
,,
,

mnh
d
d
vo i n = k + 1. Trong truo` ng
`, d
suy ra mnh
` dung
.
.
,
,
,
,
,
,
,ng ch vo,i n0 , ma` ca
` bu,o,c co, so, phai kim tra khng nhu
ho. p nay
,
,
,
,,
,
ho,n ta co th phat
biu lai
vo i n0 + 1. Tng quat
l o ph`n
.
. dinh
,,
,
truo c nhu sau:
inh
ly 2.1. Cho p la` s nguyn du,o,ng va` day
cac
mnh
d
`
.
.
P (1), P (2), . . . , P ( n ), . . .

24

,,
,,
`
quy nap
hoc
Chuong 2. Ky thut
phuong phap
. dung
. toan
.

nu
A) P(1), P(2), . . . , P( p) la` nhu ,ng mnh
d

va`
` dung
.
s tu. , nhin k p cac
B) Vo,i mi
mnh
d
` P ( k p + 1), P ( k p +
.
2), . . . , P(k ) dung,

suy ra mnh
d

dung,

` P(k+1) cung
.
,,
th` mnh
d

vo,i moi
` P(n) dung
.
. s nguyn duong n.
,
,
` hoan
` toan
` lap
. lai
Chu ng minh dinh
l nay
l 1.1.
.
.
. nhu dinh
,,
Sau dy
dang
dinh
l 2.1.
ta xet mt
.
. s v du. su dung
.
.
,
s tu. , nhin k co dang
V du. 2.1. Cho v0 = 2, v1 = 3 va` vo,i mi

thu,c sau: vk+1 = 3vk 2vk1 . Chu,ng minh rang


`
vn = 2n + 1.
,
,
,
,
` toan
dung,

Lo` i giai. Bu,o,c co, so, : Vo i n = 0 va` n = 1 kt lun


. bai
`
` da cho.
do di
u kin
. bai
, ,,
,
,
`
vk1 = 2k1 + 1; vk = 2k + 1, khi
Buo c quy nap:
. Gia su rang
d
o
vk+1 = 3(2k + 1) 2(2k1 + 1) = 2k+1 + 1.
hoc
Theo nguyn ly quy nap
dinh
l 2.1, suy ra vn =
.
. toan
. dang
.
,
,
n

2 + 1 dung
vo i moi
. s tu. nhin n.

,
,,
2
V du. 2.2. Cho x1 va` x2 la` nghim
. cua phuong tr`nh x 27x +
,
,
,
14 = 0; n la` mt
` tng Sn =
. s tu. nhin bt k`y. Chu ng minh rang
x1n + x2n khng chia ht cho 715.
,
,
,
Lo` i giai. Theo cng thu c Viet x1 + x2 = 27; x1 x2 = 14.
,
s S1 = 27; S2 = ( x1 + x2 )2 2x1 x2 = 701 va`
Bu,o,c co, so,: Cac
S3 = ( x1 + x2 )[( x1 + x2 )2 3x1 x2 ] = 27 687 d
`u khng chia ht
,
,
` toan
dung

cho 715. Suy ra mnh


d
vo i n = 1, 2, 3.
` cua bai
.
, ,,
,

Bu,o,c quy nap:


d
vo i n = k 2, n = k
` dung
.
. Gia su mnh

hoc
2.1. Mt
nguyn ly quy nap
. s dang
.
. toan
.

25

1, n = k, ta tnh
x1k+1 + x2k+1 = ( x1 + x2 )( x1k + x2k ) x1 x2 ( x1k1 + x2k1 )

= ( x1 + x2 )[( x1 + x2 )( x1k1 + x2k1 )


x1 x2 ( x1k2 x2k2 )] x1 x2 ( x1k1 + x2k1 )
= 715( x1k1 + x2k1 ) 378( x1k2 + x2k2 ).
Do d
o x1k+1 + x2k+1 khng chia ht cho 715, v` 378 khng chia ht
,
khac
mnh

cho 715, noi cach


d
vo i n = k + 1.
` dung
.

V du. 2.3. Chu,ng minh vo,i moi


s thu. ,c x > 0 va` moi
s tu. , nhin
.
.
,
n bt dang

thu,c sau dung

1
1
1
(2.1)
x n + x n2 + x n4 + + n4 + n2 + n n + 1.
x
x
x
,
,
,
,
,

Lo` i giai. 1a) Vo i n = 1 bt dang


thu c (2.1) co dang
.
1
x + 2.
(2.2)
x
,
,
,
,
,
`, bt dang

Bt dang
thu c (2.2) suy ra tu
thu c hin nhin: ( x
1)2 0.
,
,
,

thu c (2.1) co dang


1b) Vo i n = 2 bt dang
.
1
(2.3)
x2 + 1 + 2 3.
x
,
,
,
,

Bt dang
thu c (2.2) dung
vo i moi
dung
vo i x2 ,
. x > 0, vy
. no cung
1
2.
x2
,
,
,,
,
`
thu,c sau cung
Cng
vo i 1, ta nhn
o. c (2.3).
. du
. hai v cua bt dang
,
, ,,
,
,

2) Gia su bt dang
thu c (2.1) dung
vo i n = k, ma` k la` mt
. s
,
` d
tu. nhin nao
o
x2 +

x k + x k 2 + x k 4 + +

1
x k 4

1
x k 2

1
k + 1,
xk

(2.4)

26

,,
,,
`
quy nap
hoc
Chuong 2. Ky thut
phuong phap
. dung
. toan
.

,
,
,
,

ta se chu ng minh khi d


thu c (2.1) dung
vo i n = k + 2,
o bt dang
hay la`
1
1
1
(2.5)
x k+2 + x k + x k2 + + k2 + k + k+2 k + 3.
x
x
x
,, k+2
,,
Tht
o. c
. vy,
. trong (2.2) th x boi x , ta nhn
. du
1
2.
(2.6)
x k +2
,
,
,,
,
,
bt dang

Cng
v tuong u ng cua cac
thu c (2.4) va` (2.6), ta se co
.
(2.5).
,
,
Tom lai:
Bu,o,c co, so,: Trong 1a) va` 1b) ta da chu ng minh bt
.
,
,

dang
thu c dung
cho n = 1 va` n = 2.
,
,
`, gia thit dung

Bu,o,c quy nap:


Trong 2) ta da chu ng minh tu
.
,
,
,
,

cua (2.1) voi n = k suy ra no dung


vo i n = k + 2. Kt qua la`
,
,
,
,
` 1a) va` 2) cho ta khang

+ Tu
dinh
la` bt dang
thu c (2.1) dung
.
,
,

voi moi
. s le n.
,
,
,
`, 1b) va` 2) cho ta khang

+ Tu
dinh
la` bt dang
thu c (2.1) dung
.
,
n.
voi moi
. s chan
,
,
,
,
,

Nhu vy,
thu c (2.1) dung
vo i moi
. bt dang
. s tu. nhin n.
,
,
V du. 2.4. Chu,ng minh rang
`
vo,i moi

thu,c
. s tu. nhin n dang
sau dung:


 

12 n
17 n1
a)
.2
.2
= 2n 1 ;
7
7

 

17 n
12 n2
b)
.2
.2
= 2n +1 ,
7
7
, ,
,
,
o, dy
[ a] la` s nguyn lo n nht nho hon a.
x k +2 +

,
,
,
,
,
,
,ng dang

Lo` i giai. Bu,o,c co, so,: Vo i n = 1, 2, 3 nhu


thu c trn dung
,
,
`
kim tra tru. c tip.
bang
cach

hoc
2.1. Mt
nguyn ly quy nap
. s dang
.
. toan
.

27

,
,
,
,
`

hai dang
Bu,o,c quy nap:
thu c dung
vo i ba s
. Gia thit rang
,
,
,
,
dang

tu. nhin lin tip k, k + 1, k + 2. Ta se chu ng minh cac


thu c
,

trn dung
vo i n = k + 3.
12
12
12 k+3
`,
2a) Tu
= (1 + 7)2k = 12.2k + .2k ;
.2
7
7
7
17
17
17 k+2
.2
= (1 + 7)2k1 = 17.2k1 + .2k1 ,
7
7
7
suy ra

 


 

12 k+3
17 k+2
12 k
17 k1
k
k 1
.2

.2
= 12.2 17.2
+
2
2
.
7
7
7
7
,
,

Nhung v` a) dung
vo i n = k

 

12 k+3
17 k+2
.2

.2
= 12.2k 17.2k1 + 2k1 = 2k+2 .
7
7
,
,
,

Vy
thu c a) dung
vo i n = k + 3.
. dang
17 k+3
17
`,
2b) Tu
.2
= 17.2k + .2k ,
7
7
12 k+1
12
k 2
.2
= 12.2
+ .2k2 ,
7
7
suy ra

 


 

17 k+3
12 k+1
17 k
12 k2
k
k 2
.2

.2
= 17.2 12.2
+
2
.2
.
7
7
7
7
,
,
Nhung v` b) vo i n = k, ta co
 


17 k+3
12 k+1
.2

.2
= 17.2k 12.2k2 + 2k+1 = 2k+4 .
7
7
,
,
,

Vy
thu c b) dung
vo i n = k + 3.
. dang
,
,
hoc

Theo nguyn ly quy nap


vo i moi
. toan
. a), b) dung
. s tu. nhin
n.
V du. 2.5. Chu,ng minh rang
`
n +1 n +1
un =
,
(2.7)

28

,,
,,
`
quy nap
hoc
Chuong 2. Ky thut
phuong phap
. dung
. toan
.

2 2
3 3
u1 =
, u2 =
( 6= )

,
s tu. , nhin k > 2 co dang
va` vo,i mi

thu,c sau:
nu

uk = ( + )uk1 uk2 .
,
,
,

Lo` i giai. 1) Vo i n = 1 va` n = 2, (2.7) dung


do di
`u kin
. da cho.
, ,, ,
,
,

2) Gia su dang
thu c dung
vo i n = k 1 va` n = k 2
u k 2 =

k k
k 1 k 1
, u k 1 =

khi d
o

k k
k 1 k 1
k +1 k +1

=
.

,
Mt
nguyn ly quy nap
hon nguyn ly quy nap
. dang
.
. manh
.
. ta da
,
,

`
bit cung
rt du
o. c hay dung.
inh
l 2.2 Cho mt
mnh
d
`
. day
.
.
uk = ( + )

P (1), P (2), . . . , P ( n ), . . .
Nu
,
A) P(1) la` khang

dinh
dung,

va`
.
,
s tu. , nhin k 1, nhu ,ng khang
B) vo,i mi

dinh
.
,
P(1), P(2), . . . , P(k ) dung

suy ra khang

dinh
P
(
k
+
1
)
c
ung

d
ung,

.
,
,
,

th` P(n) dung

vo i tt ca s tu. nhin n 1.
,
,,
, ,,
` khac
vo,i cac
dang
Dang
nay
truo c la` gia thit manh
hon o
.
.
.
,
,
,

bu,o,c quy nap.


dinh
P (1), P (2), . . . , P ( k )
.
. Ta gia thit tt ca khang
,

dung
suy ra P(k + 1) cung
dung.
D dang
chu ng minh hai cach
,
,
,
,
,
,
biu dinh
phat
l 1.1. va` dinh
l 2.2 tuong du
ong nhau. Nhung
.
.
,
,
,
dung
` bai
` toan
cu. th dung
`
trong thu. c t ap
vao
dinh
. l 2.2 d giai
.
,
hon.

hoc
2.1. Mt
nguyn ly quy nap
. s dang
.
. toan
.

29

1
1
V du. 2.6. Chu,ng minh rang
` nu x + la` s nguyn th` x n + n
x ,,
x
,
cung

la` s nguyn vo,i moi


. s tu. nhin duong n.
,
,
,
,

d
Lo` i giai. Bu,o,c co, so,: Khi n = 1 mnh
` hin nhin dung.
.
, ,, ,
1
,
`, 1 d
Bu,o,c quy nap:
n k, x k + k la`
. s tu. nhin tu
. Gia su vo i moi
x
1
,
,
k
+
1
`

`
ng s nguyn. Ta cn chu ng minh rang

nhu
x
+ k+1 cung
la`
x
s nguyn.
1
1
1
k +1 + 1
Tht
= ( x + )( x k + k ) ( x k1 + k1 ).
. vy,
. x
k
+
1
x
x
x
x
,
,
,
,
1 k
1
1 k 1
,

Theo gia thit ca 3 biu thuc x + , x + k , x


+ k1 d
u biu
x
x
x
1
k +1 +
cac

s nguyn. Vy
cung
la` mt
din
. s nguyn.
. x
x k +1
,
,
,
,
V du. 2.7. Chu,ng minh rang
`
moi
. s tu. nhin lo n hon 1 co th
,
,
du,o,i dang
tch cua nhu ,ng s nguyn t.
biu din
.

,
,
,
,
,

Lo` i giai. Bu,o,c co, so,: Hin nhin mnh


d
vo i moi
` dung
.
. s
,
,
,
. bit
nguyn t, truo` ng ho. p dac
. n = 2.
,
,,
,
,
,
,

Buo c quy nap:


d
vo i moi
s tu. nhin k,
` dung
.
. Gia su mnh
,.
du,o,i dang
ma` 2 k < n. Ngha la` moi
`u biu din
. s 2 k < n d
.
`,a s nguyn t. Ta xet hai tru,o`,ng ho.,p
thu
tch cac

1) Nu n la` s nguyn t th` mnh


d
` dung.
.
,
,
2) Nu n la` ho. p s th` theo dinh
ngha ho. p s t`n tai
hai s
.
, .
nguyn n1 < n va` n2 < n sao cho n = n1 n2 . Theo gia thit quy
,
,,
du
`
s nguyn t. Do
nap
tch cac
`u biu din
o. c thanh
. n1 va` n2 d
,
,
,
du

`
s nguyn t.
d
biu din
tch cac
o suy ra n cung
o. c thanh
,
,
,
,
trong bai
y:
Ta cung

` nay
`
Chu
co th chu ng minh su. biu din
,
cho moi
. s tu. nhin la` duy nht.

30

,,
,,
`
quy nap
hoc
Chuong 2. Ky thut
phuong phap
. dung
. toan
.

cap
V du. 2.8. Chu,ng minh rang
`
mi
. s nguyn n 1 va` b > 1
,
,
,
duo i dang
t`n tai
.
. biu din
n = c s b s + c s 1 b s 1 + + c 1 b + c 0 ,

(2.8)
,,
,
o dy
s 0 la` mt
. s nguyn, va` 0 ci b 1 vo i moi
. i =
0, 1, . . . , s 1 va` 0 < cs b 1.
,
,
,,
dung
chu,ng
Lo` i giai. Ta ly s bt ky` b > 1 va` ap
phuong phap
.
hoc.
minh quy nap
. toan
.
,, ,
,
,
,
Buo c co so: Vo i n = 1, ta ly s = 0, c0 = 1 b 1. Ta nhn
.
,
,,
,

du
dang
thuc (2.8) dang
1 = c0 .
o. c dang
.
.
, ,, ,
,
,
,
,
(2.8) dung

n
vo i moi
Buo c quy nap:
. s tu. nhin
. Gia su biu di
, ,
,
,
,
,
,
k nho hon n. Theo dinh
l co ban cua s hoc
.
. vo i n va` b co th t`m
,,
du
o. c s nguyn khng m n1 va` r, sao cho
n = bn1 + r,

0 r b 1.

D thy n1 < n. tht


. vy,
. nu ta co n1 n, th` v` b > 1, r 0 ta

co n = bn1 + r > n, v ly.


,,
,
Ta xet hai truo` ng ho. p
,
,,
,
,
vo,i
1) Nu n1 = 0, th` n = r, th` (2.8) tuong u ng vo i biu din
s = 0, c0 = r.

,
,
2) Nu n1 1, th` 1 n1 < n, theo gia thit quy nap
. biu
,
,
,
(2.8) dung

din
vo i moi
. s tu. nhin k n. Ngha la` vo i n1 ta co
n 1 = r t b t + r t 1 b t 1 + + r 0
,
` d
vo i mt
y t va` 0 ri b 1 (i = 0, 1, .., t), rt > 0. Khi
. s nao
d
o
n ,= bn1 + r = rt bt+1 + rt1 bt + + r0 b + r,
(2.8) tu,o,ng u,ng vo,i s = t + 1, cs = rt , . . . , c1 =
` biu din
ngha la,
r0 , c0 = r.

hoc
2.2. Mnh
d
` trong nguyn ly quy nap
.
. toan
.

31

,
,
, ,
trong bai
y:
Ta cung
` nay
`

Chu
co th chu ng minh su. biu din
,
,
,

`
cho moi
l v su. biu din
.
. s tu. nhin la` duy nht. y la` dinh
,
,

mt
. s tu. nhin n theo co s b.
,
,a cua nguyn ly quy nap
nu
hoc
C`on mt
khac
. s dang
.
. toan
.

chung
ta se xet sau.

2.2. Mnh
d
y quy nap
` trong nguyn l
.
. toan
hoc
.
`
dung
v du. tru,o,c ta thy rang
da
nguyn
Trong cac
s vic
. ap
,
,.
,
,
hoc

. biu thu,c toan


ly quy nap
i cng thu c hoac
. toan
. , la` su. bin d
` chung

dung
hoc.
ti nhn manh
d
n vic
. Trong muc
. nho nay
.
. ap
.
,
,
`

`
nguyn ly quy nap
trn
c
ac
m
nh
d
khng
ph
ai
l
a
cng
th
u c

.
.
,
,,
,
,
hoc.
bu,o,c
. biu thu,c toan
hoac
Trong truo` ng ho. p nhu vy
. cac
, .
,
,
,,
,

P(k ) cua mnh


d
dinh
dinh
` khang
o. c xac
. du
. m`m deo hon thng
.
v du. sau:
qua cac
,
,
,,
,
V du. 2.9. Chu,ng minh rang
` tng lp
. phuong cua ba s tu. nhin
lin tip chia ht cho 9.
,
,
,
,
Lo` i giai. Bu,o,c co, so,: Tng 13 + 23 + 33 chia ht cho 9. Ngha la`
,
,
` toan
la` dung,

mnh
d
khi s d
` cua bai
`u tin cua 3 s lin tip
.
la` 1.
,
, ,,
,
` toan
dung

Bu,o,c quy nap:


d
dinh
` khang
. cua bai
.
. Gia su mnh
,
,
3
3
3
voi k, ngha la` k + (k + 1) + (k + 2) chia ht cho 9. Ta se chu ng
,
,
,
`
d
`, (k + 1) khang

minh rang
vo i ba s tu. nhin lin tip bat
`u tu
,
3
` toan
cung

khac
( k + 1) + ( k + 2)3 +
dinh
cua bai
dung,
noi cach
.
(k + 3)3 se chia ht cho 9. Tht
. vy,
.
3
3
3
( k + 1) + ( k + 2) + ( k + 3) = ( k 3 + ( k + 1)3 + ( k + 2)3 ) + 9( k 2 +
3k + 3).

32

,,
,,
`
quy nap
hoc
Chuong 2. Ky thut
phuong phap
. dung
. toan
.

,
,
,
,
d
nhu, tng cua hai
`, k + 1 biu din
Tng ba s lin tip bat
`u tu
,
` cung

s hang
d
chia ht cho 9.
`u chia ht cho 9, th` tng nay
.
V du. 2.10. Chu,ng minh rang
`
moi
`ng (ti`n Vit
. s nguyn d
.
,
,
,
,
,
,
1
`
Nam ) lo n hon 6 co th d

nhu ,ng d
i ra ti`n le khng du bang
`ng
ti`n g`m nhu ,ng t`o, 2 d
`ng.
`ng va` 5 d
,
,
,, ,
,
,
,
,

d
dinh
Lo` i giai. Buo c co so: Vo i s ti`n 7 d
`ng, mnh
` khang
.
.

dung:
7=5+2.
,
, ,,
,
,
,

dinh
dung
vo i s k 7 d
Buo,c quy nap:
`ng.
.
. Gi,a su khang
,
,

chu ng minh di
dinh
dung
vo i s k + 1 d
`u khang
`ng. Ta xet
. cung
,

hai kha nang:


,
,
,,
,
`
1) k du
mt
o. c d
i ch bang
`ng.
. loai
. ti`n to` 2 d
,
,,
,
`
loai
2) k du
cac
o. c d
i bang
`ng.
. ti`n, t nht co mt
. to` loai
. 5 d
,
, ,,
,
`

loai
Ta phai chu ng minh k + 1 d
d
cac
`ng cung
i du
o. c bang
.
,
,
,
`

`
tin da cho. Vo i s (k + 1) d
ng th` ta d
i nhu sau:
,
,, , ,
,
,
- Nu k d
`ng o truo` ng ho. p 1), th` t nht phai co 4 to` 2 d
`ng,
, ,
,
,
,
`
v` k > 6. d
1 to`
i k + 1 d
`ng, ta ly 2 to` loai
`ng d
i thanh
. 2 d
loai
`ng.
. 5 d
, ,
,,
,
- Nu k d
`ng trong truo` ng ho. p 2), th` d
d
i k + 1 d
`ng, ta ly
,
,
,
mt
`ng d
i ly 3 to` loai
`ng.
. to` loai
. 5 d
. 2 d
,
,
hoc

Theo nguyn ly quy nap


dinh
dung
vo i moi
.
. toan
. khang
.
,

`
s n d
ng voi n > 6.
,
,,
V du. 2.11. Chu,ng minh rang
`
n du

khac
nhau trn
o` ng thang
,
,
,
mt
. phang

di
. phang

ra 2n ph`n.
qua mt
m chia mat
. mat
. di
,
,
,
,
,

Lo` i giai. Bu,o,c co, so,: Vo i n = 1 mnh


d
dinh
la` dung,
v`
` khang
.
.
,
,
,`,

. phang

mt
chia mat
ra hai ph`n.
ong thang
. du

11

,
,,
,
d
`ng o dy
ta hiu la` 1000 d
`ng trn thu. c t.

hoc
2.2. Mnh
d
` trong nguyn ly quy nap
.
. toan
.

33

, ,,
,

` d
Gia su mnh
d
Bu,o,c quy nap:
vo i s n nao
` dung
o, ngha la`
.
.
,
,
,
,`,
nhau di

. phang

n du
khac
di
ra 2n
ong thang
qua mt
m chia mat
.
,
,
,
,
`
`

phn. chu ng minh mnh


d
dinh
cung
dung
vo i n + 1
khang
.
.
,
,
,
,`,
,
,
,
`

du
ta chu y rang
nu du. ng du
di
ong thang,
ong thang
qua di
m
,
,,
,,
,
`
` trong s cac
du

da cho va` khng trung


vo i du
nao
o` ng thang
o` ng
,
,
,a cua mat

.
thang
c`on lai,
ta se nhn
thm 2 ph`n nu
. th` chung
.
,
,
,
,
,

. phang

phang.
Nhu vy
da co la` 2n cng
voi 2,
. s phn cua mat
.
. la` 2(n + 1).
hoac
V du. 2.12. Trong thanh
`
ph co n nha.
` T`m s lo,n nht nhu ,ng
,
,,
hang
`
rao
` khep kn khng cat
nhau co th xy du. ,ng du
o. c trong
hang
thanh
`
ph, nu mi
`
rao
` vy quanh t nht mt
. nha` va`
khng co hai hang
`
rao
` nao
` vy quanh mt
`
. cum
. nha.
,
,
,
`
` c`n tnh la` X1 = 1.
rao
Lo` i giai. Bu,o,c co, so,: Khi n = 1 s hang
,
nha` mt
`
` sau d
Khi n = 2 ta co th quy mi
rao
o lai
. hang
.
,
,
,

`
`
`
`
`
du. ng mt
. hang rao quy ca hai nha. Nhu vy
. s hang rao X2 = 3.
,
nha` mt
`
` sau d
Khi n = 3 ta co th quy mi
rao,
o quy
. hang
`
`
` va` sau cung
`
`
`
hai nha` bt ky` bang
mt
rao
la` mt
rao
. hang
. hang
,
` Ta co X3 = 5.
quy ca ba nha.
,
,
,
Do d
o gia thit quy nap:
. Xn = 2n 1. chu ng minh cng
,
,

`
thu c la` dung,
ta co nhn
ph n nha`
. xet sau: i vo i mt
. thanh
,
,
,
di
` lun xy du. ng du

`
`
theo cac
n hang
rao
`u kin
`u bai,
o. c dung
. d
,
,
,

`
` chung cho ca thanh
`
ring cua mi nha` va` ch co mt
rao
. hang

ph.
, ,,
,
,

Bu,o,c quy nap:


vo i moi
.
. Gia su cng thu c Xn = 2n 1 dung
,
,
`

n k va` ta cn chung minh no cung


dung
voi n = k + 1.
,
,
`
` vo i s hang
`
` lo,n nht co th
Ta xet h. thng hang
rao
rao
,
,
,,
`
cac

du. ng du
ph co k + 1 nha` va` thoa man
o. c trong mt
. thanh

34

,,
,,
`
quy nap
hoc
Chuong 2. Ky thut
phuong phap
. dung
. toan
.

,
` Theo nhn
di
`u bai.
o lun co
`u kin
. , xet trong h. th,ng d
. , cua d
,,
,

`
` lo n quy ca thanh
`
`
`
1 (va` ch 1) hang
rao
ph. Gia su hang
rao
,
,
,
`

d
`
`

d
ph du
2 khu bang
2
o bi. bo di
th` luc
o thanh
o. c quy thanh
,
,
,

`
` Khu thu nht chang
` khu thu hai co l

hang
rao.
han
. la` m nha,
` m 1; l 1; m + l = k + 1.
nha:
,
,
`
` quy khu thu, nht cung

H. thng hang
rao
la` lo n nht tu c la`
,
`
` va` quy khu thu, hai co 2l 1 hang
`
`
co tt ca 2m 1 hang
rao,
rao
,
,
,
`
` nao
` co th quy
(theo gia thit quy nap).
Khng th co hang
rao
.
,
,
,
,
ng ngi nha` tu
` 2 khu dang
d
xet d
`ng tho` i nhu

o. Do d
o ch c`on
,
`
` duy nht. o la` hang
`
` chung quy ca thanh
`
lai
rao
rao
. mt
. hang
,
ph. Nhu vy
. ta co
Xk+1 = (2m 1) + (2l 1) + 1 = 2(m + l ) 1

= 2(k + 1) 1.

V du. 2.13. Tu`, 2n s 1, 2, 3, . . . , 2n ta ly ra mt


bt k`y n + 1
. cach
,
s. Chu ng minh rang
`
trong s cac
s ly ra d
o co t nht mt
. s
chia ht cho mt

. s khac.
,
,
,
,
,,
giai sau dy
Lo` i giai. (Phuong phap
la` cua M. Fritman).2 Khi
,

n = 1 mnh
d
la` hin nhin.
` dung
.
, ,,
,
`, 2(n 1) s

Gia su mnh
d
vo i n 1 ngha la` tu
` dung
.
,
,,
,,
1, 2, . . . , 2(n 1) (o dy
n 2) co th chon
o. c n s sao cho
. ra du

trong d
o co t nht mt
. s chia ht cho mt
. s khac.
,
,
,
`, 2n s 1, 2, . . . , 2n
vo,i n. Gia su, tu
Ta chu ng minh mnh
d
` dung
.
,
,,
` la`
ta co th chon
o. c n + 1 s sao cho trong d
o khng co s nao
. du
,
,

Ta ky hiu
bi
ac.
o la` Xn+1 . i
. s cua s kh
. tp
. tt ca n + 1 s d
,
,`,
,
,
voi tp
. Xn+1 xay ra 4 truong ho. p.
2 Bai
`

, ,
,,
`
` cung

irichlet trong [1]


nay
co th giai bang
phuong phap

hoc
2.2. Mnh
d
` trong nguyn ly quy nap
.
. toan
.

35

, ,
1. Xn+1 khng chu a ca 2n 1 va` 2n,
,
,
2. Xn+1 chu a 2n 1 va` khng chu a 2n,
,
,
3. Xn+1 khng chu a 2n 1 va` chu a 2n,
, ,
4. Xn+1 chu a ca 2n 1 va` 2n.
,
,,
,
`, Xn+1 mt
` c`on lai
Truo` ng ho. p 1: Ta bo di
tu
. s bt ky,
. n s ma`
,
,
s d
`
mi
`u khng lo n hon 2n 2 va` trong s d
o khng co s nao
,

la` bi
. cua mt
. s khac.
, ,
,`,
,
ma` moi
` Xn+1 s 2n 1, c`on lai
Truong ho. p 2: Ta bo tu
. la` n s
.
,
,
,
` la` bi
s d
c
ua
m
t
s
`u khng lo n hon 2n 2 va` khng co s nao
.
.

khac.
, ,
,,
,
` Xn+1 s 2n, c`on lai
Truo` ng ho. p 3: Ta bo tu
a` moi
. la` n s m
. s
,
,
,
` la` bi
d
`u khng lo n hon 2n 2 va` khng co s nao
. cua mt
. s

khac.
,
,,
,,
,
Truo` ng ho. p 4: Truo c ht ta thy trong Xn+1 khng chu a s
, ,
,,
,
` Xn+1 hai s 2n 1 va` 2n
` ta bo tu
n do d
o trong truo` ng ho. p nay
,
,
,
` s n ta cung

thm vao
nhn
o. c n s ma` moi
`u khng lo n
. du
. s d
,
,
`
`
hon 2n 2. Ta se chu ng minh rang
trong n s d
o khng co s nao
,
,
,
,

`
`
chu ng minh di
` ta ch cn chu ng
chia ht cho s khac.
u nay
minh:
`, s n khng co s nao
` chia ht cho n va`
1) Trong s d
o tru
` khac,
ngoai
` n.
2) S n khng chia ht cho s nao
,
,
,
,
s d
i`u thu nht la` hin nhin v` tt ca cac
o d
`u khng lo n
,
hon 2n 2.
,
,

i`u thu hai cung


la` hin nhin v` trong Xn+1 s 2n khng
` khac.

chia ht cho mt
. s nao

Vy
d
cho 2n s th` cung
dung
cho
` khng dung
. nu mnh
.

36

,,
,,
`
quy nap
hoc
Chuong 2. Ky thut
phuong phap
. dung
. toan
.

,
vo,i gia thit quy nap.
` mu thun
2(n 1) s. i`u nay
. Vy
. mnh
.
,
,
,
`

`
d
vo i 2n s 1, 2, . . . , 2n vo i n la` s tu. nhin bt ky.
da cho dung

,
,,
,,
c quy nap
2.3. Buo
o. c xy du. ng trn P(k)
. du
,
,,
`
quy nap
hoc,
Trong chu ng minh bang
phuong phap
. toan
. kho
,
,
,
,
` mnh
nht la` buo c quy nap
khan
d
` P(k) sang mnh
. chuyn tu
.
.
,
,
` cung

v du. o, muc
d
nhu cac
sau
ta
` P(k + 1). Trong ph`n nay
.
,
,
,
,
,
` khang
kha nang

xem xet ky cac


bin d
i quy nap
. ,tru. c tip tu
,
,

dinh
dung
cua P(k ) sang khang
dinh
dung
cua P(k + 1).
.
.
V du. 2.14. Chu,ng minh rang
`
2n 1 ( a n + b n ) > ( a + b ) n ,

(2.9)

,
o, dy
a + b > 0, a 6= b, n > 1.
,
,
, ,
,
,

Lo` i giai. Bu,o,c co, so,: Vo i n = 2 dang


thu c (2.9) co dang
.
(2.10)
2( a2 + b2 ) > ( a + b )2 .
,
,

V` a 6= b ta co bt dang
thu c dung
( a b)2 > 0, cng
hai v bt
.
,
,
,
2
` vo i ( a + b) , ta co (2.10).

dang
thu c nay
, ,,
,

` d
Bu,o,c quy nap:
vo i s n = k nao
o,
. Gia su (2.9) dung
2k 1 ( a k + b k ) > ( a + b ) k .

(2.11)
,
,

chu ng minh (2.9) cung


dung
cho n = k + 1, ta nhn hai v
,
,
,

(2.11) voi a + b v` a + b > 0 ta nhn


thu c dung
. bt dang
2k1 ( ak + bk )( a + b) > ( a + b)k+1 .
(2.12)
,
,
,
,
,
,

Nhu vy
vo i n = k + 1 by gio` ta ch
chu ng minh (2.9) dung
. d
,
c`n chu ng minh
2k ( ak+1 + bk+1 ) > 2k1 ( ak + bk )( a + b).

(2.13)

,,
,,
,
2.3. Buo c quy nap
o. c xy du. ng trn P(k )
. du

37

,
,
,
,,
, ,,
,

Sau khi bin d


thu c tuong
n gian hai v ta du
o. c bt dang
i va` do
,,
`, d
du
ong ak+1 + bk+1 > ak b + bk a, tu
o suy ra

( ak bk )( a b) > 0.

(2.14)

,,
,
Xet hai truo` ng ho. p:
1) Nu a > b, va` di
`u kin
. da cho la` a > b, suy ra a > |b|.
,,
k
k

V` vy
o bt phuong tr`nh (2.14) dung.
. a > b . Do d
,
,
,
k
k
2) Nu a < b, ly lun
. tuong tu. ph`n trn ta co a < b , trong
,,
,
,
` (2.14) cung

dung.
Tom lai,
vo i moi
truo` ng ho. p nay
. (2.14) dung
.
,

a 6= b, do d
vo i n = k + 1.
o (2.9) dung

V du. 2.15. Cho day


s 0 < a1 < a2 < < an , va` ei = 1, i =
n
2
1, 2, . . . Chu,ng minh rang
`

ei ai nhn
. t nht Cn+1 gia tri. khac
i =1
,
, ,
,
,
nhau khi ei thay d

xy ra.
i du trong t ho. p 2n kha nang
,
,
,

nhau
2 gia tri. khac
Lo` i giai. Bu,o,c co, so,: Khi n = 1, t`n tai dung
,
,
,
2
`

cua tng ( a va` a) va` C2 = 1, nhu vy


d
dung.
. mnh
.
k
, ,,
,
`

`
Bu,o,c quy nap:
Gi
a
s
u
m
nh
d
d
ung
v
o
i
n
=
k;
ngh

a
l
a

ei a i
.
.
i =1
, ,
,,
2
nhau. Gia su, thm mt
nhn
ph`n tu
. t nht Ck+1 gia tri. khac
.
,
,
,
`
ak+1 , ma` ak+1 > ak . Ta c`n phai ch ra rang
tng se co Ck2+2 gia
,
,
,
C2 gia tri. cua tng khac

tri.
. Theo gia thit quy nap
. da co san
k +1
,,
,
ng gia tri. khac
nhau
nhau sinh boi a1 , a2 , . . . , ak ; ta c`n t`m nhu
,
,
,
,
,
2
2
tng d
cua tng, co s luo. ng la` Ck+2 Ck+1 = k + 1. T`m cac
o
k
k
,
`
sau dy:
. S = ai (nhu, vy
bang
cach
at

. th` S ei ai vo i moi
.
i =1
i =1
,
, ,
`
tng sau S + ak+1 , S + ( ak+1
su. lu. a chon
cac
. ei ), va` chu y rang
nhau va`
ak ), S + ( ak+1 ak1 ), . . . , S + ( ak+1 a1 ) co gia tri. khac

38

,,
,,
`
quy nap
hoc
Chuong 2. Ky thut
phuong phap
. dung
. toan
.

,
,
,
,
,
,a
nhau nu
lo n hon thu. c su. S. Nhu vy
t`n tai
k + 1 gia tri. khac
.
.
,
,
cua tng da cho.

V du. 2.16. Nu a > 0 va` b > 0, th` (n 1) an + bn nan1 b, vo,i


,
,
,
n la` s nguyn du,o,ng; dang

thu,c xy ra khi va` ch khi a = b.


,
, ,,
,
,
,

Lo` i giai. Mnh


d
vo i n = 1. Gia su mnh
d
vo i
` dung
` dung
.
.
n = k,

(k 1) ak + bk kak1 b
,
,
,
`
bu,o,c
xy du. ng mnh
d
cac
` vo i n = k + 1, ta tin hanh
.
,
, ,

1) Nhn hai v bt dang


thu c vo i a
(k 1) ak+1 + bk a kak b.
,
,
` bt dang

2) Cng
thm ak+1 vao
thu c trn
.
kak+1 + bk a kak b + ak+1 .
,
,
3) Chuyn bk a sang v phai ta co
kak+1 kak b + ak+1 bk a.
,
,
, `
` hai v cua bt dang

4) Cng
thm bk+1 vao
thu c nay
.
kak+1 + bk+1 kak b + ak+1 bk a + bk+1 .
,
,
,
,,
,
`

Theo gia thit quy nap


th` bt dang
thu c trn tro thanh
dang
.
,
,
,
,
,

thu c khi va` ch khi a = b. chu ng minh P(k + 1) dung,


ta ch
,
, ,
,
,
`

ra v phai cua bt dang


thu c sau cung
thoa man
kak b + ak+1 bk a + bk+1 (k + 1) ak b
,
,
,
,,
,
,
`

va` bt dang
thu c tro thanh
dang
thu c khi va` ch khi a = b. Tht
.

,,
,,
,
2.3. Buo c quy nap
o. c xy du. ng trn P(k )
. du

39

, , ,,
` duo i ln
vy,
i tu
. ta bin d
kak b + ak+1 bk a + bk+1 (k + 1) ak b,

ak b + ak+1 bk a + bk+1 0,
ak ( a b) + bk (b a) 0,

( ak bk )( a b) 0.
,
, `

Bt dang
thu c nay
dung
(do a b va` ak bk co cung
du), suy
,
,
,,
,
,
,

nguo. c lai
bt dang
thu c ta c`n chu ng minh la` dung
va` dang
thu c
.
,
,
xy ra khi va` ch khi a = b.

V du. 2.17. Chu,ng minh rang


`
Sn = 1 2 + 3 4 + + (1)

n 1

.n = (1)

n 1

n+1
2

, ,
,
,,
,
o, dy
[ x ] la` s nguyn lo n nht nho hon x, n la` s nguyn duong.
,
,
,
,
,, `
ta chu,ng minh cng
Lo` i giai. chu ng minh du
toan,
o. c bai
,
thu c sau
hni n + 1
+
=n
2
2
,
voi moi
. n. Tht
. vy,
.
,
ta co
a) voi n = 2m la` s chan,




hni
1
n+1
+
= [m] + m +
= m + m = n.
2
2
2
,
,
b) vo i n = 2m + 1 la` s le

hni n + 1 
1
+
= m+
+ [m + 1] = m + m + 1 = n.
2
2
2


,,
1+1
,
,
,
,
0

vo i n = 1
Buo c co so: S1 = 1 = (1)
, mnh
d
` dung
.
2

,,
,,
`
quy nap
hoc
Chuong 2. Ky thut
phuong phap
. dung
. toan
.

40

, ,, ,
,
,

Bu,o,c quy nap:


thu c dung
vo i n = k, ngha la`
. Gia su dang


k 1
k 1 k + 1
Sk = 1 2 + 3 4 + + (1) .k = (1)
.
2
Khi d
o

k+1
+ (1)k (k + 1)
Sk+1 = Sk + (1) (k + 1) = (1)
2





k+1
k+2
= (1)k k + 1
= (1)k .
.
2
2
,
,
,
,
, ,,
,
`, dang
`

ang
thu c sau cung
suy ra tu
thu c o ph`n trn. ang
thu c
,
,

cua d
vo i n = k + 1.
` ra dung
k

k 1

,,
,,
,
c quy nap
2.4. Buo
o. c xy du. ng trn P(k + 1)
. du

,
,,
hoc

Buo c quy nap


ang
. trong nguyn ly quy nap
. toan
. c`n kh
,
,
`, P(k ). Nhu,ng nhi`u khi vic
dinh
P(k + 1) suy tu
i tru. c
.
. bin d
`, P(k ) sang P(k + 1) gap
. rt nhi`u kho khan
hoac
khng
tip tu
,
, . ,
,
, ,
,

Khi d
` nguo. c lai
co huong chnh xac.
o ta phai lam
biu din
. d
,
`
`
P(k + 1) ra mnh
d
quy nap.
` cua P(k ) va` tin hanh
.
. Ph`n nay
,
,
,
,
,
,
va` ph`n truo c lin quan mt
ong nhau.
. thit va` tuong du
V du. 2.18. Chu,ng minh rang
`
s zn = 32n+1 + 40n 67 chia ht
,
cho 64 vo,i moi
. s tu. nhin n.
,
,
,
Lo` i giai. Bu,o,c co, so,: z1 = 33 + 40 67 = 0 chia ht cho 64. mnh
.
,

d
vo i n = 1
` dung
, ,,
Bu,o,c quy nap:
o
. Gia su zn chia ht cho 64. Khi d
zn+1 = 32n+3 + 40n 27

= 9(32n+1 + 40n 67) 320n + 576 = 9.zn 64(5n 9)


,

` toan
dung

cung
chia ht cho 64. Bai
vo i moi
. n.

,,
,,
,
2.4. Buo c quy nap
41
o. c xy du. ng trn P(k + 1)
. du
r
q
p

V du. 2.19. Ky hiu


R
=
2
+
2
+
2 + + 2 can
bc
n
.
. hai

1
n l`n. Chu,ng minh rang
` cos n = Rn1 , sin n =
2 R n 2
2
2
2
2
,

voi moi
. n 3.

,
,,

2
,
,
,
,
,

Lo` i giai. Buo c co so: cos


= sin =
, mnh
d
vo i
` dung
.
4
4
2
n = 3 v`
v
u
p

u 1 + cos
t

2+ 2
R2

4
=
=
,
cos 3 = cos =
2
8
2
2
2
v
u
p

u 1 cos

2 2
2 R1
4
sin 3 = sin =
=
=
.
2
8
2
2
2
, ,,
,
,

d
Bu,o,c quy nap:
vo i s tu. nhin k 3. Khi
` dung
.
. Gia su mnh
d
o
v
u
u 1 + cos

t
k

2 + R k 1
R
2
=
= k,
cos k+1 =
2
2
2
2
v
u
u 1 cos

t
k

2 R k 1
2
sin k+1 =
=
.
2
2
2
,
,

Nhu vy,
d
vo i n = k + 1. Theo nguyn ly quy nap
` dung
. mnh
.
.
,
,
hoc
cng thu c dung

toan
vo i moi
. cac
. n 3.

n5
n4
n3
n
V du. 2.20. Chu,ng minh rang
`
+
+

la` s nguyn
5
2
3
30
,
vo i n = 0, 1, 2, ...
,
,
,
,

Lo` i giai. Bu,o,c co, so,: Mnh


d
vo i n = 0.
` dung
.

42

,,
,,
`
quy nap
hoc
Chuong 2. Ky thut
phuong phap
. dung
. toan
.

, ,,
,
,
,

d
Bu,o,c quy nap:
vo i n = k. Ta c`n phai chu ng
` dung
.
. Gia su mnh
minh
( k + 1)5 ( k + 1)4 ( k + 1)3 k + 1
+
+

5
3
30
, 2 ,
,
la` s nguyn. Ta khai trin biu thu c trn
k5 + 5k4 + 10k3 + 10k2 + 5k + k4 + 4k3 + 6k2 + 4k + 1
+
+
5
2
k3 + 3k2 + 3k + 1 k + 1
+

.
3
30
,
Nhom lai
xut hin
. d
. P(k)
k
k5 k4 k3
+ + ) + ((k4 + 2k3 + 2k2 + k) + (2k3 + 3k2 + 2k) + (k2 + k) + 1)
5
2
3
30
,
,
,
nhom
Nhu vy
thit la` s nguyn va` cac
. nhom thu nht theo gia
,
,

sau cung
la` s nguyn, suy ra tng cua chung
la` s nguyn va` d
o
,
`

` toan
vo i n = k + 1.
cung
la` mnh
d
bai
.

V du. 2.21. Chu,ng minh rang


` vo,i moi
. s nguyn n 2 va` | x | < 1
,
,
th` bt dang

thu c sau lun dung:

(1 x ) n + (1 + x ) n < 2n .
,
,
,
,
,
,

Lo` i giai. Bu,o,c co, so,: Khi n = 2 bt dang


thu c dung
hin nhin.
,
, ,,
,
,
,

Bu,o,c quy nap:


su bt dang
thu c dung
vo i n = k. Ta phai
. Gia
,
,
,
,
,

chu ng minh bt dang


thu c cung
dung
vo i n = k + 1; do gia thit
,
` va` gia thit quy nap
d
`u bai
. ta co

(1 x )k+1 + (1 + x )k+1 < [(1 x )k + (1 + x )k ][(1 x ) + (1 + x )]


< 2k .2 = 2k+1 .
,
,
,
,,
,
,

Nhu vy
thu c du
o. c chu ng minh vo i n = k + 1.
. bt dang
,
V du. 2.22. Vo,i moi
`
. x trong 0 x , chu ng minh rang
,,
| sin nx | n sin x, o dy
n la` s nguyn khng m.

hoc
2.5. Quy nap
. toan
. va` phep truy h`i

43

,
,
, ,
,
,

thu c dung
la` tt nhin.
Lo` i giai. Bu,o,c co, so,: Vo i n = 1 bt dang
,
,
,,
,
,

thu c dung
vo i n = k: | sin kx |
Bu,o,c quy nap:
. Gia su bt dang
,
,
,

thu,c cung
k sin x. Ta c`n chu ng minh bt dang
dung
vo i n = k + 1.
Ta xet

| sin(k + 1) x | = | sin(kx + x )| = | sin(kx ) cos x + cos(kx ) sin x |


= | sin(kx ) cos x | + | cos(kx ) sin x |
= | sin(kx )|| cos x | + | cos(kx )|| sin x |
|k sin x | + | sin x | (k + 1) sin x.
,
,,
,
,,
,ng bt dang

Nhu
thu c trn du
o. c suy ra boi 0 x nn sin x
,
,
,
,

0 va` | cos kx | 1. Nhu vy


thu c dung
. ta da chu ng minh bt dang
,

cho n = k + 1. Suy ra no dung


vo i moi
. n 1.

hoc
` phep
truy h`i
2.5. Quy nap
. toan
. va
` toan
ta da xet co lin quan d
s nhu, cp s
Nhi`u bai
n day
,
,
,,
s hang
bang
`

cng,
cp s nhn, ... mi
cua chung
du
o. c biu din
.
.
,
,
,
,
,
ng gia tri. cua s hang
ng s hang
ly nhu
` nhu
cach
truo c no, ngoai
.
.
,
,
,,
,
,
,

khoi d
u
c
ua
d
ay.
Nh
u
ng
cng
th
u
c
s
h
ang
chung
c
ua
d
ay
nhu

.
,,
,
,
Phu,o,ng phap

vy
ngha mt
o. c du
a ra va` coi nhu la` dinh
.
. du
. day.
,
,
nhu vy
hoc.
cho mt
. ,day
. rt ,ging vo i nguyn ly quy nap
. toan
.
,
,

Ta co th dung
quy nap
d
d
inh
ngh

a
m
t
kh
ai
ni
m
m
o
i.
Nh
u
ng

.
.
.
.
,
hoc
`
nha`
nha` toan
g
oi
n
o
l
a
lo
ai
d
inh
ngh

a
quy
n
ap,
ng cac

.
.
. .
. nhu
, ,
tnh goi
khoa hoc
ngha h`i quy. hiu vn d
`
. may
. no la` dinh
.
,
,
` a cua mt
` ta xet mt
nay
ngha giai thu
.
. v du.
. inh
. s nguyn,
,
P(n) = 1 va` nu
ky hiu
. la` P(n) = n!, nhu sau: nu n = 0 ta gan
,
n > 0 gan
gia tri. P(n) = n (n 1) . . . 2 1 ngha la` tch
voi mi

44

,,
,,
`
quy nap
hoc
Chuong 2. Ky thut
phuong phap
. dung
. toan
.

,,
n s nguyn duong d
`u tin. Ta tnh mt
`u
. s gia tri. d
0! = 1,
1! = 1 = 1 0!,
2! = 2 1 = 2 1! = 2,
3! = 3 2 1 = 3 2! = 6.
,
,
,
. la` n = 0 co P(n)=1; hoac
. la`
Nhu vy
. vo i moi
. s tu. nhin n, hoac
,
`
` go. i y ta dinh
n > 0 co P(n) = n (n 1)!. iu nay
ngha theo
.
,
,
quy nap
. cua P(n) nhu sau:
,
Bu,o,c co, so,: Nu n = 0, th` n! = 1 va`
,
dinh,
dinh
Bu,o,c quy nap:
th` ta co th xac
.
.
. Nu n! da xac
`
(n + 1)! bang
(n + 1) n!.
,,
,,
Buo c quy nap
ngha trn trong tin hoc
.
. trong dinh
. nguo` i ta
,
,
,,
,,
ngha kiu nhu trn la`
thuo` ng goi
.
. dinh
. la` Buo c h`i quy va` goi
,,
`
dinh
ngha theo h`i quy. Ta thy rang
dinh
ngha h`i quy o trn
.
.
, , , ,,
hoc.
` toan
` nhu, nguyn ly quy nap
hoan
. toan
. Buo c co so cho ta gia
,
,,
tri. tai
`u 0. Buo c h`i quy cho ta bit nu ta da
. s tu. nhin ban d
,
,,
,,
bit dinh
o. c tai
. ngha tai
. , ngha du
. n th` ta co th dinh
. n + 1 (buo c
,
,
tip theo). Nhu vy
ngha d
`y du cho moi
.
. dinh
. s tu. nhin n.
`, 0 va` tu
`,ng bu,o,c lin tip dinh
tu
inh
ngha xut phat
ngha
.
.
,
,
,
,
,

`
` cang
`
P(n) cho nhung s tu. nhin cang
ngay
lon. Nhung d
tnh
,
,,
,
`, mnh
P(n) ta di
toan
d
d
nguo. c lai,
` lo n nht d
n mnh
` nho
. tu
.
.
nht. V du. ta tnh
3! = 3 2! = 3 2 1! = 3 2 1 0! = 3 2 1 1 = 6
,
Theo dinh
ngha mun tnh gia tri. P(n) ta kim tra xem nu
.
, , , ,, ,
,,
`, n
dung
n = 0 th` ap
buo c co so cua dinh
. , ngha; nguo. c lai
.
. ta tru
, ,
,,
` toan
v` s nguyn nho ho,n, nhu, vy
di
1 d
du
a bai
. buo c h`i quy

hoc
2.5. Quy nap
. toan
. va` phep truy h`i

45

,
,
, , , ,,
` toan
d
xong
giam bc
n khi v` buo c co so va` tnh toan
. cua bai
` toan.
`
hoan
,,
, lp

du
Rt nhi`u ngn ngu
cho ta tnh toan
o. c cng
. tr`nh,cung
,
,

thu c theo dinh


ngha h`i quy, chang
han
sau trong
.
. nhu cu lnh
.
,
lp
ngn ngu
. tr`nh Pascal:
if n = 0 then Pn := 1 else Pn := n (n 1)!;
,,
,
`
D`ong lnh
trn noi vo i chuong tr`nh bin dich
. rang:
.
,

1. Kim tra xem n = 0 co dung


khng ?
, ,

P(n) = 1;
2. Nu tra lo` i la` ung,
th` gan
, ,
P(n) := n (n 1)!;
3. Nu tra lo` i la` Sai, th` gan
,
,
,
,a d
phai goi
tnh tip tuc
l`n nu
tnh (n
. may
. d`ong lnh
.
,
,
,
,
1)! va` cu tip tuc
khng th
. nhu vy
. to i khi n = 0. Mt
. cu lnh
.
h`i quy.
hin
. ht vic
. tnh toan
,,
hoc
Trong toan
. va` tin h, oc
. nguo` i
thut
th hin
ta tao
. ra cac
. toan
. chu
Co nhi`u cach

tr`nh tnh toan.


m
,
,
,
` ng bu,o,c
nhu lit
ta thut
. toan
. k tu
,
,
, `
`
. bang
thu. c hin
so d
khi (biu
. hoac
,

d
ta da thy thut
`). Nhu chung
, .
cung

toan
la` mt
cng
c
u
m
t
a
vic
.
.
.
,

thu. c hin
nguyn
l
y
quy
n
ap
to
an
.
.
,

`
`

hoc,
nh
t
l
a
c
ac
bi
u
d
thu
t

.
. toan

h`i quy. Ta ly v du. t`m thut


. toan
H`nh 2.1:
,
,
,
` a cua s tu. nhin cho
tnh giai thu
,,
truo c ?
,
1. Cho gia tri. s tu. nhin n;
,
. kt qua Pn := 1 va` bin d
2. at
m m := n;

,,
,,
`
quy nap
hoc
Chuong 2. Ky thut
phuong phap
. dung
. toan
.

46

,
,,
,,

di
3. Kim tra m = 0? nu dung
d
n buo c 5; nu sai d
n buo c 4;
,,
4. Tnh Pn := Pn m va` m := m 1; di
d
n buo c 3;

5. Kt thuc.
,
, `
So d
khi th hin
. qua h`nh 1.
,
,
`, cng thu,c h`i quy hoac
mnh
. cac
Nhu vy
d
`
. vic
. chuyn tu
.
,
,
va` biu d

h`i quy sang thut


` th hin
. toan
. qua tr`nh tnh toan
,
,
,
,
,
,
` nhp
liu
` va` ly kt qua ra, cung

buo c tnh
tu
vao
nhu cac
. du
, .
,
,
,
,
d`
phuong phap
quy nap

toan
oi hoi ta phai hiu thu dao
. toan

` ta se d
` tp
hoc.
nay
` cp
n mt
. Trong cun ,sach
. d
. s bai
. v`
,
va` biu d
thut
` d
minh hoa.
. toan
.
, `

Ta du
ngha h. s Newton3
a vao dinh
.
Cnk =

n!
,
k!(n k )!

(n = 0, 1, . . . ; k = 0, 1, . . . , n).

(2.15)

,
`,a va` gian u,o,c thu
`,a s chung, ta nhn
`
Dung
dinh
ngha giai thu
.
.
,,
du
o. c
Cnk =

n ( n 1) . . . ( n k + 1)
,
k!

(n = 0, 1, . . . ; k = 0, 1, . . . , n)
(2.16)

Chung
ta thit lp
tam
gi
ac

Pascal
theo
nguyn
t
ac:
C
t
d
u
. ,
. tin
,,
,
,
` s mt;
`
va` canh
huy`n ch g`m toan
thu n va`
.
. s du ng o hang
,
,
,,
,
`
ct
n 1, tai
. k, la` tng cua hai s o hang
. ct
. thu k 1 va` k.
1
1
1
1
1
...
3 Ky

1
2
3
4
...

,,
,,
hiu
ong Cnk Cnk .
. tuong du

1
3
6
...

1
4
...

1
...

hoc
2.5. Quy nap
. toan
. va` phep truy h`i

47

,
V du. 2.23. Chu,ng minh rang
`
nhu ,ng s trong bang trn la`
,
s du,ng o,, hang
nhu ,ng h. s Newton: mi
`
thu, n va` ct
. thu k
la` Cnk .
,
,
,
,
`

Lo` i giai. Chu ng minh bang


quy nap,
d
` dung.
. vo i n = 0 mnh
.
, ,,
,
,
,
,
,
0
1
k
`
s Cn , Cn , . . . , Cn va` ky
Gia su hang
thu n du
o. c tao
. boi cac
,
,
,
ng s o, hang
`
hiu
thu n + 1. Ta
. n+1,0 , n+1,1 , . . . , n+1,n+1 la` nhu
,
`

dung
se chu ng minh rang
n+1,k = Cnk +1 . Tht
nguyn tac
. vy,
. ap
.
,
,
tao
. bang s va` gia thit quy nap,
. ta co
n!
n!
n+1,k = Cnk1 + Cnk =
+
(k 1)!(n k + 1)! k!(n k)!
n!
1
1
( n + 1) !
=
(
+ )=
.
( k 1) ! ( n k ) ! n k + 1 k
k!(n k + 1)!
,
,
,
Thu. c t ta da chu ng minh cng thu c
Cnk1 + Cnk = Cnk +1 .

(2.17)

,
,,
,
V du. 2.24. Hay
vit thut
va` ve so, d
` khi tuong u ng d

. toan
tnh h. s Newton Cnk khi cho n, k(1 k n).
,
,
,
,
,

Lo` i giai. H`nh 2 th hin


` cua thut
. biu d
. toan.
1. Nhp
. thng s n va` k;
va` so, d
2.Tnh t := k! theo thut
` da bit;
. toan
,
3. Chun bi. tnh n(n 1) . . . (n k + 1), bin i nhn
`u
. gia tri. d
la` n;
,
, `
,
s, kho,,i d
4. ua vao
bin m chu a gia tri. cua mu
`u la` 1;
,
,
,
d
s k buo c vo i gia tri. n(n 1) . . . (i + 1), i < n
5. Bat
`u tnh mu
,
nhn voi i va` nhn
gia tri. m = n(n 1) . . . i.
,
,.
,
6. Gia tri. cua ch s giam di
1.
,
,
, ,
`
7. Kim tra ch s co nho hon s hang
sau cung
n k + 1 khng
.

48

,,
,,
`
quy nap
hoc
Chuong 2. Ky thut
phuong phap
. dung
. toan
.

,,
,,

? nu khng dung
quay v` buo c 5; nu dung
di
tip buo c 8.
,
bin b chu,a gia tri. thu,o,ng cua m = n(n 1) . . . (n k + 1)
8. Gan
va` t = 1, 2, . . . , k.
,
kt qua c = Cnk .
9. Kt thuc:
,
` toan
cho
Trong thu. c t nhi`u bai
s
day

a1 , a2 , . . . , a n , . . .
,
`
dinh
xac
bang
cng thu c h`i quy
.
,
an =
f ( an1 , an2 , . . . , ank ) vo i
` da
1 k n 1 va` f la` mt
. ham
,,
,

bit. Cung
nhu dinh
. ngha o trn khi

`
cho mt
ban d
`u va` ham
. s s hang
.
,

`
s hang
d
f th` cac
cua day
u tnh
.
,,
,

du
o. c. Dy s nhu vy
u goi
. d
. la` day
`
hi quy.

H`nh 2.2:

,,
V du. 2.25. Cho day
s a1 , a2 , . . . , an , . . . du
ngha theo
o. c dinh
.
,
cng thu c sau:

(n + 2)(n + 1) an+2 n2 an = 0,

(n = 1, 2, . . .)

(2.18)

va` a1 = 0, a2 = 1. Hay
t`m an ?
,
,
,
Lo` i giai. Ta vit lai
. cng thu c (2.18 )
n2
an .
(2.19)
(n + 1)(n + 2)
,
,
`
`
,ng s hang
D thy rang
nhu
mang ch s le bang
0. Tht
.
. vy,
.
,

. n = 2k 1, khi d
dat
o voi k = 1, 2, . . . ta co
a n +2 =

a2k+1 =

(2k 1)2
a
.
2k (2k + 1) 2k1

(2.20)

hoc
2.5. Quy nap
. toan
. va` phep truy h`i

49

`, (2.20) theo quy nap


V` a1 = 0, tu
. suy ra a3 = 0, a5 = 0, . . .. Khi ta
,
co cng thu,c

ng s hang
. n = 2k, th` nhu
dat
chan
.
2k2
a .
(k + 1)(2k + 1) 2k
,,
,
`, (2.21) ta tnh du
Vo i k = 1, 2, 3 tu
o. c
2
1
1
a4 =
.a2 =
.1 = ,
2.3
1.3
3
2
2.2
2.4 1
a6 =
.a4 =
. ,
3.5
3.5 3
2.32
2.4.6 1
a8 =
.a6 =
. .
4.7
3.5.7 4
,
,
,
,
s hang

Ta so sanh
mi
vo i ch s va` du
a ra gia thit
.
a2k+2 =

a2k =

2.4.6 . . . (2k 2) 1
.
1.3.5 . . . (2k 1) k

(2.21)

(2.22)

, ,,
,,
,,
`, (2.22) vo,i k = 2, 3, 4 nhn
Tu
o. c a2 , a4 va` a8 o trn. Gia su
. du
,
,
,

` d

(2.22) dung
vo i s k nao
dung
vo i
o, ta se chu ng minh no cung
k + 1:
2.4.6 . . . 2k
1
a2k+2 =
.
(2.23)
, 1.3.5 . . . (2k + 1) k + 1
Tht
. vy,
. do (2.21) va` gia thit quy nap
. ta co
2k2
2.4 . . . (2k 2) 1
.
.
(k + 1)(2k + 1) 1.3 . . . (2k 1) k
2.4 . . . (2k 2)2k
1
.
.
=
1.3 . . . (2k 1)(2k + 1) k + 1
,
,
,,
,
Nhu vy,
o. c chu ng minh. Tom lai,
. cng thu c (2.22) du
.

n = 2k 1 (k = 1, 2, . . .)
0
an = 2.4 . . . (2k 2) 1

.
n = 2k.
1.3 . . . (2k 1) k
a 2( k +1) =

50

,,
,,
`
quy nap
hoc
Chuong 2. Ky thut
phuong phap
. dung
. toan
.

,
,,
V du. 2.26. Chia mat
. phang

thanh
`
bao nhiu ph`n tu`, n du
o` ng
,
,
,,

nao
` d
thang,

di
nhau va` khng co ba du
`ng
mt
o` ng thang
. cat
quy?
,
,
,
,
,,
. F (n) la` s ph`n mat
. phang

Lo` i giai. Ta dat


do n du
o` ng thang
,
,
,,
` ra. n

bt ky` theo gia thit bai


tao
o` ng thang
. ra. Ta, xet n + 1 du
,
,
,`,
,
,

. phang

du
d
ra F (n) ph`n; c`on du
ong thang
`u chia mat
o` ng thang
,
,
,
,,
tai
nhau vo,i n du

thu n + 1, g bi. cat


n di
d
m khac
o` ng thang
`u,
.
,
,
,`,
,
,
,
,

nhu vy
du
g du
ong thang
o. c chia ra n + 1 ph`n. Suy ra du
o` ng
, .
,,
ph`n du
` di,

thang
g di
qua n + 1 ph`n da cho, mi
o. c chia lam

,
do d
o g tao
. thm ra n + 1 ph`n mo i, ngha la`
F (n + 1) = F (n) + n + 1.
,
,
`

Thay n trong dang


thu c trn bang
n 1, n 2, . . . , 2, 1 ta co
F (n) = F (n 1) + n,
F (n 1) = F (n 2) + n 1,
......
F (3) = F (2) + 3,
F (2) = F (1) + 2.
,
,
,,
dang

thu c trn ta nhn


Do F (1) = 2 va` cng
cac
o. c
. du
.
F ( n ) = 1 + (1 + 2 + + n ) = 1 +
. la`
Hoac
F (n) =

n2 + n + 2
.
2

n ( n + 1)
.
2

,
V du. 2.27. (Bai
` toan
thap
Ha` ni).
. Cho ba chic coc.
. Coc
. thu
,,
nht xu n cai
d
nhau sao cho cac
d
a co du
o` ng knh khac
a co
,
,
,
,
,
,
,
,
,
,
du

ta mun chuyn tt ca cac


d
a,
o` ng knh lo n hon o duo i. Chung

,
hoc
hoa
2.6. Quy nap
. toan
. va` tng quat

51

,
l`n mt
xp thu, tu. , tu`,
mi
d
a vn
. chic, sang coc
. thu hai ma` cac
,
,
,,
,
lo,n ln d
coc
n nho. Trong th`oi gian chuyn qua cac
o. c
. khng du
,
,
,
dat
. d
` c`n thit co coc
a lo n ln d
a nho (d
`u nay
. thu ba). S l`n
,
,
t nht d
` b. d
a trong coc
chuyn toan
. mt
. sang coc
. hai la` bao
nhiu?
,
,
,
,
`,
Lo` i giai. Ky hiu
n d
a tu
. Mn la` s l`n nho nht chuyn xong
, ,,
,
`
coc
coc
hai. Ro rang
M1 = 1, nhu vy
ta gia su n > 1.
. , mt
. sang
.
.
,
,
,
,,
,, `
chuyn du
sang ct
n n1
o. c d
a duo i cung
. hai, ta phai chuy
,
,,
,

`
d
Nhu vy
a o trn sang ct
. ba.
. ta co s ln chuyn t nht l,a`
,
,
Mn1 . Mt
a to nht sang ct
. l`n chuyn d
. thu hai, va` lai
. phai
,
,
,
,
,
`

`
`

thu. c hin
a tu ct
. Mn1 ln chuyn s n 1 d
. thu ba v ct
. thu
,
hai. Nhu vy,
.
Mn = 2Mn1 + 1, M1 = 1.
,
,,

`
`

D dang
bang
quy nap
o. c Mn = 2n 1. Tht
. ta chu ng, minh du
.
,
,
,
,
,
,

vy,
v
o
i
n
=
1
cng
th
u
c
d
ung.
Gi
a
s
u
cng
th
u
c
d
ung
v
o
i
s
n
=
k,

.
k
k
k
+
1
Mk = 2 1. Ta xet Mk+1 = 2Mk + 1 = 2(2 1) + 1 = 2
1,
,
,

do d
vo i n = k + 1.
o cng thu c dung

`
hoa

2.6. Quy nap


to
an
h
oc
v
a
t
ng quat
.
.

,
,
,
` toan
d giai ho,n o, dang
Nht
Rt nhi`u bai
tng quat.
.
,
,
,
,
,
`
quy nap
gia
la` chu ng minh bang
phuong phap
khi du. do
an
.
,
,
,
,
mnh

thit quy nap.


han
ta phai chu ng minh day
. Chang
. nhu
.
,
,
,
, ,
d
bu
o
c
quy
` P(1), P(2), . . . khng co du thng tin d
thu. c hin
,.
,`,
,
`

ho,n
nap.
Trong
tru
o
ng
h
o
p
d
o
ta
x
e
t
d
ay
m
nh
d
t
ng quat

.
.
.
,
n mnh
Q(1), Q(2), . . . ma` vo i mi
d
` Q(n) keo theo P(n), va` sau
.
,
,
dung
quy nap
d
phuong phap
o ta lai
. ap
.
. cho Q(1), Q(2), Q(3), . . .
,
` ta xet mt
Trong muc
. nay
. s v du. nhu vy:
.

52

,,
,,
`
quy nap
hoc
Chuong 2. Ky thut
phuong phap
. dung
. toan
.

,
V du. 2.28. Tnh tng
n
1
2
+ + + n.
2 22
2
,
,
,
,
Lo` i giai. Ta nhn
. xet tng trn co th vit lai
. la`
Sn =

(2.24)

Sn ( x ) = 1x + 2x2 + + nx n .
(2.25)
,
,
,
1
,,
,
Tng ta c`n tnh ch la` truo` ng ho. p ring cua (2.25) Sn ( ). Ta co
2
,
, ,,
,
,,
,
`
th dung
ky thut
tnh tng o cui Chuong 1 d
co cng thu c tnh
.
,
,
,
,
` tp).
tng (2.25)(ban
Ta du
a ra cng
. doc
. thu. c hin
. nhu mt
. bai
.
, ,,
,
`
hoc:
thu c o dy
quy nap
va` chu ng minh bang
. toan
.
n
+
1
n
+
2
x ( n + 1) x
+ nx
,
(2.26)
S( x ) =
2
(1 x )
,
vo i x 6= 1.
,
,
x 2x2 + x3
,
,

Vo i n = 1 dang
thu c (2.26) co dang
x
=
hin nhin
.
2
(1 x )

dung.
, ,,
,
,

Gia su (2.26) dung


vo i s tu. nhin n. Khi d
o
Sn+1 = x + 2x2 + 3x3 + + nx n + (n + 1) x n+1
x (n + 1) x n+1 + nx n+2
+ ( n + 1 ) x n +1 =
(1 x )2
x (n + 1) x n+1 + nx n+2 + (n + 1) x n+1 2(n + 1) x n+2 + (n + 1) x n+3
(1 x )2
x ( n + 2 ) x n +2 + ( n + 1 ) x n +3
=
.
(1 x )2
,
,

Nhu vy
vo i n + 1.
. (2.26) dung
1
p dung
(2.26) tai
.
. gia tri. x = 2 ta co
1
1
1
( n + 1 ) n +1 + n n +2
1
n+2
2
2
2
Sn ( ) =
= 2 n .
1
2
2
2
2

,
hoc
hoa
2.6. Quy nap
. toan
. va` tng quat

53

V du. 2.29. Chu,ng minh rang


`
nu A1 + + An = , 0 < Ai
, i = 1, 2, . . . , n, th`
sin A1 + sin A2 + + sin An n sin

.
n

,
,
,
,,
`
hoc
Lo` i giai. Chu ng minh bang
quy nap
n buo c quy nap
. toan
. d
.
`

k, mnh
d
P
(
k
)
d
ung
c
o
d
ang
n
u
A
+

+
A
=
,
0
<
A

1
i
k
.
.
, i = 1, 2, . . . , k, th`
sin A1 + + sin Ak k sin

.
k

,
,
,,

chu ng minh mnh


d
ta cho truo c A1 + +
` P(k + 1) dung
.
,
,
Ak + Ak+1 = , 0 < Ai , i = 1, . . . , k + 1 phai chu ng minh
sin A1 + + sin Ak + sin Ak+1 (k + 1) sin

.
k+1

,
`
Nu ta dung
gia thit quy nap
. th`

k
,
,
,
,,
,,
Ai
khng th suy ra du
o. c buo c tip theo. Do vic
. tng cua cac
,
,
`

bang
dn d
n han
. ch rt nhi`u khi chu ng minh. By gio` ta
,
xet mnh
d
hon Q(n) :
` rng
.
.
sin A1 + + sin Ak1 + sin( Ak + Ak+1 ) k sin

Nu 0 < Ai , i = 1, . . . , n, khi d
o
sin A1 + + sin An n sin

A1 + + A n
.
n

, ,,
`
`

Ta thy rang
Q(n) suy ra P(n). ro rang
Q(1) dung.
Gia su Q(k )

54

,,
,,
`
quy nap
hoc
Chuong 2. Ky thut
phuong phap
. dung
. toan
.

, ,,

dung,
va` gia su co 0 < Ai , i = 1, . . . , k + 1, khi d
o
sin A1 + + sin Ak + sin Ak+1
A + + Ak
k sin( 1
) + sin Ak+1
k
k
A1 + + A k
1
= (k + 1)[
sin(
)+
sin Ak+1 ]
k+1
k
k+1
k
A + + Ak
1
(k + 1)[sin(
( 1
)+
A )]
k+1
k
k + 1 k +1
A + + A k + A k +1
).
(k + 1) sin( 1
k+1
,
,
,
,
,
`

Trong bin d
thu c sau cung
c`n phai chu ng minh
i hai bt dang
,
,
,
,,

cho hai goc co danh


gia nhu trn co th chu ng minh du
o. c (ban
.
,
,
,
, ,

doc
d
`
. hay kim tra lai).
. Nhu vy
. tu su. dung dan cua mnh
.

Q(k + 1) suy ra P(k + 1) cung


dung.
,
V du. 2.30. Cho ui la` s hang
thu, i cua day
Fibonacci. Chu,ng
.
minh rang
`

u2n+1 + u2n = u2n+1 .


,
,
, ,,
,
,
,

Lo` i giai. Khang


dinh
dung
vo i n = 1. Gia su mnh
d
vo i
` dung
.
.
n = k. Khi d
o
u2k+2 + u2k+1 = (uk+1 + uk )2 + u2k+1

= u2k+1 + 2uk+1 uk + u2k + u2k+1


= (u2k+1 + u2k ) + (2uk+1 uk + u2k+1 )
= u2k+1 + (2uk+1 uk + u2k+1 ).
,
,,
,,
,
Theo cng thu c day
Fibonacci th` buo c quy nap
o. c chu ng minh
. du
,
`
` toan
` nu ta ch ra rang
hoan
2uk+1 uk + u2k+1 = u2k+2 . Do co cng
,
thu c d
o nn u2k+1 + (2uk+1 uk + u2k+1 ) = u2k+1 + u2k+2 = u2k+3 .
,
, ,

By gio` ch c`on chu ng minh 2uk+1 uk + u2k+1 = u2k+2 . Ta cung


tin

` tp
2.7. Bai
.

55

,
, ,,
,
,
`
hanh
theo quy nap,
hin nhin, gia su
. vo i n = 1 cng thu c dung
,

dung
vo i n = k ta co
2uk+2 uk+1 + u2k+2 = 2(uk+1 + uk )uk+1 + u2k+2

= 2u2k+1 + 2uk uk+1 + u2k+2


= (2uk+1 uk + u2k+1 ) + (u2k+1 + u2k+2 )
= u2k+2 + (u2k+1 + u2k+2 ).
,
,
,,
,

` toan
ban d

Chung
ta lai
thu c sau
`u, nu dang
. vuo ng phai bai
2
2

dung
u2k+1 + u2k+2 = u2k+3 . Nu co vy
. th` u2k+2 + (uk+1 + uk+2 ) =
,,
` hoan
` toan
` xong.
u2k+2 + u2k+3 = u2k+4 va` buo c quy nap
. ph`n nay
,
,
,

Kt qua la` nu dung


mnh
d
mnh
d
` thu nht th` dung
` thu
.
.
,,
,
` toan
xet hai day
mnh
hai va` nguo. c lai.
d
`
. Thu. c ra bai
.
P(n) : u2n+1 + u2n+1 = u2n+1 ,
Q(n) : 2un+2 un+1 + u2n+2 = u2n+2 .
,
`

P(1) va` Q(1) d


Theo ly lun
`u dung.
. cua ph`n trn thy rang
,

P(k ) va` Q(k ) dung


suy ra P(k + 1) dung,
va` vo i P(k + 1) va` Q(k )
,
,
` P(k ) va` Q(k ) suy ra P(k + 1)

lai
Nhu vy
. suy ra Q(k + 1) dung.
. tu

va` Q(k + 1) d
`u dung.

` tp
2.7. Bai
.
,ng s nguyn du,o,ng.
. 2.31. Cho x1 < x2 < . . . < xn la` nhu
,
,
,

thu c
Chu ng minh bt dang

( x15 + x25 + + xn5 ) + ( x17 + x27 + + xn7 ) 2( x13 + x23 + + xn3 )2 .


(2.27)
,
, ,
,
,
,
`

Chung minh rang


dang
thuc ch xy ra khi va` ch khi xk = k
(k = 1, 2, . . . , n).

56

,,
,,
`
quy nap
hoc
Chuong 2. Ky thut
phuong phap
. dung
. toan
.

,
,

. 2.32. Chu,ng minh bt dang


thu c

( a1 + a2 + + ak )2 k( a21 + a22 + + a2k ),


(2.28)
,,
,
,ng s thu.,c bt
o dy
k 1 la` s tu. nhin va` a1 , a2 , . . . , ak la` nhu
`
ky.
`
. 2.33. Cho f ( x ) = ( x2 1)1/2 , x > 1. Chu,ng minh rang
f (n) ( x ) >
,
,
,

0 voi n le va` f (n) ( x ) < 0 vo i n chan.


,,
`
. 2.34. Chu,ng minh rang
phuong tr`nh x2 + y2 = zn co nghim
.
,
nguyn ( x, y, z) vo i moi
n
=
1,
2,
3,
.
.
.
.

, ,
CHUONG

,
,

T`IM CNG THUC TNG QUAT


` cp s nhn . . . . . . . . . . . . . . . . . . . . . . . .
3.1. Cp s cng
va
, .
,
` s hang
......................
3.2. Tnh tng va
tng quat
.
,,
`

3.3. Phuong tr`nh truy hi tuyn tnh . . . . . . . . . . . . . . . . . . . . . .


,
,
,
,
,
s tu. nhin
3.4. Tng cua nhung l
uy th`ua c`
ung bc
. cac
` tp.
3.5. Bai
. ................................................

57
66
71
84
87

,,
,,
,
,,
quy nap
hoc
Phuong phap
toan
g`m hai buo c nhu hai chuong
.
.
,
,
,
,
,,
Bu,o,c co, so, chuyn sang bu,o,c gia thit quy
truo c ta da khao sat.
,
,

nap
no d`
giai toan,
oi hoi nhi`u kinh nghim
. la` rt quan trong,
.
.
,
,
do
dung,

va` co ly.

phan
du
dinh
chung chnh xac
an
a ra khang
.
,,
,

` ng lai
` ta du
t`m cng thu,c
Chuong nay
vic
thit lp
mt
s cach
.
.
.
.
,
,
s. Sau khi t`m ra
cho cac
mnh

dinh
la` day
tng quat
d
` khang
.
.
,
,
,
`
hoc.
tng ta co th chu ng minh bang
quy nap
. toan
.

` cp s nhn
3.1. Cp s cng
va
.

,
cng va`
chu,o,ng tr`nh ph thng ta da hoc
Trong cac
. cp s
, . ,
,,
,
lai
cp s nhn, o dy
ta nhac
. hai cng thu c tnh tng nhung
,,
,
`
du
quy nap.
o. c chu ng minh bang
.
,
V du. 3.1. Chu,ng minh dang

thu,c sau:
1 + q + q2 + + q n =

1 q n +1
,
1q

(3.1)

,,
, ,

Chuong 3. T`m cng thu c tng quat

58

,
vo,i moi
. q 6= 1 va` vo i moi
. n = 0, 1, 2, . . . (cp s nhn).
,
,
,
`
Lo` i giai. Gia thit quy nap
`u bai.
. co ngay trong d
. Sn = 1 + q + + qn . Vo,i n = 0, S0 = 1 cng thu,c (3.1)
at

dung.
,
, ,, ,
,
,
` d

Gia su vo i s tu. nhin n = k nao


thu c
o ta co dang
Sk =

1 q k +1
.
1q

,
,
,
,
,

Ta se chu ng minh dang


thu c dung
vo i s tu. nhin tip theo,
ngha la`
1 q k +2
.
S k +1 =
1q
Tht
. vy,
.
S k +1 = S k + q k +1 =

1 q k +1
1 q k +2
+ q k +1 =
.
1q
1q

,
,
V du. 3.2. Cho b va` d la` hai s. T`m s hang
tng quat
an cua
.
, , dinh theo cng thu,c sau (cp s cng):
day,
du
o. c xac
.
.
a1 = b, an = an1 + d

(n = 2, 3, . . .).

,
,
,ng s da cho b va` d, cung

Lo` i giai. Ta c`n t`m s hang


an theo nhu
.
,
,
nhu ch s n da cho. Ta tnh mt
. a1 = b, a2 = a1 + d =
. s gia tri:
b + d, a3 = a2 + d = (b + d) + d = b + 2d, a4 = a3 + d = b + 3d. D
,
,
`
dang
du
a ra gia thit quy nap
.
an = b + (n 1).d.

(3.2)

,
,
,

Ta se chu ng minh (3.2) dung


vo i moi
. s tu. nhin n. Tht
. vy,
.
,, ,
,
,
,
,

tnh ph`n trn.


Buo c co so: Vo i n = 1, (3.2) dung
vo i cach

3.1. Cp s cng
va` cp s nhn
.

59

, ,,
,
,

` d
vo i s tu. nhin n nao
Bu,o,c quy nap:
o. Khi
. Gia su ,(3.2) dung
,

` (3.2) cho ta kt qua


d
o, tu
an+1 = an + d = [b + (n 1)d] + d = b + nd.
,
,
,

Nhu vy
vo i moi
. (3.2) dung
. s tu. nhin n.
,
By gio` ta xet cp s cng
. va` cp s nhn m, a` cng sai va` cng
,
,
,
` s, chung

bi
khng la` hang
bin d
tng
i nhu s hang
. cua, chung
.
cua cp s cng
. cp s nhn.
quat
hoac
.

V du. 3.3. Cho mt


s n s hang
. day
.
a1 = a1 , a2 = a1 + d1 , a3 = a2 + d2 , . . . , an = an1 + dn1 , (3.3)
,,
o dy
s d1 , d2 , . . . , dn1 la` mt
vo,i cng sai d 6= 0
day
. cp s cng
.
(cp s cng-c
ng)
. Chu,ng minh rang
`
.
.
1
ak = a1 + (k 1)d1 + (k 1)(k 2)d,
(3.4)
2
n(n 1)(2d1 3d) n(n + 1)(2n + 1)d
Sn = n ( a1 + d d1 ) +
+
.
4
12
(3.5)
,
,
,
,
,
,,
`

Lo` i giai. Cng thu c trn co th chu ng minh bang


phuong phap
,
,
,
,
,
gon
`
tnh toan

quy nap
ng d
phuong phap
ngan
. nhu
. ta ch dung
,
,
,
s (3.3) goi
va` bin d
ng
vo i s hang
thu
i. Day
. la` cp s cng-c
.
.
.
,
`
k, ak , cng sai thu k, dk va` cng sai d. D dang
co ak = a1 + d1 +
,
,
s gia cua (3.3) ta co
d2 + + dk1 . Voi day s la` cac
dk = d1 + (k 1)d,
d1 + d2 + + d k =
Vy
.

d1 + d k
2d1 + (k 1)d
k=
k.
2
2

1
ak = a1 + (k 1)d1 + (k 1)(k 2)d.
2

,,
, ,

60
Chuong 3. T`m cng thu c tng quat
,
,
,,
,
,ng s hang
cp s cng-c
ng
tnh du
Tng cua nhu
o. c nhu sau:
.
.
.
n

Sn =

ak =

k =1

[(a1 + d d1 ) + (d1 2 d)k + 2 k2 ]

k =1

n
3
d n
= n ( a1 + d d1 ) + ( d1 d ) k + k 2 ,
2 k =1
2 k =1
,
,
` dy
Tu
suy ra cng thu c (3.5).

V du. 3.4. Cho mt


s n s hang
. day
.
a 1 = a 1 , a 2 = a 1 + d 1 , a 3 = a 2 + d 2 , . . . , a n = a n 1 + d n 1 ,
,,
,
o dy
s d1 , d2 , . . . , dn1 la` mt
day
. cp s nhn vo i cng bi
. q 6= 1
,
`

(cp s cng-nhn).
Chu ng minh rang

.
a k = a1 +
Sn = n ( a1 +

d 1 ( q k 1 1 )
q1

(3.6)

d1
d ( q n 1)
.
)+ 1
1q
( q 1)2

(3.7)

,
,
,
,
`, gia thit cua bai
` toan
va` cng thu,c cho cp s cng
Lo` i giai. Tu
.
,

va` cp s nhn ta co cng thuc


d k = d 1 q k 1 ,

d1 + d2 + + d k =

d1 ( q k 1)
.
q1

,,
`
Ta d dang
tnh du
o. c
a k = a 1 + d 1 + d 2 + + d k 1 = a 1 +

d 1 ( q k 1 1 )
.
q1

,
,
`, dang
`

Tu
thu c sau cung
suy ra
n

Sn =

ak =

k =1

= n ( a1 +

d1

d1

[(a1 + 1 q ) + q 1 qk1 ]

k =1

d ( q n 1)
d1
)+ 1
.
1q
( q 1)2

3.1. Cp s cng
va` cp s nhn
.

61

V du. 3.5. Cho day


hu ,u han
. n s
a 1 = a 1 , a 2 = a 1 q 1 , a 3 = a 2 q 2 , . . . , a n = a n 1 q n 1 ,

(3.8)

,
o, dy
s q1 , q2 , . . . , qn1 la` cp s cng
vo,i cng sai d 6= 0. Day

day
.
,
,

s (3.8) goi
vo i s hang
thu k, ak , cng bi
. la` cp s nhn-cng
.
.
.
,
,
,

thu k, qk va` cng sai d. Hay


lp
tng quat

. cng thu c tnh s hang


.
,
,
,
,

`
cua cp s nhn-cng
va` tng nhu ng s hang
d
u cua no.
.
.
,
,
Lo` i giai. V` qk = q1 + (k 1)d, th` ta co
k 1

k 1

a k = a 1 q 1 q 2 . . . q k 1 = a 1 q l = a 1 [ q 1 + ( l 1 ) d ]

a1 [q1k1

+ S1k2 dq1k2

. la`
hoac
ak =

k 2
1

Slk2 dl q1kl1 ,

l =0

a1

l =1
k 2 2 k 3
+ S2 d q 1

l =1

2 k 3 2
2 k 2
q1 + Skk
q1 ]
+ + Skk
3d
2d

,
vo i k=2, 3, . . . , n ;
,
vo i k=1 ,

,,
,
,
o dy
S0n = 1 vo i n = 0, 1, 2, . . ., c`on Sin vo i 0 < i n va` i nguyn,
,
,
,
,

,ng thu
`,a s cua mi
tch dang
la` tng cua tt ca cac
p1 p2 ..pi , Nhu
.
,
` 1 d
` toan
` khac
nhau va` nhn
tch la` i s, hoan
n
. gia tri. nguyn tu
,
,
,
n
ng s hang
khng chu a s n
n. Nu trong tng Si ta nhom nhu
.
,
,
,
`,a s, va`
s hang
nhu mt
va` cac
co chu a n nhu mt
. s hang,
.
.
. thu
,,
,
,
,,
` ngoac,
. ta se nhn
sau d
o. c
o o nhom thu hai ta du
a n ra ngoai
. du
Sin = Sin1 + nSin11 .

(3.9)

,
,
`
gia tri. Sin vao
` bang tam giac
mo, rng
Ta dung
(3.9) vit cac
d
n
.
,
`
v cung
nhu sau

,,
, ,

Chuong 3. T`m cng thu c tng quat

62
t
0
1
2
3
4
5
6
7
..
.

0
1
1
1
1
1
1
1
1
..
.

1
3
6
10
15
21
28
..
.

2
11
35
85
175
322
..
.

6
50
225
735
1960
..
.

24
274
1624
6769
..
.

120
1764
13132
..
.

720
13068
..
.

5040
..
.

...
...
...
...
...
...
...
...
...
...

,
,,
Bang trn co nhi`u tnh cht rt hay, v du. du
o` ng cheo la`
,
,
, ,
an = n!, n = 0, 1, 2, . . ., nhung ta khng nghin cu u o dy.

,
,
,
,
`,
By gio` ta t`m cng thu c tng Sn cua cp s nhn-cng.
Tu
.
,
,

(3.9) ta co dang
thu c
a1 = a1 ,
a2 = a1 S00 q1 ,
a3 = a1 (S01 .q21 + S11 .d.q1 )
...............................
2 k 2
ak = a1 (S0k2 q1k1 + S1k2 dq1k2 + + Skk
q1 )
2d
,
,
,
xp theo s mu
dang

Cng
theo v cua cac
thu c trn va` sap
.
,
,
,

tang
d`n cua q1 ta nhn
o. c
. du
2 k 2
Sk = a1 [1 + q11 (1 + S11 d1 + S22 d2 + + Skk
)
1d
2 k 3
+ q21 (1 + S12 d1 + S23 d2 + + Skk
)+
3d

+ q1k2 (1 + S1k2 d) + q1k1 ],


. la`
hoac

n 1 n i 1

Sn = a1 [1 +

i =1

k =0

Ski+k1 dk )q1i ].

3.1. Cp s cng
va` cp s nhn
.

63

V du. 3.6. Cho day


hu ,u han
. n s
a 1 = a 1 , a 2 = a 1 q 1 , a 3 = a 2 q 2 , . . . , a n = a n 1 q n 1 ,

(3.10)

,
o, dy
s q1 , q2 , .., qn1 la` cp s nhn vo,i cng bi

day
. q 6= 1. Day
,
,
,
,

s hu u han
. trn goi
. la` cp s nhn-nhn vo i thu` a s thu k la` ak ,
,
k-cng bi
lp
. la` qk va` cng bi
. q. Hay
. cng thu c tnh s hang
.
,
,
,

tng quat
va` tng n s hang
d
cp s nhn-nhn.
u tin cua day
.
,
,
Lo` i giai. Ta da bit qk = q1 qk1 , khi d
o
ak = a1 q1 q2 . . . qk1 = a1 q1k1 q1+2+3++(k2)
hay la`
ak =
Sn =

a1 q1k1 q

(k 2)(k 1)
2
,

k =1

k =1

a k = a1 q

q1k1 q

k ( k 3)
2
.

,
,
,ng day
s o, cac
bai
` tp
day
cap
. di.
Nhu
goi
la` cac

. trn
.
,,
, ,
,
,a du.,a trn co, so, cua cp s cng
mo rng
hon nu
va` cp s nhn
.
.
,`,
,a, ta ly v
cp s cap
. di
nguoi ta lai
mt
. ghep thm cac
. l`n nu
,u han
`, n s
hu
du. sau: Ta xet day
. tu
b1 , b2 = b1 + a1 , b3 = b2 + a2 , . . . , bn = bn1 + an1
,,
,
s a1 , a2 , . . . , an1 la` cp s cng-c
ng
vo i cng sai d va`
o dy
day
.
.
,
cng sai thu k la` dk . Khi d
o v` bk = b1 + a1 + a2 + + ak1 va` v`
,
,,
,
. ba
(3.4) ta t`m du
thu k cua cp s cap
o. c s hang
.
bk = b1 + (k 1)( a1 + d d1 ) +

(k 1)k(2d1 3d) (k 1)k(2k 1)d


+
.
4
12
(3.11)

,,
, ,

64
Chuong 3. T`m cng thu c tng quat
,
,
,,
,ng s hang
ta t`m du
cua day
Tng nhu
o. c
.
Sn =

k =1

k =1

bk = [(b1 + d1 a1 d)

,,
Ta nhn
o. c
. du

3
11
d
d
+ k ( a1 d1 + d ) + k 2 ( 1 d ) + k 2 ].
2
6
2
6

n(n + 1)(6a1 9d1 + 11d)


(3.12)
12
n(n + 1)(2n + 1)(d1 2d) n2 (n + 1)2 d
+
.
+
12
24
,
,
dung
` tp:
minh hoa
cng thu c trn ta xet bai
. ap
.
.
,
,
V du. 3.7. T`m s hang
tng quat
cua day
cp s cap
. ba sau dy

.
Sn = n(b1 + d1 a1 d) +

2, 5, 9, 17, 32, 57, 95, . . .

(3.13)

,
va` tnh tng n s hang
d
`u tin.
.
,
,
,,
,
,ng hiu
tao
da cho la`
Lo` i giai. Day
. ra boi nhu
. lin tip cua day
3, 4, 8, 15, 25, 38, . . .
,
,
,ng hiu
g`m nhu
ph`n tu, la`
Tip tuc
. tao
. ra day
. cua cac
1, 4, 7, 10, 13, . . .

(3.14)

dy
d
la` cp s cng
`u tin la` d1 = 1 va` cng sai d = 3.
. co s hang
.
,
,
cho day

V` b1 = 2, a1 = 3, theo cng thu c (3.11) s hang


tng quat
.
(3.13) la`
1
bn = (n3 5n2 + 14n 6).
2
,
,
,
V` tng cua n s hang
d
`u theo cng thu c (3.12) ta co
.
1
Sn =
n(3n3 14n2 + 57n + 2).
24

3.1. Cp s cng
va` cp s nhn
.

65

,
V du. 3.8. T`m cng thu,c tnh tng
Sn = 3.2 + 5.5 + 7.8 + + (2n + 1)(3n 1).
,
,
,

thu, my gia tri. ban d


. lai
Lo` i giai. Nu chung
ta lai
lap
cach
`u
.
.
,
,
nh`n ky vao
` d
va` m`y m`o t`m kt qua th` kha vt va. Ta hay
`
,
`
` va` dat
an = (2n + 1)(3n 1). Ta thy rang
. s hang
bai
tng quat
.
,
,
,
,
` a s thu nht la` cp s cng
` a s thu hai cung

thu
va` thu
la` cp s
.
,
,
,
,
` cng thu c da co cua cp s cng
. ra la` du. a vao
cng.
Cu hoi dat
.
.
,
,
,,
,, ,
,
` toan
co th phai du
co tnh du
o. c tng Sn o trn khng? Bai
a v`
,
,,
,
ho,n:
truo` ng ho. p tng quat
,
,
Tnh tng Sn = a1 b1 + a2 b2 + + an bn vo i a1 , a2 , . . . , an la` cp
s cng
co cng sai d a va` b1 , b2 , . . . , bn la` cp s cng
co cng sai
.
.
db . Ta co
n

Sn =

a k bk =

k =1

[a1 + (k 1)da ].bk =

k =1
n

= ( a 1 d a ) bk + d a
k =1

[(a1 da ) + kda ].bk

k =1

k.bk

k =1

n
2b + (n 1)db
n + d a k.[(b1 db ) + kdb ]
= ( a1 d a ) 1
2
k =1

= ( a1 d a )

n
n
2b1 + (n 1)db
n + d a ( b k + d a db k2
2
k =1
k =1

1
= ( a1 d a )(b1 db )n + [d a (b1 db ) + db ( a1 d a )]n(n + 1)+
2
1
+ d a db n(n + 1)(2n + 1).
6
. la`
Hoac
1
1
Sn = d a db n3 + (d a b1 + db a1 d a db )n2
3
2
1
+ (6a1 b1 3db a1 3d a b1 + d a db )n.
(3.15)
6

,,
, ,

Chuong 3. T`m cng thu c tng quat

66

,
` v du. trn vo,i a1 = 3, d a = 2, b1 =
p dung
cng thu c (3.15) vao
.
,
2, db = 3. Ta co kt qua
1
n(4n2 + 7n + 1).
2
,
, ,
,,
,
,
trn da tnh du
Cng thu c tng quat
o. c qua tru. c tip bin d
i. Vo i
,
, `
,
`

hoc
cng thu c nay
ta cung
co th chu ng minh bang
quy nap
. toan
.
,
,
`
`

(danh
cho ban
d
oc).
Nhu
ng
nhi
u
khi
ch
u
ng
minh
b
ang
quy
n
ap

.
.
,
,
,.
,
,

hoc
`
`
toan
d
n
d
n
bi
n
d
i
bi
u
th
u
c
v
c
ung
ph
u
c
t
ap
l
am
n
an

.
.
,
,
,
phu,o,ng phap

ch manh
` toan

l`ong chung
ta. Mi
v
o
i
m
t
l
.
. o p bai
` d
nao
o thi.
Sn =

,
,
` s hang

3.2. Tnh tng va


t
ng quat
.

,
,,
,,
cac
phu,o,ng an
khac
nhau cua
Chuong truo c ta da quan sat
,
,,
quy nap,

phuong phap
nhi`u khi di
s hang
tng quat
t`m mt
.
.
.
,
,
,
,
,
,
nguo` i ta ap
dung
quy
. mt
hoac
phuong phap
. tng cua mt
. day
.
,
,
,
hoc
hin nhin do cac
buo c h`i quy lin tip
nap
. toan
. mt
. cach
,
,
,
,
,ng v du. sau dy
Nhu
ma` ta t`m du
o. c cng thu c tng quat.
minh
,,
quy nap
`
hoa
. phuong phap
. nay.
,,
,
s a0 , a1 , a2 , . . . du
V du. 3.9. Dy
sau: Hai
o. c xy du. ng theo cach
,
s sau d
s d
`u a0 va` a1 la` nhu ng s da cho, mi
o la` trung b`nh
,
,
,
,
an theo a0 , a1 va` n.
cng
cua hai s truo c d
biu din
o. Hay
.
,
,
Lo` i giai. Ta co
a0 + a1
a1 + a2
a2 + a3
a3 + a4
a2 =
, a3 =
, a4 =
, a5 =
,...
2
2
2
2
`, d
tu
o suy ra
a0 a1
a1 a2
a2 a3
a2 a1 =
, a3 a2 =
, a4 a3 =
,...
2
2
2

,
,

3.2. Tnh tng va` s hang


tng quat
.
Suy ra

67

a1 a0
2
a2 a1
a a0
a3 a2 =
= 1 2
2
2
a3 a2
a1 a0
a4 a3 =
=
2
23
................
a2 a1 =

,,
,,
`
quy nap
hoc
D thy rang
(phuong phap
dung
o. c ap
. toan
. du
.
,,
o dy)

a1 a0
an an1 = (1)n1 n1 .
2
,
,
,
dang

Cng
theo v cua cac
thu c trn, ta co
.
a1 a0
a a0
a a0
a a0
a n a1 =
+ 1 2 1 3 + + (1)n1 1 n1
2
2
2
2
a1 a0
1
1
1
=
(1 + 2 + + (1)n2 n2 )
2
2 2
2
a1 a0
n 1 1
{(1)
1}.
=
3
2n 1
`, d
Tu
o suy ra
2a1 + a0
a a0
an =
+ (1)n1 1 n1 .
3
3.2
,
,
V du. 3.10. Day
s a1 , a2 , a3 , . . . du
dinh
theo cng thu,c
o. c xac
.

a1 = 2 va` an = 3an1 + 1.
Hay
tnh a1 + a2 + + an .
,
,
,
,

Lo` i giai. Ta xet dang


thu c ak = 3ak1 + 1. Cho k gia tri.
n
n
,,
2, 3, 4, . . . , n va` cng
o. c ak = 3 ak1 + n 1. Ta
. lai
. ta nhn
. du
k =2

k =2

. a1 + a2 + + an = S. Khi d
dat
o ta co S a1 = 3(S an ) + n 1.
Suy ra
1
S = {3an a1 n + 1}.
2

68

,,
, ,

Chuong 3. T`m cng thu c tng quat

,
,
an qua a1 . Ta co
Ta ch c`on biu din
an = 3an1 + 1, an1 = 3an2 + 1.
`, d
Tu
o suy ra an an1 = 3( an1 an2 ). V` th
a n a n 1 = 3 ( a n 1 a n 2 ) = 32 ( a n 2 a n 3 ) =

= 33 ( an3 an4 ) = = 3n2 ( a2 a1 ).


,
n2 (quy nap toan

Nhung a2 = 3a1 + 1 = 7, v` vy
. an an1 = 5.3
.
,,
,
`
`
gia tri. n bang

hoc
o dy).
Vo i cac
2, 3, 4, . . . , n ta co

. da dung
a2 a1 = 5.1,
a3 a2 = 5.3,
a4 a3 = 5.32 ,
............
an an1 = 5.3n2 .
,
,
,
,,
v cua dang

Cng
theo cac
thu c, ta tnh du
o. c
.
a n a 1 = 5 ( 1 + 3 + 32 + + 3 n 2 ) =

5 n 1
(3
1).
2

,
,
`, biu thu,c cua S ta co
Tu
1
S = {3( an a1 ) + 2a1 n + 1} =
2
1 15 n1
1
= { (3
1) + 4 n + 1} = {5(3n 1) 2n}.
2 2
4
V du. 3.11. Day
s a1 , a2 , . . . xac
dinh
theo cng thu,c
.
,
an theo a1 , k, l va` n.
an = kan1 + l (n = 2, 3, . . .). Hay
biu din

,
,
Lo` i giai. Ta co an = kan1 + l, an1 = kan2 + l. Suy ra
a n a n 1 = k ( a n 1 a n 2 ) = k 2 ( a n 2 a n 3) = . . . = k n 2 ( a 2 a 1 ),

,
,

3.2. Tnh tng va` s hang


tng quat
.
`, d
tu
o suy ra

69

a2 a1 = ( a2 a1 ),
a3 a2 = k ( a2 a1 ),
a4 a3 = k 2 ( a2 a1 ),
.........

a n a n 1 = k n 2 ( a 2 a 1 ).
,
,
dang

Cng
lai
thu c trn ta co
.
. cac
k n 1 1
l.
k1
,
,
V du. 3.12. Cho day
a1 , a2 , . . . thoa man
dang

thu,c sau:
,
an theo a1 , a2 va` n.
an+1 2an + an1 = 1. Hay
biu din
a n = k n 1 a 1 +

,
,
,
,
,,

Lo` i giai. Ta vit lai


thu c duo i dang
sau:
. dang
.
an+1 an ( an an1 ) = 1.
. an an1 = xn , (n = 2, 3, . . .). Khi d
Ta dat
o ta co xn+1 xn = 1.
,
,
`
` dang
`
thu c sau cung
2, 3, . . . , n 1 va` cng
Thay vao
gia tri. n bang
.
,,
lai,
o. c xn x2 = n 2.
. ta nhn
. du
` an an1 = xn va` cng
Ta lai
lai
. thay n = 3, 4, . . . , n vao
.
. ta
,,
nhn
o. c
. du
a n a2 = x3 + x4 + + x n .
Hay la`
a n = a2 + x3 + x4 + + x n .
,
Nhung
n

x k = ( x2 + k 2)

k =3

k =3

= ( n 2) x2 + ( n 2) + ( n 3) + + 1
(n 1)(n 2)
.
= ( n 2) x2 +
2

,,
, ,

Chuong 3. T`m cng thu c tng quat

70
`, d
Tu
o suy ra

(n 1)(n 2)
2
(n 1)(n 2)
= a2 + (n 2)( a2 a1 ) +
2
(n 1)(n 2)
=
+ ( n 1) a2 ( n 2) a1 .
2

a n = a2 + ( n 2) x2 +

V du. 3.13. Cho hai day


s
a1 , a2 , a3 , . . .
b1 , b2 , b3 , . . .
,
,
du
dinh
theo cng thu,c sau:
o. c xac
.
a n + bn
2an bn
a n +1 =
;
bn + 1 =
2
a n + bn
,
,
o, dy
a0 va` b0 la` nhu ng s da cho a0 > b0 > 0. Tnh an va` bn theo
a0 , b0 va` n.
,
,
`
Lo` i giai. D thy rang
an+1 bn+1 = an bn , va` suy ra an bn = a0 b0
,
,
voi moi
. s nguyn n. Nhung
p

a n bn
a n a n 1 bn 1
a n a n bn
p
=

a n + bn
a n + a n bn
a n + a n 1 bn 1
a n 1 + bn 1 p
!2
p

a n 1 bn 1
a

b
n

1
n

1
2
p
= a +
=
.
p
n 1 bn 1
a n 1 + bn 1
+ a n 1 bn 1
2

a n bn
= un . Khi d
.
Ta dat
o ta co
a n + bn
un1 = u2n2 ,
un2 = u2n3 ,
......

,,
3.3. Phuong tr`nh truy h`i tuyn tnh

71

u2 = u21 ,
u1 = u20 .
,
,
,
,,
,
`,a cua cac

dang

Nng bc
thu
thu c l`n luo. t vo i
. luy
n 1
,,
,
1, 2, 22 , . . . , 2n2 . Ta tnh du
o. c un1 = u20 . Nhung
p

a n 1 bn 1
an1 a0 b0
p

u n 1 =
=
,
an1 + a0 b0
a n 1 + bn 1

a0 b0
a0 a0 b0
=

.
u0 =
a0 + b0
a0 + a0 b0
,
Nhu vy
. ta co


 2n 1
an1 a0 b0
a0 a0 b0

=
.
an1 + a0 b0
a0 + a0 b0

,,
3.3. Phuong tr`nh truy h`i tuyn tnh
,,
bai
` toan
d
s va` cac
s hang
Muc
`u cho day
. truo c ta thy cac
.
,
,,
,
,
`
giai d
du
cng thu c truy h`i. Cach
o. c lin h. voi nhau bang
`u t`m
,
,
,
,,
trong cng thu c truy h`i mi lin h. d
tng
tnh du
o. c s hang
.
,
,
va` tng n s hang
nao
` ta cung

quat
d
`u tin. Khng phai luc
.
,
,
,
`n tru,o,c.
phuong tr`nh truy h`i dep
bai
` tp
co cac
d
e nhu cac
. ph
,
,.
,
` ta ch ra mt
tng quat
tnh s hang
va`
Muc
nay
cach
tng quat
.
,.
,.
,
,
,
ng day
thoa man
phuong tr`nh truy h`i.
tng nhu
,
s
Cho k s hang
d
`u cua day
.
x1 , x2 , x3 , . . . , x n , . . .
(3.16)
,
,
s hang

la` x1 = u1 , x2 = u2 , . . . , xk = uk . Mi
thu k + 1 cua day
.
(3.16) t`n tai
. mi lin h.
x k + n + a 1 x n + k 1 + a 2 x n + k 2 + + a k x n = bn ,
(3.17)
,,
,ng s da cho, c`on b1 , b2 , . . . , bn , . . . la` day

o dy
a1 , a2 , . . . , ak la` nhu
,,
,
,
da cho. Khi d
o (3.17) cho phep ta tnh du
o. c moi
. ph`n tu lin tip

,,
, ,

72
Chuong 3. T`m cng thu c tng quat
,
,
,

s hang
(3.16) va` sau d

biu din
tng quat
cua day
o ta c gang
.
,

cua chung.
Xet v du.
,
,
V du. 3.14. T`m cng thu,c tng quat
cua day
xac
dinh
nhu, sau:
.
x1 = 5, x2 = 19,

xn 5xn1 + 6xn2 = 0.

,
,
,,
`, mi lin h. h`i quy xn = 5xn1 6xn1 , ta t`m du
Lo` i giai. Tu
o. c
x1 = 5 = 32 22 , x2 = 19 = 33 23 ,
x3 = 5x2 6x1 = 65 = 34 24 , x4 = 5x3 6x2 = 211 = 35 25 ,
x5 = 5x4 6x3 = 665 = 35 25 , x6 = 5x5 6x4 = 2059 = 36 26 .
,
,
,
`
cua day
co dang
Ta gia thit rang
s hang
tng quat
.
.
(3.18)
x n = 3n +1 2n +1 .
,
,,
,
,,
`
quy nap.
Gia thit du
phuong phap
o. c chu ng minh bang
.
,
,

1) Ta da kim tra (3.18) dung


vo i n = 1 va` n = 2.
,
,

2) Ta gia thit (3.18) dung


vo i n = k va` n = k + 1, ngha la`
,,
xk = 3k+1 2k+1 va` xk+1 = 3k+2 2k+2 . Khi d
o ta t`m du
o. c
xk+2 = 5xk+1 6xk = 5(3k+2 2k+2 ) 6(3k+1 2k+1 )

= (15 6)3k+1 (10 6)2k+1 = 3k+3 2k+3 .


,
,

Nhu vy
vo i n = k + 2. Theo nguyn ly quy nap
. (3.18) dung
. toan
,

hoc
vo i moi
. suy ra (3.18) dung
. n.
,,
,,
Phuong tr`nh (3.17) goi
. phuong tr`nh h`i quy tuyn tnh bc
.
,ng h s, c`on b goi la` s
,ng s a1 , a2 , a3 , . . . , ak goi

`
k. Nhu
l
a
nh
u
n .
.
.
, do. Khi vo,i moi n, b = 0 phu,o,ng tr`nh goi la` thu`n
hang
t
u
n
.
. ,
.
.
,
s hang
` toan
dat

. ra la` t`m cach


nht. Bai
biu din
tng quat
.
,
,
,
dai
` ta ch xet
xn qua cac
`u da cho. Trong muc
. luo. ng ban d
. nay
,,
phuong tr`nh truy h`i bc
. hai.

,,
3.3. Phuong tr`nh truy h`i tuyn tnh

73

,
V du. 3.15. T`m s hang
tng quat
xn cho phu,o,ng tr`nh truy h`i
.
thu`n nht bc
. hai:
a0 xn+2 + a1 xn+1 + a2 xn = 0.

(3.19)

,
,
,
,,
Lo` i giai. Ta goi
. t1 va` t2 la` nghim
. cua phuong tr`nh bc
. hai
a0 t2 + a1 t + a2 = 0 ( a0 6= 0, a2 6= 0).
,
Khi d
o theo cng thu c Viet a0 (t1 + t2 ) = a1 , a0 t1 t2 = a2 .
,
,,
,,
Phuong tr`nh (3.19) co th vit duo i dang
.
xn+2 (t1 + t2 ) xn+1 + t1 t2 xn = 0.
(3.20)
,
,
cua day
da cho phu. thuc
` hai gia tri. t1 va`
S hang
tng quat
.
. vao
`
nhau hoac
. bang

t2 khac
nhau:
,
,
,
1) Truo` ng ho. p t1 6= t2 :
,,
`
` (3.20) ta nhn
Thay n bang
n 2 vao
o. c
. du
xn (t1 + t2 ) xn1 + t1 t2 xn2 = 0.

(3.21)

Ta vit lai
.
x n t 2 x n 1 = t 1 ( x n 1 t 2 x n 2 ).
(3.22)
`
gia tri. n 1, n 2, . . . , 4, 3 vao
` (3.22) ta nhn

Thay n bang
cac
.
,,
du
o. c
x n 1 t 2 x n 2 = t 1 ( x n 2 t 2 x n 3 )
x n 2 t 2 x n 3 = t 1 ( x n 3 t 2 x n 4 )
......

(3.23)

x4 t2 x3 = t1 ( x3 t2 x2 )
x3 t2,x2 = t1 ( x2 t2 x1 ).
,
`,a s
v tu,o,ng u,ng cua (3.22) va` (3.23), gian u,o,c thu
Nhn cac
,,
chung, ta nhn
o. c
. du
xn t2 xn1 = t1n2 ( x2 t2 x1 ).

(3.24)

74

,,
, ,

Chuong 3. T`m cng thu c tng quat

,,
,
,,
Tuong tu. vit (3.21) duo i dang
.
x n t 1 x n 1 = t 2 ( x n 1 t 2 x n 2 )
`
gia tri. n 1, n 2, . . . , 4, 3, cung

va` sau d
cac
tnh
o thay n bang
,,
du
o. c
xn t1 xn1 = t2n2 ( x2 t1 x1 ).
(3.25)
,
` (3.24) va` (3.25) ta co
Tu
t1 xn t2 xn = t1n1 ( x2 t2 x1 ) t2n1 ( x2 t1 x1 ).
,
. la` nghim
Hoac
. cua (3.19) co dang
.
xn = C1 t1n + C2 t2n ,
(3.26)
,,
t
x

x
x

t
x
2
2
2 1
1 1
o dy

, C2 =
.
(3.27)
C1 =
t1 ( t1 t2 )
t2 ( t1 t2 ) ,
,,
`
,ng hang
` kim tra tru.,c tip
Nguo. c lai,
s bt ky,
. C1 va` C,2 la` nhu
,
,
,,
vo,i s hang
(3.26) la` nghim
day
tng quat
cua phuong tr`nh n
.
.
,
,,
{ xn } du
(3.19). Gia tri. duy nht cua day
dinh,
nu cho hai
o. c xac
.
,
,
Khi d
dinh
gia tri. ban d
theo cng thu c
`u cua day.
o C1 va` C2 xac
.
(3.27).
,
,,
,,
Tro lai
. v du. trn th` t1 = 3, t2 = 2 la` nghim
. cua phuong
2
`
,ng hang
tr`nh bc
o nu C1 va` C2 la` nhu
. hai t 5t + 6 = 0. Khi, d
,
` th` day
vo i s hang

s bt ky,
tng quat
.
xn = C1 .3n + C2 .2n

,
,,
,
la` nghim
. cua phuong tr`nh truy h`i xn 5xn1 + 6xn2 = 0. Vo i
,,
,
dinh
x1 = 5, x2 = 19, ta xac
du
o. c C1 = 3, C2 = 2 va` nhu vy
.
. s
,
,
cua day
la` xn = 3n+1 2n+1 .
hang
tng quat
.
,,
,
,ng s phu,c., ngha
Trong truo` ng ho. p nghim
t1 , t2 la` nhu
.
,
la` t1 = (cos + i sin ), t2 = (cos i sin ), theo cng thu c
Moivre ta co
t1n = n (cos n + i sin n),

t2n = n (cos n i sin n)

,,
3.3. Phuong tr`nh truy h`i tuyn tnh

75

,
,
(3.26) tro, thanh
`
va` nghim
. tng quat
xn = P1 n cos n + P2 n sin n,

(3.28)

,
,,
`
`
` xac
dinh
o dy
s bt ky.
P1 va` P2 trong
P1 va` P2 la` hang
. hang
(3.28) khi cho hai gia tri. d
`u x1 va` x2 , ta co

( t1 + t2 ) x1 x2
,
t1 t2
(t1 + t2 ) x2 (t21 + t22 ) x1
,
P2 = i (C1 C2 ) = i
t1 t2 ( t1 t2 )
P1 = C1 + C2 =

. la`
hoac
P1 =

2
cotg
cos 2
1
x2
cos .x1 2 .x2 , P2 =
.x1 .
2

sin

(3.29)

,,
,
,,
,

ta
2) Tru,o`,ng ho.,p t1 = t2 : Nhu trong truo` ng ho. p truo c chung
,
,
` (3.24) va` (3.25) vo i t1 = t2
suy ra tu
xn t1 xn1 = t1n2 ( x2 t1 x1 ).

(3.30)

,
,,
,,
,a cho xn va`
dinh
xac
du
o. c xn c`n t`m mt
.
. phuong tr`nh nu
,
`, phu,o,ng tr`nh xn 2t1 xn1 + t21 xn2 = 0 co th vit du,o,i
x n 1 . Tu
dang
.

(n 2) xn 2(n 2)t1 xn1 + (n 2)t21 .xn2 = 0


. la`
hoac

(n 2) xn (n 1)t1 xn1 = t1 [(n 3) xn1 (n 2)t1 xn2 ].


(3.31)
,
,
,
`
` (3.31) ta
Sau khi th n tuong u ng bang
n 1, n 2, . . . , 4, 3 vao

,,
, ,

Chuong 3. T`m cng thu c tng quat

76
,,
nhn
o. c
. du

(n 3) xn1 (n 2)t1 xn2 = t1 [(n 4) xn2 (n 3)t1 xn3 ],


(n 4) xn2 (n 3)t1 xn3 = t1 [(n 5) xn3 (n 4)t1 xn4 ],
......
2x4 3t1 x3 = t1 ( x3 2t1 x2 ),
x3 2t1 x2 = t21 x1 .

(3.32)
,
,,
,
,
,,
,
`

v tuong u ng cua (3.32) vao


` (3.31) ta nhn
Ln luo. t th cac
o. c
. du

(n 2) xn (n 1)t1 xn1 = t1n1 .x1 .


,,
`, (3.30) va` (3.33) ta t`m du
Tu
o. c

(3.33)

(n 1) xn (n 2) xn = (n 1)t1n2 ( x2 t1 x1 ) + t1n1 x1 ,
. la`
hoac
,,
o dy

xn = (C1 + C2 .n)t1n ,

(3.34)

x2 t1 x1
2t1 x1 x2
, C2 =
.
C1 =
2
t1
t21
, ,
, ,
`
Ban
(3.34) la` kt qua cua (3.26) khi
. doc
. co th kim tra d dang
ta cho t2 = t1 .

,
,
V du. 3.16. T`m s hang
tng quat
cua day
xac
dinh
theo cng
.
.
,
thu c sau:
x1 = 10, x2 = 12, xn + 4xn2 = 0.
,
,
,,
`, phu,o,ng tr`nh t2 + 4 = 0 ta tnh du
Lo` i giai. Tu
o. c t1 = 2i =
2(cos 2 + i sin 2 ), t2 = 2i = 2(cos 2 i sin 2 ) hay la` = 2, =
,
,,

o nghim
. cua phuong tr`nh truy h`i la`
2 . Khi d
n
n n
xn = ( P1 cos
+ P2 sin
)2 .
2
2

,,
3.3. Phuong tr`nh truy h`i tuyn tnh

77

,
,
`, (3.39) suy ra P1 = 3, P2 = 5, do d
phai t`m
Tu
tng quat
o s hang
.
la`
n
n n
xn = (3 cos
+ 5 sin
)2 .
2
2
. la`
hoac

,
3.2n
vo i n = 4k

,
n
5.2
vo i n = 4k + 1
xn =
,
3.2n vo i n = 4k + 2

5.2n vo,i n = 4k + 3
,
,
V du. 3.17. By gi`o, ta quan tm cu hoi t`m nghim
. ring cua
,
phu,o,ng tr`nh truy h`i khng thu`n nht bc
`
. hai vo i h. s hang
s
a 0 x n + 2 + a 1 x n + 1 + a 2 x n = bn

( a0 6= 0, a2 6= 0).

(3.35)

,
,
,
,,
2

Lo` i giai. Ta cung


xet nghim
. t1 , t2 cua phuong tr`nh a0 t + a1 t +
a2 = 0.
,ng
1) Tru,o`,ng ho.,p t1 6= t2 : Khi d
o theo ph`n trn t1n va` t2n la` nhu
,
,,
nghim
. ring cua phuong tr`nh thu`n nht (3.17) va` co nghim
.
,

theo cng thuc (3.26).


,
,,
,,
Nghim
cua phuong tr`nh (3.35) ta se t`m theo phuong
. ring
,
,
,
`
bin d
phap
s nhu sau: Ta phai t`m nghim
i hang
. ring n co
dang
.
n = n t1n + n t2n ,
(3.36)
,
,
(3.26) la` h. s n va` n co th phu. thuc
` n. Ta co
no ch khac
. vao
n+1 = n+1 t1n+1 + n+1 t2n+1

= n t1n+1 + n t2n+1 + (n+1 n )t1n+1 + ( n+1 n )t2n+1 .


,
,
day
{n } va` { n } thoa man

. di
By gio` ta dat
`u kin
. cho cac
t1n+1 + t2n+1 = 0,

(3.37)

,,
, ,

Chuong 3. T`m cng thu c tng quat

78

,,
o dy
= n+1 n , = n+1 n . Khi d
o
n+1 = n t1n+1 + n t2n+1
n+2 = n+1 t1n+2 + n+1 t2n+2

= n t1n+2 + n t2n+2 + t1n+2 + t2n+2 .


,
,
,,
` (3.35), ta nhn
Ta th biu thu c n , n+1 , n+2 vao
o. c
. du
( a0 t1n+2 + a1 t1n+1 + a2 t1n ).n + ( a0 t2n+2 + a1 t2n+1 + a2 t2n ). n +
+ a0 (t1n+2 + t2n+2 ) = bn .
Do ( a0 t1n+2 + a1 t1n+1 + a2 t1n ).n = 0 va` ( a0 t2n+2 + a1 t2n+1 + a2 t2n ). n =
0 suy ra
bn
t1n+2 + t2n+2 = .
(3.38)
a0
,
,
,
` (3.38) va` (3.17) ta t`m du
Tu
o. c
= n+1 n =

bn
1
.
a0 t1n+1 (t1 t2 )

(3.39)
bn
1
. n +1
.
a0 t1 ( t2 t1 )
,,
`
gia tri. tu,o,ng u,ng n 1, n
Sau khi th l`n luo. t n bang
cac
,,
` (3.39) ta nhn
2, . . . , 2, 1, 0 vao
o. c
. du
= n+1 n =

bn 1
1
. n
,
a0 t1 ( t1 t2 )
bn 2
1
=
. n 1
,
a0 t1 ( t1 t2 )

n n 1 =
n 1 n 2

......
b2
1
. 3
,
a0 t1 ( t1 t2 )
b1
1
,
2 1 = . 2
a0 t1 ( t1 t2 )
3 2 =

,,
3.3. Phuong tr`nh truy h`i tuyn tnh

79

b0
1
.
.
a0 t1 ( t1 t2 )
,
,
,
dang

Ta cng
theo v cua cac
thu c trn va` chu y 0 6= 0 ta t`m
.
,,
du
o. c
1 0 =

1
(b0 t1n1 + b1 t1n2 + + bn2 t1 + bn1 ).
a0 t1n (t1 t2 )
,,
,
,,

Tuong tu. ta cung


t`m du
o. c
n =

(b0 t2n1 + b1 t2n2 + + bn2 t2 + bn1 ).


t1 )
,
`, cng thu,c (3.36) ta co
Nhu vy
. tu
n =

a0 t2n (t2

n =

f ( t1 ) f ( t2 )
,
a0 ( t1 t2 )

(3.40)

,,
o dy

f (t) = b0 tn1 + b1 tn2 + + bn2 t + bn1 .


,,
,
` trn ta cung

lam
t`m
2) Tru,o`,ng ho.,p t1 = t2 : Tuong tu. cach
,,
,
,
n

du
n = (n + n n )t1 , va` ta cung
nhn
o. c n dang
o. c
.
. du
n =

f 0 ( t1 )
.
a0

,
phu,o,ng tr`nh truy h`i bc
Trn dy
ta da khao sat

. hai.
,
,,
,
,
,
,

` tuong tu. . Ta cung

Phuong tr`nh bc
co th lam
. cao hon ta cung
,
,
`
thanh
lp
. phuong tr`nh
t k + a 1 t k 1 + a 2 t k 2 + + a k 1 t + a k = 0
(3.41)
,
,,
. tru,ng cua (3.17). Phu,o,ng tr`nh sau dy
goi

. la` phuong tr`nh dac


,
,
goi
. la` phuong tr`nh truy h`i tuyn tnh thu`n nht
xn+k + a1 xn+k+1 + + ak xk = 0.
,
Ta khng chu ng minh dinh
l sau
.

(3.42)

,,
, ,

Chuong 3. T`m cng thu c tng quat

80

,
inh
l 3.1: Nu t1 nghim
s
. bi
. s l`n cua (3.41), th` s day
.
,
,
vo i s hang
tng quat

.
t1n , nt1n , n2 t1n , . . . , ns1 t1n
,
,,
la` nghim
. ring cua phuong tr`nh thu`n nht (3.42)
,
,
,
,,
nghim
. tru,ng theo
Nhu vy
ong tr`nh dac
. tt ca cac
. cua phu
,
dinh
l trn cho ta k nghim
.
. ring cua (3.42).
(1)

(2)

(k)

xn , xn , . . . , xn .
,
,,
inh
l 3.2: Nghim
. chung cua phuong tr`nh thu`n nht
.
,,
` cng thu,c
(3.42) du
o. c cho bang
(k)

(2)

(1)

xn = C1 xn + C2 xn + + Ck xn
,,
,
o dy
`
s bt k`y.
C1 , C2 , . . . , Ck la` nhu ng hang
,
,,
inh
l 3.3: Nghim
. chung cua phuong tr`nh truy h`i tuyn
.
,,
` cng thu,c
tnh (3.17) du
o. c cho bang
(1)

(2)

(k)

xn = C1 xn + C2 xn + + Ck xn + n
,
,
`
s bt k`y va` n la` mt
o, dy
C1 , C2 , . . . , Ck la` nhu ng hang
. nghim
.
,
ring cua (3.17).
,,
`
`
Hang
s C1 , C2 , . . . , Ck du
dinh
bang
gia tri. k s hang
o. c xac
.
.
,
`

ban d
u
c
ua
d
ay.

,
,,
,,
,,
`
Nghim
cua phuong tr`nh (3.17) du
phuong
o. c t`m bang
. ring
,
`
bin d
s co dang
phap
i hang
.
(1)

(2)

(k)

n = 1n xn + 2n xn + + kn xn .
,,
,,
,
Truo` ng ho. p ring dinh
l sau rt hay du
dung
o. c ap
.
.
,,
, ,
n
inh
l 3.4: Nu bn = q .Ps (n), o dy
Ps (n) la` da
thu c cua n
.
,
bc
. cua (3.42), phu,o,ng tr`nh (3.17)
. s, c`on q la` m-l`n nghim
. lap

,,
3.3. Phuong tr`nh truy h`i tuyn tnh

81

,
,
co nghim
= qn Qm+s (n), o, dy
Qm+s (n) la` da
thu c
. ring dang
.
,
cua n, bc
. m + s.
dung
inh
Sau dy
cac
l trn
la` mt
.
. s v du. ap
.
,
V du. 3.18. Chu,ng minh rang
`
vo,i moi
. s tu. nhin n, s zn =
4n+1 + 6n + 5 chia ht cho 9.
,
,
,ng s z1 = 27, z2 = 81 va` z3 = 279 chia ht cho 9.
Lo` i giai. Nhu
,
,,
, ,

C`on zn co th nhn
o. c theo cng thu c tng quat
. du
xn = C1 .4n + C2 .n.1n + C3 .1n
,
vo i C1 = 4, C2 = 6, C3 = 5.
,
,
,,
vo,i s hang
xn la` nghim
Day
tng quat
cua phuong
.
. chung
,
,
tr`nh thu`n nht tuyn tnh vo i h. s khng d
i, ma` nghim
.
,
,,
`
.
cua phuong tr`nh thu`n nht co nghim
t
=
4
v
a
nghi
m
l
ap
1
.
.
,,
,
2
. trung la` (t 4)(t 1) = 0
t2 = t3 = 1. Suy ra phuong tr`nh dac
3
2
. la` t 6t + 9t 4 = 0, va` phu,o,ng tr`nh truy h`i la`
hoac
xn+3 6xn+2 + 9xn+1 4xn = 0.
,
,,
V` zn la` nghim
. ring cua phuong tr`nh truy h`i, ta co
zn+3 = 6zn+2 9zn+1 + 4zn .
,
,
,
`, dy,
,ng s zn , zn+1 va` zn+2 chia ht cho 9, th`
Tu
nu gia su nhu

zn+3 cung
chia ht cho 9. Nhu vy
theo nguyn ly quy nap
. toan
,.
,
,
,
` toan
da du
hoc
o. c giai d
`y du.
. bai
,
,
V du. 3.19. T`m cng thu,c tng quat
cho tng

xn = 15 + 25 + 35 + + n5 .
,
,
Lo` i giai. Ta co
x1 = 1, xn+1 xn = (n + 1)5 .

82

,,
, ,

Chuong 3. T`m cng thu c tng quat

,,
,
Ta da co phuong tr`nh truy h`i khng thu`n nht bc
mt
vo i h.
.
.
,
,,
. tru,ng t 1 = 0 co nghim
s khng d
i. Phuong tr`nh dac
. t1 = 1.
,
,
,
dinh
` la`
Theo cac
l 3.1 -3.3 nghim
.
. chung c, ua phuong tr`nh nay
,,
,
,
xn = C1 + n , o dy
n la` nghim

. ring cua phuong tr`nh ta dang


xet.
,
,
,,
Nhung v` bn = 1n .(n + 5)5 va` s 1 la` nghim
. cua phuong tr`nh
,

tru,ng, theo dinh


dac
l 3.4 (m = 1, s = 5) n la` da
thu c bc
.
. sau
,.
cua n. Khi d
o
xn = B0 + B1 n + B2 n2 + B3 n3 + B4 n4 + B5 n5 + B6 n6 .
,,
,,
` phu,o,ng tr`nh ta thit lp
Thay vao
`u, ta nhn
o. c
. o ph`n d
. du
B1 (n + 1) + B2 (n + 1)2 + B3 (n + 1)3 + B4 (n + 1)4 + B5 (n + 1)5

+ B6 (n + 1)6 B1 n B2 n2 B3 n3 B4 n4
B5 n5 B6 n6 =, (n + ,1)5 .
,
,,
,

`ng nht h. s truo c d


thu c, ta
`ng bc
. cua n cua hai v dang
co
6B6 = 1, 5B5 + 15B6 = 5, 4B4 + 10B5 + 20B6 = 10,
3B3 + 6B4 + 10B5 + 16B6 = 10,
2B2 + 3B3 + 4B4 + 5B5 + 6B6 = 5,
B1 + B2 + B3 + B4 + B5 + B6 = 1.
`, d
Tu
o suy ra
1
5
1
1
, B3 = 0, B2 = , B1 = 0.
B6 = , B5 = , B4 =
6
2
12
12
1
5
1
1
Khi d
o
xn = B0 n2 + n4 + n5 + n6 .
12 , , 12
2
6
,
`, dy
Tu
vo i n = 1 ta nhn
o. c 1 = B0 + 1, ngha la` B0 = 0 suy ra
. du
1
15 + 25 + + n 5 =
(2n6 + 6n5 + 5n4 n2 ).
12
,
V du. 3.20. Giai phu,o,ng tr`nh

xn+1 nxn = n!n5


vo,i di
`u kin
`u la` x1 = 1.
. ban d

(3.43)

,,
3.3. Phuong tr`nh truy h`i tuyn tnh

83

,
,
,,
,,
,
Lo` i giai. Ta xet phuong tr`nh thu`n nht tuong u ng
yn+1 nyn = 0.

(3.44)

,,
,,
` (3.44)
Sau khi thay n boi l`n luo. t gia tri. n 1, n 2, . . . , 2, 1 vao
ta co
y n = ( n 1 ) y n 1
y n 1 = ( n 2 ) y n 2
......
y3 = 2y2
y2 = y1 = C.
,
,
,,
dang

Nhn theo v cac


thu c trn ta t`m du
o. c yn = C (n 1)!.
,
,
,,
bin d
Nghim
i
. ring n cua (3.43) ta t`m theo phuong phap
,,
`
` (3.43), nhn
hang
s dang
n = (n 1)!un . Khi ta th vao
o. c
.
. du
,,
. la` un+1 un = n5 . Ta nhn
n!un+1 n!un = n!n5 , hoac
o. c
. du
,
,,
,,
` tp
phuong tr`nh truy h`i co nghim
. chung (kt qua bai
. truo c)
un = B0 +
,
vo i B0 = 0 ta co

1 2
n (n 1)2 (2n2 2n 1).
12

1 2
n (n 1)2 (2n2 2n 1)(n 1)!.
12
,
` toan
la`
Suy ra nghim
. chung cua bai
n =

1 2
n (n 1)2 (2n2 2n 1)](n 1)!.
12
,,
,

Vo i n = 1 cho gia tri. ban d


`u ta nhn
o. c C = 1 va` suy ra day
. du
,
`
phai t`m la:
xn = yn + n = [C +

xn = [

1 2
n (n 1)2 (2n2 2n 1) + 1](n 1)!.
12

,,
, ,

Chuong 3. T`m cng thu c tng quat

84

,
,
,
,
ng l
` a c`
s
3.4. Tng cua nhu
uy thu
ung bc
. cac
,
tu. nhin
,
,
`, cung
` v` tnh tng cua luy
thu
`
s
. mt
Ta da gap
bc
. s bai
. cac
,
, ,
,
` a bc
` ta ap
dung
tu. nhin, nhu voi luy thu
. hai,, bc
. ,ba. Ph`n nay
.
,
,
,
,

cng thu c Newton d


s
m
u
n
ao
d
o
cac
tnh du
o. c tng cua mt

.
,
,
,
,
` 1 d
s tu. nhin tu
n n. tnh tng
Sk = 1k + 2k + 3k + + n k
,
,
,
,
dung

vo i k la` mt
dang
thu c sau
. s tu. nhin, ta ap
.

(3.45)

( x + 1)k+1 = x k+1 + Ck1+1 x k + Ck2+1 x k1 + + Ckk+1 x + 1.


,,
,,
Thay x l`n luo. t cac
gia tri. 1, 2, 3, . . . , n ta nhn
o. c
. du
2k+1 = 1k+1 + Ck1+1 1k + Ck2+1 1k1 + + Ckk+1 1 + 1,
3k+1 = 2k+1 + Ck1+1 2k + Ck2+1 2k1 + + Ckk+1 2 + 1,
4k+1 = 3k+1 + Ck1+1 3k + Ck2+1 3k1 + + Ckk+1 3 + 1,
..............................................
nk+1 = (n 1)k+1 + Ck1+1 (n 1)k + + Ckk+1 (n 1) + 1,
k +1
( n + 1 ) k +1 =
+, Ck1+1 nk + Ck2+1 nk1 + + Ckk+1 n + 1.
, n
,
,,
dang

Cng
theo v cua cac
thu c va` ta nhn
o. c
.
. du

(n + 1)k+1 = Ck1+1 Sk + Ck2+1 Sk1 + + Ckk+1 S1 + n + 1


hay la`
1
[(n + 1)k+1 Ck2+1 Sk1 Ckk+1 S1 n 1]. (3.46)
k+1
,
,
`, biu thu,c (3.46) cho phep ta tnh l`n lu,o.,t tng (3.45). V du.
Tu
,,
,
`, (3.46) ta tnh du
vo i k = 1, 2, . . . tu
o. c
Sk =

S1 =

1
n ( n + 1)
1
1
[(n + 1)2 n 1] =
= n2 + n,
2
2
2
2

,
,
,
thu`,a cung
`
s tu., nhin
3.4. Tng cua nhu ng luy
bc
. cac

85

n(n + 1)(2n + 1)
1
[(n + 1)3 3S1 n 1] =
3
6
1 3 1 2 1
= n + n + n,
3
2
6
1
n ( n + 1) 2
S3 = [(n + 1)4 6S2 4S1 n 1] = [
]
4
2
1
1
1
= n4 + n3 + n2 ,
4
2
4
1
S4 = [(n + 1)5 10S3 10S2 5S1 n 1]
5
n(n + 1)(2n + 1)(3n2 + 3n + 1)
1
1
1
1
=
= n5 + n4 + n3 n,
30
5
2
3
30
1
6
S5 = [(n + 1) 15S4 20S3 15S2 6S1 n 1]
6
1
1
1
5
1
= n2 (n + 1)2 (2n2 + 2n 1) = n6 + n5 + n4 n2 ,
12
6
2
12
12
1
n(n + 1)(2n + 1)(3n4 + 6n3 3n + 1)
S6 =
42
1
1
1
1
1
= n7 + n6 + n5 n3 + n,
7
2
2
6
12
1 2
n (n + 1)2 (3n4 + 6n3 n2 4n + 2)
S7 =
24
1
1
7
7
1
= n8 + n7 + n6 n4 + n2 .
8
2
12
24
12
,
,
,
Ta se chu ng minh rang
`
tng Sk la` mt
thu c bc
. da
. k + 1 co s
1
hang
tu. , do bang
`
khng, h. s tru,o,c nk+1 bang
`
, h. s tru,o,c
.
k+1
1
k
,
,
,
,
k
k

1
`
`

n bang

, h s truo c n
bang

, h s truo c nk2 bang


` 0.
2 .
12 .
,
,
,
,ng

Tht
vy,
Gia thit di
dinh
vo i nhu
`u khang
. , trn dung
.
.
,
,

tng S1 , S2 , . . . , Sk1 . Ta c`n kt lun


dinh
cung
dung
vo i
.
. khang
,
,
,
,
`

tng Sk . Khang
dinh
Sk la` mt
thu c theo n va` h. s tu. do bang
.
. da
,
,
,
,
,
` dang
h. s ak , ak1 , ak2 tuong u,ng

0, suy ra tu
thu c (3.46). Cac
S2 =

,,
, ,

Chuong 3. T`m cng thu c tng quat

86

,,
,,
`, (3.17) ta t`m du

truo c nk , nk1 , nk2 trong Sk cung


tu
o. c
1
1
1
[Ck1+1 Ck2+1 ] = ,
k+1
k
2
1
1
k
1
a k 1 =
[Ck2+1 Ck2+1 . Ck3+1
]= ,
k+1
2
k1
12
1
k

1
1
1
a k 2 =
[Ck3+1 Ck2+1
Ck3+1 . Ck4+1
] = 0.
k+1
12
2
k2
,
,,
,
,
Nhu vy
. trong truo` ng ho. p chung ta co cng thu c
ak =

ik = k + 1 .nk+1 + 2 nk + 2 Ck1 B2 nk1 + 4 Ck3 B4 nk3 +

i =1

(3.47)
1
1 5
+ Ck B6 nk5 + Ck7 B8 nk7 +
6
8

,ng s
`
`
. n2 . Trong (3.47) nhu
s hang
cu
i
c
ung
l
a
n hoac
.
,ng s Bernoulli. Mt
B2 , B4 , B6 , B8 , . . . goi
. ,la` nhu
. s s Bernoulli
,,
du
o. c lit
. k trong bang
5
,
66
691
7
3617
43867
B12 =
, B14 = , B16 =
, B18 =
,
2730
6
510
798
174611
854513
236364091
B20 =
, B22 =
, B24 =
,
330
123
2730
8553103
23749461029
8615841276005
B26 =
, B28 =
, B30 =
,
6
870
14322
2577867858367
7709321041217
, B34 =
.
B32 =
510
6
,
,
,,
,
,ng s nay
`,a
` du cho ta tnh du
thu
Nhu
o. c cng thu c tng luy
,, , ,
, s tu., nhin d
bc
`u tin.
. mt,
. hai, ba, . . . , ba muoi tu cua n chu
,,
,
,
g` tnh du
By gio` khng kho khan
o. c cng thu c dang
.
,
,
,
,
n
k o, dy P ( t ) la

`
[
P
(
t
)]
da
th
u
c
theo
t.
V

d
u
ch
u
ng
minh
nh
u
ng

t =1
.
,
,

dang
thu c sau dy

B2 =

1
,
6

B4 =

1
,
30

B6 =

1
,
42

B8 =

1
,
30

B10 =

` tp
3.5. Bai
.

87

1
n(2n 1)(2n + 1);
3
1
2) 12 + 52 + 92 + + (4n + 1)2 = (n + 1)(16n2 + 20n + 3).
3
Tht
. vy,
.
1) 12 + 32 + 52 + + (2n 1)2 =

1) (2t 1)2 =
t =1

t =1

t =1

t =1

(4t2 4t + 1) = 4 t2 4 t + n

n ( n + 1)
n(n + 1)(2n + 1)
4
+n
= 4.
6
2
1
= n(2n 1)(2n + 1).
3
n

2) (4t + 1)2 =
t =1

t =1

t =1

t =1

(16t2 + 8t + 1) = 16 t2 + 8 t + n

n ( n + 1)
n(n + 1)(2n + 1)
+ 8.
+n+1
= 16.
6
2
(n + 1)(16n2 + 20n + 3)
=
.
3
, ,
` toan
tu,o,ng tu., nhu, vy.
Ban
. doc
. co th giai nhi`u bai
.

` tp
3.5. Bai
.
`
. 3.21. Chu,ng minh rang
Sn = a1 b1 + a2 b2 + + an bn =

q n 1 1
a1 b1 qan bn
dqb1
.
1q
(1 q )2

,
vo i a1 , a2 , . . . , an la` cp s cng
co cng sai d va` b1 , b2 , . . . , bn la` cp
.
s nhn co bi
. s q 6= 1.
,
,
cua cp s cng
. 3.22. T`m s hang
tng quat
.
.
2, 5, 10, 17, 26, 37, . . .
,
,
va` tnh tng n s hang
d
`u tin cua no.
.

,,
, ,

Chuong 3. T`m cng thu c tng quat


,
t`m cng thu,c tng
. 3.23. Hay
88

Sn = 3.2 + 5.5 + 7.8 + + (2n + 1)(3n 1).


,
t`m nghim
phu,o,ng tr`nh sau:
. 3.24. Hay
. chung cua cac
a) xn+1 + xn+1 + xn = 0;
b) xn+1 + 2xn+1 + xn = 0;
c) xn+2 xn = 0;
d) xn+3 3xn+2 + 3xn+1 xn = 0.
,
,
cho day
xac
dinh
. 3.25. T`m cng thu,c tng quat
. theo cng thu c
sau:
a) x1 = 10, x2 = 16, xn+2 4xn+1 + 3xn = 0;
b) x1 = 1, x2 = 3, x3 = 29, xn+3 9xn+2 + 26xn+1 24xn = 0;
c) x1 = 1, x2 = 7, xn+2 6xn+1 + 9xn = 4.

, ,
CHUONG

HOC
S
.
chia ht . . . . . . . . . . . . . . . . . . . . . . . . . . . . . . . . . . . . . . . . . 89
4.1. Phep
Euclide . . . . . . . . . . . . . . . . . . . . . . . . . . . . . . . . . . . . 94
4.2. Thut
. toan
,
4.3. S ph
uc . . . . . . . . . . . . . . . . . . . . . . . . . . . . . . . . . . . . . . . . . . . . . . . 99
,
. . . . . . . . . . . . . . . . . . . . . . . . . . . . . . . . . . . 105
4.4. Nhung v du. khac
` tp
4.5. Bai
. . . . . . . . . . . . . . . . . . . . . . . . . . . . . . . . . . . . . . . . . . . . . . . . 108

chia ht
4.1. Phep
,ng
Trong s hoc
. phep chia cho ta r,t nhi`u tnh cht v` nhu
` toan
phat
biu du,o,i dang
phep chia
s nguyn. Nhi`u bai
cac
.
,
,
,
lai
ng s nguyn va` k ca cac
thut
tnh toan.
Ta nhac
nhu
. toan
.
,
`

`
`

mt
s
kh
ai
ni
m.
N
u
a
v
a
b
l
a
nh
u
ng
s
nguyn,
ta
n
o
i
r
ang
.
.
..
b chia ht cho a, ky hiu
. la` b . a, ,khi t`n tai
. mt
. s nguyn c sao
,
,
cho b = ca. S c goi
. la` thuong c,ua phep chia, a nhi`u khi goi
. la`
,
,
,
,
,
,
uo c s cua b, s b goi
. la` bi
. c
. s cua a. Truo` ng ho. p khng t`n tai
`

theo dinh
ngha trn ta noi rang
b khng chia ht cho a, ky hiu
.
.
,
..
,
,
` dinh
`
tnh
b 6 . a. Tu
ngha do
loat
n gian trn ta suy ra hang
.
. cac
,
,
,
.
,,
,
cht cua phep chia, o dy
ta ch ly mt
n gian: Nu b .. a
. v du. do
.
.
,

va` c .. a, th` (ub + vc) .. a vo i moi


. s nguyn bt ky` u va` v. Hai khai
,
,

`
nim
rt hay du
o. c dung.
. sau dy
,
,
,
,
Mt
. s d goi
. la` uo c s chung lo n nht cua hai s nguyn a va`

,,
Chuong 4. S hoc
.

90

b, ky hiu
. d=(a, b), khi
1) a va` b d
`u chia ht cho d;
,
,,
cua a va` b.
2) d chia ht cho moi
. uo c s chung khac
,
,
Mt
. s m goi
. la` bi
. s chung nho nht cua hai s nguyn a va`
b, ky hiu
. m=[a, b],, khi
1) m chia ht cho ca a va` b;
,
cua a va` b d
2) Moi
`u chia ht cho m.
. bi
. s chung khac
,
,a hai khai
nim
Cng thu c lin quan giu
. trn la`
ab
[ a, b] =
.
( a, b)
3n+3
V du. 4.1. Chu,ng minh rang
` vo,i moi
. s nguyn n 0, s 3
26n 27 chia ht cho 169.
,
,
. An = 33n+3 26n 27. Khi d
Lo` i giai. Ta dat
o A0 = 33 27 = 0
suy ra A0 chia ht cho 169.
, ,,
,
,
,
` d
Gia su An chia ht cho 169 vo i n nao
o. Ta bin d
i An+1 nhu
sau
An+1 = 33(n+1)+3 26(n + 1) 27

= An + 26.33n+3 26 = An + 26[(33 )n+1 1]


= An + 26(33 1)(33n + + 1) = An + 4.169.(33n + + 1).
`, d
Tu
o suy ra An+1 chia ht cho 169.

V du. 4.2. Chu,ng minh rang


` vo,i s n nguyn du,o,ng
1) C = 7n + 3n 1 chia ht cho 9;
2) E = a4n+1 a chia ht cho 30, vo,i a la` s nguyn.
,
, ,,
,
Lo` i giai. 1) Nu n = 1, th` C1 = 7 + 3 1 chia ht cho 9. Gia su
,
n = k 1 va` Ck = 7k + 3k 1 chia ht cho 9. Khi d
o vo i n = k + 1

4.1. Phep chia ht

91

s
Ck+1 = 7k+1 + 3(k + 1) 1 = 7.7k + 3k + 2

= 7.7k + 21k 7 18k + 9 = 7(7k + 3k 1) 9(2k 1)


= 7Ck 9(2k 1)
,,

quy nap
hoc
cung
chia ht cho 9. Theo phuong phap
. toan
. C chia
,,
,

ht cho 9 voi moi


. n nguyn duong.
2) Nu n = 1, th`
E1 = a5 a = a( a2 1)( a2 + 1) = a( a 1)( a + 1)[( a2 4) + 5]

= ( a 2)( a 1) a( a + 1)( a + 2) + 5( a 1) a( a + 1).


`,a s thu, nht chia ht cho 5! = 120 = 4.30, c`on thu
`,a s thu,
Thu
hai chia ht cho 5.3! = 30. Suy ra E1 chia ht cho 30.
, ,,
,
Gia su n = k 1 va` Ek = a4k+1 a chia ht cho 30. Khi d
o vo i
n = k + 1 s
Ek+1 = a4k+5 a = a4k+5 a + a4k+1 a4k+1

= ( a4k+1 a) + ( a4k+5 a4k+1 ) = Ek + a4k .E1

cung
chia ht cho 30.

V du. 4.3. Chu,ng minh rang


`
nu vo,i nhu ,ng s tu. , nhin du,o,ng
,
k
k
k
a, b, c thoa man
a2 + b2 = c2 , th` moi
Ek = a2 + b2 + c2
. s co dang
.
k
k
k
va` Fk = ( ab)2 + (bc)2 + (ca)2 vo,i k 2 chia ht cho s
1
D = ( a4 + b4 + c4 ).
2
,
,
,
,
Lo` i giai. Ta chu ng minh d
`ng tho` i s Ek va` Fk chia ht cho D.
,
,,
Tht

Truo c tin ta khang


dinh
a4 + b4 + c4 la` s chan.
.
. vy,
. nu
,
,
,
va` suy
. d
. d
c = 2k, th` a va` b la` hoac
`ng tho` i le hoac
`ng tho` i chan
,
4
4
4
2
2
Nu c = 2k + 1, th` a + b la` le va` suy ra
ra a + b + c la` chan.

,,
Chuong 4. S hoc
.

92

,
,

la` le. Do c4 = (2k + 1)4 la` le nn


a4 + b4 = ( a2 + b2 )2 2a2 b2 cung
,
,
v` la` tng hai s le.
( a4 + b4 ) + c4 chan,
Ta xet s
F2 = ( ab)4 + (bc)4 + (ca)4

= a4 b4 + c4 ( a4 + b4 ) = 2Dc4 + a4 b4 c8
= 2Dc4 + ( a2 b2 c4 )( a2 b2 + c4 ).
c4 ( a4 + b4 )
V` 2a2 b2 = c4 ( a4 + b4 ), nn a2 b2 c4 =
c4 =
2
,

D, Tu`, d
nhu vy
o suy ra F2 chia ht cho D. Cung
. E2 chia ht
,
,
,
` quy nap
` toan
suy ra tu
hoc
cho D. Mnh
d
` cua bai
.
. toan
. vo i cac
,
k
,

dang
thu c Ek+1 + 2Fk = Ek2 va` Fk+1 + 2( a.b.c)2 Ek = Fk2 .

V du. 4.4. Chu,ng minh rang


`
vo,i s tu. , nhin du,o,ng bt k`y n, s
n
24 + 5 chia ht cho 21.
,
,
Tng quat:
Vo,i s tu. , nhin bt k`y a > 1, n 1, biu thu,c
n
Bn = a4 + a3 a 1 chia ht cho ( a 1)( a + 1)( a2 + a + 1).
,
,
Lo` i giai. Nu n = 1, th` B1 = a4 + a3 a 1 = ( a 1)( a + 1)( a2 +
a + 1).
, ,,
Gia su n = k 1 va` Bk chia ht cho ( a 1)( a + 1)( a2 + a + 1).
,,
,
Khi d
o vo i n = k + 1 ta nhn
o. c
. du
Bk+1 = a4

k +1

+ a3 a 1 = ( a4 a + a )4 + a3 a 1
k

= [ a( a3 1) + a]4 + a3 a 1 = K ( a 1)( a2 + a + 1)+


k

+ a4 + a3 a 1 = K ( a 1)( a2 + a + 1) + Bk .

. khac
Mat
k

Bk+1 = ( a4 + a a)4 + a3 a 1 = [ a( a3 + 1) a]4 + a3 a 1


k

= K ( a + 1) + a4 + a3 a 1 = K ( a + 1) + Bk .

4.1. Phep chia ht

93

,
,
,
`,a cua a + 1.) V` cac
s
(Ky hiu
. K ( a + 1) la` biu thu c luy thu
`
a 1, a + 1 va` a2 + a + 1 nguyn t cung
nhau, suy ra Bk+1 chia
,
n
,

ht cho tch cua chung.


Vo i a = 2, ta co Bn = 24 + 5.

n
V du. 4.5. 1) Chu,ng minh rang
` 32 1 chia ht cho 2n+2 va` khng
chia ht cho 2n+3 vo,i n nguyn du,o,ng.
n
2) Chu,ng minh rang
`
23 + 1 chia ht cho 3n+1 va` khng chia ht
cho 3n+2 vo,i n nguyn du,o,ng.

,
,
Lo` i giai. 1) Ta co
A n + 1 = 32

n +1

1 = (32 )2 1 = (32 1)(32 + 1).


, ,,
n
,

Vo i n = 1 mnh
d
Gia su An = 32 1 chia ht cho 2n+2 .
` dung.
.
n
V` 32 + 1 chia ht cho 2, nn An+1 se chia ht cho 2.2n+2 = 2n+3 .
, ,
n
ta gia su, An = 32 1 khng chia ht cho 2n+3 .
. khac,
Mat
n
n
n
,
Nhung 32 + 1 khng chia ht cho 4, v` (32 + 1) 2 = 32 1
n
chia ht cho 4 (thm
. ch chia ht cho 4.2 do ph`n trn. V` vy
.
n
(32 + 1) chia cho 4 du, 2) tu`, dy
suy ra An+1 khng chia ht cho
2n +4 .
,,
,
,,
,
,
2) Chu ng minh tuong tu. nhu ph`n trn. Ta su dung
.

23

n +1

+ 1 = (23 )3 + 1 = (23 + 1)(22,3 23 + 1)


n

= (23 + 1)[(23 + 1)2 3.23 ].


. vung chia ht cho 3, khng chia ht cho 9.
S trong ngoac

V du. 4.6. 1) Chu,ng minh rang


`
nu p la` s nguyn t, th` s
,,
,
a p a chia ht cho p vo i moi
. a (a la` s nguyn duong).
2) Chu,ng minh rang
`
nu p la` s nguyn t va` a khng chia ht
cho p, th` s a p1 1 chia ht cho p (dinh
ly Fermat).
.

,,
Chuong 4. S hoc
.

94

,
,
,
,,
,
Lo` i giai. 1) Vo i a = 1 mnh
d
` la` hin nhin, v` trong truo` ng
.
,
` a p a = 1 1 = 0.
ho. p nay
, ,,
,

` d
Gia su mnh
d
vo i mt
` dung
y, ngha la` a p a
.
. s a nao
,

chia ht cho p. Ta se chu ng minh mnh


d
dung
cho a + 1.
` cung
.
,
,,
dung
Tht
phn tch nhi. thu c Newton, ta nhn
o. c
. vy,
. ap
.
. du

( a + 1) p ( a + 1) = a p + p f ( a ) + 1 a 1 = ( a p a ) + p f ( a ).
,,
,
,
dung
O dy
f ( a) la` da
cng thu c h.

thu c bc
. p 1 theo a. Do ap
.
,
`,a s co p du
thu
` lp
s Newton va` nhom cac
a ra ngoai
. ra f ( a).
,

hoc
Mnh
d
vo i a + 1, theo nguyn ly quy nap
` dung
.
. toan
. no
,

dung
vo i moi
. a 1.
,
` ph`n tru,o,c suy ra a p a chia ht cho s nguyn t p.
2) Tu
,
,
`, dang

Khi d
thu c
o tu
a p a = a ( a p 1 1 ).
` a khng chia ht p, nn a p1 1 chia ht
Do di
`u kin
`u bai
. d
cho p.
, ,
y:
Nu ta dat
. a = 2, 3, . . . , p 1, th` nhu, h. qua cua kt
Chu
s 2 p1 1, 3 p1 1, . . . , ( p 1) p1 1 chia ht cho
lun
. trn cac
p
. la` 2 1 , 3 p1 , . . . , ( p 1) p1 chia cho p du, 1.
p hoac
,
,,
, `
Trong truo` ng ho. p nay
tng 2 p1 + 3 p1 + + ( p 1) p1 chia
,
ht cho p, vo i p la` s nguyn t.

Euclide
4.2. Thut
. toan
,ng s nguyn. T`m u,o,c s chung
Cho a > 0 va` b > 0 la` nhu
,
,
,,
,
cua Euclide
lo n nht cua hai s da cho du
o. c t`m theo thut
. toan
,
nhu sau:
,
,,
. r0 = a va` r1 = b. Chia s a cho s b du
cho tin
o. c
. dat

Euclide
4.2. Thut
. toan

95

,
,,
,
thuong q1 va` s du la` r2 . Ta co th vit
a = bq1 + r2 ,

(0 r2 < r1 ).

Nu b > a ta co q1 = 0 va` r2 = a. Nu r2 = 0, th` a chia ht cho


,,
,
` u,o,c s chung lo,n nht la` b. Nu r2 6= 0,
b; trong truo` ng ho. p nay
,,
`
` ky hiu
ta tin hanh
buo c tip theo: Ly b chia cho r2 , l`n nay
.
,,
,

thuong va` s du la` q2 va` r3 , ta co

(0 r3 < r2 ).
`,ng. Tru,o`,ng ho.,p ngu,o.,c lai
du
Nu r3 = 0, th` thut
. toan
. ta lai
.
,,
,,
,

ly r2 chia cho r3 du
o. c thuong q3 va` s du r4 , hay la` r2 = r3 q3 +
,
,
s du, d
r4 (0 r3 < r2 ). Cu tip tuc
cac
`u thuc
. nhu vy,
. v`
,
, .
,

s giam ngat
. chu ng to sau
[0, b) va` b > r1 > r2 > . . . 0 day
, ,
, ,
,
,
, , `

mt
. s buoc (s buoc khng ,lon hon b) se dn toi s du bang 0 va`
,,
`,ng. Kt qua ta nhn
se du
thut
o. c day
. toan
. du
r0 = r1 q1 + r2
b = r2 q2 + r3

r1 = r2 q2 + r3
r2 = r3 q3 + r3

(4.1)

...
r n 2 = r n 1 q n 1 + r n
rn1 = rn qn (0 ri < ri1 , i = 1, 2, . . . , n).
,
,
`
0.
Trong cng thu c trn rn la` s du cui cung
khac

,
V du. 4.7. Ta chu,ng minh rang
`
rn la` u,o,c s chung lo,n nht cua
a va` b.
,
,
,
,
,,
`
Lo` i giai. 1) Ta ch ra rang
rn la` uo c s chung cua a va` b.
,
Do rn1 chia ht cho rn va` cng thu c h`i quy
r i 1 = r i q i + r i +1

(i = 1, 2, . . . , n; rn+1 = 0).

(4.2)

,,
Chuong 4. S hoc
.

96

,
,
,
,
,
,,
dat
ch cua ta th` phai ch ra rn la` uo c s chung cua
., muc
. d
,
,
,
`

tt ca rn1 , rn2 , . . . , r0 . dang


thu c cui cung
cua (4.1) cho ta thy
, ,, ,
rn2 chia ht cho rn (v` rn1 da chia ht cho rn ). Gia su vo i mt
.
,

` d
s nao
o i (1 i n 1) nhung s rn1 , rn2 , . . . , ri chia ht cho
`
`, (4.2) suy ra ri1 chia ht cho rn . Nhu, vy
rn . Khi d
quy
o t u
. b,ang
,

nap
ta kt lun
r1 = b cung
chia ht cho rn , c`on dang
thu c
. chung
.
,
,
,,
d
`u tin cua (4.1) cho ta rn la` uo c s cua a.
,
,
,,
,
2) Ta chu ng minh rn la` uo c s chung lo n nht cua a va` b.
,
,,
Tht
. vy,
. Ky hiu
. d la` uo c s,chung bt ky` cua a ,va` b (ngha
,,
,
`, dang

la` r0 , r1 co uo c s chung d). Tu


thu c d
`u tin cua (4.1) suy
,
,
,

ra r2 chia ht cho d. V` th d cung


la` uo c s chung cua r1 va` r2 .
, ,,
,
,,
,
` d
Gia su d la` uo c s chung cua ri1 , ri vo i s i (1 i n 1) nao
o;
,
,
,
,
` (4.1) suy ra d la` uo c s chung cua ri+1 . Theo nguyn ly
khi d
o t u
,
,,
`
`
hoc,
nay
` ta thy rang
quy nap
cach
. toan
. d, la` uo c s cua rn . Bang
,
,,
,,

la` uo c s chung cua rn , suy ra


moi
. uo c s chung cua a va` b cung
,,
,
rn la` uo c s chung lo n nht.
,,
`, bu,o,c i + 1 va` i d
hoc
Dang
quy nap
tu
n buo c i 1
.
. toan
. di
,
,,
nhu trn goi
. la` phep quy nap
. nguo. c.
, ,
,
,,
u,o,c s chung lo,n nht bang
`
t
Nguo` i ta co th biu din
cac
,
,
` tp
ho. p tuyn tnh cua hai s a va` b thng qua bai
. sau.

V du. 4.8. Cho a va` b la` hai s nguyn du,o,ng. Khi d


o

( a, b) = sn a + tn b
,
vo,i n = 0, 1, 2 . . ., sn va` tn la` nhu ,ng s hang
thu, n cua day
{ s n },
.
,,
{tn } xac
dinh
boi
.
s0 = 1, t0 = 0, s1 = 0, t1 = 1

Euclide
4.2. Thut
. toan

97

va`
s i = s i 2 q i 1 s i 1 ,
t i = t i 2 q i 2 t i 1 ,
,
,,
,
vo,i i = 2, 3, . . . , n, o, dy

qi la` thuong s thu i trong thut


. toan
,
,
,
Euclide khi t`m uo c s chung lo n nht.
,
,
,
`
Lo` i giai. Ta se chu ng minh rang
ri = si a + ti b,

(4.3)

,
,
`, (4.3) co th suy ra lo`,i
vo i i = 0, 1, 2 . . . , n. V` ( a, b) = rn nn tu
,
giai.
,
,,
,
`
quy nap
hoc
Dung
phuong phap
chu ng minh (4.3).
. toan
. d
,

Vo i i = 0, ta co a = r0 = 1.a + 0.b = s0 a + t0 b. Do d
o (4.3) dung
,
,
b = r1 = 0.a + 1.b = s1 a + t1 b, nhu vy
. khac
voi i = 0. Mat
.
,
,
,

(4.3) cung
dung
vo i i = 1. Gia thit ri = si a + ti b dung
vo i moi
.
`, bu,o,c k trong thut

i = 1, 2, . . . , k 1. Khi d
to
an
Euclide
ta
o, tu
.
co
r k = r k 2 r k 1 q k 1
,

theo gia thit quy nap


. suy ra
r k = ( s k 2 a + t k 2 b ) ( s k 1 a + t k 1 b ) q k 1

= ( s k 2 s k 1 q k 1 ) a + ( t k 2 t k 1 q k 1 ) b
= sk a + tk b.

V du. 4.9. Cho a la` s tu. , nhin, a > 1. Hay


t`m ( am 1, an 1),
,,
,
,
o dy
m va` n la` nhu ng s tu. nhin.
,
,
`
. (n, m) = d. Ta chu,ng minh rang
Lo` i giai. at
( am 1, an 1) =
,
,
,
ta co th gia thit n m. Ta
( ad 1). Khng mt tnh tng quat

,,
Chuong 4. S hoc
.

98

,
,
,
,
,

chu ng minh quy nap


theo m. Vo i m = 0, dang
thu c hin nhin.
.
, ,,
. n = mq + r, 0 r < m. Ta co
Gia su m > 0. Ta dat
an 1 = amq ar 1 = amq ar ar + ar 1

= ar ( amq 1) + ( ar 1) = A( am 1) + ( ar 1),
,
,,
, `
`, nhu
,ng dang

o dy
thu c nay
suy ra
A la` s nguyn. Tu
( an 1, am 1) = ( am 1, ar 1)
,
hoc
va` theo gia thit quy nap
. toan
. ta co

( am 1, ar 1) = a(m,r) 1 = ad 1.

,
V du. 4.10. Chu,ng minh rang
`
bi
. s chung nho nht (BSCN)
,
,
,
cua day
1, 2, . . . , 2n bang
`
bi
. s chung nho nht cua n + 1, n +
2, . . . , 2n:
BSCN (1, 2, . . . , n) = BSCN (n + 1, n + 2, . . . , 2n).
,
,
. [1, 2, . . . , 2n] = s va` [n + 1, n + 2, . . . , 2n] = t. v`
Lo` i giai. Ta dat
,
s n + 1, n + 2, . . . , 2n, c`on t la` bi
s la` mt
bi
s chung cua cac
.
.
.
,
,
,
,,

s nho nht nn s chia ht cho t. chung minh nguo. c lai


. t chia
,
,
`
s chia ht cho n + 1, n +
ht s ta phai chu ng minh rang
mi
,

2, . . . , 2n cung
chia ht cho 1, 2, . . . , n. Ta se chu ng minh di
`u d
o
,
,
,
,
`
quy nap

phuong phap
dinh
la`
bang
.
. theo n. Vo i n = 1 khang
,
, ,,
,
,

hin nhin. Gia su mnh


d
vo i n, ta chu ng minh no cung
` dung
.
,

dung
vo i n + 1. Cho m la` s chia ht cho (n + 1) + 1, (n + 1) +
`, m chia ht cho 2(n + 1) nn m chia ht cho
2, . . . , 2(n + 1). Tu
,
n + 1 va` suy ra m chia ht cho n + 1, n + 2, . . . , 2n (ta ch thm
,
,
` d
cac
s ma` theo gia thit m da chia ht). Theo gia
vao
`u day
,,
thit quy nap
o ta da co m
. m chia ht cho 1, 2, . . . , n, c`on truo c d
chia ht cho n + 1.

,
4.3. S phu c

99

,
,
,
V du. 4.11. Chu,ng minh rang
` tng tt ca u,o,c s cua s tu. , nhin

,
n > 2 nho ho,n n n.
,
,
,
, ,,
,
,
`
Lo` i giai. Ta ky hiu
tng cua tt ca uo c s cua s n bang
D ( n ).
.
,

,,
,
,
`

Ta phai chu ng minh rang


D (n) < n n vo i n 3. Ta chon
. truo` ng
,
ho. p n = 2 ( la` s nguyn, 2). Khi d
o
D (n) = 1 + 2 + 22 + + 2

= 2+1 1 < 2+ 2 = n n.
,
,
`

Gia thit rang


n 6= 2 va` D (k ) < k k vo i moi
. 3 k, < n va` chung
,

,
,
`
ng di
ta se chu ng minh D (n) < n n. Do nhu
u k trn, ta co th
,
,,
`
xet n = mp, o dy
p 3, 1 + p <
p la` s nguyn t le. Chu y rang
1

,
,
p p (tht
. vy,
. voi p 4, 1 + p < 2 < p; voi p = 3, 1 + 3 <

,,
,
3 3). V` th, nu m = 1, th` D ( p) = 1 + p < p p; tuong tu. , nu
p

m = 2, th` D (n) = 1 + 2 + p + 2p = 3 + 3p < 2p 2p = n n, nu


,

m 3, th` theo gia thit quy nap


D (m) < m m. Nhu vy
. mi
.
,
,
,
,,
,,
. dp, o, dy
uo c s cua n co dang
d hoac
d la` uo c s cua m, nn
.
D (n) = D (m) + pD (m) = D (m)(1 + p)

< m m.p p = n n.

,
c
4.3. S phu
,
,
,
, ,
lai
nim
Ta nhac
. khai
. co ban cua s phu c. Mt
. s phu c z co
,,
,
,ng s thu.,c, c`on i la` do
dang
x va` y la` nhu
n
. , dai
. s z = x + iy, o dy
,
vi. ao co tnh cht i2 = 1; s x goi
la` ph`n thu. c, c`on s y goi
.
. la`
,
,
,
`

phn ao cua s phuc z.


,
`
Hai s phu c z1 = x1 + iy1 , z2 = x2 + iy2 la` bang
nhau khi va`
,
ch khi x1 = x2 , y1 = y2 . Nu x = y = 0, th` z = 0 + i0 = 0. Moi
.
,
,
,
,
,
,
`

s thu. c co th coi nhu la` mt


s
ph
u
c
khi
ph
n
ao
c
ua
n
o
b
ang
0.
.

100

,,
Chuong 4. S hoc
.

,
,
,
,
,
,ng phep
Nhu vy
. tp
. s thu. c ch la` tp
. con cua tp
. s phu c. Nhu
,,
,
du
toan
ngha trn tp
o. c dinh
.
. s phu c g`m:

( x1 + iy1 ) + ( x2 + iy2 ) = ( x1 + x2 ) + i (y1 + y2 ),


( x1 + iy1 ) ( x2 + iy2 ) = ( x1 x2 ) + i (y1 y2 ),
( x1 + iy1 )( x2 + iy2 ) = ( x1 x2 y1 y2 ) + i ( x1 y2 + x2 y1 ).
,
`
S z = x iy goi
la` s lin ho.,p cua z = x + iy. Ro rang
nu z1
.
,
,
,
,

la` lin ho. p cua z2 , th` z2 cung


lin ho. p cua z1 .
,
,
. che vo,i h. toa
S phu c co lin h. chat
. d. vung goc nhu h`nh
ve.
,
,
s phu,c z = x + iy biu din
nhu, mt
Mi
m ( x, y) trong
. di
h. toa
. d. vung goc.
,
`, di
` tu
. dai
m gc toa
d
n
. d
,.
,
,
di
m s phu c goi
. la` modun cua z va`
,,
du
|z|. Tu`, h`nh ve ta thy
o. c ky hiu
p .
,a truc
|z| =
x2 + y2 0. Goc giu
.
,,
`
Ox va` Oz do
theo
chi
u
ngu
o
c
kim

.
,
`
d
l
a
argumen
c
ua
z;
va` ky
`ng h` goi
.
`
hiu
. la` argz. Ro rang
(
x = |z| cos ,
y = |z| sin ,
,,
,
,
o dy
= argz. Nhu vy
. s phu c z =
H`nh 4.1:
,
,,
qua dang

x + iy biu din
luo. ng giac
.
z = (cos + i sin ), = |z|, = argz.
,,
,,
,
s phu,c, co nhi`u thun
phep
o dang
luo. ng giac
.
. lo. i trong cac
,
,

tnh. Chang
han
. ta ly phep nhn hai s phu c: z1 = 1 (cos 1 +

,
4.3. S phu c

101

i sin 1 ), z2 = 2 (cos 2 + i sin 2 ). Khi d


o
z1 z2 =

= 1 2 [(cos 1 cos 2 sin 1 sin 2 ) + i (cos 1 sin 2 + cos 2 sin 1 )]


= 1 2 [cos(1 + 2 ) + i sin(1 + 2 )].
,
,
, ,,
` kha dep.
Cng thu c nhn va` chia s phu c o dang
nay
Kt qua
.
.
,

tng quat:
V du. 4.12. Cho nhu ,ng s phu,c
z1 = 1 (cos 1 + i sin 1 ),
z2 = 2 (cos 2 + i sin 2 ),
........................
zn = n (cos n + i sin n ).
Khi d
o
z1 z2 . . . zn = 1 2 . . . n (cos(1 + 2 + + n )+
i sin(1 + 2 + + n )) (4.4)
vo,i n = 2, 3, . . .
,
,
,
,
,,
,
`
quy

thu c trn bang


phuong phap
Lo` i giai. Chu ng minh dang
,
,
hoc

nap
theo phep
. toan
. theo n., Vo i n, = 2, cng thu c (4.4) dung
,,
,
,

` d
thu c dung
nhn hai s phu c. Gia su dang
cho mt
o. Ta
. s n nao
,
,
,
,

thu c (4.4) dung


se chu ng minh dang
cho n + 1. Tht
. vy,
. vo i n + 1,
,,
,
s phu c z = (cos + i sin ), = 1, 2, . . . , n + 1. Su dung
gia
.
thit quy nap
.
z1 z2 . . . zn zn+1 = 1 2 . . . n [cos(1 + 2 + + n )

+ i sin(1 + 2 + + n )].n+1 (cos n+1 + i sin n+1 )

,,
Chuong 4. S hoc
.

102

= 1 2 . . . n n+1 [cos(1 + 2 + + n ) cos n+1


sin(1 + 2 + + n ) sin n+1
+ i [cos(1 + 2 + + n ) sin n+1
+ sin(1 + 2 + + n ) cos n+1 ]
= 1 2 . . . n n+1 [cos(1 + 2 + + n+1 )
+ i sin(1 + 2 + + n+1 )].

,,
,
Truo` ng ho. p ring 1 = 2 = . . . = n = va` 1 = 2 = . . . =
,,
, `,
,
`
n = cng thu c vu
a chu ng minh tro thanh
zn = n (cos n + i sin n).
,
,,
,
` trn du
Nu di
m z nam
o` ng tr`on do
n vi. th` z = cos + i sin va`

(cos + i sin )n = (cos n + i sin n).


,
,
,
`
Cng thu c sau cung
goi
Cng thu c Moivre
. la` cng thu c Moivre.
,
,
,
,

` toan
lin quan
co nhi`u u ng dung
trong thu. c t cung
nhu giai bai
.
,
,
` toan
rt hay nhung khng lin quan d
d
n s phu c. Nhi`u bai
n
,,
,,
1
quy nap
hoc
.
phuong phap
n o dy

. toan
. ti khng xet d
,
,
V du. 4.13. Chu,ng minh rang
` nu kt qua thu. ,c hin
. mt
. s hu u
han
phep tru`,, phep nhn, phep chia) trn
. phep tnh (phep cng,
.
,
,
day
s x1 , x2 , . . . , xn la` mt

cac

. s u, th` kt qua thu. c hin


. cung
,
,
,

phep tnh d
lin ho. p x1 , x2 , . . . , x n se nhn
o. c s u
o trn day
. du
,
,
lin ho. p cua u.
,
,
,
,

Lo` i giai. Bu,o,c co, so,: `u tin ta se chu ng minh mnh


d
` dung
.
,
, `,

voi tung phep tnh trn hai s phu c. Cho x1 = a + ib, x2 = c + id.
1 Ban

,
,
,
,
doc
cua s phu c trong [2].
. co th t`m thy mt
. s u ng dung
.

,
4.3. S phu c

103

khi d
o
x1 + x2 = ( a + c) + (b + d)i = u,

x1 + x2 = ( a ib) + (c id) = ( a + c) (b + d)i = u.


,
` phu,o,ng phap
`,, nhn, chia hai
tu,o,ng tu., ta kim tra phep tru
Bang
,
,

s phu c, khang
dinh
d
`u dung.
.
,
,
,,
, ,
,
s phu,c x1 , x2 , . . . , xn .
By gio` gia su cho biu thu c hu
u han
. cac
,
,
,
biu thu,c nhu, vy
cac

Thu. c hin
. tnh toan
. la` thu. c hi, n
. mt
. day
,
,
,
,

phep tnh trn hai s phu c, vic


s thu tu.
. thu. c hin
. co th danh
,
,
,,
,

du
han
o. c. Chang
. biu thu c
x1 x2 + x3 x4
u=
.
x1 + x2 x3
,
,
,
,,
bu,o,c sau:
thu. c hin
. cac
. u nguo` i ta thu. c hin
1) x1 x2 = u1 ,

4) u3 x3 = u4 ,

2) x3 x4 = u2 ,

5) u1 + u2 = u5 ,

3) x1 + x2 = u3 , 6)u5 : u4 = u.
, ,,
,
,
,

biu thu,c ma` trong su., tnh


Gia su mnh
vo i tt ca cac
` dung
. , d
,,
,,
,
d`
bu,o,c thu.,c hin
toan
oi hoi khng qua k buo c thu. c hin,
. cac
. o
`,, nhn hoac
` cng,
. chia hai s phu,c. Ta se chu,ng minh
dy
tru
la:
.
,
,
,
,,
,
,

mnh
d
vo i cac
biu thu c d`
` dung
oi hoi k + 1 buo c thu. c hin.
.
.
, ,
,

`
`
Tht
v
y,
bu
o
c
th
u
c
hi
n
cu
i
c
ung
k
+
1
trn
hai
s
u
v
a
u
,
m
a`
i
j
. .
.
. , ,
,
,
ng s nay
` la` kt qua cua vic
nhu
. thu. c hin
. khng qua k phe, p
,
`
s lin ho.,p cua
cac
tnh. Kt qua khi ta thay x1 , x2 , . . . , xn bang
,
`

s ui va` u j cung

chung
th` theo gia thit quy nap
thay bang
. cac
,
,
,
,
s lin ho. p cua chung

cac
trong kt qua thu. c hin
. phep tnh. Khi
,
,,
,
,
`

d
thay bang
s lin ho. p cua no u trong buo c thu. c hin
o u cung
.
,
thu k + 1.

104

,,
Chuong 4. S hoc
.

,
,,
2
V du. 4.14. Cho z la` nghim
. cua phuong tr`nh bc
. hai x + x +
,
,
1 = 0. Chu ng minh rang
` vo i n = 0, 1, . . . ta co
(
0, nu n khng chia ht cho 3,
1 + zn + z2n =
3, nu n chia ht cho 3.

,
,
1
3
,
,
,
Lo` i giai. Nghim
. cua phuong tr`nh la` b1 = 2 + i 2 , b2 =

,
1
3
,,
i
. Ta ky hiu
z la` nghim
bt ky` cua phuong tr`nh,
.
.
2
2

3 ,,
1
, o dy
ngha la` z = + i
= 1 khi z = b1 va` = 1 khi
2
2
,
1
. = arg z( ); bo,i v` |z| = 1 nn cos =
z = b2 . at
2

,
2
3
,

` dy
va` sin =
, tu
. Suy ra z co biu din
ta co =
2
3
,,
sau
luo. ng giac
2
2
z = cos
+ i sin
.
3
3
,
p dung
cng thu c Moivre
.
n = 1 + zn + z2n
2n
4n
4n
2n
+ i sin
) + (cos
+ i sin
).
= 1 + (cos
3
3
3
3
, ,,
,
,
cho tng cos va` sin ta
p dung
cng thu c bin d
i luo. ng giac
.
,,
nhn
o. c
. du
n
n
+ 2i sin n cos

n = 1 + 2 cos n cos
3
3
n
= 1 + (1)n 2 cos
.
3
Ta chu y cos n = cos n = (1)n , sin n = 0. Cho n chia ht
,
cho 3. Khi d
o n = 3k (k la` s tu. nhin) va`
3k
n = 3k = 1 + (1)3k .2 cos
3
= 1 + (1)k .2(1)k = 1 + 2 = 3.

4.4. Nhu ng v du. khac

105

,,
Nguo. c lai
n = 3k + s ( s = 1 va` s = 2) va`
. n co dang
.

(3k + s)
3
s
k+s
k
= 1 + (1) .2(1) . cos
3
1
= 1 + (1)k+s .2.(1)k .(1)s+1 . = 0.
2
,
,,
` toan
co th vit lai
Vy
. bai
. duo i dang
.
(
0, s 6= 0, (k = 0, 1, . . . ; s = 1, 2)
3k+s = 1 + z3k+s + z6k+2s =
3, s = 0,
(4.5)
n = 3k+s = 1 + (1)3k+s .2 cos

V` 1 + z + z2 = 0, nn 0 = (1 z)(1 + z + z2 ) = 1 z3 suy
,
ra z3 = 1. Vo i k = 0 ta co s = 1 + zs + z2s (s = 0, 1, 2); 0 =
1 + 1 + 1 = 3, 1 = 1 + z + z2 = 0, 2 = 1 + z2 + z4 = 1 + z2 (1 + z2 ,
,,
,
,
dung

o dy
1 + z2 = z. Va` nhu vy
da ap
.
. vo i k = 0, (4.5) dung.
, ,,
,
,
,

` d
Gia su by gio (4.5) dung
voi k 0 nao
o, ta se chung minh
`

no cung dung cho k + 1. Tht


rang
. vy,
.
3(k+1)+s = 1 + z3k+s+3 + z6k+2s+6 = 1 + z3k+s z3 + z6k+2s z6

= 1 + z3k+s + z6k+2s = 3k+s ,


,,
,,
z3 = 1, z6 = (z3 )2 = 1.
o dy
ta da su dung
.

,
ng v du. khac

4.4. Nhu
V du. 4.15. Cho n va` k la` nhu ,ng s tu. , nhin, k 2. Chu,ng minh
,
s trong chung
rang
` t`n tai

phn tch
. n s tu. nhin lin tip, mi
,

`
ra tch t nht k thua s nguyn t.
,
,
,
,
,
` toan
co ngha la` t`n tai

Lo` i giai. Vo i k = 2 khang


dinh
. cua bai
. n
,
,
,
,
ng ho. p s. Day
nhu vy
lin tip nhu
. v du. nhu (n + 1)! + 2, (n +

106

,,
Chuong 4. S hoc
.

,
,
` d
1)! + 3, . . . , (n + 1)! + (n + 1). Gia thit vo i s k nao
o ta da t`m
,,
,

du
o. c n s tu. nhin lin tip
N, N + 1, . . . , N + n 1
s nay
`,a s nguyn t. Khi d
` phn tch ra tch t nht k thu
mi
o
,
s trong nhu
ng s sau
mi

( N + n 1)! + N, ( N + n 1)! + N + 1, . . . , ( N + n 1)! + N + n 1


,,

`,a s nguyn t. Tht


phn tch du
o. c ra t nht k + 1 thu
. vy,
. mi
s ( N + n 1)! + N + i (i = 0, 1, . . . , n 1) chia ht
s trong cac
,
,,
,ng s nay
` phn tch du
cho N + i, ma` theo gia thit quy nap
o. c
. nhu
( N + n 1) ! + N + i
`, s nguyn t, c`on s
ra tch t nht k thu
N+i
,
1.
hin nhin la` khac

V du. 4.16. Cho m va` n la` nhu ,ng s tu. , nhin. Chu,ng minh rang
`

t nht mt
s n m, m n khng vu,o.,t qua 3 3.
. trong cac
,
,
,
,
,
,

Lo` i giai. Ta se chu ng minh vo i moi


. s tu. nhin n co bt dang
,
,
,
hoc.
thu c sau 3n n3 . Chu ng minh theo quy nap
. to,an
. ,Vo i n =
,,

1, 2, 3, 4 ta co 31 13 , 32 23 , 33 33 , 34 43 . Gia su khang
dinh
.
,
,

dung
vo i n = k, (k 4). Khi d
o theo gia thit quy nap
.
3k+1 = 3.3k 3.k3 = k3 + 3k2 + 3k + (k 3)k2 + (k2 3)k > (k + 1)3 .
,
,
,
V` k > 3, (k 3)k2 1, (k2 3)k > 1. Nhu vy
. vo i moi
. s tu. nhin

. la` 3 3 n n.
n, ta co 3n n3 hoac
,
Cho m va` n s tu. nhin va` n m. Khi d
o

3
n
m n n 3.

,
V du. 4.17. B. ba Pythagore la` mt
. b. ba s tu. nhin ( x, y, z),
sao cho x < y < z va` x2 + y2 = z2 . Chu,ng minh rang
`
vo,i s

4.4. Nhu ng v du. khac

107

tu. , nhin bt k`y n, s 2n+1 co mat


. trong n b. ba Pythagore khac

nhau.
,
,
,
`
hoc
Lo` i giai. Chu ng minh bang
quy nap
. toan
. theo n,
,
,
b.
. 1 l`n trong cac
1) Vo i n = 1 kt lun
. hin nhin: S 4 gap
.
, ,,
,
`

2) Gia su mnh
d
vo i bt ky` k n. Khi d
dung
o nu b.
.
,
,
n
+
2

ng s x, y, z khng
ba Pythagore ( x, y, z) co chu a s 2 , va` nhu
,

ng s nay
`,a s chung 2 va`
`
` co thu
nguyn t cung
nhau, th` nhu
,
x y z
,
( , , ) la` b. ba Pythagore trong d
o chu a s 2n+1 . Theo gia thit
2 2 2
, ,,
,,
,ng b. ba nhu, vy
quy nap
co s luo. ng n. Gia su ( x, y, z) b. ba
. nhu
.
,
s cua chung

`
Pythagore , ma` cac
x, y, z nguyn t cung
nhau, va`
n
+
2
`

mt
. trong chung bang 2 .
2
2
2
th` x va` y
Theo di
`u kin
. x + y = z . V` th nu z la` s chan
(do chung

`
khng chan
nguyn t cung
nhau). Va` suy ra x2 va` y2
,
,
khi chia cho 4 cho du 1. Khi d
o x2 + y2 khi chia cho 4 cho s du 2,
,
d
Suy ra z la`
nhung trong khi d
o z2 chia ht cho 4. Dn
n v ly.
,
,
,
,
n
+
2
s le va` truo` ng ho. p ring z 6= 2 .
,
Nhung x2 = (z y)(z + y), va` nu x = 2n+2 , th`
`, d
x2 = 22n+4 = (z y)(z + y), tu
o z y = 2k , z + y =
1 k
22n+4k (0 k 2n + 4). Suy ra z =
(2 + 22n+4k ).
2
,
,
. la` k = 1, hoac
. la` k = 2n + 3.
Nhu vy
. z la` le, th` hoac
,`,
,
,
2n
Trong truong ho. p thu nht z = 1 + 2 +2 , y = 22n+2 1,
,
n+2 < 22n+2 1 < 22n+2 + 1 vo
,i n 1,
x = 2n+2 va` nhu vy
. 0<2
,,
th` (2n+2 , 22n+2 1, 22n+2 + 1) b. ba Pythagore . Trong truo` ng
,
,
,,
,
ho. p thu hai z = 1 + 22n+2 , y = 1 22n+2 . Nhu vy
. trong truo` ng
,
`
ho. p cui cung
y < 0, th` t`n tai
. mt
. b. ba ( x, y, z) sao cho x, y, z
n
+
2
`
nguyn t cung
nhau va` x = 2 .

,,
Chuong 4. S hoc
.

108

,,
,
,
`
Cui cung
cho y = 2n+2 , khi d
o ly lun
. tuong tu. nhu ph`n
,,
. la` x = 22n+2 1, hoac
. la`
trn ta nhn
o. c z = 2n+2 + 1 va` hoac
. du
,
,
,
,
,
,
2n
+
2

x = 12
. Nhung vo i n 1 trong truo` ng ho. p thu nht x > y,
,`,
,
,
c`on truong ho. p thu hai x < 0.
,
Nhu vy,
x, y, z
. t`n tai
. mt
. b. ba s Pythagore ( x, y, z) sao cho
,
n
+
2
`
. b.
nguyn t cung
nhau va` co mt
. s la` 2 . V` th tt ca cap
,
n
+
2
`
ba trong no chu a 2 , bang
n + 1.

k
k
V du. 4.18. Chu,ng minh rang
` nu a b (mod m), th` am bm
,
(mod mk+1 ), o, dy
k = 0, 1, 2, . . .

,
,
,
,
`
Lo` i giai. Ta chu ng minh bang
quy nap theo k. Vo i k = 0 th`
,
, ,, , .
k

` d
mnh
d
hin nhin. Gia su vo i mt
` dung
o, ta co am
.
. k nao
k
. l = mk . Ta co
bm (mod mk+1 ). Ta dat
k +1

k +1

bm = ( al bl )( al (ml ) + al (m2) bl + + bl (m1) ).


,
`,a s thu, nht chia ht cho mk+1 ,
V` theo gia thit quy nap,
. thu
,
,
`,a s thu, hai chia ht cho m. Nhu,ng
vy
. ch c`on chu ng minh thu
`, d
a b (mod m), tu
o al bl (mod m) va` al (m1) + al (m2) bl +
+ bl (m1) al (m1) + al (m1) + + al (m1) mal (m1) 0
(mod m).
am

` tp
4.5. Bai
.
,,
,
n +1 +
`
. 4.19. Chu,ng minh rang
vo i moi
. s nguyn duong n, 11
122n1 chia ht cho 133.
,,
,
`
. 4.20. Chu,ng minh rang
vo i s n nguyn duong
1) A = n7 + 6n chia ht cho 7;
2) B = 26n+1 + 32n+2 chia ht cho 11;
3) D = 22n+1 9n2 + 3n 2 chia ht cho 54.

` tp
4.5. Bai
.

109

,,
,
`
. 4.21. Chu,ng minh rang
vo i n nguyn duong, s 23n+3 7n + 41
,
,ng s nguyn du,o,ng,
Vo,i a, n la` nhu
chia ht cho 49. Tng quat:
,
,
biu thu c An = ( a + 1)n an 1 chia ht cho a2 .
`
chu
, s, chu
, s d
. 4.22. Chu,ng minh rang
mt
`u
. s co s chan
,
s cui cung
`
s c`on lai
tin va` chu
la` 1, cac
. la` 0, th` no chia ht
cho 11.
,, n
`
, s 1, chia ht cho
. 4.23. Chu,ng minh rang
mt
. s tao
. boi 3 chu
3n .
,
,
`
,ng s co
. 4.24. Chu,ng minh rang
vo i s tu. nhin n 2, nhu
n
, s cui cung
, s 7 (S Fermat).
`
dang
N = 22 + 1 co chu
la` chu
.

. 4.25. Cho day


s a1 , a2 , a3 , . . . sao cho a1 = a2 = 1 va` an+2 =
,
,
`
an + an+1 . Chu ng minh rang
a5n chia ht cho 5 vo i n = 1, 2, . . ..

, ,
CHUONG

S
DAY
,
5.1. D
ay s tu. nhin . . . . . . . . . . . . . . . . . . . . . . . . . . . . . . . . . . . . . .
,
5.2. D
ay tri
. hon .,. . . . . . . . . . . . . . . . . . . . . . . . . . . . . . . . . . . . . . . . .
,
,
,
ng th
5.3. Nhung bt da
uc ni ting . . . . . . . . . . . . . . . . . . . . .
,
5.4. D
ay do
n di
u
. .......................................
5.5. S e . . . . . . . . . . . . . . . . . . . . . . . . . . . . . . . . . . . . . . . . . . . . . . . . . .
5.6. D
ay s Fibonacci . . . . . . . . . . . . . . . . . . . . . . . . . . . . . . . . . . . .
` tp
5.7. Bai
. ...............................................

110
117
121
128
131
134
139

,,
,,
t nhi`u
quy nap
hoc
Phuong phap
dung
o. c ap
. toan
. du
. , cho r
,
,
,
,
` toan
v` day
s. Kt ho. p vo i cac
tnh cht cua bt dang

bai
thu c
,
,
,
,,
`

nay
` rt ngan

thu c th` chu ng minh bang


va` dang
phuong phap
,
gon
. va` d hiu.

,
y s tu. nhin
5.1. Da
,
V du. 5.1. Cho day
v han
. s tu. nhin a1 = 1, a2 , a3 , . . . , an , . . . va`
,
,
thoa man
bt dang

thu,c sau
a n 1 + a 1 + a 2 + + a n 1
,
,
,
vo,i moi
` vo,i moi
. s tu. nhin n 2. Chu ng minh rang
. s tu. nhin
,
,
,
,
,,
nhu, tng cua mt
du,o,ng co th biu din
` s du
o. c chon
. vai
. trong
day.

,
,
,
,
,
`

Lo` i giai. Ta se chu ng minh rang


moi
. s tu. nhin N thoa man

s tu., nhin
5.1. Day

111

,
, ,
,

bt dang
thu c 0 < N < 1 + a1 + a2 + + an , co th biu din
,
,
,
` s trong day
a1 , a2 , . . . , a n .
nhu mt
. tng cua mt
. vai
,
,

Voi n = 1 mnh
d
v` a1 = 1, khi d
` dung
o 0 < N < 1 + 1 ch
.
la` N = 1 = a1 .
, ,,
,
,

Gia su mnh
d
vo i s tu. nhin n = k 1, ngha la` moi
` dung
.
.
,
,
,
,
bt dang

s tu. nhin N thoa man


thu c 0 < N < 1 + a1 + a2 +
, ,
,
,
nhu, mt
` s trong day

+ ak , co th biu din
. tng cua mt
. vai
,
,
`

a1 , a2 , . . . , ak . Ta se chu ng minh rang


no cung
dung
vo i n = k + 1.
,
,
,`,
,
Ta ch xet truong ho. p sau dy
la` du
1 + a 1 + + a k N < 1 + a 1 + + a k + a k +1
,
`, giai

v` nu 0 < N < 1 + a1 + + ak , th` mnh


d
suy ra tu
` dung
.
thit quy nap.
.
,,
` ta nhn
Do di
`u kin
`u bai,
o. c
. d
. du
0 1 + a 1 + + a k a k +1 N a k +1 < 1 + a 1 + + a k .
,

Nu N ak+1 = 0, th` mnh


d
vo i n = k + 1; nu N
` dung
.
,
, ,

ak+1 > 0, th` theo gia thit quy nap


N ak+1 co th biu din
.
,
,
, ,

` s trong a1 , a2 , . . . , ak va` khi d


nhu tng cua mt
o N biu di
n
. vai
, ,
` a k +1 .
nhu tng trn va` thm vao

V du. 5.2. Cho p1 < p2 < . . . < pn < . . . la` day


s nguyn t.
,
,
Chu ng minh rang
` giu a hai s p1 + p2 + + pn va` p1 + p2 + +
,,
pn+1 lun lun co mt
. s chnh phuong.
,
, ,,
,
s nguyn t.
Lo` i giai. Gia su 2 = p1 < p2 < p3 . . . day
,
,
,
`
1) Ta chu ng minh rang
vo i n 7, pn > 2n + 1: Ta chu ng minh
,,
,
`
quy nap.
bang
phuong phap
. Vo i n = 7, ta co p7 = 17 > 15, mnh
.
, ,,
,
`

d
Gia su mnh
d
d
ung
cho
n
=
k,
p
>
2k
+
1.
Ta
ch
u
ng
` dung.

k
.

,,
s
Chuong 5. Day

112

minh mnh
d
dung
vo i n = k + 1, tht
` cung
.
. vy,
. do pn la` s
, ,
le vo i moi
. n > 1, nn pk+1 pk 2, ngha la` pk+1 pk + 2 >
,
,

2k + 1 + 2 = 2(k + 1) + 1. Nhu vy
cho
. ta da chu ng minh dung
moi
. n 7, pn > 2n + 1.
,
,
,
2
2) Ta chu ng minh vo i moi
. n, yn n : Ky hiu
. yn la` tng
y n = p1 + p2 + + p n .
Ta co y1 = 2 > 12 , y2 = 5 > 22 , y3 = 10 > 33 , y4 = 17 > 42 , y5 =
28 > 52 , y6 = 41 > 66 va` y7 = 2 + 3 + 5 + 7 + 11 + 13 + 17 =
,
,
58 > 49 = 72 . Ta se chu ng minh quy nap
. theo n, vo i n 7 ta co
y n > n2 .
, ,, ,
,
2
Tht
. vy,
. Gia su vo i n = k, ta co yk > k . Vo i n = k + 1 ta tnh
,,
yk+1 = yk + pk+1 > k2 + (2k + 1) = (k + 1)2 , ta da su dung
kt
.
,
,
,
qua ph`n trn va` gia thit quy nap.
. Ngha la` ta da chu ng minh
,
2
voi moi
. n, yn n .
,
,,
,
,
,
Ly m2 la` s chnh phuong lo n nht khng lo n hon yn (co th
,,
,
, ,
ly du
o. c theo chu ng minh trn va` tin d
` thu tu. ). Khi d
o theo
,
,
,
. la` m = n + k, k 0. Nhu vy,
chu ng minh trn m n hoac
. vo i
moi
. n 1 t`n tai
. s k 0 sao cho

( n + k )2 y n ( n + k + 1)2 .
, ,,
,
,,
Ta se chu ng minh pn+1 > 2(n + k ) + 1. Gia su nguo. c lai,
. ta co
,
,
pn+1 2(n + k ) + 1. Nhung voi n 2, pn+1 pn + 2. Suy ra
pn 2(n + k ) + 1 2 = 2(n + k ) 1,
pn1 2(n + k ) + 1 4 = 2(n + k ) 3,
...............................
pn j 2(n + k ) + 1 2( j + 1) = 2(n + k ) (2j + 1),
...............................

s tu., nhin
5.1. Day

113

3 = p2 2(n + k ) + 1 2(n 1) = 2(n + k ) (2n 3),


2 = p1 2(n + k ) + 1 2n = 2(n + k ) (2n 1).
,
,
`,ng v cac
bt dang

Cng
tu
thu c trn, ta co
.
yn = p1 + p2 + + pn 2n(n + k ) (1 + 3 + + (2n 1))

= 2n(n + k) n2 = n2 + 2nk + 1 1 = (n + k)2 1.


,
,
Ngha la` y (n + k )2 1. Nhung theo gia thit y (n + k )2 va`
n

d
Suy ra pn+1 > 2(n + k ) + 1. Khi
yn la` s nguyn. Dn
n v ly.
d
o
y n +1 = y n + p n > ( n + k )2 + 2 ( n + k ) + 1 = ( n + k + 1 )2 > y n ,
ngha la`
Suy ra (n + k

+ 1)2

y n < ( n + k + 1 )2 < y n +1 .
` giu
,a yn va` yn+1 .
nam

V du. 5.3. Cho day


s chia thanh
`
tu`,ng nhom nhu, sau:
,
(1), (2, 3), (4, 5, 6), (7, 8, 9, 10), (11, 12, 13, 14, 15), . . . Tnh tng S1 +
,
,
,
,
,
S3 + S5 + + S2n1 , o, dy
Sk la` tng nhu ng s cua nhom thu k.
,
,
,
,
`
Lo` i giai. S d
`u tin cua nhom thu k bang

(1 + 2 + + (k 1)) + 1 =
,
,
,,
,
C`on tng cua k s o nhom thu k la`

k ( k 1)
+ 1.
2

k ( k 1)
( k + 1) k
+1+
)
k3 + k
2
2
Sk =
=
.
2
2
,
`
`
hoc
Bang
quy nap
. toan
. theo n, ta chu ng minh rang
k(

S1 + S3 + + S2n1 = n4 .
,
1) Vo i n = 1, ta co S1 = 1 = 14 .

(5.1)

,,
s
Chuong 5. Day

114

, ,,
,
,

` d
vo i n nao
2) Gia su (5.1) dung
y, ta chu ng minh cho n + 1.
S1 + S2 + + S2(n+1)1 = (S1 + S2 + + S(2n1) ) + S2n+1

(2n + 1)3 + (2n + 1)


2
= n4 + (2n + 1)(2n2 + 2n + 1)
= n4 +

= n4 + 4n3 + 6n2 + 4n + 1 = (n + 1)4 .


,
,
,,
,

ang
thu c (5.1) dung
vo i moi
. n nguyn duong.
, , dinh theo cng
V du. 5.4. Cho day
s F1 , F2 , F3 , . . . , Fn , . . . du
o. c xac
.
,
,
thu c sau: F1 = 1, F2 = 1, Fn = Fn1 2Fn2 vo i n 3. Chu,ng
minh rang
` vo,i n 2 s 2n+1 7Fn21 la` s chnh phu,o,ng.
,
,
,
`
hoc
Lo` i giai. Chu ng minh bang
quy nap
d
`
. toan
. theo n, mnh
.
,
sau: Voi n 2,

2n+1 7Fn21 = (2Fn + Fn1 )2 .


, ,,
3
2

Tht
d
` dung
. vy,
. n = 2 ta co 2 7 = (2 + 1) . Gia su mnh
.
,,
,
voi moi
n 2. Khi d
o
. k n, o dy

(2Fn+1 + Fn )2 = (2Fn 4Fn1 + Fn )2 = ( Fn 4Fn1 )2


= Fn2 + 8Fn Fn1 + 16Fn21
= 2(4Fn2 + 4Fn Fn1 + Fn21 ) + 14Fn21 7Fn2
= 2(2Fn + Fn1 )2 + 14Fn21 7Fn2
= 2(2n+1 7Fn21 ) + 14Fn21 7Fn2
= 2n+2 7Fn2 .
,
,

Nhu vy
d
vo i k = n + 1.
` dung
. mnh
.
,
V du. 5.5. Cho n 1 la` mt
ngha day
s
. s tu. nhin. inh
.
x1 , x2 , . . . va` y1 , y2 , . . . theo cach
sau:
n
xi + yi
] , y i +1 = [
] (i = 1, 2, . . .),
x1 = n, y1 = 1, xi+1 = [
2
x i +1

s tu., nhin
5.1. Day

115

[ x ] s nguyn lo,n nht khng lo,n ho,n x.

Chu,ng minh rang


` min{ x1 , x2 , . . . , xn } = [ n].
,

,
,
,
i (i = 1, 2, . . .), xi [ n].
Lo` i giai. Ta se chu ng minh vo i mi
,
,
, ,,
,
,
,
,

Bt dang
thu c dung
vo i i = 1. Gia su bt dang
thu c dung
vo i

i = k. Khi d
o xk = [ n] + t, t 0,

n
[ n ]2
[ n ]2 t2
yk = [
][
][
] = [ n] t
[ n] + t
[ n] + t
[ n] + t
suy ra

xk + yk
[ n] + t + [ n] t
x k +1 = [
][
] = [ n ].
2
2
,
hoc
thu,c c`n chu,ng
Khi d
o theo nguyn ly quy nap
. toan
. bt dang
,

minh dung
vo i moi
. i = 1, 2, . . ..
, ,, ,

,,
` d
Gia su vo i mt
s s nao
t 1
o co xs = [ n] + t, o dy

.
,
, ,,
` co th du
` ra nu ly

(di
han ly s = 1). Ngoai
`u nay
o. c, chang
, .

2
,,
0 p 2[ n] (ta cung

ly
n = [ n] + p, o dy
s p thoa man
,
,,
,
,
,
,
nhu vy
du
o. c do p la` s tu. nhin va` p thoa man
. v` nu nguo. c lai
.

,
2

th` dn toi di
`u v ly n ([ n + 1] ).
Khi d
o

n
[ n ]2 + p
[ n ]2 + 2[ n ]

]=[
][
] = [ n]
ys = [
[ n] + t
[ n] + t
[ n] + 1

v`

[ n ]2 + 2[ n ]

] < [ n + 1].
[ n] < [
[ n] + 1

,,
`, ys [ n] < [ n] + t = xs ta nhn
Tu
o. c ys xs 1 suy ra
. du

x s +1 = [

xs + ys
2xs 1
[
] = xs 1 < xs .
2
2

116

,,
s
Chuong 5. Day

` ngha la` khi xs > [ n] vo,i s = 1, 2, . . . day


x1 , x2 , . . . se
i`u nay
,
, ,
, ,
,
, ` ,
,
giam thu. c su. va voi s du lo n (nhung nho hon n, v` x1 = n) th` se
,

co dang
thu c xs = [ n].

,
,,
ngha theo dang

V du. 5.6. Cho day


s a1 , a2 , . . . , an du
o. c dinh
.
thu,c sau: ak = k 1 vo,i k = 1, 2, 3, 4 va` a2n1 = a2n2 + 2n2 , a2n =
,
a2n5 + 2n vo,i moi
n 3. Chu,ng minh rang
` vo,i moi
. s tu. nhin n
, .
khac
khng dang

thu,c sau dung:





12 n1
17 n1
a) 1 + a2n1 =
.2
;
b) 1 + a2n =
.2
.
7
7

,
,
,
,
,
,
, `
kim

Lo` i giai. Vo i n = 1 va` n = 2 dang


thu c dung
vo i bang
cach
,
, ,,
,
,
,
,

tra tru. c tip. Gia su hai dang


thu c trn dung
vo i hai s tu. nhin
,
,
,

lin tip n 1 va` n. Ta se chu ng minh hai dang


thu c dung
cho
gia tri. tip theo n + 1.
,
`, dinh
1 + a2n+1 = 1 + a2n + 2n1. Chu y to,i
Tu
ngha cua day
.
,
17 n1
,

thu c b) cho gia tri. n, ta co 1 + a2n+1 =


dang
.2
+ 2n1 . V`
7 

,
12 n
,
` .2.4 nn co 1 + a2n+1 =

thu c a) trong bai


dang
.2 suy ra
7
,
,
,
` toan
vo,i gia tri. n + 1.

dang
thu c a) cua bai
,
,
`, dinh
1 + a2n+2 = 1 + a2n3+ 2n+1 . Ch
Tu
. ngha cua day
 u y to i
,
12 n2
,

dang
thu c a) cho gia tri. n 1, ta co 1 + a2n+2 =
.2
+ 2n +1 .
7

,
17 n
,
` .2.4 nn co 1 + a2n+2 =

V` dang
thu c b) trong bai
.2 suy ra
7
,
,
,
` toan
vo,i gia tri. n + 1.

dang
thu c b) cua bai

,
tri
5.2. Day
. hon

117

,
y tri
5.2. Da
. hon

Cho hai day


a1 , a2 , a3 , . . .

(5.2)

(5.3)
,
,
,
(5.2) tri
(5.3) nu chung

bt dang

Ta goi
thoa man
. day
. hon day
,
thu c:
bn an , (n = 1, 2,, . . .)
(5.4)
,
,
,

`
hoc
`
` dac

. bit
Trong toan
loai
thuc nay,
. thuong dung
. bt dang
. cac
,
`

` toan
v danh

bai
gia mt
. qua tr`nh, t`m gioi han,
. ...
,
V du. 5.7. Chu,ng minh bt dang

thu,c
4n
(2n)!
, (n = 2, 3, . . .).
(5.5)
<
n+1
(n!)2
b1 , b2 , b3 , . . .

,
,
,
,
,

Lo` i giai. Chu ng minh quy nap


thu c (5.5) dung
. theo n. Bt dang
,
,
`
voi n = 2 suy ra tu
42
(2.2)!
16

=
6 < 0.
2
2+1
(2!)
3
, ,,
,
,
`

` d
Gia su (5.5) dung
vo i mt
no
o. Ta se chu ng minh rang
. s n nao

cung
dung
cho n + 1,
4n +1
(2n + 2)!
<
.
n+2
((n + 1)!)2
,
cua (5.6) du,o,i dang
Tht
. vy,
. ta vit v trai
.
4n +1
4n 4 ( n + 1 )
=

n+2
n+1 n+2

,
`, gia thit quy nap
Tu
. ta co

4n 4 ( n + 1 )
(2n + 2)!
2( n + 1)2
4n +1
=
<
.
,
n+2
n+1 n+2
((n + 1)!)2 (2n + 1)(n + 2)

(5.6)

,,
s
Chuong 5. Day

118

. khac
mat
0<

2( n + 1)2
2n2 + 4n + 2
= 2
=
(2n + 1)(n + 2)
2n + 5n + 2
(2n2 + 5n + 2) n
n
=
= 1 2
<1
2n2 + 5n + 2
2n + 5n + 2

,
vo i n = 1, 2, . . . va` suy ra
2( n + 1)2
(2n + 2)!
(2n + 2)!
4n +1
.
.
<
<
n+2
((n + 1)!)2 (2n + 1)(n + 2)
((n + 1)!)2

,
V du. 5.8. Chu,ng minh bt dang

thu,c

1
1
1 + + + > n,
n
2

(n = 2, 3, . . .).

(5.7)

,
,
,
1
,
,
,

Lo` i giai. Vo i n = 2 ta co bt dang


thu c 1 + > 2, bt dang
,
, ,, 2
, `
,
,

thu c nay
dung
qua kim tra tru. c tip. Gia su (5.7) dung
vo i mt
.
,
,
,

` d

thu c cung
dung
gia tri. n nao
o va` ta se chu ng minh bt dang
,
. la`
vo i n + 1, hoac

1
1
1+ ++
> n + 1, (n = 2, 3, . . .).
(5.8)
n+1
2
,
1
,
,

Tht
hai v bt dang
thu c (5.7) vo i s hang
, ta
. vy,
. cng
.
.
n+1
co

1
1
1
1
1+ ++ +
> n+
.
(5.9)
n
n+1
n+1
2
,
Nhung

1
n2 + n + 1
n2 + 1
=
>
= n + 1.
n+
n+1
n+1
n+1
,
` dy
Tu
va` (5.9) suy ra (5.8).

,
tri
5.2. Day
. hon

119

V du. 5.9. Chu,ng minh rang


`
a2 b2
a n bn
( a1 + + an )(b1 + + bn )
a1 b1
+
++

,
a1 + b1 a2 + b2
a n + bn
( a1 + + an ) + (b1 + + bn )
(5.10)
,,
,
,
,
o dy
a1 , a2 , . . . , an , b1 , b2 , . . . , bn la` nhu ng s duong.
,
,
,
,
,
,

Lo` i giai. Vo i n = 1 bt dang


thu c dung.
Vo i n = 2 ta co dang
.
a1 b1
a2 b2
( a1 + a2 )(b1 + b2 )
+

a1 + b1
a2 + b2
( a1 + a2 ) + (b1 + b2 )

(5.11)

. la`
hoac

( a1 + a2 )(b1 + b2 )( a1 + b1 )( a2 + b2 ) a1 b1 ( a2 + b2 )( a1 + a2 + b1 + b2 )
a2 b2 ( a1 + b1 )( a1 + a2 + b1 + b2 ) 0
,
,
, `

. la` ( a1 b2 a2 b1 )2 0 bt dang

hoac
thu c nay
hin nhin dung.
, ,,
,
,
,

` d
Gia su (5.10) dung
vo i s tu. nhin n 2 nao
o. Ta se chu ng
,,
,

minh no dung
vo i n + 1. Su dung
(5.10) va` (5.11) ta co
.
a1 b1
a2 b2
a n bn
a .b
+
++
+ n +1 n +1
a1 + b1 a2 + b2
a n + bn
a n + 1 + bn + 1
( a1 + + an )(b1 + + bn )
a .b

+ n +1 n +1
( a1 + + an ) + (b1 + + bn ) an+1 + bn+1
( a1 + + an + an+1 )(b1 + + bn + bn+1 )
.

( a1 + + an + an+1 ) + (b1 + + bn + bn+1 )

V du. 5.10. Cho 0 < x1 x2 . . . xn . Chu,ng minh rang


`
x1
x2
x
xn
x2
x3
xn
x
+
+ + n 1 +

+
++
+ 1.
x2
x3
xn
x1
x1
x2
x n 1
xn
,
,
,
,
,
,

Lo` i giai. Vo i n = 2 bt dang


thu c dung.
Vo i n = 3 ta co
x1
x2
x3
x2
x3
x
( x3 x2 )( x3 x1 )( x2 x
+
+

1 =
0
x2
x3
x1
x1
x2
x3
x1 x2 x3

,,
s
Chuong 5. Day

120

,
,
, ,,
,
,
,

bt dang
thu c dung.
Gia su bt dang
thu c dung
vo i n = k 1,
x2
x
x2
x3
x
x1
+
+ + k 1
+
++ 1 .
x2
x3
x1
x1
x2
x k 1
,
,
,
,
,
Do chu ng minh vo i truo` ng ho. p n = 3 nn ta co
x1
x
x
x
x
x
+ k 1 + k k 1 + k + 1 .
x k 1
xk
x1
x1
x k 1
xk
,
,
,
,
,
,
`

Cng
thu c sau cung
ta nhn
thu c
o. c bt dang
. , hai bt dang
. du
,
phai chu ng minh cho n = k.

V du. 5.11. Chu,ng minh rang


`
nu tch n s thu. ,c du,o,ng
,
,
,
bang
`
1, th` tng cua chung

khng nho ho,n n. Noi cach

khac,

,
,
,
,
`

cho x1 , x2 , . . . , xn la` nhu ng s duong, chu ng minh rang

nu
,
x1 x2 . . . xn = 1 suy ra x1 + x2 + + xn n vo i moi
. n = 1, 2, . . .
,
,
,
,
,
`, x1 x2 = 1 suy ra
Lo` i giai. Vo i n = 2, ta c`n phai chu ng minh tu
,
,
,
`, bt dang

x1 + x2 > 2. Tht
thu c hin nhin ( x1 1)2 0,
. vy,
. tu
1
,,
suy ra x12 + 1 2x1 , chia hai v cho x1 ta nhn
2,
o. c x1 +
. du
x
1
,
,
,

thu c xy ra khi x1 = 1, do d
ngha la` x1 + x2 2, dang
o x 1 =
x2 = 1.
, ,,
,
,

Gia su mnh
d
cho n 2. Ta se chu ng
` da chu ng minh dung
.
,
`,

minh dung
cho n + 1, ngha la` se chu ng minh tu
x 1 x 2 . . . x n x n +1 = 1

(5.12)

,
,

suy ra bt dang
thu c
x1 + x2 + + xn + xn+1 n + 1.
(5.13)
,
, ,
,
,
,
,

ang
thu c (5.12) ch xy ra hai truo` ng ho. p sau:
,
`
`,a s bang
thu
I. Tt ca cac
nhau x1 = x2 = . . . = xn+1 = 1.
,
`
s d
II. Khng phai cac
nhau.
`u bang

,
,
,
,

5.3. Nhu ng bt dang


thu c ni ting

121

,,
,
Trong truo` ng ho. p I. Ta co x1 + x2 + + xn+1 = n + 1.
,,
,
`,a s co thu
`,a s lo,n ho,n 1 th` cung
thu

Truo` ng ho. p II. Trong cac


,
,
,
,
` a s nho hon 1. Nu khng co d
co thu
`ng tho` i hai s co tnh cht
,
,

1. Chang

trn th` tch cua chung


se khac
han
. x1 < 1, xn+1 > 1.,
,,
. y1 = x1 xn+1 . Do gia
Khi d
o ta co y1 x2 x3 . . . xn = 1, o dy
ta dat
,

thit quy nap


vo i n, nn ta co y1 + x2 + + xn n. Khi d
o
. dung
x 1 + + x n +1 = ( y 1 + x 2 + + x n ) + x n +1 y 1 + x 1

n + x n +1 y 1 + x 1 = ( n + 1 ) + x n +1 y 1 + x 1 1
= ( n + 1 ) + x n +1 x 1 x n +1 + x 1 1
= (n + 1) + ( xn+1 1)(1 x1 ).
,
Do ta co xn+1 > 1 va` x1 < 1 suy ra di
`u c`n chu ng minh.

,
,
,
,
c ni ting
ng bt da
ng thu
5.3. Nhu
,
,
,
` tp
`

Bai
thu c v cung
phong phu va` chung loai
. v` bt dang
.
`
`

`
nhau, da co nhiu sach
d
` Trong muc
khac
cp
n vn d
nay.
. d
,
,
,
, .
,
,
,

ng bt dang
` chung

nay
ti ch lit
thu c co ban, tt ca d
`u
. k nhu
,,
,
,
,
`
quy nap.
du
phuong phap
o. c chu ng minh bang
.
,
V du. 5.12. (Bt dang

thu,c Cauchy). Cho day


s du,o,ng bt k`y
x1 , x2 , . . . , xn chu,ng minh rang
`

x1 + x2 + + x n
n x1 x2 . . . x n .
(5.14)
n
,
,
ang

thu,c xy ra khi va` chi khi x1 = x2 = . . . = xn .


,
,
. a =
Lo` i giai. Cach

chu,ng minh thu, nht: Ta dat


,

x1 + x2 + + x n
,
. g = n x1 x2 . . . xn . Khi d

va` dat
thu c
o dang
n,
, ,
n
` .5.11. Tht
(5.14) ch la` h. qua cua bai
. vy,
. g = x1 x2 . . . x n

122

,,
s
Chuong 5. Day

,
x1 x2
xn
x
` toan
tru,o,c ta so 1 +
. ...
= 1. Do kt qua bai
g g
g
g
,
, ,
x2
xn
,
,

++
n. Tu` d
thu c ch xy ra khi
o suy ra (5.14). ang
g
g
x2
xn
x1
=
= . . . = , ngha la` x1 = x2 = ... = xn .
g
g
g
,,
,
chu,ng minh theo
Cach

chu ng minh thu, hai: Phuong phap


,
vo,i b`nh thu,o`,ng do chnh Cauchy du
quy nap
dang
khac
a ra. Co
.
.
,
,
`,ng do
chu,ng minh quy nap
th noi dy
la` cach
an
. cho tu
. chu a cac
,
s tu. nhin.
,
,
,
,

Vo i n = 1 = 20 bt dang
thu c (5.14) dung.
Vo i n = 2 = 21 ,
,

, ,
`, ( x1 x2 )2 0 va` du dang

(5.14) suy ra tu
thu c xay ra khi
,
, ,,
,

va` ch khi x1 = x2 . Gia su (5.14) dung


vo i s n. Khi d
o
suy ra

x1 + x2
x3 + x4
x
+ x2n
+
+ + 2n1
x1 + x2 + + x2n
2
2
2
=
2n
n
r
x + x2 x3 + x4
x2n1 + x2n
n 1
.
...
2
2
2
q

n x1 x2 x3 x4 . . . x2n1 x2n

= 2n x1 x2 . . . x2n .
,
,
,
,
,

thu c (5.14) dung


vo i 2n. Suy ra no dung
vo i tt ca cac
Bt dang
,
,
n

1
s co dang
2
vo i moi
.
. s tu. nhin n.
,
,
,
`
Voi m la` s tu. nhin. Nu m co dang
2n vo i n la` mt
ao
.
. s n
,
,
,
`

trong khoang
d
V` vy
o, th` (5.14) dung.
. ch c`on kim tra m nam
,
n

1
n
n

1
n
a 2
. m + q = 2n . Khi
giu
va` 2 . ngha la` 2
< m < 2 . Ta dat
d
o
x1 + x2 + + x m + q

m+ p x1 x2 . . . x m + q .
m+q
x + x2 + + x m
,
. xm+1 = xm+2 = . . . = xm+q = 1
By gio` ta dat
,
m

,
,
,
,

5.3. Nhu ng bt dang


thu c ni ting

123

khi d
o
x1 + x2 + + x m
x1 + x2 + + x m + q + q
x1 + x2 + + x m
m
=
m
m+q
r
x1 + x2 + + x m q
m+q

x1 x2 . . . x m (
)
m
. la`
hoac




x1 + x2 + + x m q
x1 + x2 + + x m m + q
x1 x2 . . . x m
,
m
m


x1 + x2 + + x m m
x1 x2 . . . x m .
m
,
,
,
`, dy

Tu
Nhu vy
vo i moi
suy ra (4.13) dung.
. no dung
. s tu. nhin
n.
,
,
,
,
,
,

By gio` ta chu ng minh (5.14) xy ra dang


thu c ch khi x1 =
, ,,
x2 = . . . = xn , tht
vy,
gia su t nht co hai s trong x1 , x2 , . . . , xn ,
.
.
,
`

chang
han
nhau. Khi d
o
. x1 va` x2 khng bang
x1 + x2
x + x2
+ 1
+ x3 + + x n
x1 + x2 + + x n
2
2
=
n
n
s

2
x1 + x2
n

x3 x4 . . . x n .
2
,
`, x1 6= x2 suy ra
Nhung tu

x1 + x2
> x1 x2
2
,
,
`, hai bt dang

Tu
thu c trn ta suy ra

x + x2 + + x n
( 1
) > n x1 x2 . . . x n .
n
,
,
,
,
Nhu vy
d
` du
o. c chu ng minh.
. mnh
.

,,
s
Chuong 5. Day

124

,
x1 + x2 + + x n
s
goi
la` trung b`nh cng
cua cac
.
.
n
,

x1 , x2 , . . . , xn . C`on s n x1 x2 . . . xn goi
. la` trung b`nh nhn cua cac
s da cho.
,
V du. 5.13. (Bt dang

thu,c Bernoulli). Chu,ng minh rang


`
vo,i
,
,
,
moi

thu,c
. x > 1, x 6= 0 va` vo i moi
. s tu. nhin n 2 bt dang
sau dung

(5.15)
(1 + x )n > 1 + nx.

y:
s
Chu

,
,
,
,
,

Lo` i giai. Vo i n = 2 bt dang


thu c (5.15) co dang
1 + 2x + x2 >
.
,
, ,,
,

1 + 2x va` dung
la` hin nhin. Gia su (5.15) dung
vo i mt
. s n 2.
,
,
,

Ta se chu ng minh no dung


cho n + 1, ngha la` phai chu ng minh

(1 + x )n+1 > 1 + (n + 1) x.
Tht
. vy,
. ta co

(1 + x )n+1 > (1 + nx )(1 + x ) = 1 + (n + 1) x + nx2 ,


,
do nx2 > 0 suy ra di
`u c`n chu ng minh.
,
,
,
y:
Bt dang

Chu
thu c Bernoulli c`on dung
cho moi
. s thu. c:

(1 + x ) > 1 + x, x 1, > 1,
,,
, ,
,
o dy
la` mt
. s thu. c lo n hon 1.
,
V du. 5.14. (Bt dang

thu,c Cauchy-Bunyakovski). Chu,ng minh


rang
`
( x12 + x22 + + xn2 )(y21 + y22 + + y2n ) ( x1 y1 + x2 y2 + + xn yn )2
(5.16)
,
,
,
vo i x1 , x2 , . . . , y1 , y2 , . . . la` nhu ng s thu. c va` n = 1, 2, 3, . . .
,
,
,
,
,

Lo` i giai. Vo i n = 1, (5.16) dung


hin nhin. Vo i n = 2, ta co
( x1 y1 + x2 y2 )2 = ( x12 + x22 )(y21 + y22 ) ( x1 y2 x2 y1 )2

,
,
,
,

5.3. Nhu ng bt dang


thu c ni ting
125
,
, ,,
,
,
`, dy

tu
thu c (5.16) dung
vo i n = 2. Gia su bt
suy ra bt dang
,
,
,
,
,
,

dang
thu c dung
vo i n = k, ta phai chu ng minh no cung
dung
vo i
,
,
,
,

n = k + 1. Tht
thu c dung
vo i n = 2 va` gia thit
. vy,
. do bt dang
quy nap
. ta co

( x12 + x22 + + xk2+1 )(y21 + y22 + + y2k+1 )


q
q
( x12 + x22 + + xk2 y21 + y22 + + y2k + xk+1 yk+1 )2
( x 1 y 1 + x 2 y 2 + + x k y k + x k +1 y k +1 )2 .

,
V du. 5.15. (Bt dang

thu,c Chebychev). Cho day


s x1 , x2 , ..., xn
,
va` y1 , y2 , ..., yn la` 2n s, sao cho thoa man

x1 x2 . . . x n

(5.17)
y1 y2 . . . y n .
,
Chu,ng minh rang
`
tch cua trung b`nh cng
cac
s x1 , x2 , . . . , xn
.
,
,

vo i trung b`nh cng


cua cac
s y1 , y2 , . . . , yn khng vu,o.,t qua trung
.
,
b`nh cng
cua cac
s x1 y1 , x2 y2 , . . . , xn yn , hoac
. la`
.
x1 + x2 + + x n y1 + y2 + + y n
x y + x2 y2 + + x n y n
.
1 1
.
n
n
n
(5.18)
,
,
.
Lo` i giai. Ta dat
A n = x1 + x2 + + x n ,
Bn = y1 + y2 + + yn ,
Cn = x1 y1 + x2 y2 + + xn yn ,

(5.19)

Dn = nCn An Bn .
,
,
,

Khi d
thu c (5.18) co th vit lai
o bt dang
.
,
Dn 0 vo i n = 1, 2, . . .

(5.20)

,,
s
Chuong 5. Day

126

,
,,
,
`
quy nap.
Ta chu ng minh bang
phuong phap
o
. Vo i n = 1. Khi d
D1 = 1C1 A1 B1 = x1 y1 x1 y1 = 0.
,
,
,
,

Nhu vy
vo i n = 1. Ta kim tra vo i n = 2,
. (5.20) dung
D2 = 2C2 A2 B2 = 2( x1 y1 + x2 y2 ) ( x1 + x2 )(y1 + y2 )

= x1 y1 + x2 y2 x1 y2 x2 y1 = ( x2 x1 )(y2 y1 ) 0,
v` x2 x1 0 va` y2 y1 0 theo (5.17).
,
,
,
,
Dn theo cac
s
Nhu vy
. ny ra cu hoi la` biu din
,
,
`
`
x1 , x2 , . . . , xn va` y1 , y2 , . . . , yn nhu th nao?
Ta phai lam
thm
,`,
,
,
,
a vo i n = 3. Ta co
truong ho. p ring nu
D3 = 3C3 A3 B3

= 3( x1 y1 + x2 y2 + x3 y3 ) ( x1 + x2 + x3 )(y1 + y2 + y3 )
= (2x1 y1 x1 y2 x1 y3 ) + (2x2 y2 x2 y1 x2 y3 )
+ (2x3 y3 x3 y1 x3 y2 )
= x1 (y1 y2 ) + x1 (y1 y3 ) + x2 (y2 y1 ) + x2 (y2 y3 )+
+ x3 ( y3 y1 ) + x3 ( y3 y2 ).
,,
`, dy
Tu
ta nhn
o. c
. du
D3 = ( x2 x1 )(y2 y1 ) + ( x3 x1 )(y3 y1 ) + ( x3 x2 )(y3 y2 ).
, ,
,,
,
`

So sanh
hai truo` ng ho. p n = 2, 3 ta co th gia thit rang
Dn = Dn1 + ( xn xn1 )(yn yn1 ) + + ( xn x1 )(yn y1 ).
(5.21)
,
,
,
` quy nap
` bang
hoc.
Ta chu ng minh cng thu c nay
. toan
. Vo i n = 2, 3
, ,,
,
,

` d
cng thu c (5.21) dung.
Gia su no dung
vo i mt
o. Ta
. s n nao

,
,
,
,

5.3. Nhu ng bt dang


thu c ni ting

127

tnh Dn+1
Dn+1 = (n + 1)Cn+1 An+1 Bn+1

= (n + 1)(Cn + xn+1 yn+1 ) ( An + xn+1) )( Bn + yn+1 )


= (nCn An Bn ) + (Cn Bn xn+1 ) + (nxn+1 yn+1 An yn+1 ).
,

. khac
Nhung nCn An Bn = Dn mat
Cn Bn xn+1 =

= x 1 y 1 + + x n y n y 1 x n +1 y 2 x n +1 y n x n +1
= y 1 ( x n +1 x 1 ) y 2 ( x n +1 x 2 ) y n ( x n +1 x n )
va`
nxn+1 yn+1 An yn+1 = nxn+1 yn+1 x1 yn+1 xn yn+1

= ( x n +1 x 1 ) y n +1 + ( x n +1 x 2 ) y n +1 + + ( x n +1 x n ) y n +1 .
,
,,
Va` nhu vy
o. c
. ta nhn
. du
Dn+1 = Dn + [y1 ( xn+1 x1 ) y2 ( xn+1 x2 ) yn ( xn+1 xn )]

+ [yn+1 ( xn+1 x1 ) yn+1 ( xn+1 x2 ) yn+1 ( xn+1 xn )]


= Dn + ( xn+1 xn )(yn+1 yn ) + + ( xn+1 x1 )(yn+1 y1 ).
,
,
,
`
`

Nhu vy,
quy nap
dang
thu c (5.21) dung
. bang
. ta kt lun
. rang
,
voi moi
. n = 2, 3, . . .
,
,
`

Chu y rang
di
thu c
`u kin
. (5.17) suy ra bt dang
( xn+1 xn )(yn+1 yn ) + + ( xn+1 x1 )(yn+1 y1 ) 0
,
,,
,
`, dy
vo i moi
va` (5.21) ta nhn
o. c bt
. s tu. nhin n > 1. Tu
. du
,
,

dang
thu c
Dn Dn1 , (n = 2, 3, . . .).
,
`, (5.22) va` tru,o`,ng ho.,p n = 1.
By gio` (5.20) suy ra tu

(5.22)

128

,,
s
Chuong 5. Day

,
y do
5.4. Da
n di
u
.
s
Cho day
a1 , a2 , . . .,, an , . . .
(5.23)
`
`

Ta goi
d
ay
(5.23)
b
i
ch
an

bn
ph
ai,
khi
t
n
t
ai
h
ang
s
a
sao
cho
.
.
.
.
,,
,

. trai
an a, (n = 1, 2, . . .). Tuong tu. ta cung
dinh ngha bi. chan
, .
,
phai goi
bi. chan
ln
. ca trai
(khi d

o an a). Nu mt
. day
. la` day
,
`
bi. chan
. khi va` ch khi t`n tai
bi. chan.
. Suy ra mt
s
. day
. hang
(5.23) goi
K > 0 sao cho | an | K, (n = 1, 2, . . .). Day
tang,

. la` day
khi
a1 a2 . . .
(5.24)
,
giam khi
va` la` day
a1 a2 . . .
(5.25)
,
ma` no co tnh cht (5.24) hoac
. (5.25) goi
Mt
do
n di
u.
. day
. la` day
.
,
`

goi
`

Ta bit rang
mt
th` day
. day
. la` hi
. tu. khi n, tin to i ,v cung
,
,

u han.
biu mt
d
o tin toi mt
.
. gia tri. hu
. Ta co th phat
. mt
. dinh
,
,
ly co ban:
,
inh
da cho la` tang

(giam) va` bi. chan


. bn
. day
. , l 5.1: Nu mt
phai (bi. chan
. bn trai),
th` no hi
t
u.
. .
,
,
di

Theo dinh
ly trn nu cac
ly thoa man
`u kin
.
.
. cua dinh
,
,
ng s hang

th` t`n tai


an . Theo dinh
ly trn th` nhu
cua day
.
. nlim
.
,
tang
tnh cht bi. chan,
. th` lim an = vo,i day

khng thoa man


n
,
,
giam.
va` lim an = vo i day
n
,
,
,ng bai
` toan
chu,ng minh t`n tai

Nhu
gio i han
cua mt
.
.
. , day,
,
,
,
,
da bit d
ng quy tac
co th dung
`
tnh gioi han
nhu
tnh.
. cua, day
,
,
,
,
,
dung
Nhi`u khi ta phai ap
dinh
nhung phai chu ng
. ly co ban trn
.
,
,
,
,ng bai
ta xet la` do
` toan

. giai nhu
minh day
n di
u
. va` bi. chan.
,
,
,
,
,
,
do
dung
quy nap
v` day
tru. c tip phuong phap
n di
u
. thuo` ng ap
.
.
hoc.
toan
.

,
do
5.4. Day
n di
u
.

129

, , dinh theo cng thu,c


V du. 5.16. Day
s a1 , a2 , . . . du
.
o. c xac
an+1 = an (2 an ),
(5.26)
,,
,,
1
o dy

la` mt
. s duong, c`on a1 la` mt
. s bt k`y trong (0, ). Hay
tnh lim an .
n

,
,
`, (5.26) ta vit lai
da cho la` bi. chan:
. Tu
Lo` i giai. 1) Day
.
1
[1 (an 1)2 ].
(5.27)

,,
`, (5.27) suy
Boi v` > 0 va` 0 < a1 < 1 , th` (a1 1)2 < 1 va` tu
,,
,
`

nhu, vy
bang
phuong phap
ra (vo i n = 1) 0 < a2 < 1 . Cung
.
,
1
`
`

chu ng minh rang


nu an trong do
(
0,
)
,
th`

a
c
ung
n
am
an
n
+
1
.

,
,,
`

hoc
trong do
n
ay.
Theo
nguyn
l
y
quy
n
ap
to
an
tt ca ph`n tu
an
.
.
.
,
1
` trong do
s d
da cho la` bi.
cua day
`u nam
an
. (0, ). Suy ra day
.
chan.
,
2) Tnh do
n di
u:
. Ta lai
. co
a n +1 =

an+1 an = an (2 an ) an = an (1 an ) > 0,
,
,
,
do
dung

v` 0 < an < 1 . Nhu vy


day
tang
va` ta co th ap
n di
u
.
.
.
,
,
,
da cho hi
dinh
ly co ban, day
.
. tu. ngha la` co gio i han.
.
,
,
ng ly lun
3) T`m gioi han:
. Do nhu
. ph`n trn ta goi
. lim an = l
n

dung
` (5.26) lim an+1 = ( lim an ).(2 lim an ).
ap
vao
.
n
n
n
,
,,
,,
` la`
Ta nhn
du
o
c
l
=
l
(
2

l
)
.
Nghi
m
c
ua
phu
ong tr`nh nay
. .
.
,
,
1
s la` tang
. la` l = . Nhu,ng day

l = 0 hoac
nn ta ch co th ly

1
l= .

, , dinh theo cng thu,c


V du. 5.17. Day
s a1 , a2 , . . . du
o. c xac
.

a n +1 =

( a n + ),
2
an

(5.28)

,,
s
Chuong 5. Day

130

,
,,
,,
o, dy
tnh
la` mt
. s duong, c`on a1 la` mt
. s duong bt k`y. Hay
lim an .
n

,
,
,
,
`, bt dang
da cho bi. chan
. du,o,i: Tu

Lo` i giai. 1) Day


thu c >
,
,
,
0, a1 > 0 va` (5.28) vo i n = 1 suy ra a2 > 0. Gia thit vo i mt
. n
,
,
`
` d
nao
o an > 0; khi d
o tu > 0 va` (5.28) suy ra an+1 > 0. Nhu
,
,
,
hoc

vy
thu c an > 0 vo i
. theo nguyn ly quy nap
. toan
. bt dang
n = 1, 2, . . .
,
,
,
,
`, (5.28) va` bt dang
da cho la` do

2) Day
thu c
n di
u
. giam: Tu
trung b`nh cng
va` trung b`nh nhn ta co
.
an +
a n +1 =

an

an .

= .
an

,
,
, ,
`
` chu,ng to rang
s hang
i`u nay
tt ca cac
khng nho hon .
.
ta co
. khac
Mat
a n +1 a n =

a2n
( an + ) an =
,
2
an
2an

,
an > 0(n = 1, 2, . . .) va` a2n < 0 ( t nht vo i n = 2, 3, . . .) suy ra
,
giam.
an+1 < an . Ngha la` day
, `
,,
,
,

3) Tnh gio i han:


nhu bai
truo c cho gio i han
. Cung
. trong (5.28)
,,
,,
ta nhn
o. c phuong tr`nh
. du
l=

( l + ),
2
l

,,
,,
,
,
`, d
tu
o t`m du
o. c gio i han
. l = ( c`on truo` ng ho. p l = khng
,,
du
o. c).

5.5. S e

131

5.5. S e
`
hoc
Mt
s toan
trong
sau s la` s e. S e
. hang
. rt quan
.
,
,,
, ` ,

du
ngha nhu la gioi han
o. c dinh
.
. cua day
1
an = (1 + )n , n = 1, 2, . . .
(5.29)
n
. la`
hoac
1
e = lim (1 + )n .
(5.30)
n
n
V du. 5.18. Day
(5.29) la` day
tang.

,
,
,
,
nhin: Khi n tang

Lo` i giai. i`u khang


dinh
khng phai ngu
.
,
,
,
,

s mu trong (5.29) cung


tang,
nhung phn co s giam (no tin to i
,
, `
,
,
ta du
1 va` co gia tri. lo n hon 1). thun
s
a vao
. tin
. tnh toan
1
2
k1
)(1 ) . . . (1
),
(5.31)
n
n
n
,
voi n = 2, 3, . . . ; k = 2, 3, . . . , n.
,

,
thu,c 0 < 1 < 1
<1
Vo i (0 < < n) ta co bt dang
n
n
+
1
,
` d
tu
o suy ra
pn,k = (1

0 < pn,k < pn+1,k < 1, (n = 2, 3, . . . ; k = 2, 3, . . . , n).


(5.32)
,
,
Khai trin theo nhi. thu c Newton ta co
1
1
1
1
an = (1 + )n = 1 + Cn1 + + Cnk k + + n
n
n
n
n
n
n ( n 1) . . . ( n k + 1) 1
= 2+
. , (n = 2, 3, ...).
k!
nk
k =2
`
. khi ta dung
Hoac
(5.31), (1 +
,,
vit (5.29) duo i dang
.
n

an = 2 +

k =2

pn,k
k!

n
,
p
1 n
) = 2 + n,k . Khi d
o ta co th
n
k!
k =2

(n = 1, 2, . . . ; k = 2, 3, . . . , n).

(5.33)

,,
s
Chuong 5. Day

132
Ta co hiu
. an+1 an theo (5.33)
n +1

a n +1 a n = ( 2 +

k =2
n

k =2

n
pn+1,k
p
) (2 + n,k )
k!
k!
k =2

pn+1,k pn,k
p
+ n+1,n+1
k!
( n + 1) !

va` theo (5.32) ta co


a n +1 a n >

pn+1,n+1
> 0.
( n + 1) !

,
e1 , e2 , . . . theo phu,o,ng phap
sau:
By gio` , ta dinh
ngha day
.
n

e1 = 2, en = 2 +

k! ,

(n = 2, 3, . . .).

(5.34)

k =2

,
` day
`, (5.32) va` (5.33) suy ra bt dang
nay
` la` tang.

D thy rang
Tu
,
thu c
an < en .
(5.35)
,

` tp

`
`

Bai
ti
p
sau
ch

ra
d
ay
e
,
e
,
.
.
.
l
a
b
i
ch
an
v
a
suy
ra
n
o hi
1 2
.
.
.
.
,
`
`
`

tu.
T
u
dy
v
a
(5.35)
suy
ra
a
,
a
,
.
.
.
c
ung
b
i
ch
an.
B
ang
c
ach
n
ay

1 2
.
, . .
,
,,
,
,

(5.30).
ta chu ng minh du
gi
o
i
h
an
c
ua
day
o. c su. t`n tai
.
.
,
V du. 5.19. Chu,ng minh rang
`
nhu ,ng s hang
cua day
(5.34)
.
,
thoa man

1
en < 3 n1 , (n = 3, 4, . . .).
(5.36)
2
,
, ,,
1
,
,
Lo` i giai. Vo i n = 3 ta co e3 < 2, 67 < 3 31 = 2, 75. Gia su
2
,
,
,
,,
,
vo,i s n nao
` d

(5.36) thoa man


bt dang
thu c hin
y. Ta su dung
.
,
,,
,
nhin (n + 1)! > 2n vo i n > 1 va` gia thit quy nap
o. c
. ta nhn
. du
1
1
1
e n +1 = e n +
< (3 n 1 ) +
( n + 1) !
( n + 1) !
2
1
1
1
< 3 n 1 + n = 3 n .
2
2
2

5.5. S e

133

,
(5.34) la` s e.
V du. 5.20. Gio,i han
. cua day
,
,
,
, , `
a1 , a2 , . . . va` e1 , e2 , . . .
Lo` i giai. Ta da chu ng minh du
day
o. c rang
,

` (5.30) va` (5.35) suy ra


. e = lim en . Tu
la` hi
. tu.
. at
n

e e .

(5.37)

,
,
`

Ta se chu ng minh cung


vo i (5.37) cung
co
e e,
,
,
,
Tht
. vy,
. ta chon
. s tu. nhin s trong khoang (1, n). Nu bn phai
,
,
,
pn,s+1
pn,n
,,
cua (5.33) ta bo di
,...,
, ta se nhn
du
s nho

o. c mt
.
.
( s + 1) !
n!
,
hon an , ngha la`
s

pn,k
< an , (n = 3, 4, . . . ; s = 2, 3, . . . , n 1).
(5.38)
k!
k =2
,
,
,

Ta xet dang
thu c (5.31). Khi n tin to i v cung,
c`on k c dinh,
mi
.
`,a s tin to,i 1. V` th lim pn,k = 1. Nu ta cho n tin to,i v
thu
n , ,
`
cung
trong (5.38) ta se nhn
o. c
. du
s
1
e (s = 2, 3, . . .).
es = 2 +
k!
k =2
,
,
`, pha phai s e, tu
`, d
tang
. tu

Nhu vy
e1 , e2 , . . . . bi. chan
o suy
. , day
,
,
,
` (5.36) va` (5.37).

dinh
suy ra tu
ra bt dang
thu c (5.38). Khang
.
2+

,
,
d
nao
` d
Nhu vy
n d. chnh xac
y theo
. s e co th tnh toan
,
cng thu c trn
1
1
1
en = 2 + + + + , (n = 2, 3, . . .).
2! 3!
n!
,
,
,
`
n c dinh
`, bt dang

Tu
thu c an < en < e ta thy rang
vo i mi
s
.
,
,
,

en g`n dung
e. S en goi
. la` xp x thu n cua e, c`on hiu
.
n = e en

(5.39)

,,
s
Chuong 5. Day

134
,
goi
. la` sai s cua e.

,
V du. 5.21. Chu,ng minh nhu ,ng bt dang

thu,c sau
ei e n

2
1
.(1 i n ),
( n + 1) !
2

n = 2, 3, . . . . ; i = n, n + 1, . . .
(5.40)

,
,
,
,
,
`
Lo` i giai. Ta se chu ng minh (5.40) bang
quy nap
i voi i. Vo i
. d
, ,,
,

` d
i = n th` 0 0 (5.40) dung.
Gia su (5.40) dung
vo i s i nao
o, ta
,
,
,

c`n phai chu ng minh no cung


dung
vo i n = i + 1. Tht
v
y,
. .


1
1
e n = ( ei e n ) +
e i +1 e n = e i +
( i + 1) !
( i + 1) !


2
1
1

1 i n +
( n + 1) !
( i + 1) !
2


1
2
1
1
=
1 i n + .
.
( n + 1) !
2 (n + 2)(n + 3) . . . (i + 1)
2
thu
`,a s, mi
`,a s lo,n ho,n
Tch (n + 2)(n + 3) . . . (i + 1) co i n thu
,
,
2. Khi d
o 2(n + 2)(n + 3) . . . (i + 1) > 2in+1 , o dy
e i +1 e n <
2
1
1
2
1
(1 i n + i n +1 ) =
(1 (i+1)n ). Nhu, vy,
.
( n + 1) !
( n + 1) !
2
2
2
,

(5.40) dung
vo i moi
. i = n, n + 1, . . .

y s Fibonacci
5.6. Da
,,
,
`
s u1 , u2 , . . . du
Mt
ngha bang
cng thu c
o. c dinh
.
. day
u n +2 = u n +1 + u n ,

(n = 1, 2, . . .)

(5.41)

va`
u1 = 1, u2 = 1.
(5.42)

,ng s u1 , u2 , u3 , . . . goi
`

`
Nhu
l
a
s
Fibonacci.
D
ay
s
n
ay
c
o rt
.
,
,
,
,
` toan
thu. c t. Theo dang

nhi`u u ng dung
trong bai
thu c (5.41)
.

s Fibonacci
5.6. Day

135

, ,
,
,
,,
` s thu, ba d
moi
`u la` tng cua hai s truo c d
o.
. s k tu
,

`
ng s d
Nhu
u tin la` 1, 1, 2, 3, 5, 8, 13, 21, 34, 55, 89,
,ng bai
` tp
tnh
144, 233, 377. Nhu
n cac
. sau lin quan d
,
,
`

s nay
` co chung minh bang
hoc,

cht cua day


quy nap
. toan
.
,
,

tnh cht khac


cua day
s nay
` th` rt nhiu va` chung
c`on cac
`
cach
khac
nhau.

minh bang
cac
,
,
V du. 5.22. Tng cua n s d
`
s thu, n + 2 tru`, di
1,
`u tin bang
hoac
. la`
u 1 + u 2 + + u n = u n +2 1 ( n > 1 ).
(5.43)
,
,
,
,
,

Lo` i giai. Vo i n = 2 dang


thu c dung
v` theo (5.41) va` (5.42) ta co
u1 + u2 = 1 + 1 = 3 1 = u4 1.
,
, ,,
,
,
`, gia thit quy nap

` d
Gia su (5.43) dung
vo i s tu. nhin n nao
o. Tu
.
,
,
, ,
va` cng thu c (5.41) vo i tng cua n + 1 s Fibonacci ta co :
u 1 + u 2 + + u n +1 = ( u 1 + u 2 + + u n ) + u n +1

= ( u n +2 1 ) + u n +1
= un+1 + un+2 1 = un+3 1.
,
,
,
hoc

Theo nguyn ly quy nap


thu c dung
vo i moi
. toan
. dang
. n 2.
,
V du. 5.23. Chu,ng minh rang
` dang

thu,c

u n + m = u n 1 u m + u n u m +1
(5.44)
dung

vo,i s tu. , nhin bt k`y n > 1 va` vo,i moi


. m = 1, 2, . . .
,
,
,
,,
`
quy nap
theo
Lo` i giai. Ta chu ng minh bang
phuong phap
. toan
,
m. Voi m = 1 ta co
u n 1 u 1 + u n u 2 = u n 1 + u n = u n +1 ,

,,
s
Chuong 5. Day

136
,
va` vo i m = 2:
un1 u2 + un u3 = un1 + 2un

= ( u n 1 + u n ) + u n
= u n +1 + u n = u n +2 ,
,
,
, ,, ,
,
,

` d
dang

dang
thu c (5.44) d
Gia su vo i s m nao
thu c
`u dung.
o cac

sau dung
u n + m = u n 1 u m + u n u m +1

(5.45)

u n + m +1 = u n 1 u m +1 + u n u m +2
,
,
,

Ta se chu ng minh dang


thu c sau dung
u n + m +2 = u n 1 u m +2 + u n u m +3 .
(5.46)
,
,
,
`,ng v cua hai dang

Tht
tu
thu c trong (5.45) ta nhn
. vy,
. cng
.
.
,,
du
o
c
.
u n + m +1 + u n + m = u n 1 ( u m +1 + u m ) + u n ( u m +2 + u m +1 )
,
,
,
dung

dy
thu c (5.46) khi ta bin d
(5.41).
chnh la` dang
i va` ap
.

, ,

V du. 5.24. Chu,ng minh rang


` moi
. s Fibonacci co th biu din
,
,
duo i dang
.
!n
!n
1+ 5
1 5

2
2

, (n = 1, 2, . . .)
(5.47)
un =
5
,
,
,
,,
,
Lo` i giai. Vo i n = 1 va` n = 2 ta nhn
o. c u1 = 1 va` u2 = 1. Gia
. du
,, ,
,
` d
su vo i mt
o co un va` un+1 theo cng thu c (5.47) la` s
. s n nao
,
,
`
Fibonacci. Ta se chu,ng minh rang
hang
thu n va` n + 1 cua day
.

s Fibonacci
5.6. Day

137

,
,
,
theo cng thu,c (5.47).
cung

s hang
thu n + 2 cua day
biu din
.
,
cho gon
. ta ky hiu
.

1+ 5
1 5
=
,
=
.
2
2
n n
n +1 n +1

dat
. ta co
Khi d
, u n +1 =
. Do cach
o u n =
5
5
,
,,
+ = 1 va` = 1. Suy ra va` la` nghim
. cua phuong tr`nh
,
x2 x 1 = 0. Khi d
o 2 = + 1, 2 = + 1. Nhu vy
. ta co
n n
n +1 n +1

+
5
5
n 2 n 2
n +2 n +2
n ( + 1) n ( + 1)

=
=
.
=
5
5
5
,
,
un bang
`
hoc

Theo quy nap


(5.47) dung
vo i moi
. toan
. biu din
. n.
u n +2 = u n +1 + u n =

V du. 5.25. Chu,ng minh rang


`
n = u n + u n 1
,
, ,, ` tp tru,o,c.
,
vo i moi
. s tu. nhin n 2. nhu o bai
.
,
,
, ,, ,
,
,
,
,

Lo` i giai. Vo i n = 2, 3 dang


thu c dung.
Gia su dang
thu c dung
,
vo i n = k va` n = k + 1, ngha la`
k = u k + u k 1 ,
k+1 = uk+, 1 + uk .
,
,
,,

thu c lai
Ta cng
hai dang
o. c
. kt qua nhn
. du
.
k + k +1 = ( u k + u k +1 ) + ( u k 1 + u k ),
. la`
hoac
k +2 = u k +2 + u k +1 .

,,
s
Chuong 5. Day

138

,
,
hoc
` cho la` da

Theo nguyn ly quy nap


thu c trong bai
. toan
. dang

dung.
` toan
` tu,o,ng tu., ta cung

Hoan
co

n = u n + u n 1 .
V du. 5.26. Chu,ng minh rang
`
u2n un1 un+1 = (1)n+1 vo,i n >
1.
,
,
,
,
,

Lo` i giai. Vo i n = 2, ta co u22 = u1 u3 1 dang


thu c dung.
,
, ,,
,
`

Gia su rang
u2k uk1 uk+1 = (1)k , ta c`n chu ng minh dang
,

thu c trn cung


dung
cho n = k + 1, ngha la` u2k+1 uk uk+2 =
(1)k+2 . Tht
. vy,
.
u2k+1 uk uk+2 = u2k+1 uk (uk+1 + uk )

= uk+1 (uk+1 uk ) u2k = uk+1 uk1 u2k

= (u2k uk+1 uk1 ) = (1)k+1 = (1)k+2 .


V du. 5.27. Chu,ng minh rang
`
nu n chia ht cho m, th` un chia
ht cho um .
,
,
,
Lo` i giai. V` n chia ht cho m, nn ta co th vit n = mk. Ta se
,
chu ng minh quy nap
. theo k.
,
,
Voi k = 1, khi d
o n = m nhu vy
. un chia ht cho um la`
,
, ,,
,
hin nhin. Gia su umk chia ht cho um , ta xet um(k+1) . Nhung
,
u
=u
va` theo cng thu c (5.44) ta co
m ( k +1)

mk +m

um(k+1) = umk1 um + umk um+1 .


,
,
S hang
thu nht co chu a um nn no chia ht cho um , c`on s hang
.
.,
,
,
,

thu hai theo gia thit quy nap


u
chia
h
t
cho
u
.
Nhu
v
y
t
ng
m
mk
.
.
,
cua hai s hang
chia ht cho um , suy ra um(k+1) chia ht cho um .
.

` tp
5.7. Bai
.

139

` tp
5.7. Bai
.

,
t`m u,o,c s chung lo,n nht cua cac
s
. 5.28. Hay
2

22 + 22 + 1; 22 + 22 + 1; . . . ; 22

n +1

+ 22 + 1; . . .

,
`
. 5.29. Chu,ng minh rang
nu pn la` s nguyn t thu n, th` pn <
n
22 .
`
. 5.30. Chu,ng minh rang
nu 0 1 1 2 . . . n

n , th`
2
n

i =1

i =1

(sin i sin i ) sin( ( i i )).

,
,
. 5.31. Chu,ng minh bt dang

thu c
n

13

n + i > 24 ,

n = 2, 3, . . .

i =1

. 5.32. T`m gio,i han


. cua day
r
q
q

a1 = c, a2 = c + c, a3 = c + c + c, . . .
,,
,,
`
o dy
s duong.
c la` hang

, ,
CHUONG

H`INH HOC
.
hoc
6.1. V du. quy nap
. toan
. cho h`nh hoc
. ..............
` tp
6.2. Bai
.
.
.
.
.
.
.
.
.
.
.
.
.
.
.
.
.
.
.
.
.
.
.
.
.
.
.
.
.
.
.
................
.

140
154

,,
quy nap
hoc
dung

Phuong phap
ap
rt nhi`u trong cac
. toan
.
,.
,

ng kha
` tp
bai
. h`nh hoc.
. Ban
. doc
. co th t`m thy trong [7] nhu
,
,,
`

nay
` trong h`nh hoc.
canh
cn thit cho phuong phap
Ta ch lit
.
.
.
,
, ,

` rt di
k duoi dy
mt
n h`nh.
. s bai

hoc
6.1. V du. quy nap
. toan
. cho h`nh hoc
.

,
cap
V du. 6.1. Trong mat
. phang

cho n h`nh l`i ( n > 3 ), mi


. ba
,
,
,
trong chung

co di
`
t`n tai
m chung. Chu ng minh rang
m, ma`
. di
,
no nam
` trn tt ca cac
h`nh.
,
,
,
`
,ng h`nh bang
Lo` i giai. 1. Vo i n = 4, ta ky hiu
C1 , C2 , C3 , C4 .
. nhu
Cho C1 C2 C3 = A4 , C1 C2 C4 = A3 , C1 C3 C4 = A2 ,
C2 C3 C4 = A1 .
,
` trong tam giac
A1 A2 A3 , th` bo,i v` A1 A2 A3
a. Nu A4 nam
,
,
C4 , ta co A4 C4 va` suy ra A4 C, o, dy
C ky hiu
. la` giao cua
h`nh C1 , C2 , C3 , C4 .
cac
,
,
, ` `
,,
cua cac
du
b. Nu A1 A2 A3 A4 la` tu giac
li va A la` di
m cat
o` ng
,
`

cheo cua chung,


th` d thy rang
A C.

hoc
6.1. V du. quy nap
. toan
. cho h`nh hoc
.

141

,
, ,,
,
,

2. Gia su da chu ng minh cho khang


dinh
dung
vo i n 1.
.

Ta xet n h`nh C1 , C2 , . . . , Cn . Ly C = Cn1 Cn . Ta xet day


,
h`nh nay
`
. ba cac
C1 , C2 , . . . , Cn1 , C. Ta se chu ng minh moi
. cap
,

nhau. Tht
nhau khng la` C,
d
u giua ba h`nh cat
u cat
. vy,
. n
,
` trong s ba
`

th` di
Nu C nam
u kt lun
. trn hin nhin dung.
,
,
ng h`nh C1 , C2 , Cn1 , Cn
h`nh va` v du. nhu C1 , C2 , C, th` v` th nhu
,
,
,
co di
di
m chung X (theo chu ng minh tai
m 1.). Suy ra X
.
,
,
`
`, dy
` toan
suy ra bang

Cn1 Cn = C. Tu
dinh
cua bai
quy
khang
.
nap.
.
,
V du. 6.2. Cho n h`nh vung bt k`y. Chu,ng minh rang
` ta co th
, ,
,
cat
chung

ra thanh
`
mt
ph`n d
tu` cac
o ta co th ghep
. s ph`n d
,
lai
`
mt
. thanh
. h`nh vung mo i.
,
,
,
,

Lo` i giai. Khi n = 1, di


dinh
la` hin nhin.
`u khang
.

A1

B1
B

A
Q
A2

O
Q

B2
D2

N
D1

C1

P
A2

D2

C2

B2
D

C2

H`nh 6.1:
,
,
`

Ta chu ng minh rang


khi n = 2, di
dinh
d
`u khang
o cung
.
,

` cac
canh
dung.
Goi
cua hai h`nh vung A1 B1 C1 D1 va`
. d. dai
.
, ,,
,,
,
canh
A2 B2 C2 D2 tuong u ng la` x1 va` x2 . Gia su x1 x2 . Trn cac
.
,
,

cua h`nh A1 B1 C1 D1 voi canh


x
ta
d
at
c
ac
do
an
A
M
=
B
N
=

1
1
1
.
.
.
x1 + x2
,
,
h`nh vung d
du
C1 P = D1 Q =
va` cat
o theo cac
o` ng
2

142

,,
Chuong 6. H`nh hoc
.

,
nhau tai
`
MP va` NQ, ro rang
MP va` NQ cat
. O cua h`nh vung
,,
,
du
va` tao
o` ng d
o chia h`nh vung
. vo i nhau mt
. goc vung. Cac
,
`
`

`
` h`nh vung

thanh
4 phn bang
nhau nhung h`nh d
o ghep vao
,
,,
A2 B2 C2 D2 nhu h`nh bn. H`nh nhn
o. c se la` h`nh vung v`
. du
gia tri. goc M, N, P, Q bu` nhau, cac
goc A, B, C, D la` vung va`
cac
AB = BC = CD = DA.
, ,,
,,
,
,
Gia su mnh
d
` da du
o. c chu ng minh d
i vo i n h`nh vung va`
.
, ,,
gia su ta co n + 1 h`nh vung V1 , V2 , . . . , Vn , Vn+1 . Ta ly ra bt
,
,,
,
,

ky` hai h`nh vung, chang


han
. Vn va` Vn+1 nhu da chu ng minh o
mt
` h`nh vung thu,
trn sau khi da cat
. h`nh vung va` ghep vao
,,
,
hai ta du
mo i V 0 . Do vy
o n h`nh vung
o. c mt
. h`nh vung
. ta c
,
,
,,
0
ra du
V1 , V2 , . . . , Vn1 , V theo gia thit quy nap
o. c cac
. co th cat
,
`, cac
ph`n d
`
ph`n va` tu
mt
o co th ghep lai
. thanh
. h`nh vung
,
moi.

,
,
,
V du. 6.3. Trong mat
. phang

cho n 3 di
`
m, tt ca khng nam
,
,
,
,
,
,
,
,

ni

Chu ng minh rang


`
tt ca cac
du
trn du
o` ng thang
o` ng thang.
,
,
,
,
,
hai di
di

khac
nhau
o` ng thang
m trong cac
m da cho tao
. ra s du
, ,
khng nho hon n.
,
,
,
,
,
,

Lo` i giai. Vo i n = 3 di
Ba di
m, mnh
` hin nhin dung:
m
. , d
,,
,
` trn mt
`,ng di

khng nam
du
ni tu
o` ng thang
vo i nhau tao
.
. ra
,
,`,
nhau.

khac
ba du
ong thang
, ,,
,
,
,

Gia su mnh
d
vo i n 3 di
` dung
m. Ta chu ng minh no
.
,
,
,
,
`

cung
dung
vo i n + 1 di
t`n
m. Ta co th chu ng minh rang
,
,
,
,
,
,

tai
ch chu a hai di
o` ng thang
m. Ta ky hiu
. t nh,t mt
. du
.
,
,`,
,ng

du
di
ong thang
qua hai di
m An va` An+1 la` An An+1 . Nu nhu
,
,
,,
,,
` trn mt

di
du
th` s luo. ng cac
m A1 , A2 , . . . , An nam
o` ng thang,
.
,
,
,`,
,
,
,

du
se dung
la` n + 1: G`m n du
ni An+1 vo i
ong thang
o` ng thang

hoc
6.1. V du. quy nap
. toan
. cho h`nh hoc
.

143

,
,
,,
di

cac
chung
ni chung. Nu
o` ng thang
m A1 , A2 , . . . , An va` du
,
,
,
,
` trn mt

A1 , A2 , . . . , An khng nam
du
th` theo gia
o` ng thang,
.
,
,,
nhau. By gio`, ta thm

khac
thit quy nap
o` ng thang
. , co n du
,
,,
,,
,
du

cac
ni An+1 vo i cac
di
o` ng thang
m A1 , A2 , . . . , An . V` du
o` ng
,
,
,
` trong A1 , A2 , . . . , An1 ,

thang
An An+1 khng chu a mt
di
m nao
.
,
,
,`,
,,
` khac
hoan
` toan
` vo,i n du

th` du
nay
tao
ong thang
o` ng thang
. ra
,
,,
,
,`,

boi A1 , A2 , . . . , An . Nhu vy
tao
khng
ong thang
. s du
. ra cung
, ,
nho hon n + 1.
,
,
,
,,
V du. 6.4. Trong mat
. phang

cho n 3 di
m. uo` ng knh cua
,
,
,
mt

ni hai di
m la` do
an
m trong h. thng va`
. h. thng di
. thang
,
,,
d. dai
` d cua do
` la` lo,n nht. Chu,ng minh rang
`
s du
an
o` ng
. nay
knh khng vu,o.,t qua n.

,
,
,
,
`, mt
tu
Lo` i giai. Nu xut phat
m A cua h. da cho, ta co ba
. di
,
,,
du
o` ng knh AB, AC va` AD. Khi d
o d thy ba di
m B, C va` D se
,
,
,
,,
` trn du
,ng di
nam
o` ng tr`on k1 ( A, d). Tt ca nhu
m c`on lai
. cua h.
,
` hoac
do
. trn k1 hoac
. bn trong no. bo,i v` mi
thng se nam
an
.
,
,
,
,
,
ng di

thang
BC, BD va` CD khng lo n hon d, th` nhu
m B, C va` D
,
` trn cung cua k1 , tu,o,ng u,ng goc khng lo,n ho,n 600 . Goi
se nam
.
_
_
,
,`,
,
0
di
m C bn trong cung BD, voi no BD 60 .Ta ve du
ong tr`on
,
,
, ,,
,
,

ng di
k2 (C, d); Nhu
m cui cua tt ca du
o` ng knh cua h. da cho
_
,
,
,
` trn cung MN cua k2 (M, N la` giao
` C, phai nam
tu
xut phat
,
,
,
` trong h`nh tr`on k1 . Nhu,ng mi
di
di
m cua k1 va` k2 ) va` nam
m
_
,
,
,ng di
` A, du,ng cach
xa nhu
. D mt
cua cung MN, ngoai
m B hoac
.
,
,
,
,
,
,

`
lon hon d, tu d
khoang cach
o suy ra CA la` du
ong knh duy nht,
,

`
tu C.
xut phat
,
,
,
`
Nhu vy
ta kt lun
rang
vo i mt
h. da cho n di
m, t`n tai
.
.
.
.
,
,
,

. la` trong h. co mt
hai kha nang:
hoac
di
m,
t
u
n
o
xu
t
ph
at

144

,,
Chuong 6. H`nh hoc
.

,
,,
di
`, mi
phat

. la` tu
khng qua mt
o` ng knh, hoac
m d
`u xuat
. du
,
,

dung
hai du
o` ng knh.
,
,
`
` toan
chu,ng minh bang

Khang
dinh
quy nap
. , cua bai
. theo n.
,
, ,,
,
,

Voi n = 3 khang
dinh
hin nhin dung.
Gia su no dung
vo i n =
.
,
,
,
,

k 3. Ta se chu ng minh no c`on dung


ca vo i h. co n = k + 1 di
m.
,
,
Tht
m: A1 , A2 , . . . , Ak+1 co di
m,
. vy,
. nu trong h. co k + 1 di
,,
`, no khng xut phat
`, no xut
du
. tu
v du. A1 , tu
o` ng knh hoac
,
,
,,
,,
,,
,ng du
ch mt
phat
o` ng knh, th` s luo. ng nhu
o` ng knh cua
. du
,
,,
` nhi`u ho,n s lu,o.,ng du
h. nay
o` ng knh cua h. A2 , A3 , . . . , Ak+1
,
,,
,,
xet
nhi`u nht la` 1, ngha la` s luo. ng du

o` ng knh cua h. ta dang


,
,
,
,
`,
khng lon hon k + 1. Nu mt
m nhu vy
. di
. khng t`n tai,
. th` tu
,
,,
di
`, d
dung

mi
hai du
m Ai xut phat
o` ng knh va` tu
o suy ra s
,
2
(
k
+
1
)
,,
,`,
,
`
luo. ng du
= k + 1, Nhu vy
kt lun
cua
ong knh bang
.
.
2
,
` toan
dung

bai
vo i moi
. n.
,
,,

trong mt
.
V du. 6.5. Chu,ng minh rang
`
n du
o` ng thang
. mat
,
,
,
,
nhau, co th t mau
`
phang

chia mat
. phang

ra nhu ng mi`n khac


,
mi`m sao cho nhu ng mi`n canh
trang

hoac
. den
nhau
cho mi
.
khac
mau
` nhau.
,
,
,
,,
,
,,
.

. phang

AB chia mat
P ra hai nua mat
Lo` i giai. 1) uo` ng thang
,
,
,

` trang,
` den

phang
P1 va` P2 . T P1 mau
P2 mau
thoa
va` nhu vy
.
,
,,
,
d`
` toan.
Vo,i n = 1 mnh
man
d
oi hoi bai
` du
o. c chu ng minh.
.
,
, ,,
,,
,

. phang

2) Gia su mnh
d
vo i n = k va` mat
P du
` dung
o. c t
.
,
,
` nhu, yu c`u bai
` toan.
u,o`,ng thang

.
mau
thu k + 1, CD chia mat
,
,
,,

`
,
phn trong Q1 ta giu

phang
P ra hai nua Q1 va` Q2 . Tt ca cac
,
ph`n ta thay trang

` da t, c`on trong nu,a Q2 mi


nguyn mau

`
`
thanh
den
trang.
va` den
thanh
, ,,
,
Gia su O1 va` O2 la` hai ph`n bt ky` canh
nhau, sau khi ke
.

hoc
6.1. V du. quy nap
. toan
. cho h`nh hoc
.

145

,
,
,
,,

du
sau xy ra:
o` ng CD. Ch co mt
. trong hai kha nang
,
` trn ph`n khac
nhau cua CD,
a) O1 va` O2 nam
,
` trn cung
`
b) O1 va` O2 nam
mt
. pha cua CD.
,
,
,,
,,
,

Truo` ng ho. p a) O1 va` O2 sau khi ke k du


d
o` ng thang
`u tin,
,
,
,
,
,

`
nhung CD chua ke th` chung
la` mt
mt
o. c t cung
. mi`n va` du
.
,
,
,
`
nguyn mau
` Nhung sau khi ke CD th` mt
` ,
mau.
. min Q1 giu
,
,,
,
`
` theo cach
du. ng. Ngha la` O1 va` O2
c`on phn kia O2 du
o. c d
i mau
mau
` nhau.
khac
,
,`,
,,
,
,

Truong ho. p b) Sau khi ve k du


ma` CD c`on chua
o` ng thang,
,
,
,,

ke , khi d
nhau do k du
o O1 va` O2 la` hai mi`n canh
o` ng thang
.
,
,

mau
` nhau. Sau khi ke
tao
gia thit quy nap
chung
khac
. ra, theo
.
,
,,
,
` cung
`
`

CD, nu O1 va` O2 nam


pha vo i Q1 , th` mau
du
o` ng thang
,
,
khac
` cung

mau
` nhau. Nu chung

`
cua chung
khng d
nam
i, vn
,
,
,
,
,
,
mi`n d
` cua mi
pha voi Q2 , th` mau
`u d
i. Nhu vy
. moi
. truo` ng
,
` khac
nhau.
ho. p O1 va` O2 d
`u co hai mau

,
,
V du. 6.6. Chu,ng minh rang
` nu n mat
. phang

di
di
qua mt
m
.
,
,
,,
sao cho khng co ba mat
. phang

nao
` co chung mt

o` ng thang,
. du
th` chung

chia khng gian ra An = n(n 1) + 2 ph`n.


,
,
,
` hai ph`n va`

. phang
chia khng gian lam
Lo` i giai. 1) Mt
. mat
,

A1 = 2. Voi n = 1 mnh
d
` dung.
.
,
, ,,
,

. la` k mat
. phang

2) Gia su mnh
d
vo i n = k, hoac
chia
` dung
.
,
`

khng gian ra k (k 1) + 2 phn. Ta se chung minh k + 1 chia


khng gian ra k(k + 1) + 2 ph`n.
,
,
, ,,
,
mat
. phang

. phang

Tht
thu k + 1. Mi
. vy,
. Gi, a su P la` mat
,
,
,,
mat
` d
. phang

. phang

trong k mat
cat
P mt
du
nao
o` ng thang
o sao
.
,
,
,
,
,
,
ng ph`n tu
` k du

. phang

cho mat
P bi. chia ra nhu
khac
o` ng thang

146

,,
Chuong 6. H`nh hoc
.

,
,
,,
` tru,o,c mat
`
. phang

nhau di
mt
P du
qua cung
m. Theo bai
o. c
. di
,
, ,
ph`n la` goc trong mat
. phang

chia ra 2k ph`n, mi
vo i dnh la`
,
di
m da cho.
,
`
. phang

k mat
d
mt
`u tin chia khng gian thanh

. s, goc da
,,
` bi. chia ra lam
` hai ph`n boi mat
. phang

din.
P.
. Mt
. s goc nay
,
,
,
,
,
. chung cua hai ph`n co ph`n cua mat
. phang

Vo i mat
gio i han
.
,
,,
nhu
,ng mat
. cua goc da
boi hai tia ma` theo no P cat
m
t
trong
din
.
.
,
,
,,
. phang,

. phang

2k goc mat
ma` no chia ra boi mat
P.
` co ngha la` s goc da
` hai ph`n
i`u nay
din,
. ma` bi. chia lam
, ,
,,
,
boi P, khng th lo n hon 2k.
ph`n trong 2k ph`n, ma` no bi. chia do P cat
k
mi
. khac,
Mat
,
,
. phang

. chung cua hai goc da


mat
d
`u tin, la` mat
din
. va` ngha
,
,,
` hai ph`n.
. phang

la` chia goc da


ra lam
din
. tao
. boi k mat
,ng goc da
` co ngha s lu,o.,ng nhu
i`u nay
din,
. ma` no bi. chia
, , ,
,,
` hai ph`n boi P, khng th nho hon 2k.
ra lam
,
,
,,

. phang

V` mat
P chia dung
2k ph`n cua khng gian tao
b oi
.
,
,
. phang.

. phang

k mat
V` th nu k mat
chia khng gian ra k (k
,
. phang

1) + 2 ph`n, th` k + 1 mat


se chia no ra

[k(k 1) + 2] + 2k = k(k + 1) + 2
ph`n.

,
,
,
V du. 6.7. 8n 4 di
` o, dang
chu , thp
m nam
.
. (h`nh ve vo i n = 4).
,
,
,
,
Co bao nhiu kha nang

d
chon
m trong chung
tao
. bn di
.
,
thanh
`
dnh h`nh vung ?
,
,
,
,,
Lo` i giai. Ta chu ng minh s luo. ng h`nh vung c`n t`m la` 10n 9.
,
,

1) Vo i n = 1, hin nhin dung


v` ta co 1 = 10.1 9 h`nh vung.

hoc
6.1. V du. quy nap
. toan
. cho h`nh hoc
.

147

,
,,
vo,i n = 2, ta co th chon
lam
2) Khng kho khan
o. c 11 =
. du
10.2 9 h`nh vung.
, ,,
,

3) Gia su mnh
d
vo i
` dung
.
n = k, ma` k 2.
Ta xet h`nh g`m 8(k + 1)
,
,
4 di
m (trong h`nh vo i k = 3)
(h`nh 5)
,
Theo gia thit quy nap
.
,
,,
ta co th chon
o. c 10k
. du
9 h`nh vung, khng co mt
.
` trong chung

h`nh vung nao


co
H`nh 6.2:
,
,
,
`
di
mt
vo i cac
m
. dnh trung
,
,
,ng h`nh vung co di
A, B, C, D, E, F, G, H. By gio` ta xet nhu
m
,
,,
,
`
ta ch t`m du
trung
vo i A. Khng kho khan
o. c 3 h`nh vung
,
,

ABB1 A1 , ACED, AC1 F1 H. Nhu vy,


ch co 10 h`nh vung khac
.
, ,
,
,
nhau khi ta thay d
i dnh A voi cac di
m B, C, . . . , H: 4 h`nh
vung dang
AC1 F1 H, 4 h`nh vung dang
ABB1 A1 va` hai h`nh
.
,
,.

vung ACEF, BDFH. Tng cng


tt ca la` 10k 9 + 10 = 10(k +
.
1) 9 h`nh vung.

V du. 6.8. Chu,ng minh rang


`
vo,i moi
.
. s nguyn n 0 t`n tai
,
,
,
,
,
,
mt
. phang

chu a trong no n di
o` ng tr`on trn mat
m vo i toa
. du
. d.
nguyn.
,
,
,
,
Lo` i giai. Chu ng minh quy nap
0, th` thu. c
. theo n. Nu n =
,
,,
,
` co toa
cht du
o` ng tr`on K0 khng chu a trong no mt
m nao
. di
.
,
,
,
,
d. nguyn, d
co di
`u d
o ta ly du
o` ng tr`on bt ky` vo i tm
,
,,
O( x0 , y0 ), o dy
ta chon
. mt
. trong x0 , y0 khng phai la` s nguyn
knh r thch ho.,p. Nu, v du. nhu, x0 khng nguyn, th`
va` ban

148

,,
Chuong 6. H`nh hoc
.

,
,
co th chon
. r la` , s nho nht trong { x0 } va` 1 { x0 }. Tht
. vy,
.
,
,
,
,

` a du. ng K0 , th`
nu M ( x, y) di
m bt ky` trong du
o` ng tr`on vu
x x0 + |OM| < x0 + r x0 + 1 { x0 } = [ x0 ] + 1 va` x
x0 |OM| > x0 r x0 { x0 } = [ x0 ] hay la` [ x0 ] < x < [ x0 ] + 1
va` v` th x khng la` s nguyn.
, ,,

Gia su chung
ta da xy
,
,,
,`,
,
du. ng du
o. c du
ong tr`on Kn vo i
,
,
tm On , chu a trong no n di
m
,

voi toa
. d. nguyn. Ta se tang
,
,
knh du
ban
o` ng tr`on d
o ln va`
,
,,
nguyn tm, tang

giu
du
o` ng
,
,
tr`on d
n mu c khng chu a
,
di
m co toa
. d. nguyn bn
,,
` Kn . Ta nhn
trong ngoai
o. c
. du
,
H`nh 6.3:
v`ong tr`on L1 d
`ng tm vo i Kn ,
,
,
,
,
chu a n di
m co toa
o` ng tr`on Kn va` t nht
. d. nguyn trong du
,
,`,
mt
m co toa
ong tr`on.
. di
. d. nguyn A trn du
, ,,
,
,,
,
(On A) vo,i du
,a
Gia su P va` Q la` di
m cat
o` ng tr`on Kn , hon nu
,,
, ,`,
` giu
,a P va` A. Ta du.,ng du
On nam
o` ng tr`on L2 vo i du
ong knh [ PA].
,
,
,
,

Khi d
n + 1 di
d. nguyn: n di
o L2 chu a dung
m vo i toa
m bn
.
,
,

trong va` mt
khng nhi`u ban
m trn bin. Ta co th tang
. di
, ,,
,`,
,`,
,
`
knh du
ong tr`on L2 d
du
o. c du
ong tr`on Kn+1 d
ng tm voi L2 ,
,
,
,
ma` no chu a n + 1 di
m co toa
. d. nguyn va` khng chu a toa
. d. co

` khac.

s nguyn nao

,
,
V du. 6.9. Ho.,p cua mt
. s h`nh tr`on co din
. tch 1. Chu ng minh
,
,,
,
rang
`
tu`, trong chung

co th chon
o. c mt

. du
. s h`nh tr`on, tu` ng di
, , 1
mt
nhau, ma` din
. khng cat
. tch chung khng nho hon 9 .

hoc
6.1. V du. quy nap
. toan
. cho h`nh hoc
.

149

,
,
,,
`, mnh
` toan
du
Lo` i giai. Bai
d
o. c suy ra tu
` sau:
.
,
,
,
,,

phu mat
.
B d
. phang

cho n du
o` ng tr`on, chung
`: Trong mat
,
,
,
,
,

. mt
phang

voi din
o co th chon
o. c mt
. hoac
.
. tch S. Khi d
. du
,
,

s h`nh tr`on di
nhau, tng din

mt
. khng cat
. tch cua chung
, , S
khng nho hon .
9
,,
,
,,
,

quy nap
Chung minh. Ta su dung
phuong phap
theo n. Vo i
.
.
,
,
,
,
`

n = 1 khang
dinh
dung
hin nhin. Ta gia thit rang
b d
` dung
.
,
,
,
`
.
voi moi
k < n. Ta xet tp
. ho. p R gm n h`nh tr`on, ma` no phu mat
, . ,
,
,
knh

phang
vo i din
o` ng tr`on co ban
. tch S va` ta ky hiu
. K la` du
S
,
,
,
` Nu S(K ) ,
lon nht r. Goi
o` ng tr`on nay.
. S(K ) la` din
. tch du
9
,
,
,
,
kt lun
th` h`nh tr`on K thao man
`. V` vy
. cua b d
. vo i S(K ) <
S
mt
`, R co ban
knh khng lo,n ho,n r, va` suy
. Mi
. h`nh tr`on tu
9
,
,
,,
,
` trong du
ra, nu no co di
m chung vo i K, th` no phai nam
o` ng
,
,,
,
knh 3r. V` din
tr`on d
`ng tm vo i K co ban
o` ng tr`on
. tch cua du
,
,
,
,
,
` la` 9S(K ), tp
lo n nay
ho. p R1 cua tt ca h`nh tr`on ma` co di
m
.
,
,
,
,
,
`n cua mat
. phang

chung vo i K, phu mt
vo i din
. ph
. tch khng
,
,,
,
,
,
lon hon 9S(K ), suy ra nho hon S, v` 9S(K ) < S. Khi d
o co du
o` ng
,
,
,,
,
,ng du
tr`on trong R khng co di
m chung vo i K. Tt ca nhu
o` ng
,
,

`
rng
tr`on nhu vy
anh
mt
tp
khac
R2 = R \ R1 va` phu
. tao
. th
.
.
,
,
,
,ng
. phang

mt
vo i din
. ph`n mat
. tch S2 , S 9S(K ); Vo i ,nhu
,,
,
`, R2 s lu,o.,ng nho ho,n n. Theo gia thit
du
o` ng tr`on nhu vy
. tu
,
,,
,
`, tp
. mt
quy nap
o. c mt
. tu
. ho. p R2 co th chon
., du
. ho, ac
. s h`nh

tr`on di
khng
mt
. khng giao nhau, tng din
. tch cua chung
, , S2
, , 1
S
nho hon
, suy ra khng nho hon (S 9S(K )) =
S ( K ).
9
9
9
,
,
,
,,
,
ng di
H`nh tr`on K khng co nhu
m chung vo i bt cu mt
o` ng
. du
,,
,
,ng du
` trong nhu
` tp
tr`on nao
xet. Thm vao
o` ng tr`on ta dang

. ho. p

150

,,
Chuong 6. H`nh hoc
.

,,
,,
,,
,ng du
nhu
xet du
o` ng tr`on dang

o` ng tr`on K, ta se nhn
o. c tp
. du
.
,
,
,
,
,
ng du
ho. p nhu
o` ng tr`on di
mt
. khng giao nhau, ma` tng din
.
,
, , S

tch cua chung


khng nho hon .
9
,
,

V du. 6.10. Day


nhu ng s tu. nhin a1 , a2 , . . . , an , . . . xac
dinh
. theo
,
,
dang

thu c

a1 = 2, an+1 = (n + 1) an + 1, n = 1, 2, . . .
,
,
Trong mt
. phang

cho an + 1 di
nhau, khng co ba
m khac
. mat
,
,
,
,
,
,
`

Tt ca cac
do

ni
di
` nam
trn mt
an
m nao
o` ng thang.
. thang
. du
,
,
,
,
`
mt
` da cho. Chu,ng
nhu ng di
` du
m nay
o. c t bang
. trong n mau
,
minh rang
`
vo,i moi
vo,i nhu ,ng dnh
. n = 1, 2, . . . t`n tai
. tam giac
,
,
,
,,
`
trong cac
di
cua no d
m da cho, ma` nhu ng canh
`u du
o. c t cung
.
mt
`
. mau.
,
,
,
,
,

Lo` i giai. Chu ng minh quy nap


theo n. Vo i n = 1 khang
dinh
.
.
,
,
,
`

dung
hin nhin. Ta gia thit rang
no cung
dung
voi n = k. Cho
,
,
`

`
ak+1 + 1 di
bai toan va O la` mt
trong
m thoa man di
u kin
,.
,
,
, .
,
,

nhung di
m d
o. Tt ca do
an
m c`on lai
. thang ni O voi mi di
.
,
,
,
,,
`

`
`
tu ak+1 di
m, la ak+1 = (k + 1) ak + 1. Nhung do
an
o. c t
. nay du
`
`, nhu
,ng do
` khac
nhau. Suy ra tu
nhi`u nht bang
k + 1 mau
an
.
,
,
,
` O, co t nht ak + 1 do
tu
`

thang
xut phat
an
th
ang
t
c
ung
m
t
.
,
,.
,
ng di
` (nguyn ly Dirichlet), ly d
` do. Ta xet nhu
mau
o la` mau
m
,
,
,
`

` do.

A1 , A2 , . . . , A ak +1 , ma` chung
ni vo i O bang
do
mau
an
. thang
,
`
,a chung

Nu giu
co hai Ai va` A j cung
ni bang
do
thang
mau
an
.
,
,
OAi A j la` cung
`
` Nu moi
. hai di
do, th` tam giac
mt
mau.
cap
m
.
.
,
,
,
,,
`
`

cua A1 , A2 , . . . , A ak +1 du
do
thang
khng phai mau
o. c ni bang
an
.
,
,
,
,
,
,
,
`
ng do
a chung

do, ta co ak + 1 di
giu
du
m, nhu
an
o. c t bang
. thang
,
` Theo gia thit quy nap
` d

k mau.
la`
an
o trong chung
. ba do
. nao
,
,
cung
`
`
dnh cua mt
mau.
. tam giac

hoc
6.1. V du. quy nap
. toan
. cho h`nh hoc
.

151

,
,
V du. 6.11. Trong mt
. phang

cho 2000 di
m, khng co ba
. mat
,
,
,,
,,

Mt
s trong chung

du
di
` nam
` trn mt
du
o. c
m nao
o` ng thang.
.
.
,
,
,,
ni thanh
`
do

theo nguyn tac


sau: Nu di
an
m A du
o. c ni
. thang
,
,
,
,,
,,
,
vo,i di
m C, th` A khng du
o. c
o. c ni vo i di
m B va` di
m B du
,
,
,,
ni vo,i di
`
vo,i cach
ni trn ta thu du
m C. Chu ng minh rang
o. c
,
khng qua 1 000 000 do

an
. thang.
,
,
,
`
`
Lo` i giai. Bang
quy nap
. theo n ,ta se , chu ng minh rang
,
di
. phang

nu trong mat
cho 2n di
m thoa man
`u kin
.
,
,
,
,
,
2 (n =
ng do
` th` nhu

d
th
ang
k
e
du
o
c
khng
qu
a
n
`u bai,
an

. ,
. ,
,
,

2, 3, . . .). Vo i n = 2 khang
dinh
la` hin nhin. Gia thit
.
,
,
,
,
`

rang
khang
dinh
dung
vo i 2n di
m, va` ta xet 2n + 2 di
m.
.
,
,
,
, , `
`, nhu
,ng di

Ly hai di
do
thang.
m A va` B tu
m ni du
o. c bang
an
.
,
,
,
,
,ng do

C`on lai
nhu
thang
ke
m, theo gia thit quy nap
an
. 2n di
.
.
,
,
,
,,
,a 2n di
,ng do
` khng lo,n ho,n n2 . Nhu

du
thang
ke
o. c giu
m nay
an
.
,
,
`, A va` B to,i 2n di
tu
m c`on lai,
m
. khng qua 2n (v` nu mt
. di
,
,,
,,
,,
,
,
du
Ch c`on
o. c ni vo i A, no se khng du
o. c ni vo i B va` nguo. c lai).
.
,
,
,
,
do

thm mt
ke
an
an
. do
. ni A va` B. Nhu vy
. tt ca cac
. thang
,,
du
o. c khng qua n2 + 2n + 1 = (n + 1)2 .
,
,
V du. 6.12. a) im O nam
` trong ph`n trong cua da
l`i
giac
,
,,
,
A1 A2 . . . An . Ta xet tt ca cac
goc Ai OA j , o dy
i va` j la` nhu ng
s tu. , nhin khac
nhau giu ,a cac
s 1, 2, . . . , n. Chu,ng minh rang
`
,
,
,
giu a nhu ng goc nay
` co t nht n 1 goc phai la` goc nhon
. (ngha
la` hoac
. la` goc vung, goc tu` hoac
. la` goc bet).
.
,
,
b) Cung
`
bai
` toan
cho da
din
. vo i n dnh.

,
,
,
,
,
`

Lo` i giai. Ta chu ng minh bang


quy nap
. , theo n. Vo i n = 3 khang
,,
,
,
`
`

dinh
du
Ta gia thit rang
no dung
vo i
o. c chu ng minh d dang.
.
,,
,
` d
l`i co
n = k, o dy
k la` mt
o va` ta xet da
giac
. s tu. nhin nao

152

,,
Chuong 6. H`nh hoc
.

,
, ,,

s canh
k + 1, A1 A2 . . . Ak+1 . Ta ke du
l di
o` ng thang
qua O vung
.
,
goc voi OAk+1 (h`nh 7).
,
nht mt
,ng dnh
It
trong nhu
.
,
` trong nu,,a
nam
cua (k + 1)-da
giac
,
pha vo,i Ak+1 ngan
. phang

mat
khac
,
,
,,
,`,

boi du
l. Cho d
ong thang
o la` dnh
`
A2 . Khi d
o goc A2 OAk+1 la` tu.
,

Theo gia thit quy nap


giac
. k-da
l`i A1 A2 . . . Ak co t nht k 1 goc

khng nhon
Ai OA j va` chung
. dang
.
,
`
H`nh 6.4:
vo i goc A2 OAk+1 . Bang
d
khac
cach
o
,
,
,
,
,
,
`
ng goc Ai OA j khng nhon
chu ng minh du
s luo. ng nhu
o. c rang
.
A2 . . . Ak+1 t nht la` k.
trong (k + 1)da
giac

V du. 6.13. a) Chu,ng minh rang


` trong moi
(n 4) co t
. n giac
,
,
` tron

nht mt
giac.
o` ng cheo nam
. ven
. bn trong da
. du
,
,,
,
b) S nho nht cac
du
la` bao
o` ng cheo nhu vy
. trong n giac
nhiu?
,
,
,
la` l`i th` bai
` toan
la` hin nhin. By gio`,
Lo` i giai. a) Nu da
giac
, ,,
,
,
,,
,
,
tai
gia su goc trong cua da
dnh A lo n hon 1800 . Boi goc nh`n
giac
.
,
,
,
`, dnh A lun du,o,i mt
tu
tron
cua da
goc nho
giac
. ven
. mt
. canh
.
.
,
,,
,
`, dnh A se nh`n du
hon 1800 , cho nn tu
o. c tron
. ven
. t nht, hai
,
,
` di
tu
canh.
Do d
o t`n tai
m A ma` trn d
o xy ra
.
. tia xut phat
,
,
,
,
,

`
`
phn) cac
canh
vic
du
i (cac
o. c nh`n tu di
m A (h`nh 9). Mi
. d
.
,`,
` tron

tia d
o cho mt
ong cheo nam
giac.
. du
. ven
. bn trong da
,
,
` h`nh 8 ta thy cach

co dung

b) Tu
du. ng mt
n3
. n giac
,
,`,
,
` tron
du
ong cheo nam
. ven
. trong no. C`on lai
. ta phai chu ng minh
,,
`
` bn
co t nht n 3 du
rang
trong moi
o` ng cheo nam
. n giac

hoc
6.1. V du. quy nap
. toan
. cho h`nh hoc
.

H`nh 6.5:

153

H`nh 6.6:

, ,,
,

trong. Vo i n = 3 mnh
d
Gia su mnh
d
` d
o dung.
` d
o dung
.
.
,
,
,
,
k giac,
vo i k < n, ta phai chu ng minh no cho
cho tt ca cac
,
, ,,
,,
Theo kt qua bai
` trn, n giac
co th du
ngiac.
o. c chia boi mt
.
,,
` tron
`

du
v
en
bn
trong
th
anh
hai
da
gi
ac
(
k
+
1
)
o` ng cheo nam

.
.
va` (n k + 1) giac,
vo,i k + 1 < n va` n k + 1 < n. Trong
giac
,,
da
d
cac
giac
o co t nht (k + 1) 3 va` (n k + 1) 3 du
o` ng
` bn trong tu,o,ng u,ng. Do d
co t nht
cheo nam
o trong ngiac
,`,
` bn trong.
1 + (k 2) + (n k 2) = n 3 du
ong cheo nam

,
V du. 6.14. Chu,ng minh rang
`
moi
co th cat
ra thanh
`
. ngiac
,
,
cac
tam giac
bang
` cac
du
nhau.
o` ng cheo khng cat
,
,
,
,
`
` bang
Lo` i giai. Ta chu ng minh mnh
d
quy nap
` nay
. , theo n. Vo i
, ,,.

vo,i
n = 3 mnh
d
Gia su mnh
d
cho tt ca k giac,
` dung.
` dung
.
.
,
Mt
bt
k < n, ta c`n chu ng minh no cho moi
. n giac.
. n giac
, ,,
,,
,
,
`
(bai
`
ky` co th du
hai da
o. c chia boi mt
o` ng cheo ra thanh
giac
. du
, ,
, ,
,
da
co s canh

truoc), trong dos


nho hon n, tu c la` chung
mi
giac
.
, ,,
,
`
tam giac
theo gia thit quy nap.
co th du
cac
o. c chia ra thanh
.

,
,
V du. 6.15. Chu,ng minh rang
` tng cac
goc trong cua mt

. ngiac
0
bt k`y bang
`
(n 2)180 .

154

,,
Chuong 6. H`nh hoc
.

,
,
,
,
` quy nap.
d
Vo i n = 3 mnh
Lo` i giai. Ta chu ng minh mnh
` bang
.
.
.
,
, ,,
,
,
,
`
`

d
la` hin nhin. Gia su mnh
dung
da du
o. c chu ng minh cho
. , d
,
,
,

k giac,
vo i k < n, ta phai chu ng minh no cho moi
tt ca cac
. n
, ,,
,,
,
,

Mt
bt ky` co th du
giac.
du
o. c chia boi mt
o` ng cheo ra
. n giac
,.
,
,

` 2 da
(Xem bai
` truo c). Nu s canh
d
lam
cua mt
giac
giac
o
.
. da
,
,
`
`

,a
kia bang

bang
k + 1, th` s canh
cua da
n k + 1, hon nu
giac
.
,
, ,
,
,
goc cua cac
da

ca hai s d
o d
`u nho hon n. Do d
o tng cac
giac
,,
,
0
0
`

d
(k 1)180 va` (n k 1)180 . Cung
ro rang
o tuong u ng bang
,
,
,
,
`
`
goc cua ngiac
bang
goc cua cac
da

rang
tng cac
tng cac
giac
, ` `
0
0

d
o, tuc la bang (k 1 + n k 1)180 = (n 2)180 .

V du. 6.16. Chu,ng minh rang


`
moi
l`i vo,i n 5 d
`u co
. ngiac
,
th cat
ra thanh
`
ngu giac
l`i.
,
,
,
`
`
l`i vo,i
Lo` i giai. Ta chu ng minh bang
quy nap
moi
. rang
. n giac
,
ra thanh
`
ngu giac
l`i. Vo,i n = 5 di
n 5 d
cac
`u co th cat
`u d
o
,
,
,
la` hin nhin, c`on vo i n = 6 va` 7 co th xem h`nh 10 va` 11.
, , ,, `
l`i vo,i 5 m < n
n 8 va` moi
By gio` gia su rang
. m giac
,
,
ra thanh
ra
`, n giac
`
ngu giac.
Tu
co th cat
d
cac
`u co th cat
,
,,
tao

mt
o c`on lai
. ngu giac
. boi 5 dnh lin tip. Khi d
. ,(n 3) giac.
,,
ra thanh
lai
`
Boi v` 5 n 3 < n, nn (n 3) giac
. co th cat
,
ngu giac
theo gia thit quy nap
hoc.
cac
. toan
.

` tp
6.2. Bai
.

,
,
,
`

. phang

. 6.17. Trong mat


cho n 3 di
mi
m. Chu ng minh rang
,
,
,
,
,
,
ng do

di
m co th ni vo i mt
m c`on lai
an
. s di
. sao cho nhu
. thang
,,
,
`
`
li gep
nhn
mt
o. c khng tu. cat nhau va` tao
giac
. du
. thanh
. da
,,
,
ng tam giac.

boi nhu
,
,
,
do
` cua mi

. 6.18. . dai
trong n 3 do
da
an
an
. thang
. thang

` tp
6.2. Bai
.

H`nh 6.7:

155

H`nh 6.8:

,
,
`
cho lo n hon 1. Bit rang
khng co k ( k = 3, 4, . . . , n ) s do
an
. co
,
,
,
`
canh
Chu ng minh rang
`
th tao
ra cac
mt
tng d. dai
giac.
.
. da
, .
,
,
n

1
do
cua cac
an
. lo n hon 2 .
,
,
,,
,,
manh
. phang

. 6.19. Trn mat


bi. chia boi n du
o` ng tr`on ra cac
,
,
`
`
nhau. Chu,ng minh rang
`
. phang

khac
mat
co th t bang
hai mau
,
,

`
` duy nht va` hai manh lin nhau
sao cho mi manh t mt
. mau
` khac
nhau.
co mau
,,
,,
,,
nu,,a
. 6.20. O d
`u du
o` ng knh mt
o` ng tr`on ta vit s 1. Mi
. du
,
,
,
,,
,,
,a ta vit tng nhu
,ng s o,
du
m giu
o` ng tr`on lai
va` o di
. chia di
,,
ph`n tu, cung ta lai
` di
hai d
chia lam
`u cung. Sau d
o mi
va` o
.
,
, ,
,a vit s tng o, hai d

` nay
` lap
. lai
di
lam
m giu
`u cung. Cach
. n
,
`

tnh tng cac


s da vit ra.
ln. Hay

, ,
CHUONG

,
C
A THU
,
,
7.1. Phn tch da
uc ra th`ua s . . . . . . . . . . . . . . . . . . . . . . . .
th

h. s . . . . . . . . . . . . . . . . . . . . . . .
7.2. Nguyn l
y so sanh
cac
,
,
`
7.3. ao
h
am
c
ua
da
th
u
c. . . . . . . . . . . . . . . . . . . . . . . . . . . . . . .

.
,
7.4. a th
uc Chebychev . . . . . . . . . . . . . . . . . . . . . . . . . . . . . . . . .
` tp
7.5. Baii
. ...............................................

156
160
169
172
174

,
,
c ra thu
` a s
7.1. Phn tch da
thu
,
` s co dang
a thu c bc
. n goi
. la` ham
.
P ( x ) = a 0 x n + a 1 x n 1 + + a n , ( a 0 6 = 0 )

(7.1)

,,
,
`
,ng hang
o dy
s (h. s da
a0 , a1 , . . . , an la` nhu
thu c), c`on n 0
,
,
,
,
,
` do
la` mt
s nguyn (bc
thu c). a thu c la` mt
n
. cua da
. lo p ham
, .
,
,
,
hoc.
gian, nhung co rt nhi`u u ng dung
trong toan

.
. Vo i n = 0 da
,
,
,
,
`
`
` tuyn

thu c (7.1) la` hang


s a0 , vo i n = 1, P( x ) tro thanh
ham
,
,
tnh P( x ) = a0 x + a1 , c`on voi n = 2, P la` tam thu c bc
. hai P( x ) =
,
,
2
a0 x + a1 x + a2 . da
thu c co bc
`u kin
. la` n th` lun c,o di
. a0 6= 0.
,`,
,,
,
,

Trong truong ho. p nguo. c lai


thu c P la` n.
. th` bc
. cao nht cua da
,
,ng da
Nu P( x ) va` Q( x ) la` nhu
thu c, th` P( x ) + Q( x ), P( x )
,
,
,

Q( x ) va` P( x ).Q( x ) cung


la` da
thu c, nhung phep chia hai da
thu c
,
cho nhau khng lun lun la` mt
thu c.
. da
,
,
,
S goi
thu c P( x ), nu P() = 0. Nhu
. la` nghim
. cua da

,
,
7.1. Phn tch da
thu c ra thu` a s

157

,
2
2
vy,
. Nu tam thu c, bc
. hai P( x ) = ax + bx + c, ma` b 4ac 0
,
,
` x1 , x2 du
th` hai nghim
a ta d
n phn tch
. cua tam th,u c nay
,
hon ta co
P( x ) = a( x x1 )( x x2 ). Tng quat
,
V du. 7.1. Nu P( x ) la` mt
thu c bc
. da
. n 1 va` la` mt
. s
,
,
,
,
thu. c, th` la` nghim
thu c
. cua P( x ) khi v, a` ch khi t`n tai
. mt
. da
,
Q( x ) bc
dang

thu,c sau
. n 1, ma` no thoa man
P( x ) = ( x ) Q( x )

(7.2)

vo,i moi
. x.
,
,
,
,

Lo` i giai. i`u kin


Ta chu ng minh di
`u
. du la` tt nhin dung.
,
kin
thu c bc
. c`n, nu P( x ) la` da
. n
n

a n i x i

(7.3)

ani i = 0.

(7.4)

P( x ) =

i =0

va` P() = 0, ngha la`

P() =

i =0

,
,,
,

Ta su dung
dang
thu c an bn = ( a b)( an1 + an2 b + +
.
,,
`, (7.3) va` (7.4) ta nhn
bn1 ), tu
o. c
. du
n

P( x ) = P( x ) P() =

a n i ( x i i ) =

i =0
n

= ( x ) a n i ( x i 1 + x i 2 + + n 1 ) = ( x ) Q ( x ),
i =1

,,
o dy
Q( x ) =

ani (xi1 + xi2 + + n1 ) hin nhin la` da

i =1
,
thu c bc
. n 1 (v` a0 6= 0).

,
,
V du. 7.2. Khng co mt
thu c bc
. da
. n co nhi`u hon n nghim
.
s khac
nhau.

158

,,
,
Chuong 7. a thu c

,
,
,
,
,,
`
quy nap
phuong phap
Lo` i giai. Chu ng minh bang
., theo n. Gia
,,
,
su P la` mt
thu c bc
. da
. n va` 1 , 2 , . . . la` nghim
. cua no ( i 6= j
,
,
voi i 6= j). Voi n = 1, P( x ) = a0 x + a1 ( a0 6= 0) co mt
. nghim
.
, ,,
a1
,
,

duy nht 1 = . Gia su mnh


d
vo i s n. Ta se chu ng
` dung
.
a0
, ,
,
,

` ta gia su, t`n


minh no cung
dung
vo i n + 1. Vo i muc
ch nay
. d
,
nhau
tai
o co n + 2 nghim
thu c Q bc
. mt
. da
. n + 1, m, a` n
. khac
,
,
,
duo i dang
1 , 2 , . . . , n+2 . Khi d
(do .7.1)
o Q co th biu din
.
Q ( x ) = ( x n +2 ) Q 1 ( x )
,,
,
`,a s x n+2 khng co nghim
o dy
Q1 la` da
thu c bc
. n. V` thu
, .

`
la` mt
trong
c
ac
s

,
.
.
.
,

,
th`

ch
ung
l
a
nghi
m
c
ua
Q1 .
2
1
n
+
1
.
.
,
,
` co ngha la` mt
Nhung di
`u nay
thu c bc
. , da
. n co n + 1 nghim
.
,
` toan
` khac
nhau, trai
vo i gia thit quy nap.
hoan
.

,
da
V du. 7.3. Chu,ng minh rang
` mi
thu c
P ( x ) = a 0 x n + a 1 x n 1 + + a n , ( a 0 6 = 0 )
, ,
du,o,i dang
co th biu din
.
P( x ) = a0 ( x 1 )( x 2 ) . . . ( x n ),
,
,
,
o, dy
1 , 2 , . . . , n la` nghim
thu c.
. cua da
,
,
,
`
Lo` i giai. Ta chu ng minh bang
quy nap
. theo n. Nu n = 1, th`
,
a1
P( x ) = a0 x + a1 co mt
. nghim
. duy nht 1 = a va` hin nhin
0
a1
P ( x ) = a0 ( x + ) = a0 ( x 1 ).
a0
,
, ,,
,
,

dinh
Gia su mnh
d
vo i da
` khang
thu c bc
. dung
.
. n 1 va` cho
,
`
` tp
deg P( x ) = n. Ta bit rang
P( x ) t`n tai
m
bai
. nghi
. nhu cac
.
,
,
trn da chu ng minh, ly 1 la` nghim
c
ua
P
(
x
)
.
Khi
d
o
P
(
x
)
=

.
,
,
( x 1 ) Q( x ), d thy deg Q( x ) = n 1 va` h. s truo c bc
. cao

,
,
7.1. Phn tch da
thu c ra thu` a s

159

,
,
, `
,
`, d
,ng
`
trung
vo i a0 cua P( x ). Tu
nht cua da
thu c nay
o suy ra nhu
,
,
,
,ng nghim
nghim
. cua P( x ) la` 1 va` nhu
. cua Q( x ). Theo gia thit
quy nap
.
Q( x ) = a0 ( x , 2 )( x 3 ) . ,. . ( x n ),
,
,,
o dy
2 , 3 , . . . , n la` tt ca nghim
cua Q( x ). Khi d

o tt ca
.
,
nghim
. cua P( x ) la` 1 , 2 , . . . , n va`
P( x ) = ( x 1 ) Q( x ) = a0 ( x 1 )( x 2 ) . . . ( x n ).

,
,
,
,
Ky hiu
t
. F la` m
. tp
. s: tp
. ho. p s phu c, ,tp
. ho. p s thu. c va`
,
,
,
,
`
,u ty. Nhu
,ng da
tp
s P( x ) vo i
thu c khng phai hang
. ho. p s hu
,
,,
h. s trong tp
hop F goi
tch du
o. c trn F, nu no
. la` khng phn
, ., .
,
,
,,
,
,
du
khng th biu din
o. c nhu tch cua hai da
thu c (khng phai
, ,
, `
,
,
da
s) co h. s thuc
F, co bc
thu. c su. nho hon bc
thu c hang
.
.
.
,
cua P( x ).
,
,
, ,
V du. 7.4. Chu,ng minh rang
`
moi
thu c thu. c su. vo i h.
. da
,
,
,
nhu, tch cua
s thuc
nhu ,ng thu`,a s
. F co th biu din
,
,,
,
nay
khng phn tch du
` la` duy
o. c trn F. Su. biu din
,
,
,
nht theo ngha day
cua cac
thu` a s co th khac
nhau
,
,
,
,
,
tuong u ng vo i hang
`
s khac
khng cua F, noi cac
khac,

nu
P( x ) = P1 ( x ).P2 ( x ) . . . , Pr ( x ) = Q1 ( x ).Q2 ( x ) . . . Qs ( x )
,
,
cua
la` hai biu din
P( x ) nhu, tch cac
thu`,a s khng phn tch
,,
,,
du
o. c trn F, th` r = s va` Pi ( x ) = i Qki ( x ), o dy
0 6= i F va`
,
,
,
k1 , k2 , . . . , kr la` nhu ng s nao
` d
o xp thu tu. theo 1, 2, . . . , r.
,
,
,
, `
,
Lo` i giai. Cho P( x ) khng phai la` da
trong tp
thu c hang
. ho. p F
va` n = deg P( x ). Nu n = 1, th` P( x ) = a0 x + a1 la` khng phn
,
,,
nhu, tch duy nht thu
`,a s khng phn
tch du
o. c va` no biu din
,,
tch du
o. c.

160

,,
,
Chuong 7. a thu c

,
,
,
`
Cho n la` s tu. nhin bt ky` va` gia thit rang
moi
da
thu c bc
.
.
, ,
, ,
,
,

` a s khng phn tch


thu
nho hon n co th biu din nhu tch cac
,,
du
o. c trn F.
,
,,
Nu da
thu c da cho P( x ) la` khng phn tch du
o. c trn F, th`
,
,
,
,
nhu, tch cua mt
co th cng nhn
thu c khng
. no biu din
. da
,,
phn tch du
o. c.
,
,,
Nu da
thu c phn tch du
o. c, th` no co dang
.
P( x ) = Q( x ).H ( x ),
,,
,
,
,ng da
o dy
Q( x ) va` H ( x ) la` nhu

thu c vo i h. s trong F va`


,
,
deg Q( x ) < n va` deg H ( x ) < n. Nhung khi d
o theo gia thit quy
,
,
,
nhu, tch cua cac
`,a s da
thu
nap
thu c
. Q( x ) va` H ( x ) biu din
,

trn F. Suy ra cung


dung
cho P( x ). Ngha la` moi
thu c trn F
. da
, ,
,,
nhu, tch cac
`,a s khng phn tch du
thu
co th biu din
o. c co h.
,
`
s trong F. Chu ng minh duy nht danh
cho ban
. doc.
.

h. s
7.2. Nguyn l
y so sanh
cac
` s P1 ( x ) va` P2 ( x ) xac
dinh
Cho hai ham
trn mt
.
. tp
. con D
,
,
`

cua s thu. c. Chung


ta noi rang
P1 ( x ) va` P2 ( x ) trung
nhau trong
,
,
,
,

D, nu dang
thu c P1 ( x ) = P2 ( x ) dung
vo i moi
Tp
ho. p cac
. x D.
.
,
,
,
,
da
` dac
dinh
. bit.
da
cua mi
thu c la` mt
thu c
.
. lo,p ham
. Mi`n xac
,
,
,
,
la` tp
thu c P( x ) va` Q( x )
. ho. p con cua s thu. c. Nhu vy
. hai da
,
,
,
`

trung
nhau, nu P( x ) = Q( x ) dung
vo i moi
x s thu. c. Lo p cac
da

.
,
,
,

. bit
thu c co mt
cho hai da
thu c P( x ) va`
. tnh ch,t rt dac
. ,la` d
,
`

Q( x ) trung
nhau ch cn thit kim tra P( x ) = Q( x ) dung
vo i
,
,u han
mt
. s hu
. gia tri. cua x.
V du. 7.5. Cho
P ( x ) = a 0 x n + a 1 x n 1 + + a n


h. s
7.2. Nguyn ly so sanh
cac

161

va`
Q( x ) = b0 x m + b1 x m1 + + bm
,
,
la` hai da
`
nu t`n tai
thu c va` n m. Chu ng minh rang
. n+1
,
,
nhu ng s tu` ng di
nhau 1 , 2 , . . . , n+1 (i 6= j vo,i i 6=
mt
. khac
j) sao cho P(i ) = Q(i ), i = 1, 2, . . . , n + 1, th` n = m va` a0 =
b0 , a1 = b1 , . . . , an = bn .
,
,
,
Lo` i giai. Do khng co d`
oi hoi g` v` b0 6= 0 va` lai
. co n m, ta co
, ,
`
danh

h. s)
th gia thit (bang
cach
s lai
. cac
Q( x ) = b0 x n + b1 x n1 + + bn
,
Bu,o,c co, so,: nu n = 1, th` P( x ) = a0 x + a1 , Q( x ) = b0 x + b1 va`
,
,
,
cac
dang

thoa man
thu c sau
a0 1 + a1 = b0 1 + b1
a0 2 + a1 = b0 2 + b1
,,
`, theo v ta nhn
Tru
o. c
. du
a0 (1 2 ) = b0 (1 2 )
, , ,,
,
Nhung theo di
`u kin
. 1 6= 2 ta co th gian uo c cho 1 2 6= 0.
,,
`, d

Ta nhn
suy ra a1 = b1 .
o. c a0 = b0 , tu
o cung
. du
, ,,
,
,
,

Buo c quy nap:


d
vo i n 1. Ta co
` dung
.
. Gia su mnh
n

P( x ) P(n+1 ) =

=
=

ai xni ai nn+1i

i =0
i =0
n
1
a0 ( x n+1 ) + a1 ( x n1 nn
+1 ) + + a n 1 ( x n +1 )
( x n+1 )( a0 x n1 + a10 x n2 + + a0n1 ) = ( x n+1 ) P1 ( x )
n

,,
o dy
P1 ( x ) = a0 x n1 + a10 x n2 + + a0n1
,,
,,
,
Tuong tu. Q( x ) Q(n+1 ) = ( x n+1 ) Q1 ( x ), o dy
Q1 ( x ) =
,
n

1
0
n

2
0
b0 x
+ b1 x
+ + bn1 . Khi d
o voi i = 1, 2, . . . , n ta nhn
.

,,
,
Chuong 7. a thu c

162
,,
du
o. c
P1 (i ) =

Q ( i ) Q ( n +1 )
P ( i ) P ( n +1 )
=
= Q1 ( i )
i n +1
i n +1

,
0
0
0
0
.
va` theo gia thit quy nap
. a0 = b0 , a1 = b1 , . . . , an1 = bn1 . Ta dat
P2 ( x ) = a1 x n1 + a2 x n2 + + an
Q2 ( x ) = b1 x n1 + b2 x n2 + + bn
,
,

vo i i = 1, 2, . . . , n, n + 1 se thoa man
P2 (i ) = P(i ) a0 in = Q(i ) b0 in = Q2 (i )
,
dung
Lai
gia thit quy nap
. ap
.
. ta co a1 = b1 , a2 = b2 , . . . , an = bn .

,
,
,
,
V du. 7.6. T`m tt ca cac
da
dang

thu,c
thu c P( x ) thoa man
P ( x ) = P ( x + 1).
,
,
,
`
Lo` i giai. D thy, nu P( x ) la` da
thu c hang

,
,
,
,,
,

s, th` no thoa man


dang
thu c trn. Ta gia su
deg P( x ) 1 va` cho P( x ) = a0 x n + a1 x n1 + + an ,
,
,
,
dang

thu c P( x ) = P( x + 1). Khi


a0 6= 0 va` P( x ) thoa man
d
o
a 0 ( x + 1 ) n + a 1 ( x + 1 ) n 1 + + a n = a 0 x n + a 1 x n 1 + + a n
,,
,,
,
`, dy

tu
h. s truo c x n1 , ta nhn
so sanh
o. c na0 + a1 = a1 , tu c
. , du
,
, `
,
,ng da
la` a0 = 0 no v ly vo i gia thit. Suy ra ch co nhu
thu c hang
,
di
` ra.
s la` thoa man
`u kin
`u bai
. d

,
V du. 7.7. Chu,ng minh rang
`
vo,i moi
. s tu. nhin k t`n tai
. duy
,
k
nht mt
thu c Pk ( x ) bc
. da
. k + 1 sao cho Pk (0) = 0 va` x =
Pk ( x + 1) Pk ( x ).


h. s
7.2. Nguyn ly so sanh
cac

163

,
,
,
,
Lo` i giai. Vic
thu c Pk ( x ) ta se chu ng minh quy nap
. t`n tai
. da
.
,
theo k. Voi k = 0 va` k = 1 ta co
x0 = 1 = ( x + 1) x;
1
1
1
1
x = ( ( x + 1)2 ( x + 1)) ( x2 x )
2
2
2
2
1 2 1
Ngha la` P0 ( x ) = x, P1 ( x ) = x x.
2
2
,
,
,
,

`
Gia thit voi mi l k tn tai
thu c Pl ( x ), vo i no Pl (0) = 0
. da
va` x l = Pl ( x + 1) Pl ( x ). Ta co
x k+1 = x.x k = xPk ( x + 1) xPk ( x ),
`, d
tu
o co
x k+1 + Pk ( x + 1) = ( x + 1) Pk ( x + 1) xPk ( x ).
Cho thm
Pk ( x + 1) = ak+1 x k+1 + ak x k + + a1 x + a0 .
Khi d
o
k

(1+ ak+1 ) x k+1 = ( x + 1) Pk ( x + 1) xPk ( x ) al Pl ( x + 1) al Pl ( x )


i =0

l =1

= (( x + 1) Pk ( x + 1) al Pl ( x + 1)) ( xPk ( x ) al Pl ( x )).


l =0

l =0

,
,
,
Vo i bc
thu c H ( x ) = xPk ( x )
. cua da

al Pl (x) cao nht ch la`

l =0
,
,
,,
`
cua
k + 1 va` d thy rang
h. s truo c x k+1 trong dang
chun tac
.
, ,,
,,
`

H ( x ) dung
la` ak+1 . Nu gia su rang
1 + ak+1 = 0, ta nhn
o. c
. du
`
` trn), no
H ( x + 1) = H ( x ) va` khi d
s (bai
o ta co H ( x ) la` hang
,

vo i ak+1 6= 0. Suy ra 1 + ak+1 6= 0 va` nu ta dat


.
trai

Pk+1 ( x ) =

k
1
( xPk ( x ) al Pl ( x )),
1 + a k +1
l =0

,,
,
Chuong 7. a thu c

164
,,
Ta nhn
o. c
. du

x k+1 = Pk+1 ( x + 1) Pk+1 ( x ).


,
`
Chu ng minh duy nht danh
cho ban
. doc.
.

,
,
,
V du. 7.8. Hay
t`m tt ca da
di
thu c P( x ) thoa man
`u kin
.
P( x2 2) = ( P( x ))2 2.
,
,
, `
,,
,
,
Lo` i giai. Ta chu y nhu bai
truo c vo i moi
. s tu. nhin n t`n tai
.
,
,
`

nhiu nht mt
thuc Pn ( x ) bc
. da
. n thoa man
Pn ( x2 2) = ( Pn ( x ))2 2.

(7.5)
,
,,
,,
,

`
Vo i vic
h. s truo c s mu cung
bc
. so sanh
. cua x trong phuong
,,
tr`nh trn ta t`m du
o. c
P1 ( x ) = x, P2 ( x ) = x2 2, P3 ( x ) = x3 3x,
P4 ( x ) = x4 4x2 + 2, P5 ( x ) = x5 5x3 + 5x.
,,
` ra khng kho khan
g` thit lp
Ngoai
o. c quan h.
. du
P3 ( x ) = xP2 ( x ) P1 ( x );
P4 ( x ) = xP3 ( x ) P2 ( x );
P5 ( x ) = xP4 ( x ) P3 ( x ).
,
,
` go.,i y cho ta du
i`u nay
a ra mt

. gia thit sau dy:


,
Moi

thu c trong day


. da
P1 ( x ), P2 ( x ), P3 ( x ), . . . , Pn ( x ), . . .
,
, , dinh theo cac
du
dang

thu,c sau
o. c xac
.
P1 ( x ) = x, P2 ( x ) = x2 2, . . . , Pn+1 ( x ) = xPn ( x ) Pn1 ( x )
,
thoa man
di
`u kin
. (7.5).


h. s
7.2. Nguyn ly so sanh
cac

165

,
,
,,
` tp
V` da
thu c Pn ( x ) bc
. n,,theo bai
. truo,c, nu gia thit trn
,

da
di
`
la` dung
th` chung
la` tt ca cac
thu c thoa man
`u kin
. bai

toan.
,
,
,
`
hoc
By gio` ta chu ng minh gia thit bang
quy nap toan
. theo
,
, ,, , .
,
,

` d
n. Voi n = 1 va` n = 2 gia thit dung.
Gia su vo i n nao
o da
thu c
,
,
di
Pn ( x ) va` Pn+1 ( x ) thoa man
`u kin
o d
i vo i Pn+2 ( x )
. (7.5). Khi d
ta co
Pn+2 ( x2 2) ( Pn+2 ( x ))2 + 2 =

= ( x2 2) Pn+1 ( x2 2) Pn ( x2 2) ( xPn+1 ( x ) Pn ( x ))2 + 2


= ( x2 2)(( Pn+1 ( x ))2 2) (( Pn ( x ))2 2) x2 ( Pn+1 ( x ))2 +
+ 2xPn+1 ( x ).Pn ( x ) ( Pn ( x ))2 + 2
= 2( Pn+1 ( x ))2 2( P( x ))2 + 2xPn+1 ( x ).Pn ( x ) 2x2 + 8
= 2Hn ( x ),
,,
.
o dy
ta dat
Hn ( x ) = ( Pn+1 ( x ))2 + ( Pn ( x ))2 xPn+1 ( x ).Pn ( x ) + x2 4.

. khac
Mat
Hn ( x ) = ( xPn ( x ) Pn1 ( x ))2 + ( Pn ( x ))2 x ( xPn ( x )

Pn1 ( x )).Pn ( x ) + x2 4
= ( Pn ( x ))2 + ( Pn1 ( x ))2 xPn ( x ).Pn1 ( x ) + x2 4
= Hn1 ( x ) = Hn2 ( x ) = . . . = H1 ( x )
= ( P2 ( x ))2 + P1 ( x ))2 xP2 ( x ).P1 ( x ) + x2 4
= ( x2 2)2 + x2 x ( x2 2) x + x2 4.
Suy ra Pn+2 ( x2 2) = ( Pn+2 ( x ))2 2.

, ,
,
,
V du. 7.9. Cho P( x ) la` da
thu c vo i h. s thu. c nhn
. gia tri. s hu u

,,
,
Chuong 7. a thu c

166

,
, ,
,
,
, ,
ty vo,i moi
. s x v ty. Chu ng
. s x hu u ty va` gia tri. s v ty vo i moi
,
, ,
,
minh rang
`
P( x ) la` da
thu c tuyn tnh vo i h. s hu u ty.
,
,
,
,
`
,u
h. s cua P( x ) la` hu
Lo` i giai. 1) Ta se chu ng minh rang
cac
,
,
,
,
`
ty. Chu ng minh bang
quy nap
. theo bc
. n cua, P( x ). Tht
. vy,
. vo i
,
`
`
,u ty (v` bang
n = 0, P( x ) la` hang
s va` no la` mt
v du. nhu
. s hu
,
,
,
,
,
,

da

P(0)). Gia thit khang


dinh
dung
vo i tt ca cac
thu c bc
.
. nho
,
,
,
di
` va` cho
hon s tu. nhin n (tt nhin thoa man
`u kin
`u bai)
. d
n
n

1
P ( x ) = a0 x + a1 x
+ + an1 x + an . D thy an = P(0) la` s
,
,
u ty va` nu ta dat
.
hu
P( x ) an
Q ( x ) = a 0 x n 1 + a 1 x n 2 + + a n 1 =
,
x
,
,
,
,u ty vo,i bin hu
,u ty x. Theo gia thit
th` Q( x ) se nhn
. gia tri. hu
,
,ng s a0 , a1 , . . . , an1 la` hu
,u ty.
quy nap
o nhu
. khi d
,
,
,
,u ty. Vo,i di
Nhu vy
`u d
o P( x ) khng la`
. h. s cua P( x ) la` hu
,
,
,
,
,
,
`
,u ty vo,i moi
hang
s, v` trong truo` ng ho. p nguo. c lai
. P( x ) se la` hu
.
x. Cho P( x ) = a0 x n + a1 x n1 + + an , n > 0. Khng mt tnh
,
,
,
`
co th cho rang
` ra da
tng quat
ai la` nguyn. Ngoai
thu c
Q( x ) = a0n1 .( P( x ) an )

= ( a0 x )n + a1 ( a0 x )n1 + + an1 a0n2 ( a0 x ).


,
Ngha la` da
thu c
H (y) = yn + a1 yn1 + + an1 a0n2 y

,
di
`
thoa man
`u kin
`u bai.
. d
, ,
,
,,
,
`
Ta se chu ng minh rang
vo i moi
. s nguyn du lo n m phuong
tr`nh H (y) = m co nghim.
Tht
.
. vy,
. ly m > H (0) va` (y) =
,,
H (y) m. Khi d
o (0) < 0 va` lim (y) = +, v` th phuong
y
,,
tr`nh H (y) = m co nghim
. duong ym . Ly m = p la` s nguyn t


h. s
7.2. Nguyn ly so sanh
cac

167

, ,
,
`, di
,u ty va`
du lo n. Ta co H (y p ) = p. Tu
suy ra y p la` s hu
`u kin
.
,
` ra
v` h. s bc
y p la` nguyn va` ngoai
. cao nht cua H (y) la` 1, th`
,
,,
,,
,
,

. la` y p la` uo,c s


y p du
tu. do cua (y), hoac
o. c chia ht boi s hang
.
,
,
,
. la` y p = p. Nhu,ng dang

cua p. Ngha la` y p = 1 hoac


thu c y p = 1
,
,
,
,

ch co kha nang
nhi`u nht vo i mt
p. Ngha la` y p = p cho tt ca
.
, ,
,,
khac,
ta da nhn
s nguyn t du lo n p. Noi cach
du
o. c H ( p) = p
.
,
, ,
,
`, nguyn ly so sanh

h. s
vo i tt ca s nguyn t du lo n. Tu
cac
suy ra khi d
o H (y) = y va` ngha la` P( x ) = a0 x + a1 .

, ,
,
V du. 7.10. Cho P( x ) la` da
thu c vo i h. s nguyn, vo i no P(0) =
P(1) = 1. va` a0 la` s nguyn bt k`y. Ta dinh
ngha an+1 = P( an )
.
,
,
,
,
`

voi n 0. Hay
chung minh rang

voi m 6= n co dang

thu,c sau
( am , an ) = 1.
,
,
,
Lo` i giai. Ta chia da
thu c P( x ) cho x ( x 1). Ly
P( x ) = x ( x 1) Q( x ) + ax + b,
,
,
,
`, qua tr`nh chia da
Tu
thu c suy ra Q( x ) la` da
thu c vo i h. s
,
,

nguyn. Trong dang


thu c trn ta cho x = 0 va` x = 1 va` chu
,,
,,
,
`
y rang
P(0) = P(1) = 1, d
i vo i a va` b ta nhn
o. c h. phuong
. du
,
`, dy
tr`nh b = 1, a + b = 1, tu
suy ra b = 1 va` a = 0, va` nhu vy
.
,,
,
,
ta co P( x ) = x ( x 1) Q( x ) + 1, o dy
Q( x ) la` da
thu c vo i h. s
nguyn.
,
,
,
`

Ta se chu ng minh bang


quy nap
thu c
. theo n, dang
an 1

(mod a0 a1 . . . an1 ),
,
`, dy
` toan.
Tht
tu
suy ra kt lun

. cua bai
. vy,
. cho m < n va`
( am , an ) = d, Khi d
o a0 a1 . . . an1 chia ht cho d va` suy ra an 1
,
`, dy
chia ht cho d. Nhung an chia ht cho d, tu
an 1 an chia
. la` d = 1.
ht cho d hay la` 1 chia ht cho d hoac

,,
,
Chuong 7. a thu c

168

,
,
,
`
an 1 (mod a0 a1 a2 . . . an1 ) vo i moi
Ch c`on chu ng minh rang
.
,
n. Voi n = 1 ta co a1 = a0 ( a0 1) Q( a0 ) + 1 va` suy ra a1 1
,
,

` d
(mod a0 ). Gia thit dung
vo i n nao
o an 1 (mod a0 a1 . . . an1 )
,,
,
. la` an = 1 + ka0 a1 . . . an1 , o dy
hoac
k la` nguyn. Vo i an+1 ta t`m
,,
du
o. c
an+1 = an ( an 1) Q( an ) + 1 = ka0 a1 . . . an1 an Q( an ) + 1,
`, dy
tu
ta co an+1 1 (mod a0 a1 . . . an1 an ).

,
V du. 7.11. Cho day
cac
da
thu c P0 ( x ), P1 ( x ), . . . , Pn ( x ), . . . trong
,
d
o P0 ( x ) = 2, P1 ( x ) = x va` vo i moi
. n 1 th`
Pn+1 ( x ) + Pn1 ( x ) = xPn ( x ).
,
Chu,ng minh rang
`
t`n tai
. ba s a, b va` c sao cho vo i moi
. n 1 ta
d
`u co

( x2 4)[ Pn2 ( x ) 4] = [ aPn+1 ( x ) + bPn ( x ) + cPn1 ( x )]2 .

(7.6)

,
, ,,
,
,
,

Lo` i giai. Gia su t`n tai


vo i moi
(7.6) dung
o
. a, b, c d
. n. Khi d
,

` (7.6) suy ra
(7.6) dung
vo i n = 1. Thay n = 1 vao

( x2 4)( x2 4) = [ a( x2 2) + bx + 2c]2 .
,
,

Ta nhn
thu c trn
. ,thy nu chon
. a = 1, b = 0 va` c = 1 th` dang
,,

du
o. c thoa man.
,
,
By gio` ta chu ng minh: nu chon
a = 1, b = 0, c = 1 th` (7.6)
, . ,
,
,

dung
vo i moi
. n 2, tu c la` ta phai chu ng minh
( x2 4)( Pn2 ( x ) 4) = ( Pn+1 ( x ) Pn1 ( x ))2 .
(7.6)
,,
,,
,
V` Pn+1 ( x ) = xPn ( x ) Pn1 ( x ) nn (7.6) tuong du
ong vo i
x2 Pn2 ( x ) 4Pn2 ( x ) 4x2 + 16 = ( xPn ( x ) 2Pn1 ( x ))2

= x2 Pn2 ( x ) 4xPn ( x ) Pn1 ( x ) + 4Pn21 ( x ).

,
,
` cua da
7.3. ao
thu c
. ham

169

,,
,,
,
Tuong du
ong vo i
Pn2 ( x ) + x2 4 = Pn1 ( x )( xPn ( x ) Pn1 ( x )) = Pn1 ( x ) Pn+1 ( x ).
(7.7)
,
,
,
,
` phuong phap
quy nap.
Ta se chu ng minh (7.7) bang
. , Vo i n = 1 th`
,,
2
`

P2 ( x ) = x 2, nn d dang
thy (7.7) dung.
Gia su (7.7) dung
,
,
voi n = k, tu c la`
Pk2 ( x ) + x2 4 = Pk1 ( x ) Pk+1 ( x ).
,
,
,

Ta phai chu ng minh (7.7) dung


vo i n = k + 1. Ta co

(7.8)

xPk+1 ( x ) Pk ( x ) = xPk ( x ) Pk+1 ( x )

( Pk+2 ( x ) + Pk ( x )) Pk ( x ) = ( Pk+1 ( x ) + Pk1 ( x )) Pk+1 ( x )


Pk2 ( x ) + Pk ( x ) Pk+2 ( x ) = Pk2+1 ( x ) + Pk1 ( x ) Pk+1 ( x )
Pk2+1 ( x ) = Pk2 ( x ) Pk1 ( x ) Pk+1 ( x )) + Pk ( x ) Pk+2 ( x ).
`, (7.8) suy ra
Tu
Pk2+1 ( x ) = ( x2 + 4) Pk ( x ) Pk+2 ( x ).
,
d
o la` di
`u c`n chu ng minh.

,
,
c
` cua da
7.3. ao
thu
. ham

,
Cho da
thu c
P ( x ) = a 0 x n + a 1 x n 1 + + a n , ( a 0 6 = 0 ).
,
a thu c
P0 ( x ) = na0 x n1 + (n 1) a1 x n2 + + 2an2 x + an1 , ( a0 6= 0)
,
,
,
` cua dao
goi
` bc
ham
thu c P( x ). ao
. la` dao
.
.
. ham
. nht cua da
,
0 ( x ) goi la

` bc
`
`
`

ham
nh
t
P
d
ao
h
am
b
c
hai
c
ua
P
(
x
)
v
a
k
y
hi
u

.
.
.
,.
, .
`
la` P00 ( x ). Ta co th dinh
ngh

a
theo
quy
n
ap:
d
ao
h
am
b
c
k
c
ua

.
.
.
.

,,
,
Chuong 7. a thu c

170

,
,
,
` cua dao
` bc
` P( x ) va`
da
thu c P( x ) la` dao
. ham
. ham
. k 1 cua ham
(
k
)
(
k
)
(
k

1
)
0
` P (x) = (P
. la,
ky hiu
( x )) .
. la` P ( x ). Hoac
,
,
,
` dinh
` cua da
tnh cht
Tu
ngha dao
thu c ta d thy cac
.
. ham

sau dung:
,
,
0
1. Nu bc
. cua P( x ) la` n, th` bc
. cua P ( x ) la` n 1 va`
P(n+1) ( x ) = 0.
,
,ng
phep tnh d
`
2. Cac
nu P( x ) va` Q( x ) la` nhu
i vo i dao
. ham:
,
` c`on la` mt
` th`
da
thu c bt ky,
. s bt ky,
a) ( P( x ) Q( x ))0 = P0 ( x ) Q0 ( x ); (P( x ))0 = P0 ( x );
b) ( P( x ).Q( x ))0 = P0 ( x ) Q( x ) + P( x ) Q0 ( x ).
V du. 7.12. Chu,ng minh rang
`
(( P( x ))n )0 = n( P( x ))n1 .P0 ( x ).
,
,
Lo` i giai. 1) Nu n = 1 th` P0 ( x ) = P0 ( x ).( P( x ))0 .
, ,,
2) Gia su

(( P( x ))n1 )0 = (n 1)( P( x ))n2 .P0 ( x ).


Khi d
o theo tnh cht b) ta co

(( P( x ))n )0 = ( P( x ))n1 .P0 ( x ) + (n 1)( P( x ))n2 .P0 ( x ).P( x )


= n( P( x ))n1 .P0 ( x ).

,
V du. 7.13. Chu,ng minh rang
` nu P( x ) la` da
thu c bt k`y bc
. n,
c`on a la` mt
. s bt k`y, th`
P00 ( a)
P(n) ( a )
P0 ( a)
( x a) +
( x a )2 + +
( x a)n .
1!
2!
n!
(Cng thu,c Taylor).
P( x ) = P( a) +

,
,
,
,
,
` quy nap
Lo` i giai. Chu ng minh bang
thu c P( x ).
. theo n bc
. cua da

,
,
` cua da
7.3. ao
171
thu c
. ham
, ,,
1) Nu n = 1, gia su P( x ) = A0 + A1 ( x a). Khi d
o P 0 ( x ) =
,
0
A1 ( x a)0 = A1 va` nhu vy
. P( a) = A0 va` P, ( a) = A1 , suy cng
,,
,
,
,

thu c Taylor dung


cho da
thu c bc
. nht. Gia su cng thu c dung
,
,
cho da
thu c bc
thu c bc
. n 1. Nu P( x ) la` da
. n va` nu
P ( x ) = A 0 + A 1 ( x a ) + + A n 1 ( x a ) n 1 + A n ( x a ) n .
,
Khi d
o Q( x ) = A0 + A1 ( x a) + + An1 ( x a)n1 la` da
thu c
,
bc
. n 1 va` theo gia thit quy nap
.
A0 = Q ( a ), A1 =

Q0 ( a)
Q ( n 1) ( a )
, . . . , A n 1 =
.
1!
( n 1) !

` ra ta c`on co
Ngoai
P (i ) ( x ) = Q (i ) ( x ) + n ( n 1) . . . ( n i + 1) A n ( x a ) n i
,
`, d
tu
o suy ra vo i i < n: P(i) ( a) = Q(i) ( a) va` P(n) ( a) = n!An . Cui
,,
`

cung
chung
ta nhn
o. c:
. du
P0 ( a)
P ( n 1) ( a )
P(n) ( a )
, . . . , A n 1 =
An =
.
A0 = P ( a ), A1 =
1!
( n 1) !
n!
,
,

Suy ra cng thu c Taylor dung


vo i moi
. gia tri. n.

V du. 7.14. Chu,ng minh rang


`
nu P( x ) va` Q( x ) la` nhu ,ng da

,
,

thuc bt k`y va` k la` mt


. s tu. nhin, th`

( P( x ).Q( x ))(k) = Ck0 P(k) ( x ).Q( x ) + Ck1 P(k1) ( x ) Q0 ( x )+


+ Ckk P( x ) Q(k) ( x ).
(Cng thu,c Leibniz).
,
,
,
,,
`
quy nap
Lo` i giai. Chu ng minh bang
phuong phap
. theo k.
,
1) Vo i k = 1 ta co
( P.Q)0 = P0 .Q + PQ0 = C10 P0 Q + C11 P.Q0 .
,
,
Cng thu c dung
vo i k = 1.

,,
,
Chuong 7. a thu c

172

, ,,
,
,
,
,
vo i s tu. nhin k. Khi d
2) Gia su cng thu c dung
o vo i k + 1
,,
ta nhn
o. c
. du

( P.Q)(k+1) = (( P.Q)(k) )0 = (Ck0 P(k) .Q + + Cks P(ks) .Q(s) + )0


= Ck0 ( P(k+1) .Q + Pk Q0 ) + Ck1 ( P(k) .Q0 + P(k1) .Q00 ) +
+ Cks ( Pks+1 Q(s) + P(ks) Q(s+1) ) + + Ckk ( P0 Q(k) + PQ(k+1)
= Ck0 P(k+1) .g + (Ck0 + Ck1 ) P(k) .Q0 + . . .
+ (Cks1 + Cks ) P(k+1s .Q(s) + + Ckk P.Q(k+1)
= Ck0+1 P(k+1) .g + Ck1+1 P(k) .Q0 +
+1
( k +1)
+ Cks+1 P(k+1s) .Q(s) + + Ckk+
.
1 PQ
,
,
,
,
Nhu vy
vo i moi
. cng thu c dung
. s tu. nhin k.

,
c Chebychev
7.4. a thu
,
,
, vai
` ta xet mt
. bit
Trong ph`n nay
dac
thu c giu
. dang
.
. cua da
,
` toan
v` ly thuyt cung

tr`o rt quan trong


trong nhi`u bai
nhu
.
ky thut.
.
, ,
,
nhu, da
V du. 7.15. Ham
` s cos n, (n N ) vo,i th biu din
thu c
,
bc
`
. n cua cos . Ngha la,
n

cos n =

ani cosi , a0 6= 0.

(7.9)

i =0

,
,
,
,

Lo` i giai. 1) n = 0 va` n = 2 mnh


d
Vo i n = 2
` hin nhin dung.
.
,,
,
,,
,
va` n = 3 ta nhn
o. c da
thu c tuong u ng bc
. du
. hai, bc
. ba theo
,
,
,

cng thu c luo. ng giac.


cos 2 = 2 cos2 1,
cos 3 = 4 cos3 3 cos .

,
7.4. a thu c Chebychev

173

, ,,
,

d
2) Gia su mnh
vo i n 1 va` n, ngha la`
` dung
.
n 1

bn1i cosi , b0 6= 0.

cos(n 1) =

(7.10)

i =0
n

cos n =

cni cosi , c0 6= 0.

i =0

,
,
,,
,

Ta se chu ng minh trong truo` ng ho. p d


co th
o cos(n + 1) cung
,
, ,
nhu, da
biu din
thu c cua cos co bc
. n + 1.
n +1

cos(n + 1) =

dn+1i cosi , d0 6= 0.

(7.11)

i =0

,
dung
Ta ap
cng thu c
.
cos n = 2 cos cos(n 1) cos(n 2),
,
`, (7.11) va` (7.10) suy ra

dung
vo i moi
. n va` . Tu
cos(n + 1) = 2 cos cos n cos(n 1)
n

= 2 cos cni cosi


i =0
n +1

n 1

bn1i cosi

i =0

+ d1 cos +
,
,
Ta nhn
la` da
thu c bc
. , thy ngay dy
. n + 1 cua cos, V` d0 2c0 6= 0
theo gia thit quy nap.
.
,
, ,
,
Trong da
mt
thu c (7.9) cua cos, ta co th du
a v` dang

.
. da
,
,
,

`
`
dat
. x = cos va` ta ky hiu
bang

thu c chun tac


cach
thu c nay
. da

= d0 cos

nay
`
la` Tn ( x ). Theo cach
Tn ( x ) =

a n i x i .

(7.12)

i =0

,
,
,
,
,
a thu c (7.12) goi
thu c thu n-cua Chebychev. Nhu vy
. la` da
. do

,,
,
Chuong 7. a thu c

174

,
,
,
,
cng thu c (7.9) th` da
thu c thu n cua Chebychev Tn ta co:
T0 ( x ) = 1, T1 ( x ),
Tn ( x ) = 2xTn1 ( x ) Tn2 ( x ), (n = 2, 3, . . .).
,
,
,,
,,
, ,
`, dang

Tu
thu c trn ta t`m du
da
o. c l`n luo. t cac
thu c cua Cheby,
chev vo i n = 2, 3, . . ..
T0 ( x ) = 1,
T1 ( x ) = x,
T2 ( x ) = 2x2 1,
T3 ( x ) = 4x3 3x,
T4 ( x ) = 8x4 8x2 + 1,
T5 ( x ) = 16x5 20x3 + 5x,
......
,
,
,
,
V du. 7.16. Cho Tn la` da
thu c thu n cua Chebychev. Chu ng minh
,,
mnh
d
` sau: H. s o d
i s co s mu cao nht la` 2n1 (n > 0).
.
,
,
,
, ,, `
,,
,
`, cac
dang

Lo` i giai. Tu
thu c o bai
truo c mnh
d
vo i n =
` dung
.
, ,,
,
,

` d
1, 2, 3, 4, 5. Gia su mnh
d
vo i s n nao
` dung
o. Ta chu ng minh
.
`
`, cng thu,c

suy ra tu
dung
cho n + 1 bang
cach
Tn+1 ( x ) = 2xT( x ) Tn1 ( x )
,
,

h. s cua da
va` nguyn ly so sanh
cac
thu c.

` tp
7.5. Baii
.

, ,
,
. 7.17. Cho n la` s tu., nhin va` P( x ) la` da
nho hon n.
thu c bc
.
,
,
`
,u ty R( x ) sao cho
` hu
Chu ng minh rang
t`n tai
. mt
. ham
P( x ) = x ( x + 1) . . . ( x + n)( R( x + 1) R( x )).

` tp
7.5. Baii
.

175

,
,ng da
. 7.18. Ky hiu
thu c
. P0 ( x ), P1 ( x ), P2 ( x ), . . . , Pn ( x ) la` nhu
x ( x 1)
,...
P0 ( x ) = 1, P1 ( x ) = x, P2 ( x ) =
1.2
x ( x 1)( x 2) . . . ( x n + 1)
.
Pn ( x ) =
1.2.3 . . . n
, ,
,
,
`
du,o,i
Chu ng minh rang
moi
thu c P( x ) bc
. da
. n co th biu din
dang
.
P( x ) = b0 P0 ( x ) + b1 P1 ( x ) + + bn Pn ( x ),
,,
,ng s nao
` d
o dy
b0 , b1 , . . . , bn la` nhu
o.
s Fibonacci u1 = 1, u2 = 1, ui+2 = ui+1 + ui , dat
.
. 7.19. Cho day
n
,
`
F (x) =
u xi . Chu ng minh rang
n

i =1

u n x n +2 + u n +1 x n +1 x
, ( x 2 + x 1 6 = 0)
x2 + x 1
, ,
sin(n + 1)
`
nhu,
`
. 7.20. Chu,ng minh rang
ham
co th biu din
sin

,
,
,
,
,
da
thu c Un bc
thu c loai
. n cua cos (goi
. la` da
. hai bc
. thu n cua
Chebychev).
,
`
,ng da
. 7.21. Chu,ng minh rang
nhu
thu c loai
. hai Chebychev
,
,
,
,
ng dang
nhu

thoa man
thu c sau:
Fn ( x ) =

U0 ( x ) = 1, U1 ( x ) = 2x, Un+1 ( x ) = 2xUn ( x ) Un1 ( x ).


,
,
. 7.22. Cho da
thu c P( x ) = a0 + a1 x + + an x n bc
. n vo i h.
,
,
,
`
t nht mt
s thu. c va` a 3 la` mt
. s thu. c. Chu ng minh rang
.
2
n
+
1
s |1 P(0)|, | a P(1)|, | a P(2)|, . . . , | a
trong cac
P(n + 1)|
, ,
khng nho hon 1.
,
,
`
. 7.23. Chu,ng minh rang
vo i da
thu c Pn ( x ) = x n + x n2 + x n4 +
,
,
,

Bt dang
thu c sau dung
vo i moi
. x > 0 va` n = 1, 2, . . .
1
1
Pn ( x ) + Pn ( ) n + 1 + (1 + (1)n ).
x
2

, ,
CHUONG

,
, ,
,

`
T HO
P
V
A

ANG
TH
U
C
.
, ,
,
8.1. Mt
uc t ho. p . . . . . . . . . . . . . . . . . . . . . . . . . . . .
. s cng th
,
,
ng th
uc . . . . . . . . . . . . . . . . . . . . . . . . . . . . . . . . . . . .
8.2. Mt
. s da
` tp
8.3. Bai
. ...............................................

176
186
193

,
,,
,
` ta chu,ng minh mt

Trong chuong nay


s dang
thu c va` dinh
.
.
,
,
,
,,

ly lin quan d
thu c, c gang
chu ng minh bang
phuong
n dang
,
gon
quy nap
hoc.
dung
phap
nguyn ly quy
. toan
. ngan
. khi ap
.
,
,
,
,
,
hoc
` l`n luo. t cac
buo c mt

nap
tu. nhin,
. toan
. ta tr`nh bay
. cach
,
,,
, , ,
khng nhn manh
nhu cac
chuong truo c nu
a.
.

, ,
,

8.1. Mt
s
cng
th
u
c
t
ho. p
.
,
,u han
` ta quan tm to,i tp
Trong muc
. nay
. ho. p g`m hu
.
,,
,
,
`
`
ph`n tu, v du. nhu, tp
cac
g
m
n
ph
n
t
u
k
y
hi
u
X
=
.
.
,
tp
` chung

{ a1 , a2 , . . . , an }. Khi xem xet cac


ta quan tm to i
. nay
,
,
,,
xp cua cac
ph`n tu,. Khi d
vi. tr sap
o ta noi tp
o. c
. X la` tp
. du
,
Nhu
xp
,ng bai
` toan
t ho.,p quan tm to,i s lu,o.,ng cach
sap
sap.
,
,
,ng ph`n tu, trong mt
,u han.

nhu
Chung
ta quan tm
., tp
. ho. p hu
,
, ,.
,
,
,
,
ng dang
` toan
t ho. p nhu sau:
toi nhu
co ban cua bai
.
,
,,
,
xp theo
n ph`n tu khac
nhau cua tp
Mt
. day
. ho. p, X sap
,
,
,
,
mt
du
vi. cua X.
o. c goi
.
. thu tu. nht dinh
. la` mt
. hoan
,
,
,,
,,
vi. cua n ph`n tu. Ta co th xet mt
Goi
. Pn la` s hoan
. s truo` ng

, , ,
8.1. Mt
. s cng thu c t ho. p
,
,
ho. p cu. th sau

177

,
hoan
vi. cua X
vi.
Cac
S hoan

1 = 0!
( a)
1 = 1!
( a1 , a2 ); ( a2 , a1 )
2 = 2!
( a1 , a2 , a3 ); ( a1 , a3 , a2 );
6 = 3!
( a2 , a1 , a3 ); ( a2 , a3 , a1 );
( a3 , a1 , a2 ); ( a3 , a2 , a1 );
...
...
...
...
,,
,
,
,
quy nap
hoc
Vo i phuong phap
an va` chu ng minh
. toan
. ta du. do
,
,
,
vi. cua n ph`n tu, co th tnh bang
` cng
V du. 8.1. S lu,o.,ng hoan
,
thu c
Pn = n!.
(8.1)
,
,
,
,
,,
`
quy
Lo` i giai. Ta chu ng minh cng thu c (8.1) bang
phuong phap
hoc:
nap
. toan
.
,
,
,

1) Theo bang trn cng thu c (8.1) dung


vo i n = 1.
, ,,
,
,

vi. cua k + 1 ph`n


2) Gia su (8.1) dung
vo i n = k 1. Hoan
,
,,
,
,
ph`n tu,,
tu co th lp
vi. tr thu nht cho mi
.
. nhu sau: c dinh
k ph`n tu,, c`on lai

` cac
vi. tr
(ngha la` co k + 1 cach)
r`i sap
. vao
,

tip theo (theo gia thit co Pk cach).


Do d
o
n
0
1
2
3

Tp
. X

{ a}
{ a1 , a2 }
{ a1 , a2 , a3 }

Pk+1 = (k + 1) Pk = (k + 1)k! = (k + 1)!


,
,
,

Nhu vy
vo i n = k + 1.
. cng thu c (8.1) dung
,
,
,

m ph`n tu, khac


nhau (m n) cua tp
Mt
ho. p X sap
. day
.
,
,
,
,
,
,
dinh
xp theo mt
du
o. c goi
.
. thu tu. xac
. la` mt
. chnh ho. p chp
.
,
,,
`
m cua n phn tu trong X.
,
,
,,
,,
m
chnh ho.,p chp
Ky hiu
. An la` s luo. ng cac
. m cua n ph`n tu.
Ta xet mt
. s v du. sau

178
m
1
2

3
...

,
, ,
,,
,

Chuong 8. T ho. p va` dang


thu c
,
,
,
chnh ho.,p cua X = { a1 , a2 , a3 , a4 } S chnh ho.,p
Cac
( a1 ); ( a2 ); ( a3 ); ( a4 );
4=4
( a1 , a2 ); ( a2 , a1 ); ( a1 , a3 ); ( a3 , a1 );
12 = 4.3
( a1 , a4 ); ( a4 , a1 ); ( a2 , a3 ); ( a3 , a2 );
( a2 , a4 ); ( a4 , a2 ); ( a3 , a4 ); ( a4 , a3 );
( a1 , a2 , a3 ); ( a1 , a2 , a4 ); ( a2 , a1 , a4 )
24 = 4.3.2
( a1 , a3 , a4 ); ( a2 , a3 , a4 ); ( a3 , a1 , a4 )
...
...

,
,
Ta chu ng minh cng thu c sau:
,
,
,, , ,
V du. 8.2. S lu,o.,ng chnh ho.,p chp
o. c tnh
. m cua n ph`n tu du
,

theo cng thuc sau:


Am
n = n ( n 1) . . . ( n m + 1).

(8.2)

,
,
,
`
hoc
Lo` i giai. Chu ng minh bang
quy nap
. toan
.
,
,
,
1

1) Voi m = 1 ta co An = n, suy ra cng thu c (8.2) dung


vo i
m = 1.
, ,,
,

2) Gia su (8.2) dung


vo i m = k 1, ngha la`
Akn = n(n 1) . . . (n k + 1)
,
,
, , `,
,ng chnh ho.,p chp
chnh ho.,p chp
Cac
nhu
o. c tu
. k + 1 nhn
. du
.
,,
`

`
thm vao
` cui day
mt

k bang
cach
. trong n , k phn tu c`on lai.
.
,
,
,
,
Nhu vy
. mt
. chnh ho. p chp
. k se cho n k chnh ho. p chp
. k + 1.
Do d
o
Akn+1 = (n k ) Akn = n(n 1)(n 2) . . . (n k + 1)(n k )
,

Suy ra (8.2) dung


vo i m = k + 1.
,
y:
Co th vit cng thu,c (8.2) du,o,i dang

Chu
khac
.
Am
n =

n!
.
(n m)!

(8.3)

, , ,
8.1. Mt
. s cng thu c t ho. p

179

,
,,
,,
tp
nhau cua tp
Mi
X (m n) du
o. c goi
. con m ph`n tu khac
.
.
, ,
,
,, ,
`
la` t ho. p chp
. m cua n phn tu cua X.
m
m
ph`n tru,o,c da ky hiu
. nhu, cac
Goi
. Cn (hoac
. la` Cn ). Ta xet
mt
. s v du. sau
,
,
,
t ho.,p cua X = { a1 , a2 , a3 , a4 } S t ho.,p
m Cac
1
( a1 ); ( a2 ); ( a3 ); ( a4 );
4
2
( a1 , a2 ); ( a1 , a3 ); ( a1 , a4 );
6
( a2 , a3 ); ( a2 , a4 ); ( a3 , a4 );
3
( a1 , a2 , a3 ); ( a1 , a2 , a4 );
4
( a1 , a3 , a4 ); ( a2 , a3 , a4 );
4
( a1 , a2 , a3 , a4 );
1
...
...
...
,
,
,,

chnh ho.,p chp


Chung
ta nhn
la` cac
m cua n ph`n tu
. ra ngay
.
, ,
,
, , `,
`

vi. m ph`n tu,


nhn
cac t ho. p chp
cach
hoan
o. c tu
. du
. m bang
` V` vy
nay.
. ta co lin h. sau
m
Am
n = Cn .Pm .

`, d
Tu
o suy ra

Am
n!
.
Cnm = n =
P
m!
(
n
m)!
m
,
,
,,
,
quy nap
chu ng minh theo phuong phap
. ta chu ng minh
,
,
,,
V du. 8.3. S lu,o.,ng t ho.,p chp
o. c tnh theo cng
. m cua n du
thu,c sau
n ( n 1) . . . ( n m + 1)
Cnm =
.
(8.4)
1.2 . . . m
,
,
,
,
`
Lo` i giai. 1) Ta chu y rang
Cn1 = n, ngha la` vo i m = 1 cng thu c

dung.
, ,,
2) Gia su ta co
n ( n 1) . . . ( n k + 1)
Cnk =
.
1.2 . . . k

,
, ,
,,
,

Chuong 8. T ho. p va` dang


thu c

180

,
`
Ta se chu ng minh rang
n(n 1) . . . (n k + 1)(n k )
.
Cnk+1 =
1.2 . . . k (k + 1)
,
, , ,
,,
,,
nhn
n ph`n tu: d
`u tin
. tt ca t h, o. p k +,1 ph`n tu trong
,
,,
,`,
,
t ho. p chp
nguoi ta vit tt ca cac
. k cua n ph`n tu va` thm
, ,
,,
,,
,

` mi t ho. p nay
` mt
vao
t trong n k
. ph`n tu thu k + 1 b,oi m
, .,
,,
,
,
,
ph`n tu c`on lai.
o. c tt ca t ho. p chp
. Nhu vy
. ta nhn
. du
. k+1
,
,,
,
,
,
cua n ph`n tu, nhung se nhn
o. c bi
. du
. k + 1 l`n. Tht
. vy,
. ,
, ,
,
,
`

t ho. p a1 , a2 , . . . , ak , ak+1 se cung


nhn
du
o
c
theo
c
ach;
khi
t

.
.
, ,
,,
,
,
`
` phn tu a1 ; cung

ho. p a2 , a3 , . . . , ak , ak+1 thm vao


nhu khi t ho. p
, ,
,,
`

` phn tu a2 ; . . . ; cui cung


`
a1 , a3 , . . . , ak , ak+1 thm vao
khi t ho. p
` ak+1 . Ngha la`
a1 , a2 , a3 , . . . , ak thm vao

n(n 1) . . . (n k + 1)(n k )
mk
=
.
k+1
1.2 . . . k (k + 1)
,
V du. 8.4. Chu,ng minh rang
` h. s Newton Cnk la` nhu ,ng s le khi
,
va` ch khi n = 2s 1.
,
, ,,
,
,
,
,

Lo` i giai. Vo i n 8 mnh


d
Gia su
` khim tra tru. c tip dung.
.
,
, ,,
,
,

n la` s tu. nhin bt ky` va` gia su mnh


d
dinh
dung
vo i
` khang
.
.
, ,
,
,
moi
thu c la`
. s tu. nhin nho hon n. D thy h. s da
Cnk = Cnk

n n(n 1) n(n 1)(n 2)


n(n 1)(n 2) . . . 3.2
,
,
,...,
,
1
1.2
1.2.3
1.2.3 . . . (n 1)
,
,
,
,
` cung
`
tt ca la` s le khi va` ch khi h. s da
(ma` no
thu c ngoai
,
,
,
,
,
,
`
ng s nhn
` cac
h. s da
bang
n) la` s le va` nhu
o. c tu
thu c c`on
. du
,
,
,
,
,
` cach
s va` tu, s, cung
`,a s le o, mu

bo di
thu
la` le. Ta
lai
cac
. bang
,
. n = 2m + 1. Trong tru,o`,ng ho.,p nhu, vy
dat
thu c khng
. h. s da
,
bang
`
s trong day

co s cui biu din


cac
m m(m 1) m(m 1)(m 2)
m(m 1)(m 2) . . . 3.2
,
,
,...,
.
1
1.2
1.2.3
1.2.3 . . . (m 1)

, , ,
8.1. Mt
181
. s cng thu c t ho. p
,
,
`, dy
,ng h. s da
hoc
Tu
toan
nhu
theo gia thit quy nap
thu c cui
.
.
,
,
,
,
`
cung,
c`on suy ra tt ca h. s da
thu c se la` le khi va` ch khi m co
dang
2k 1 ngha la` khi d
o n2(2k 1) + 1 = 2k+1 1.
.

,
V du. 8.5. Chu,ng minh rang
`
tu`, cac
chu , s 1 va` 2 ta co th lp
.
,
,
n
+
1
n

2
s ma` mi s d
chu ,
u co 2 chu s va` cu hai s mt
. th` cac
,
s o, hang
`
tu,o,ng u,ng (vi. tr tu,o,ng u,ng) khac
nhau khng t ho,n
2n 1 .
,
,
,
,,
,
`
Lo` i giai. Ta chu ng minh bang
quy nap
theo n. Vo i n = 1 ta du
o. c
.
,
mnh
bn s 11; 21; 12; va` 22 thoa man
d
`.
.
,,
0
`
, s 1 bang
, s 2 va`
Goi
chu
o. c khi thay trong a chu
. a la` s co du
, ,,
`
`
thay 2 bang
1 va` ab la` s tao
khi vit b canh
a. Gia su da
. thanh
.
,
,,
,
s co 2n chu
`, 2n+1 s, mi
, s, ngoai
`
xy du. ng du
o. c tp
. ho. p An tu
,
,
n

1
nhau khng t hon 2
`
ra cu hai s mt
vi. tr hang
s.
. khac
,
,
s aa va` aa0 trong d
Xet tp
o a An . Tt ca
. ho. p An+1 g`m cac
,
,ng s nay
, s va` tt ca co 2n+2 s. Ngoai
` co 2n+1 chu
` ra bt cu,
nhu
` d
nhau khng t ho,n 2n hang
`

hai s nao
s. Tht
`u khac
. vy,
. cac
,
,
0
0

s aa va` bb vo i moi
nhau
s aa va` aa , cung
nhu cac
`u khac
. a, b d
n

`
hang
`
` a va` b khac
nhau, a0 va`
dung
2 hang
s (trong cac
s nay
,
,,
`
s aa va` bb theo gia thit quy
b0 trung
nhau, va` nguo. c lai).
. Cac
`
nhau khng t ho,n 2n hang
nap
s.
. khac

V du. 8.6. Vo,i cac


s nguyn m, n (0 m n) xy du. ,ng cac
s
,
d(n, m) theo cng thu c sau:
1) d(n, 0) = d(n, m) = 1, vo,i moi
. n 0;
2) m.d(n, m) = m.d(n 1, m) + (2n m).d(n 1, m 1), vo,i moi
.
0 < m < n.
,
Chu,ng minh rang
` tt ca cac
s d(n, m) d
`u nguyn.

,
, ,
,,
,

Chuong 8. T ho. p va` dang


thu c

182

,
,
,
`
`
quy nap
d(n, m) = (Cnm )2 .
Lo` i giai. Ta se chu ng minh bang
. rang
,
Tht
. vy,
. voi n = 1 th`
2
2
d(1, 0) = 1 = C10 ; d(1, 1) = 1 = C10 .
, ,,
,
,
` toan
dung

Gia su bai
vo i n = k: d(k, m) = (Ckm )2 , vo i moi
. 0m
,
k. Khi d
o voi n = k + 1, ta co
1) Nu 0 < m < k + 1 th`
m.d(k + 1,m) = m.d(k, m) + [2(k + 1) m]d(k, m 1)

2
= m. (Ckm )2 + (2k + 2 m) Ckm1

2
k!
=
[m(k + 1 m)2 + (2k + 2 m)m
m!(k + 1 m)!
2
= m. Ckm+1 .
. k + 1, th` d(k + 1, 0) = 1 = (Ck0+1 )2 va`
2) Nu m = 0 hoac
+1 2

d(k + 1, k + 1) = 1 = (Ckk+
. theo nguyn ly quy nap
. toan
1 ) . Vy
hoc,
. ta co
d(n, m) = (Cnm )2 , 0 m n.

V du. 8.7. Cho s lu,o.,ng 3n d


`ng xu ( n 1 ), mt
`ng xu trong
. d
,
,
d
o la` gia va` nhe. hon s c`on lai.
a khng co
. Cho mt
. chic cn d
,
,
,
`
`
`
qua cn. Chu ng minh rang

bang

n ln cn co th phat
hin
. ra
,
,
`
`
`
`
d

n ln cn lun lun phat


hin
ng tin gia. Co th bang
`ng
. ra d
,
, , n
,
,
ti`n gia hay khng, nu s luo. ng d
`ng xu khng nho hon 3 + 1?
,
,
,,
,
bn
. hai d
Lo` i giai. Xet truo` ng ho. p n = 1. Ta dat
`ng xu ln mi
,
`

mt
bang,
th` d
a cn. Nu cn thang
`ng xu gia la` d
`ng xu c`on
. d
,
,,
`
`
` cn, c`on trn cn khng cn bang,
la. o ngoai
d
`ng ti`n gia nam
,,
,
` nhe. hon.
o bn nao

, , ,
8.1. Mt
183
. s cng thu c t ho. p
,
, ,,
,
,
,
` toan
dung

vo i n 1. By gio` ta
Ta gia su khang
dinh
cua bai
.

` ba nhom theo 3n1 d

co 3n . Ta chia chung
ra lam
`ng xu va` dat
, . ,,
,
`

`
` ng d

hai nhom ln tu
th` d
a cn. Nu cn cn bang,
ng xu gia o
,,
,
,,
,

nhom thu 3, c`on nguo. c lai


th` o nhom trn d
a cn nhe. hon. Va`
.
,
,
,
,,
,
` toan
suy ra theo quy nap.

ca hai truo` ng ho. p khang


dinh
cua bai
.
.
,
,
,
,
,
n
`
Ta se chu ng minh rang
nu s luo. ng d
`ng xu lo n hon 3 , th`
,
,
`
nao
` cung

hin
khng phai luc
phat
n l`n cn
`ng xu gia bang
. ra d
,
,
,
va` thm
`ng xu tht
. s d
. tr trong truo` ng ho. p da bit mt
. r`i.
,
,
,
,,
,

Truo` ng ho. p n = 1, khang


dinh
trn dung
hin nhin. Gia
.
,
,
,,
,
,

su khang
dinh
dung
vo i tt ca s tu. nhin k (k n 1) va` xet
.
,
,,
,
,
d
truo` ng ho. p k = n. Ta ky hiu
F tp
ho. p tt ca cac
`ng xu da
.
.
,
,,
,
`
d

cho, c`on J tp
s luo. ng
`ng xu tht.
. ho. p tt ca cac
. Chu y rang
,,
,
,
` co
d
`ng xu trong F lo n hon 3n , c`on s luo. ng trong J la` bt ky,
,
,

`
`

. s d
th la` 0. D thy rang
trn hai d
a cn phai dat
`ng xu bang
,
`

` hai d
. ngoai
nhau. Bang
cach
nhu vy
a cn hoac
a
. trn mt
. d
,
,
n

1
` F. Ta ky hiu
cn co mt
d
`ng xu tu
. nhom nhi`u hon 3
. nhom
`

d
N.
o bang
` hai d
Nu N ngoai
a cn, th` theo di
`u kin hai d
a cn cn
, ,,
,
`
`

bang,
d
ng xu gia o nhom N va` theo gia thit quy nap
. no khng
,
, ,,
`
`
`
`
d

th tach
n 1 ln cn.
ng tin gia du
o. c bang
Nu nhom N la` mt
trong hai d
a cn va` d
o la` d
a cn nhe.
,
, .
,
` trong no, khi d
hon, th` d
`ng xu gia nam
o theo gia thit quy nap
.
,
,
,

du
cung
khng tach
o. c d
`ng xu gia.
,
V du. 8.8. Cho bang h`nh vung cac
s
a11 a12 . . . a1n
a21 a22 . . . a2n
..
..
..
.
.
...
.

an1

an2

...

ann .

,
, ,
,,
,

Chuong 8. T ho. p va` dang


thu c

184

Chu,ng minh rang


` nu M la` mt
` s sao cho
. hang
n

|x1 a j1 + x2 a j2 + + xn a jn | M

j =1

,
vo,i moi
chon
. cach
. nhu ng s xi = 1, th`

| a11 | + | a22 | + + | ann | M.


,
,
,
,
`
Lo` i giai. Ta chu ng minh mnh
d
quy nap
` bang
. theo n. Vo i
, . ,,
,

n = 1 mnh
d
Gia su mnh
d
vo i n 1. Cho
` dung.
` dung
.
.
,

`
x1 , x2 , . . . , xn1 la` mt
chon
o ta dung
. cach
. bt ky` cua 1. Khi d
,
,

bt dang
thu c sau
2|| = |2| = |( + ) + ( )| | + | + | |,
,,
ta nhn
o. c
. du
n 1

|x1 a j1 + + xn1 a j,n1 | + 2|ann |

j =1

n 1

|x1 a j1 + + xn1 a j,n1 + ann |+

j =1

n 1

|x1 a j1 + + xn1 a j,n1 ann |+

j =1

+ | ann + x1 a j1 + + xn1 a j,n1 |+


+ | ann x1 a j1 xn1 a j,n1 |
n 1

|x1 a j1 + + xn1 a j,n1 + ann |+

j =1

n 1

|x1 a j1 + + xn1 a j,n1 ann | 2M.

j =1

, , ,
8.1. Mt
. s cng thu c t ho. p
,
Tu c la`

185

n 1

|x1 a j1 + + xn1 a j,n1 | M |ann |,

j =1

,
`, dy
tu
theo gia thit quy nap
. suy ra

| a11 | + | a22 | + + | an1,n1 | M | ann |,


ngha la` | a11 | + | a22 | + + | ann | M.

V du. 8.9. Cho cac


s tu. , nhin a1 , a2 , . . . , an (n > 1), sao cho ak
,
k, (k = 1, 2, . . . , n) va` tng a1 + a2 + + an la` chan.
Chu,ng minh
,
rang
` mt
tng dai
` 0.
. trong cac
. s a1 a2 a3 . . . an bang
,
,
,
`
Lo` i giai. Ta chu ng minh bang
quy nap
. theo n. Khi n = 2, ta d
,
,
thy a1 = a2 = 1. Do d
o a1 a2 = 0. i vo i n + 1 s tu. nhin
,
,
cac
di
` toan,
ta
a1 , a2 , . . . , an+1 (n 2) thoa man
`u kin
. cua bai
,`,
,
xet hai truong ho. p sau:
. a0n = | an an+1 |, khi d
1. an 6= an+1 . at
o do 1 an n va`
,
0
tng
. khac,
1 an+1 n + 1 nn 1 an (n + 1) 1 = n. Mat
ring an + an+1 co cung

`
a1 + a2 + + an + an+1 chan,
tnh chan
, , 0
,

le vo i a = | an an+1 |, do d
o tng a1 + a2 + + an1 + a0n cung
,
,
,
`

cac
di
` toan.
Theo gia thit quy nap
thoa man
cua bai
u kin
. ta
, .
0
tng a1 a2 . . . an1 an = a1 a2 . . .
co mt
. trong cac
,
,
`
`, d
an1 | an an+1 | bang
0. Tu
o suy ra di
`u phai chu ng minh.
va`
nay
` do a1 + a2 + + an + an+1 chan
2. an = an+1 . Luc
nn a1 + a2 + . . . + an1 cung
Vy

an + an+1 = 2an chan


chan.
.
,
u

cac
di
`

n 1 s a1 , a2 , . . . , an1 cung
thoa man
b
ai
to
an
n
`u kin
.,
,
,
` la` hin nhin v` nu
ch c`n n 1 > 1 hay n 3. Nhung di
`u nay
,
,
,
,
`
` ma` thi v` n 2),
n = 2, (ro rang
ch c`n chu y truo` ng ho. p nay
,
,
,

` gia thit a1 1, a2 2 va` a3 3 va` tng a1 + a2 + a3 chan,


th` tu
,
,
,
,
,
` truo` ng ho. p 1. da xet. Do
ta suy ra a1 = 1 va` a2 6= a3 tu c la` roi vao

,
, ,
,,
,

Chuong 8. T ho. p va` dang


thu c
, ,,
,
,
`
t nht mt
gia thit quy nap
d
d
suy ra rang
o co th su dung
.
.
.
,
,
`
biu thu c a1 a2 . . . an1 bang

trong cac
0 va` do d
o mt
. trong
,
`
tng a1 a2 . . . an1 + an an+1 cung

cac
bang
0.
186

,
,

d
a
8.2. Mt
s
ng thu
.

,
,
,
`
` ta chu,ng minh mt

Ph`n nay
dang
thu c dang
nho .
. s hang

V du. 8.10. Chu,ng minh nhi. thu,c Newton

( a + b)n =

Cni ai bni ,

(8.5)

i =0

,
,,
o, dy
n la` s nguyn duong.

,
,
,
,

vo i n = 1.
Lo` i giai. Bu,o,c co, so,: D thy (8.5) dung
, ,, ,
,
,
,

thu c (8.5) dung


vo i n, ta se chu ng
Bu,o,c quy nap:
. Gia su dang

minh no cung
dung
cho n + 1. Tht
. vy,
.

( a + b ) n +1 = ( a + b ) n ( a + b ) =
= [ an + Cn1 an1 b + + Cnk ank bk + + bn ]( a + b)
= an+1 + Cn1 an b + + Cnk an+1k bk + + abn +
+ an b + Cn1 an1 b2 + + Cnk ank bk+1 + + bn+1 .
Suy ra
h
i
h
i
( a + b)n+1 = an+1 + 1 + Cn1 an b + Cn1 + Cn2 an1 b2 +
h
i
+ Cnk1 + Cnk an+1k bk + + bn+1 .
,
,ng h. s trong cng thu,c trn rut
gon
Nhu
. theo cng thu c (2.8)
va` ta co
n +1

( a + b)n+1 = Cni +1 an+1i bi .


,
,
, i =0

Vy
thu c (8.5) dung
vo i n + 1.
. dang

,
,

8.2. Mt
thu c
. s dang

187

`
V du. 8.11. Vo,i a1 , a2 , . . . , an la` nhu ,ng s thu. ,c, chu,ng minh rang

( a1 + a2 + + an )2 = a21 + a22 + + a2n + 2( a1 a2 + a1 a3 + + an1 an )


(8.6)
,
,
vo i moi
. s tu. nhin n 2.
,
,
,
,
,
,
,
`

Lo` i giai. Vo i n = 2 cng thu c (8.6) la` hang


dang
thu dang
nho .
, ,,
,
,

Gia su cng thu c (8.6) dung


vo i n = k 1, ngha la`

( a1 + a2 + + ak1 )2 = a21 + a22 + + a2k1 + 2S


,
,
,
,
,,
`,ng di
kha nang

o dy
S la` tng tt ca cac
tu
cua day

a1 , a2 , . . . , ak1 . Ta se chung minh


( a1 + a2 + + ak )2 = a21 + a22 + + a2k + 2S1
,,
o dy
S1 = S + ( a1 + a2 + + ak1 ) ak . Tht
. vy,
.
( a1 + a2 + + ak )2 = [( a1 + a2 + + ak1 ) + ak ]2
= ( a1 + a2 + + ak1 )2 + 2( a1 + + ak1 ) ak + a2k
= ( a21 + a22 + + a2k1 ) + 2S + 2( a1 + + ak1 ) ak + a2k

= ( a21 + a22 + + a2k1 ) + 2S1 .


V du. 8.12. Cho s nguyn du,o,ng n va` s thu. ,c x, chu,ng minh
rang
`
1
2
n1
[x] + [x + ] + [x + ] + + [x +
] = [nx ].
n
n
n
,
,
,
` ra khng ro cho ta phai quy nap
Lo` i giai. Bai
s
. theo thng
,
,
,
,
,,
,
,
` y tuong d
nao.
chu ng minh la` ch ly gia tri. x trong khoang nho
k k+1
[ ,
) vo,i k = 0, 1, 2, . . .
n n
, ,,
,
` trong khoang con [0, 1 ). Khi d
`u tin gia su x nam
o [ x +
n
n 1
i
i

] = 0 vo,i i = 0, 1, 2, . . . , n 1, nhu, vy
co
[x + n ] = 0. Cung
.
n
i =0

,
, ,
,,
,

Chuong 8. T ho. p va` dang


thu c
,
,
,

[nx ] = 0, nhu, vy
cho khoang
. ta da chu ng minh kt qua dung
con d
`u tin.
,
, ,,
,
k 1 k ,,
,

By gio` ta gia su khang


dinh
dung
cho khoang [
, ), o
.
n n,
,,
,
dy
k la` s nguyn duong, va` cho x s thu. c bt ky` trong khoang
` Khi d
nay.
o

188

2
n1
1
] = [nx ].
[x] + [x + ] + [x + ] + + [x +
n
n
n
1
,,
,,
`
` x (ta lam
` du
nay
` ta nhn
Cng
thm vao
cach
du
o. c v` bang
o. c
.
.
n
,,
k k+1
s hang

ngoai
` s hang
)), mi
o bn trai
s bt ky` trong [ ,
.
.
n ,n
,
`
cui cung,
d
bn phai no, va` s hang
cui
`u chuyn sang s hang
.
.
n1
`
` [ x ]. Nhu,
cung
la` [ x +
] tu,c la` [ x + 1] thu.,c cht cng
1 vao
.
n
,
1
,
`
` v trai,
dang

vy
x + vao
thu c trn tang
ln 1.
. thay x bang
n
,
,,
1
,
`
`ng tho` i khi d
x + , gia tri. cua no
o khi x o [nx ] thay bang
n
,
,
,
bn cua dang

cung
tang
ln 1. Do mi
thu c d
ln 1 khi
`u tang
,
,
1
c`on dung
`

s trong
thay x bang
x + , kt qua vn
cho tt ca cac
n
,
k k+1
).
khoang [ ,
n n
,
,
,

Theo gia thit quy nap


kt qua c`on dung
cho tt ca gia tri.
.
,
,
,,
` toan
` tu,o,ng tu., cung

gia tri.
duong cua x. Hoan
dung
cho tt ca cac
,
m cua x.

,
V du. 8.13. Chu,ng minh rang
` vo,i moi
. s tu. nhin n 1

(n + 1)(n + 2) . . . (n + n) = 2n .1.3.5 . . . (2n 1).


,
,,
,
,,
la` Tn . Dung
`
quy nap
Lo` i giai. Ky hiu
phuong phap
. tch o v trai
.
,
,
1.

hoc

toan
theo
n.
V
o
i
n
=
1
cng
th
u
c
d
ung
v`

T
=
(
1
+
1
)
=
2

1
.

,
,

8.2. Mt
thu c
. s dang
, ,,
,
,
vo i n = k. Ta co
Gia su cng thu c dung

189

Tk = (k + 1)(k + 2) . . . (k + k ) = 2k .1.3.5 . . . (2k 1).


,
Ta c`n chu ng minh

[(k + 1) + 1][(k + 1) + 2] . . . [(k + 1) + k][(k + 1) + (k + 1)] = 2k+1 .1.3.5 . . . (2k +


. la`
Hoac
(k + 2)(k + 3) . . . (k + 1 + k)(2k + 2) = 2k+1 .1.3.5 . . . (2k + 1).
Tht
. vy,
.
Tk+1 = (k + 2)(k + 3) . . . (k + 1 + k )(2k + 2)

(k + 1)(k + 2)(k + 3)(k + k)


.(k + 1 + k )(2k + 2)
( k + 1)
(2k + 1).2(k + 1)
= Tk
.
k+1
,
V du. 8.14. Chu,ng minh dang

thu,c vo,i moi


. s nguyn n 0
=

sin 2n+1
.
2n+1 sin
,
,
sin 2
,
,
,

Lo` i giai. 1) Vo i n = 0 dang


thu c dung,
v` cos =
.
2 sin
, ,, ,
,
,
,

2) Gia su dang
thu c dung
vo i n = k, tu c la`
sin 2k+1
cos cos 2 cos 4 . . . cos 2k = k+1
.
2
sin
,

Khi d
dung
vo i n = k + 1. Tht
o no cung
. vy,
.
sin 2k+1 cos 2k+1
cos cos 2 cos 4 . . . cos 2k cos 2k+1 =
2k+1 sin
sin 2k+2
= k +2
.
2
sin
,
V du. 8.15. Chu,ng minh rang
` vo,i moi
. s tu. nhin n 1
n+1
sin
x
nx
2
sin x + sin 2x + + sin nx =
sin
.
x
2
sin
2

cos cos 2 cos 4 . . . cos 2n =

,
, ,
,,
,

Chuong 8. T ho. p va` dang


thu c

190

,
,
,
,

Lo` i giai. 1) Vo i n = 1 khang


dinh
trn la` dung.
.
2) Cho
k+1
x
kx
2
sin .
x
2
sin
2

sin
sin x + sin 2x + + sin kx =
Khi d
o

sin x + sin 2x + + sin kx + sin(k + 1) x =


k+1
sin
x
kx
2
=
sin
+ sin(k + 1) x =
x
2
sin
2
k+1
x
sin
kx
k+1
k+1
2
sin
+ 2 sin
x cos
x
=
x
2
2
2
sin
2
k+2
sin
x
k+1
2
=
sin
x,
x
2
sin
2
v`
2 cos

k+1
x
k+2
kx
x sin = sin
x sin .
2
2
2
2

,
V du. 8.16. Chu,ng minh rang
` vo,i moi
. s tu. nhin n 1
sin x + 2 sin 2x + 3 sin 3x + + n sin nx =

(n + 1) sin nx n sin(n + 1) x
.
x
4 sin2
2

,
,
,
,

Lo` i giai. 1) Vo i n = 1 khang


dinh
dung,
v`
.
2 sin x (1 cos x )
2 sin x sin 2x
=
= sin x.
x
x
4 sin2
4 sin2
2
2

,
,

8.2. Mt
thu c
. s dang
,
, ,,
,
,

2) gia su khang
dinh
dung
vo i n = k tu c la`
.
sin x + 2 sin 2x + 3 sin 3x + + k sin kx =

191

(k + 1) sin kx k sin(k + 1) x
.
x
4 sin2
2

Khi d
o
sin x + 2 sin 2x + 3 sin 3x + + k sin kx + (k + 1) sin(k + 1) x =

(k + 1) sin kx k sin(k + 1) x
+ (k + 1) sin(k + 1) x
x
4 sin2
2
(k + 1) sin kx k sin(k + 1) x + 2(k + 1) sin(k + 1) x (1 cos x )
x
4 sin2
2
(k + 2) sin(k + 1) x + (k + 1) sin kx 2(k + 1) cos x sin(k + 1) x

x
x
4 sin2
4 sin2
2
2
(k + 2) sin(k + 1) x + (k + 1) sin kx (k + 1)[sin(k + 2) x + sin kx ]

x
x
4 sin2
4 sin2
2
2
(k + 2) sin(k + 1) x (k + 1) sin(k + 2) x
.
x
4 sin2
2

V du. 8.17. Chu,ng minh rang


`
1 x
1
x
1
x
1
x
tg + 2 tg 2 + + n tg n = n cotg n cotg x
2 2 2
2
2
2
2
2
,
vo i x 6= m.
,
,
,
,

Lo` i giai. 1) Vo i n = 1 khang


dinh
dung,
v`
.
x
2 x
tg2
x
1
x 1 tg 2
1
2 = 1 tg x .
cotg cotg x = cotg
=
x
x
2
2
2
2
2 2
2 tg
2 tg
2
2

,
, ,
,,
,

Chuong 8. T ho. p va` dang


thu c

192

,
, ,,
,
,

2) Gia su khang
dinh
dung
vo i n = k, tu c la`
.
1
x
1
x
1
x
1 x
tg + 2 tg 2 + + k tg k = k cotg k cotg x.
2 2 2
2
2
2
2
2
Khi d
o
x
1
x
1
x
1 x
1
tg
+ + k tg k + k+1 tg k+1 =
tg +
2 2 22 22
2
2
2
2
1
x
1
x
= k cotg k cotg x + k+1 tg k+1
2
2
2
2
x
2
1 cotg 2k+1 1
1
= k +1
+
x
x cotg x
2
k
+
1
cotg k+1
2
cotg k+1
2
2
1
x
= k+1 cotg k+1 cotg x.
2
2
V du. 8.18. Cho a va` A > 0 la` nhu ,ng s bt k`y va` dat
.
a1 =

1
A
1
A
1
A
( a + ), a 2 = ( a 1 + ), . . . , a n = ( a n 1 +
).
2
a
2
a1
2
a n 1

Chu,ng minh rang


`

A
=
an + A
an

! 2n 1
A

,
a1 + A
a1

vo,i moi
. s nguyn n 1.
,
,
,
,
, , , ,,
,

Lo` i giai. 1) Buo c co so: D thy dang


thu c dung
vo i n = 1.
,
,
,
,,
,

2) Buo c quy nap:


thu c dung
vo i n. Ta c`n
. Gia thit dang
,
,

chu ng minh no cung


dung
vo i n + 1. Tht
. vy
.

A
1

( an + ) A
a n +1 A
a2n 2 Aan + A
an A 2
2
an
=
= a2 + 2 Aa + A = ( a + A ) .
1
A
a n +1 + A
n
n
n
( an + ) + A
2
an

` tp
8.3. Bai
.

193

,
,
Nhung theo gia thit quy nap
.
! 2n 1

a1 A
an A
=

.
an + A
a1 + A
V` vy
.

A
=
a n +1 + A

a n +1

!2
A

=
an + A

an

!2.2n1
A

=
a1 + A
a1

! 2n
A

a1 + A
a1

` tp
8.3. Bai
.
`
. 8.19. Chu,ng minh rang
cos x + 2 cos 2x + + n cos nx =

(n + 1) cos nx n cos(n + 1) x 1
.
x
4 sin2
2

`
. 8.20. Chu,ng minh rang
n

(1 + i )n = 2 2 (cos

n
n
+ i sin
).
4
4

,
s a1 , a2 , . . . va` b1 , b2 , . . . Chu,ng minh dang

. 8.21. Cho hai day


,
thu c
n

n 1

=1

=1

a b = an Bn (a+1 a ) B , (n = 2, 3, . . .),

,,
o dy
Bk =

bj , k = 1, 2, . . . , n.

j =1

,
t`m tng
. 8.22. Hay
k
k ( k 1)
k (k 1) . . . 2.1
1
+
+ (1)k
,
m + 1 (m + 1)(m + 2)
(m + 1)(m + 2) . . . (m + k)
,,
,
,
`
k la` s tu. nhin bt ky.
o dy
m la` s tu. nhin c dinh,
.

, ,
CHUONG

LIN PHN S
nim
9.1. Khai
. lin phn s . . . . . . . . . . . . . . . . . . . . . . . . . . . . . .
, ,
`
9.2. Phn tch s huu ty thanh
lin phn s . . . . . . . . . . . . .
,
9.3. Phn s xp x . . . . . . . . . . . . . . . . . . . . . . . . . . . . . . . . . . . . . . .

198

9.4. Lin phn s v han


. .................................

203

9.5. V du. . . . . . . . . . . . . . . . . . . . . . . . . . . . . . . . . . . . . . . . . . . . . . . . . .

204

` tp
9.6. Bai
. ...............................................

210

194
196

nim
9.1. Khai
. lin phn s
,
,
Mt
. biu thu c co dang
.

q0 +
q1 +

(9.1)

1
q2 +
..

.+

1
qn

,,
trong d
o q1 , q2 , . . . , qn la` s duong, c`on q0 la` s khng m, goi
. la`
,
,
,
ng s q0 , q1 , . . . , qn goi
lin phn s . Nhu
. la` ph`n thuong khng
,
,,
d
`y du (ph`n tu), c`on lin phn s

nim
9.1. Khai
. lin phn s

195
1

q0 +
q1 +

(9.2)

1
q2 +
..

.+

1
qn

, ,
,,
goi
`y du cua phn s (9.1).
. la` thuong d
,
,,
sau:
thun
o. c vit theo cach
. tin
. lin phn s (9.1) du
(9.3)
,
,,
,
`
mt
D thy rang
vo i n 1 lin phn s (9.3) biu din
s duong
.
,
` d
nao
o , goi
. la` gia tri. cua no. Ta ky hiu
.

( q0 , q1 , . . . , q n ).

= ( q0 , q1 , . . . , q n ).
, ,
ph`n tu, cua no
Cho mt
. lin phn s ( ngha la` cho cac
,
,
q0 , q1 , . . . , qn ) vo i gia tri. . Ta ky hiu
k (0 k n) ph`n d
`y du
.
,
cua (9.2). Khi d
o
1

k = qk +
q k +1 +

1
q k +2 +
..

.+

1
qn

, ,
,
`
vai tr`o nhu, ph`n khng d
Ta thy rang
k co th giu
`y du cui
,
,,
,
`
vit sau:
cung
(thu k). Boi vy
n cach
. ta co th chu y d

( q 0 , q 1 , . . . , q k 1 , q k , . . . , q n ) = ( q 0 , q 1 , . . . , q k 1 , k );

(9.4)

k = (qk , qk+1 , . . . , qn ), k = 0, 1, . . . , n.
, ,
,
,
`
D thy vo i k = 0 ph`n d
vo i lin phn s
`y du cua (9.2) trung
,
,
`
da cho, vo i k = n la` ph`n khng d
qn , ngha la`
`y du cui cung
0 = , n = qn .

,,
Chuong 9. Lin phn s

196

,
nhu, phn s b`nh thu,o`,ng.
V du:
. a) S = (1, 2, 2) biu din
Tht
. vy
.
1
2
7
= 1+ = .
5
1
5
5
2+
2
2
, ,
,
88
,
,

,ng ph`n tu,


b) S
co th biu din nhu lin phn s vo i nhu
67
nguyn. Tht
. vy
.
= 1+

= 1+

88
21
= 1+ ,
67
67
67
4
= 3+ ,
21
21
21
1
= 5+ ,
4
4

( q0 = 1),
( q1 = 3),
(q3 = 5, q4 = 4).

,
Vy
. ta co th vit
88
= (1, 3, 5, 4).
67

, ,
u ty thanh
`
9.2. Phn tch s hu
lin phn s
,, ,
,,
Theo dinh
ngha ph`n truo c nu s ph`n tu cua lin phn s
.
,
,
,u han
`
la` hu
mt
. th` ta co th chuyn lin phn s thanh
. ,phn
,
,
,
,
,
,
,
s b`nh thuo` ng. Nguo. c lai,
. mt
. phn s b`nh thuo` ng co th biu
du,o,i dang
din
lin phn s.
.
,
, , ,,
V du. 9.1. Chu,ng minh rang
`
moi
`u co th
. s hu u ty duong d
phn tch thanh
`
lin phn s.
,
a ,,
,
,
`
Lo` i giai. Cho = , o dy
a va` b la` s tu. nhin nguyn t cung
b
,
,
,
,
Euclide chuong truo c ta co
nhau. Theo thut
. toan

, , `
9.2. Phn tch s hu u ty thanh
lin phn s

197

a = bq0 + r1
b = r1 q1 + r2
......

(9.5)

r n 2 = r n 1 q n 1 + r n ,
r n 1 = r n q n ,
,
,,
`, dy
,ng dang

o dy
b > r1 > r2 > . . . > rn1 > rn = 1. Tu
va` nhu
,
,
,,
`
thu c (9.5) suy ra qn 2. Ta se chu ng minh rang
phn tch du
o. c
`
thanh
lin phn s
= ( q0 , q1 , . . . , q n ).
r
. r0 = b, i = i1 , (i = 1, 2, . . . , n).
Mun vy
. ta dat
ri
,
,
,

ang
thu c d
`u tin cua (9.5) cho ta

(9.6)

1
a
r
= q0 + 1 = q0 +
= ( q 0 , 1 ) .
b
r0
1
,,
, `, ,
,
,
,,
Tuong tu. tu
dang thu c thu hai ta t`m du
o. c = (q0 , q1 , 2 ). Ta
,
`
chu ng minh rang
=

= (q0 , q1 , . . . , qi1 , i ), i = 1, 2, . . . , n.
(9.7)
, ,, ,
,
,

` d

thu c (9.7) dung


vo i s i nao
Tht
o ( 1 i
. vy,
. gia su dang
,
,

n 1). Ta se chu ng minh khi d


dung
ca cho i + 1. Tht
o no cung
.
,
r i 1
,
,
,
,
thu c ri1 = ri qi + ri+1 vo i ri , ta nhn
vy,
=
o. c
. ta chia dang
. du
ri
,
r i +1
1
qi +
, theo dinh
ngha cua i la` i = qi +
= ( q i , i +1 ).
.
ri
i +1
,
Suy ra = (q0 , q1 , . . . , qi1 , i ) = (q0 , q1 , . . . , qi1 , qi , i+1 ), nhu
,
`, (9.7) suy ra (9.6) vo,i i = n.
vy
. (9.7) da chu ng minh. Tu
V du. 9.2. Chu,ng minh rang
`
su. , phn tch thanh
`
lin phn s
,
,
,
s hu u ty la` duy nht.
cua mi

,,
Chuong 9. Lin phn s

198

,
, ,,
,
,
,
, ,

`
vo i su. khai trin (9.6) c`on co biu din
Lo` i giai. Gia su cung

khac
= (q00 , q10 , . . . , q0m ),
q,0m > 1.
(9.8)
0
`

Ta cho rang
m n. Cho i la` ph`n d
`y du
i0 = (qi0 , qi0+1 , . . . , q0m ),

(i = 1, 2, . . . , m).

,
Hin nhin ta co

= q0 +

1
1
= q00 + 0 ,
1
1

,
`
`
`, dy

tu
suy ra rang
chung
bang
nhau ph`n nguyn cung
nhu

,
,
,,
,

ph`n phn s o hai v cua dang


thu c. Ngha la` q0 = q00 , 1 = 10 .
,
,
,

ang
thu c sau cung
co th vit:
1
1
= q10 + 0 ,
q1 +
2
2
`, d
nay
` (phu,o,ng phap
quy
tu
o suy ra q1 = q10 , 2 = 20 . Theo cach
,
,
0

hoc)

nap
thu c qn1 = qn1 va`
n dang
. toan
. ta se dn d
n = n0 ,

(9.9)

,
,,
1
`
o dy
m > n. Khi d
n = qn . Ta gia thit rang
o n0 = q0n + 0 ,
n +1
,,
0
o dy
n+1 > 1 va` (9.9) suy ra
qn = q0n +

.
n0 +1

,
, ,
,
,
,
,

Nhung dang
thu c d
o khng th xy ra, v` v phai khng phai la`
mt
o suy ra m = n, n0 = q0n = qn .
. s nguyn. i`u v ly d

,
9.3. Phn s xp x
lin phn s
Cho lin phn s (q0 , q1 , q2 , . . . , qn ). Ta xet day
0 = ( q0 ), 1 = ( q0 , q1 ), . . . , n = ( q0 , q1 , . . . , q n ).

(9.10)

,
9.3. Phn s xp x

199

,
, ,
`

,u ty. V` th chung

Ta bit rang
s i la` s hu
co th biu din
,
,
,
,
,
,
,
ng phn s ti gian (D ( a, b) la` uo c s chung lo n nht cua
nhu nhu
a va` b)
i = ( q0 , q1 , . . . , q i ) =

Pi
, ( D ( Pi , Qi ) = 1; i = 0, 1, 2, ..., n). (9.11)
Qi

, ,
Pi
goi
la` i-phn s xp x cua lin phn s (q0 , q1 , . . . , qn ).
.
Qi
,
, vai tr`o quan trong trong ly thuyt lin phn
Phn s xp x giu
,
,
,ng v du. sau dy

s. Nhu
ch ra mt
. s tnh cht cua chung:

Phn s

V du. 9.3. Chu,ng minh rang


` vo,i moi
`u co
. lin phn s ta d
Pi+1 = Pi qi+1 + Pi1 ,

(9.12)

Q i +1 = Q i q i +1 + Q i 1 ,

(9.13)

Pi+1 Qi Pi Qi+1 = (1)i ,

(9.14)

vo,i (i = 1, 2, . . . , n 1).
,
,
,
,
Lo` i giai. Ta chu ng minh quy nap
. theo i. Vo i i = 1, ta tnh Pi va`
P0
q0
,,
,
`, (9.11) ta t`m du
Qi vo i i = 1, 2. Tu
=
va` v` phn s
o. c q0 =
Q0
1
,
P0
ti gian (theo dinh
ngha), nn
.
Q0
P0 = q0 , Q0 = 1.

(9.15)

1
q0 q1 + 1
,
Vo i i = 1 ta co (q0 , q1 ) = q0 +
=
. S q0 q1 + 1 va` q1
q1
q1
,
`
nguyn t cung
nhau. Ta d chu ng minh
D ( a + c, b) = D ( a, b)

(9.16)

,
vo i di
(9.16) cho c = q0 q1 , b =
`u kin
. c chia ht cho b. p dung
.
,,
,
q1 , a = 1, ta se nhn
o. c D (q0 q1 + 1, q1 ) = D (q1 , 1) = 1. by gio`
. du

,,
Chuong 9. Lin phn s

200
q q +1
P
`, (9.11) ta co 0 1
tu
= 1 va`
q1
Q1

P1 = q0 q1 + 1, Q1 = q1 .

(9.17)

P2
q0 ( q1 q2 + 1) + q2
,
.
=
Vo i i = 2 ta co (q0 , q1 , q2 ) =
q1 q2 + 1
Q2
,
,
,
,
,
,a cac
`
phn s ti gian.

ang
thu c sau cung
la` dang
thu c giu
,
P2
ta se ap

Tht
ngha, c`on bn v trai
.
. vy
. Q ti gian theo dinh
2
dung
hai l`n (9.16)
.
D (q0 (q1 q2 + 1) + q2 , q1 q2 + 1) = D (q1 q2 + 1, q2 ) = D (q2 , 1) = 1.
Suy ra
P2 = q0 (q1 q2 + 1) + q2 , Q2 = q1 q2 + 1.
(9.18)
,
,,
,
, ,

`
Su dung
(9.17)
v
a
(9.18)
v
o
i
s
u
ki
m
tra
tr
u
c
ti
p
suy
ra
(9.12),
.
.
.
,
(9.13) va` (9.14) vo i i = 1.
,
, ,,
,

` d

Gia su khang
dinh
dung
vo i s i nao
o ( ( 1 i n 2 ) .
.
,
,
,
`

Ta se chu ng minh rang


chung
cung
dung
vo i i + 1 ngha la` thoa
,
,
,ng dang
nhu

man
thu c sau
Pi+2 = Pi+1 qi+2 + Pi ,

(9.12a)

Q i +2 = Q i +1 q i +2 + Q i ,

(9.13a)

Pi+2 Qi+1 Pi+1 Qi+2 = (1)i+1 ,

(9.14a)

`, dinh
Tu
ngha (9.11) va` theo (9.4) ta co
.
Pi+2
= (q0 , . . . , qi , qi+1 , qi+2 ) = (q0 , . . . , qi , qi+1 ),
Q i +2
1
,,
.
qi+1 = qi+1 +
o dy

qi+, 2
,
,
,

Ta so sanh
(9.19) vo i (9.11), du
thu c
a d
n dang
P
Pi+2
= ( i +1 ) ,
Q i +2
Q i +1

(9.19)
(9.20)

(9.21)

,
9.3. Phn s xp x
201
,
,
,
,,
,,
`
`
o dy
thuong khng d
qi+1 cua
du * ch ra rang
`y du cui cung
,
,,
`
`, (9.20). Theo gia
. c`n du
phn s trong ngoac
qi+1 tu
o. c th bang
`, (9.12) va` (9.13), suy ra
thit quy nap,
. ngha la` tu
Pi+1
P q + Pi1
= i i +1
.
Q i +1
Q i q i +1 + Q i 1

(9.22)

`
`, (9.11) thy rang
` q i +1 .
Tu
Pi1 , Qi1 , Pi va` Qi khng phu. thuc
. vao
,,
,
,
dung
tu *, ta nhn
Khi d
trn (9.22) toan
o ap
o. c
.
. du


Pi+1
Q i +1

Pi qi+1 + Pi1
Q i q i +1 + Q i 1

Pi (qi+1 +

1
q i +2

Q i ( q i +1 +

) + Pi1
1

q i +2

+ Q i 1
)

qi+2 ( Pi qi+1 + Pi1 ) + Pi


.
q i +2 ( Q i q i +1 + Q i 1 ) + Q i

,
,
`, kt qua nay
` cung
`
Tu
vo i (9.12), (9.13) va` (3.21) cho ta
P q + Pi
Pi+2
= i +1 i +2
.
Q i +2
Q i +1 q i +2 + Q i

(9.23)

,
,
Pi+2
,
la` phn s ti gian theo dinh
ngha, chu ng minh
.
Q i +2
,
,
,

dinh
(9.12a) va` (9.13a) ch c`on khang
v phai (9.23) cung
la` phn
.
,
, ,,
,,

s ti gian. Gia su nguo. c lai,


o Pi+1 qi+2 + Pi va` Qi+1 qi+2 + Qi
. khi d
,
,
, ,
,,

co uoc s chung d > 1. D thy d cung


la` uo c s chung cua ca s
V`

Pi+1 ( Qi+1 qi+2 + Qi ) Qi+1 ( Pi+1 qi+2 + Pi ) = Pi+1 Qi Qi+1 Pi .


,
,
`
Nhung theo gia thit quy nap
la` (1)i va` khng
. , hiu
. sau cung
,
,,
,
,,
,
vo,i di
chia ht cho d, trai
Nhu vy
phn s o
`u gia su nguo. c lai.
.
.
, ,
,

v phai cua (9.23) cung


la` ti gian, suy ra (9.12a) va` (9.13a) da
,
chu ng minh.

202

,,
Chuong 9. Lin phn s

,
,
,,
,
`
kt qua nhn
Ta dung
cac
o. c d
chu ng minh (9.14a). Ta co
. du
Pi+2 Qi+1 Pi+1 Qi+2 = ( Pi+1 qi+2 + Pi ) Qi+1 Pi+1 ( Qi+1 qi+2 + Qi )

= ( Pi+1 Qi Pi Qi+1 ) = (1)i = (1)i+1 ,


,,
, ,
`
o dy
(9.14) nhu gia thit quy nap.
ta da dung
.

,
V du. 9.4. Chu,ng minh nhu ,ng dang

thu,c sau
Pi1
P
1
a)
,
( i 1);
i = (1)i .
Q i 1
Qi
Q i Q i 1
b) Qi Pi2 Pi Qi2 = (1)i1 .qi ,
( i 2);
Pi2
P
qi
c)
i = (1)i1 .
,
( i 2);
Q i 2
Qi
Q i Q i 2
Qi
d)
= ( q i , q i 1 , . . . , q 1 ),
( i 1).
Qi 1
,
,
P
P Q Pi Qi1
,
, Pi1
`, dang

i = i 1 i
,
Lo` i giai. a) Suy ra tu
thu c
Q i 1
Qi
Q i Q i 1
,
,,
, ,,
`
vo i tu s o v phai dung
(9.14).
b) Theo (9.12) ta co Pi2 = Pi qi Pi1 ,
Q i 2 = Q i
qi Qi1 .Khi d
o Qi Pi2 Pi Qi2 = Qi ( Pi qi Pi1 ) Pi ( Qi
,,
`
qi Qi1 ) = qi ( Pi Qi1 Qi Pi1 ) = (1)i1 qi , o dy
(3.5).
ta da dung
c) p dung
ph`n b).
.
,
Q1
,
,
`

d) Vo i i = 1 dang
thu c da cho co dang
= (q1 ), di
`u nay
.
Q
, ,, 0,
,

dinh
dung
vo i
dung
v` Q0 = 1, Q1 = q1 , (q1 ) = q1 . Gia su khang
.
,
,
`

i (1 i n 1). Ta se ch ra rang
khi d
dung
vo i i + 1
o no cung
hay la`
Q i +1
= ( q i +1 , q i , . . . , q 1 ).
Qi

9.4. Lin phn s v han


.

203

,,
`, (9.13) ta nhn
Tht
o. c
. vy,
. tu
. du
Q
1
Qi
Q i +1
= q i +1 + i 1 = q i +1 +
).
= ( q i +1 ,
Qi
Qi
Qi
Q i 1
Q i 1

9.4. Lin phn s v han


.
,ng s thu.,c a0 , a1 , . . . ky hiu
s nhu
Cho day
.
a0 +

1
1
a1 +
a2 +

(9.24)
..

.
,,

goi
s v han,
c`on s a0 , a1 , . . . goi
la` ph`n thuong
. la` lin phn
.
.
, ,
,
,,

khng d
ta vit (9.24) duo i
`y du cua (9.24). thun
. tin
. chung
dang
.
( a0 , a1 , . . . ).
(9.25)
,
,
,

`
u han
Nhu ta da bit, moi
u biu din mt
. lin phn s hu
. d
. s
,
,
,
,
,
,
u han
` nhn
(gia tri. phn s), gia tri. nay
du
buo c thu. c
o. c qua hu
.
.
,
,
,
,u t trn ph`n thu,o,ng khng d
hu
hin
`y du. Nhung
. tnh toan
lin phn s v han
`u d
o.
. khng co di
,,
,
,
,
u han
Tuong tu. nhu phn s hu
.
i = ( a0 , a1 , . . . , a i )
,
,
lin phn s v han
goi
. la` phn s xp x thu i, c`on mi
.

(9.26)

( a k , a k +1 , . . . )
(k = 0, 1, . . .)
,
,
,
dinh
goi
. theo cng thu c (9.11)
. la` ph`n du cua (9.24). S (9.26) xac
Pi

` moi

i =
. Theo cach
nay
. phn s v han
. (9.24) t`n tai
. day
Qi
,
phn s xp x
P0 P1
Pn
,
,...,
,...
(9.27)
Q0 Q1
Qn

,,
Chuong 9. Lin phn s

204

,
,
mt
(9.27) hi
s thu. c. Nu day
Mi
s xp x la` mt
.
. tu. va`
. phn
,
,

gioi han
goi la` hi
. cua ,no la` mt
. s th` phn s (9.24)
. tu,
. c`on
, .

la` gia tri. cua lin phn s v han.


. Ta co th vit
= ( a0 , a1 , . . . )
,,
,,
,
`
Trong truo` ng ho. p nguo. c lai,
. lin phn s (9.24) goi
. la` phn ky.

9.5. V du.
V du. 9.5. Cho lin phn s
a1

b0 +
b1 +

a2
b2 +
..

.+

an
bn

Ta dat
.
P0 = b0 , Q0 = 1, P1 = b0 b1 + a1 , Q1 = b1 , . . .
va` cng thu,c chung
Pk+1 = bk+1 Pk + ak+1 Pk1 ,
Q k + 1 = bk + 1 Q k + a k + 1 Q k 1 .
Chu,ng minh rang
`
Pn
= b0 +
Qn

a1
b1 +

a2
b2 +
..

.+

an
bn

,
,
,
,
,
`

Lo` i giai. D thy rang


vo i k = 0, 1 cng thu c dung.
Gia thit
,
,
`

no dung
vo i k = n 1, ta se chu ng minh rang
no cung
dung
cho

9.5. V du.

205

,
,
k = n. Nhu vy
. gia thit co
a1

b0 +
b1 +

a2
b2 +
..

.+

Pn1
Q n 1

a n 1
bn 1

,
`, cng thu,c cho Pk va` Qk ta co
Nhung tu
Pn1
b Pn2 + an1 Pn3
= n 1
Q n 1
bn 1 Q n 2 + a n 1 Q n 3
,,
` an1 va` bn1 .
o dy
Pn2 , Pn3 , Qn2 , Qn3 khng phu. thuc

. vao
,
,
,

dung
`
. khac,
Mat
gia thit quy nap
o. c
. ta nhn
. du
an
) Pn2 + an1 Pn3
bn
=
an
( bn 1 + ) Q n 2 + a n 1 Q n 3
bn

( bn 1 +

a1

b0 +
b1 +

a2
b2 +
..

.+

a n 1
bn 1 +

an
bn

an
Pn2
bn Pn1 + an Pn2
Pn
bn
=
=
=
.
an
b
Q
+
a
Q
Q
n n 1
n n 2
n
Q n 1 + Q n 2
bn
Pn1 +

s trong lin phn s bang


V du. 9.6. Chu,ng minh rang
` (s mu
`
n).
r
r n +1 r
= r +1
r
1
r
r+1
r+1
r
..
.
r+1

,,
Chuong 9. Lin phn s

206

,
Pn
,
Lo` i giai. Ta ky hiu
. lin phn s theo Q . Ta co
n
P1 = r;
Q1 = r + 1;
P2 = r (r + 1);
Q2 = r2 + r + 1;
,
,,
`
quy nap
Ta chu ng minh bang
phuong phap
.
rn 1
r n +1 1
; Qn =
.
r1
r1
,
, `
,
,

Vo i n = 1 cng thu c nay


dung.
Giai thit no dung
vo i n = m, ta
,
,

se chu ng minh no dung


vo i n = m + 1. Ta co (theo v du. trn)
Pn = r

Pm+1 = bm+1 Pm + am+1 Pm1 .


,`,
, ,

Trong truong ho. p cua chung


ta th`
r m 1 1
r m 1 1
r m 1 1
r2
=r
.
r1
r1
r1
r m +2 1
,,
,

Tuong tu. ta cung


co Qm+1 =
.
r1
V du. 9.7. Chu,ng minh rang
`
Pm+1 = (r + 1)r

1
1
1
+
++
=
u1
u2
un

u21

u1
u1 + u2

u22
u2 + u3
..

u2n1
u n 1 + u n

,
1
1
1
,
,,
.
Lo` i giai. Ta dat
+
=
. Khi d
o ta t`m du
o. c xr =
ur
u r +1
ur + xr
u2r
1
1
1
,
,a,

. V` th
+
=
. Hon nu
2
u r + u r +1
u1
u2
u1
u1
u1 + u2
1
1
1
1
1
1
+
+
=
+
=
,
u1
u2
u3
u1
u2 + x2
u1 + x20

9.5. V du.

207

u21
,
. Nhu vy
.
u1 + u2 + x2
1
1
1
1
+
=
=
+
u1
u2
u3
u21
u1
u1
u1 + u2 + x2

,,
o dy
x20 =

1
u21

u22
u1 + u2
u2 + u3
,
,,
, `
,,
,
quy nap
Tuong tu. bang phuong phap
. ta co th chu ng minh cng
,
thu c chung.
,
V du. 9.8. Hay
chu,ng minh dang

thu,c sau
a1
a1 c1
=
,
a2
a2 c1 c2
b1 +
b1 c1 +
b2 +
b2 c2 +
. . an
. . a n c n c n 1
.+
.+
bn
bn c n
,,
,
o dy
0.
c1 , c2 , c2 , . . . , cn la` nhu ng s bt k`y khac

,
,
Pn
,
`
`
bang
Lo` i giai. Ta ky hiu
phn s
, bn phai bang
. bn trai
Qn
,
P0
Pn
P0
,
phn s n0 . Ta phai chu ng minh
= n0 vo,i moi
. s nguyn
Qn
Qn
Qn
,,
duong n. Ta co
P1
a P2
a1 b2
= 1;
=
;
Q1
b1 Q2
b1 b2 + a2
P10
c a P0
c1 c2 a1 b2
= 1 1 ; 20 =
;
0
Q1
c1 b1 Q2
c1 c2 (b1 b2 + a2 )
,
P1 = a1 ; Q1 = b1 ; P2 = a1 b2 ; Q2 = b1 b2 + a2 va` khi
Ta co th dat
,.
,
dang
` 9.6)

d
thu c sau (do bai
o ta co cac
Pn+1 = bn+1 Pn + an+1 Pn1 ,

Q n + 1 = bn + 1 Q n + a n + 1 Q n 1 .

.
Ta lai
. dat
P10 = c1 a1 ; P20 = c1 c2 a1 b2 ; Q10 = c1 b1 ; Q20 = c1 c2 (b1 b2 + a2 ).

,,
Chuong 9. Lin phn s

208

,
,
,
,
`
dang

Ta se chu ng minh rang


vo i moi
thu c sau dung
. n cac
Pn0 = c1 c2 . . . cn Pn ; Q0n = c1 c2 . . . cn Qn .
,
,
,
,
`
dang

Tht
quy nap,
thu c
. vy,
. ta chu ng ,minh bang
. gia thit cac
,
,
,
`

dung
vo i moi
n, ta se chu ng minh no cung
. s nho hon hay bang
,

dung
vo i n + 1. Ta co
Pn0 +1 = cn+1 bn+1 Pn0 + cn cn+1 an+1 Pn0 1 ,
Q0n+1 = cn+1 bn+1 Q0n + cn cn+1 an+1 Q0n1 .
`, d
Tu
o suy ra
Pn0 +1 = cn+1 bn+1 c1 c2 . . . cn Pn + cn cn+1 an+1 c1 c2 . . . cn1 Pn1

= c1 c2 . . . cn+1 (bn+1 Pn + an+1 Pn1 ) = c1 c2 . . . cn+1 Pn+1 .


,,
,
,
,,

Tuong tu. ta cung


chu ng minh du
o. c Q0n+1 = c1 c2 . . . cn+1 Qn+1 .
,
V du. 9.9. Chu,ng minh cac
dang

thu,c sau
1
sin(n + 1) x
1)
= 2 cos x
sin nx
1
2 cos x
2 cos x +
1
..
.+
2 cos x

lin phn s bc
. n.
2)

b2
1

b2 b3

b2 b3 . . . bn

b2

1
b2 + 1

b3
b3 + 1
..

.+

bn
bn + 1

,
,
Pn
,
`
. lin phn s bn phai bang
Lo` i giai. 1) Ta dat
. D thy
Qn

9.5. V du.

209

,
P1
sin 2x
sin x
,
. P1 =
= 2 cos x. V` th ta co th dat
; Q1 =
. Vo i
Q1
sin x
sin x
n=2, th`
1
P2
4 cos2 x 1
= 2 cos x
=
.
2 cos x
2 cos x
, Q2
.
Suy ra co th dat
P2 =

sin 3x
;
sin x

Q2 =

sin 2x
.
sin x

sin(n + 1) x
sin nx ,
,
`
Ta se chu ng minh rang
Pn =
; Qn =
vo i moi
.
sin x
sin x
,
,
,
,

x. Tht
vo i n, ta se chu ng minh no
. vy,
. gia thit cng thu c dung
,

` .9.5 )
dung
vo i n + 1. Ta co (theo bai
Pn+1 = 2 cos x

sin(n + 1) x sin nx
1

=
sin(n + 2) x.
sin x
sin x
sin x

sin(n + 1) x
` toan
` tu,o,ng tu., ta chu,ng minh cho Qn+1 =
Hoan
, v`
sin x
,
th ta co cng thu c
Pn
sin(n + 1) x
=
Qn
sin nx
,,
,
vo i moi
. s nguyn duong n.
,
, ,
,
Pn
,

thu c. Ta c`n phai


2) Ta ky hiu
. Q cho v phai cua dang
n
,
chu ng minh
Pn
= 1 + b2 + b2 b3 + + b2 b3 . . . bn
Qn
,,
,
vo i moi
. n nguyn duong.
Tht
. vy,
.
P1
1 P2
b2 + 1
= ;
=
Q1
1 Q2
1
`
v` th P1 = 1, Q1 = 1, P2 = b2 + 1, Q2 = 1. Khi d
quy nap
o bang
.

,,
Chuong 9. Lin phn s

210
,
,
,,
ta co th chu ng minh du
o. c

Pn = 1 + b2 + b2 b3 + + b2 b3 . . . bn
Qn = 1.
,
,
,

Suy ra dang
thu c c`n chu ng minh la` dung.

V du. 9.10. Chu,ng minh rang


`
nu mt
. lin phn s co n ph`n
,,
un+1 ,,
tu bang
`
1, th` gia tri. phn s nay
` bang
`
, o dy
u1 = u2 =
un
,
1, u = 2, . . . la` nhu ng s Fibonacci.
3

,
,
,,
,
,

Lo` i giai. Ta su dung


quy nap
.
. theo n. Vo i n = 1 va` n = 2 khang

dinh
dung:
.
1
u2
1
2
u3
= , 2 = (1, 1) = 1 + = = .
1
u1
1
1
u2
,
,
,

` d

Gia thit khang


dinh
dung
vo i n nao
o, ngha la` n =
.
u n +1
,
,,
,
,
(1, 1, . . . , 1) =
. Ta se chu ng minh trong truo` ng ho. p nhu vy
.
|
{z
}
un
1 = ( 1 ) = 1 =

no cung
dung
cho n + 1. Tht
. vy,
.
n+1 = (1, 1, . . . , 1) = (1, n )
|
{z
}
n +1

= 1+

` tp
9.6. Bai
.

u n + u n +1
u n +2
1
=
=
.
n
u n +1
u n +1

,,
. 9.11. Cho lin phn s v han
q0 q1 > 0, . . .
. (q0 , q1 , . . .), o dy
,
,
,
,
,

ng s tu. nhin. Hay


ng khang
chu ng minh nhu

la` nhu
dinh
sau:
.
P0 P2

tang.

a) Day
,
, . . . , la` day
Q0 Q2

` tp
9.6. Bai
.

b) Day

211

,
P1 P3
giam.
, . . . , la` day
,
Q1 Q3

,
` trn. Chu,ng minh
. 9.12. Cho (q0 , q1 , . . .) la` lin phn s o, bai
mnh
d
` sau
.
a) (q0 , q1 , . . .) hi
. tu.
.

,
b) S = (q0 , q1 , . . .) la` s v t.

1
+
5
,
`
. 9.13. Chu ng minh rang
(1, 1, . . .) =
.
2
`
. 9.14. Cho = (q0 , q1 , . . . , qn ). Chu,ng minh rang
nu ta ly
,
Pi
Pi
,
,
,
phn s xp x thu i
|(i =
d
i vo i , th` vo i sai s i = |
Qi
, Qi
danh

0, 1, . . . , n 1) thoa man
gia sau
1
1
< i <
.
Q i ( Q i + Q i +1 )
Q i Q i +1

, ,
CHUONG

10

`
MT
.
. S THI V ICH
,
,
,, `
d
nam
Rt nhi`u nuo c hang
`u t chu c thi v dich
. quc gia v`
,
,
,
,ng ky` thi d
cho cac
hoc
mn toan
sinh truo` ng ph thng. Nhu
o
,.
,
,
,
`
` giai bang
quy nap.

co rt nhi`u bai
phuong phap
ch la`
. Sau dy
,
,
,
d
` la` tn nuo c va` nam
ra
mt
s bai
` da co, bn canh
. s nho cac
.
d
` thi.
V du. 10.1. (Hungari 1932). Chu,ng minh rang
`
nu a, b va` n la`
,
,
nhu ng s tu. nhin va` b chia ht cho an , th` s ( a + 1)b 1 chia ht
cho an+1 .
,
,
,
,,
,
`
quy nap
Lo` i giai. Ta chu ng minh bang
phuong phap
. theo n. Vo i
,

n = 0 mnh
d
dinh
dung,
v` ( a + 1)b 1 chia ht cho a.
` khang
.
,
, ,, . ,
,

` d

Gia su khang
dinh
vo i s k nao
o, ngha la` ta gia thit nu
. dung
.
.
,
b .. ak , th` (( a + 1)b 1) .. ak+1 . Cho b0 la` s tu. nhin chia ht cho
.
b0
. b = . Khi d
ak+1 . Ta dat
o b .. ak va` v`
a
0

( a + 1)b = ( a + 1) ab 1 = [( a + 1)b ] a 1 =
= [( a + 1)b 1][( a + 1)(a1)b + ( a + 1)(a2)b + + ( a + 1)b + 1],
,
0
nn ( a + 1)b 1 chia ht cho ak+2 . Tht
vy,
theo gia thit quy
.
.
,
`,a s thu, nht trong biu thu,c sau cung
`
nap
chia ht cho ak+1 ,
. thu
,
,
,
,
,
duo i dang
` a s thu hai biu din
c`on thu
.
[( a + 1)(a1)b 1] + [( a + 1)(a2)b 1] + + [( a + 1)b 1] + a
212

213
.
0
`, d
tu
o d thy no chia ht cho a. Suy ra (( a + 1)b 1) .. ak+2 .

,
,
V du. 10.2. (Hungari 1979). Nhu ,ng dnh cua mt
l`i co
giac
. da
,
, ,,
` sao cho moi
. hai di
nhau
s canh
la` s le du
m canh
o. c t mau
. cap
.
.
,
,
,
`
`
co mau
` khac
nhau. Chu ng minh rang

bang

mt
o` ng cheo
. s du
,
,

khng cat
nhau cua da
nay
` co th cat
thanh
`
nhu ,ng h`nh
giac
,
,
,,
,
,
tam giac
tam giac,
ma` nhu ,ng dnh cua mi
du
o. c t vo i nhu ng
mau
` khac
nhau.
,
,
,
,,
,
`
quy nap
Lo` i giai. Chu ng minh bang
phuong phap
i vo i s
. d
,
,
,
,
Vo,i n = 3 khang
` toan
la` hin

canh
n cua da
dinh
cua bai
giac.
.,
.
nhin. cho n > 3. D thy t`n tai
. ba dnh k` nhau V1 , V2 va` V3
,
,
,
,
,
,
du
` khac
nhau A, B va` C.
cua da
giac
o. c t tuong u ng ba mau
` A; hoac
. la`
Nu V4 khng t mau

` A, c`on V5 khng t
nu V4 t mau
` B; hoac
` A,
. la` nu V4 co mau
mau
,
` B, c`on V6 khng phai
V5 co mau
,
` C, c`on lai
dung
mau
gia
. ta ap
.
`
thit quy nap
. . Nu V4 co mau
` B va` V6 co mau
` C,
A, V5 co mau

h`nh luc
giac V1 V2 V3 V4 V5 V6 ta co
, .
,
th chia ra nhu h`nh ve (h`nh 12),
,
ta ap

ph`n c`on lai


cua da
giac
.
,
dung
gia thit quy nap.
.
.

V du. 10.3. (Moscow 1945). Mt


s a1 , a2 , . . . , an bang
`
. s trong cac
,
+1, s c`on lai
` -1. Chu ng minh rang
`
. bang
r
q

a1 a2
a1 a2 . . . a n
2 sin( a1 +
++
)
=
a
2
+
a
2
+

+
a
2.
n
2
1
2
4
2n 1

,,
Chuong 10. Mt
` thi v dich
.
. s d

214

,
,
,
,,
,
`
quy nap
hoc.
phuong phap
Lo` i giai. Chu ng minh bang
. toan
. Vo i

,
,

,
n = 1 ta c`n kim tra cng thu c 2 sin a1 = a1 2, hin nhin
4
, ,,
,
,

` d
dung.
Gia su cng thu c sau dung
vo i s k nao
o
r
q

a1 a2
a a2 . . . a
2 sin( a1 +
+ + 1 k1 k ) = a1 2 + a2 2 + + ak 2.
2
4
2
Khi d
o
r
q

2 + a1 2 + a2 2 + + a k 2 =
a1 a2
a a2 a3
a a2 . . . a
+ 1
+ + 1 k 1 k )
2
4
4
2
a1 a2
a1 a2 a3
a1 a2 . . . a k

+
++
= 2 2 cos( + ( a1 + +
) )
2
2
4
4
2k 1
a1
a1 a2
a1 a2 a3
a1 a2 . . . a k
= 2(1 cos2(1 + + +
+
++
) )
2
4
8
4
2k
a1
a1 a2
a1 a2 a3
a1 a2 . . . a k
2
= 4 sin (1 + +
+
++
) .
2
4
8
4
2k
,
a1 a1 a2
a1 a2 . . . a k
,
,

V` 0 < 1 + +
++
< 2, tu` bt dang
thu c trn
k
2
4
2
suy ra
r
q

a1 a1 a2
a a2 . . . a
2 + a1 2 + + ak 2 = 2 sin(1 + +
++ 1 k k) .
2
4
4
2
,
,
,,
,
` sin la` ham
` le, vo i a0 = 1, ta nhn vao
` hai v cua
Boi v` ham
,
,
,

dang
thu c trn va` bin d
i
r
q

a0 2 + a1 2 + + a k 2 =
a
a a2
a a2 a3
a a2 . . . a
= a0 2 sin(1 + 1 + 1 + 1
++ 1 k k) =
2
4
8
4
2
a0 a1
a0 a1 a2
a0 a1 a2 a3
a0 a1 a2 . . . a k
= 2 sin( a0 +
+
+
++
) .
2
4
8
4
2k
,
,
,
,
,
,
`

di
cng thu c du
vo i n =
`u d
o ch ra rang
o. c chu ng minh dung
,
,

k + 1. Theo nguyn ly quy nap


vo i moi
. cng thu c dung
. n.

= 2 + 2 sin( a1 +

215
V du. 10.4. (Moscow 1984). Cho X = ( x1 , x2 , . . . , xn ) la` day
n,
,
,
,
`

n 4, nhu ng s khng m, tng cua chung

bang

1.
,
a) Chu ng minh rang
`
x1 x2 + x2 x3 + + x n x1

1
.
4

, ,
,
b) Chu,ng minh rang
`
t`n tai
. mt
. t ho. p Y = (y1 , y2 , . . . , yn ) cua
X, sao cho
1
y1 y2 + y2 y3 + + y n y1 .
n
,
,
,,
,
,
quy nap
Lo` i giai. a) p dung
phuong phap
i vo i n, ta chu ng
.
. d
minh

( x1 + x2 + + x n )2 4( x1 x2 + x2 x3 + + x n x1 ),

(10.1)
,
,,
,,
,
`, d
o dy
xi 0 va` n 4. Tu
o vo i xi = 1 ta nhn
o. c kt qua.
. du
n

i =1

,
,
,
,,
,,
,

Nu n = 4, bt dang
thu c (10.1) tuong du
ong vo i bt dang
,
thu c
( x1 x2 + x3 x4 )2 0
,
,
, ,

thu c xay ra khi va` ch khi x1 + x3 = x2 + x4 .


Ta chu y dang
,
,
,
,

thu c (10.1) dung


vo i mt
bt ky`
By gio` bt dang
.
. s c dinh
,
` d
nao
o n = k 4. Ta c`n chu ng minh

( x 1 + x 2 + + x k + x k +1 )2 4 ( x 1 x 2 + x 2 x 3 + + x k x k +1 + x k +1 x 1 ).
(10.2)
,
,
,
,
,

V` tng hai v cua bt dang


thu c (10.2) la` v`ong tr`on theo ch
, ,
,
,
`, gia
s, ta co th gia thit xk+1 xi vo i i = 1, 2, . . . , k. Khi d
o tu
thit quy nap
. suy ra
( x1 + x2 + + xk1 + ( xk + xk+1 ))2
4( x1 x2 + x2 x3 + + xk1 ( xk + xk+1 ) + ( xk + xk+1 ) x1 ). (10.3)

,,
Chuong 10. Mt
` thi v dich
.
. s d

216
,,
Boi v`

( x 1 x 2 + x 2 x 3 + + x k 1 ( x k + x k +1 ) + ( x k + x k +1 ) x 1 ) =
( x1 x2 + x2 x3 + + xk xk+1 + xk+1 x1 ) + xk1 xk+1 + xk ( x1 xk+1 ))
`, (10.3) suy ra (10.2).
va` x1 xk+1 0, th` tu
,
, , ,
.
b) Vo i t ho. p bt ky` Y = (y1 , y2 , . . . , yn ) cua X ta dat
SY = y1 y2 + y2 y3 + + yn y1 .
,
,
,
vi. cua X). Vo,i
Ta ky hiu
tt ca n! hoan
. SY = S (tng tnh theo
,
,
,
,
,,
,ng t ho.,p cua X, trong d
su. c dinh
i va` j, i 6= j s luo. ng cua nhu
o
.
,
,
,
,
` dy
.
xi du ng truo c x j (xp theo v`ong lap!),
la` n(n 2)!. Tu

S = n ( n 2) !

i,j=1,i 6= j

xi x j = n(n 2)!(1 xk2


i =1

1
1
n(n 2)!(1 ( xk )2 ) = n(n 2)!(1 ) = (n 1)!.
n k =1
n
,
,,
Suy ra s nho nht trong SY khng vuo. t qua

S
( n 1) !
1

= .
n!
n!
n
V du. 10.5. (Ba lan 1952-1953). Chu,ng minh rang
` nu n la` mt
.
,

s tu. nhin, th`


( 2 1) n = m m 1
,
, ` do m.
vo,i mt

. s tu. nhin thch ho. p nao


,
,
,
,
`
hoc

Lo` i giai. Ta se chu ng minh bang


quy nap
dinh
.
. toan
. khang
,
,
,
,

`
sau, voi moi
s
t
u
nhin
n
t
n
t
ai
nh
u
ng
s
t
u
nhin
a
v
a
b
sao
n
n
.
.
.
.
cho
(

( 1 2 ) n = a n bn 2
(10.4)
a2n 2bn2
= (1)n .

217
,,
,
,

`
Tht
d
vo i n = 1, v` trong truo` ng ho. p nay
` dung
. vy,
. mnh
.
,
,
,
,
,
`

a1 = b1 = 1 du
thu c trn bang
nhau. Ta gia
a cac v cua dang
,
`
,ng s thch ho.,p an va` bn ,
thit rang
vo i n c dinh
bt ky` ta co nhu
.
,
,
,
,ng dang

vo i chung
(10.4) dung.
Khi d
thu c
o nhu

(1 2 ) n +1 = ( 1 2 ) n (1 2 )

= ( an bn 2)(1 2)

= ( an + 2bn ) ( an + bn ) 2,
,
,
,ng s tu,o,ng u,ng
suy ra su. t`n tai
. cua nhu
an+1 = an + 2bn va` bn+1 = an + bn ,
tai
. v`
a2n+1 2bn2 +1 = ( an + 2bn )2 2( an + bn )2 = ( a2n 2bn2 ) = (1)n+1 .
,
,
,

Nhu vy
dinh
dung
vo i moi
`u khang
.
. di
. n.
,
va` m = 2bn2 vo,i n la` s
. m = a2 vo,i n la` s chan,
Ta ch c`on dat
,
,
,
,, ,
` toan.

le, d
nhn
o. c lo` i giai bai
. du

V du. 10.6. (Lin x 1976). Cho x0 va` x1 la` nhu ,ng s tu. , nhin
,
nho ho,n 1000, va` dat
.
x2 = | x0 x1 |, x3 = | x1 x2 |, x4 = | x2 x3 |, . . .
,
Chu,ng minh rang
`
t nht mt
. trong nhu ng s x2 , x3 , . . . , x1500
bang
` 0.
,
,
,
,
,
`
Lo` i giai. Chu ng minh bang
quy nap
hon
. theo n kt qua manh
.
, ,
d
s nhu, d
` ra s x0 va` x1 nho hon 2n, th` t
` ra: Nu trong day
,
nht mt
` 0.
. trong nhu ng s x1 , x2 , . . . , x3n bang
, ,,
,
,,
,
,
Gia su mnh
d
s nguyn nho
` da du
o. c chu ng minh vo i moi
.
.
,
,
,
thu.,c su., co bt dang

hon n. Nu trong day


thu c x3 < 2n 2, x4 <

218

,,
Chuong 10. Mt
` thi v dich
.
. s d

,
,
,
`, gia thit quy nap
2n 2, th` tu
d
` c`n chu ng
. suy ra kt qua mnh
.
minh.
,,
,
Boi v` theo di
`u kin
. co x0 2n 1, x1 2n 1, th` vo i x2 1
ta co x3 2n 2, x4 2n 3.
,
,
Nu x3 6= 2n 2, th` lai
a v` gia thit quy nap.
. du
. Nu x3 =
,
2n 2, th` x2 = 1, x1 = 2n 1, x0 = 2n 2. Ta se chu ng minh
,
,,
,
` Ta co

khang
dinh
trong truo` ng ho. p nay.
.
x3 = 2n 2, x4 = 2n 3, x5 = 1, x6 = 2n 4, x7 = 2n 5,

x8 = 1, . . . , x3k = 2n 2k, . . . , x3n = 0.


V du. 10.7. (Canada 1979). Cho a, b, c, d va` e la` nhu ,ng s nguyn
,
,
thoa man
di
`
`u kin
. 1 a < b < c < d < e. Chu ng minh rang
1
1
1
1
15
+
+
+
,
[ a, b] [b, c] [c, d] [d, e]
16
,
,
,,
o dy
[m, n] ky hiu
. la` bi
. s chung nho nht cua m va` n.
,
,
,
,
, ,

Lo` i giai. Ta chu ng minh theo quy nap


thu c tng quat
. bt dang
,
hon.
1
1
1
1
+
++
1 n
(10.5)
[ a0 , a1 ] [ a1 , a2 ]
[ a n 1 , a n ]
2
,,
,ng s tu., nhin. Vo,i n = 1 th`
o dy
0 < a0 < a1 < . . . < an la` nhu
,
, ,,
,
`

` d
(10.5) dung
hin nhin. Ta gia su rang
(10.5) dung
vo i n nao
o,
,
,
ng s tu. nhin bt ky` 0 < a0 < a1 < . . . < an < an+1 .
va` ta xet nhu
`, (10.5) suy ra
Nu an+1 2n+1 , th` [ an , an+1 ] 2n+1 va` tu
1
1
1
1
+
++
+

[ a0 , a1 ] [ a1 , a2 ]
[ a n 1 , a n ] [ a n , a n +1 ]
1
1
1
(1 n ) + n +1 = 1 n +1 .
2
2
,
,
, 2 ,
`
By gio` cho an+1 < 2n+1 . Ta chu y rang
vo i nhu
ng s tu. nhin bt
ky` p va` q, p < q, ta1co
( p, q)
qp
1 1
=

=
[ p, q]
pq
pq
p q

219
,
,
dung

(Ta da ap
dang
thu c [ p, q]( p, q) = pq va` q p chia ht cho
.
( p, q)). Suy 1ra
1
1
1
+
++
+

[ a0 , a1 ] [ a1 , a2 ]
[ a n 1 , a n ] [ a n , a n +1 ]
1
1
1
1
1
1
)=
( )+( )++(
a0
a1
a
a2
a
a n +1
11
1
1n
=

< 1 n +1 ,
a 0 , a n +1
,
, 2
,
,
,
,
`

Bt dang
thu c (10.5) tro thanh
dang
thu c vo i ai = 2i , i =
0, 1, . . . , n.
,
V du. 10.8. (Canada 1982). Cho a, b va` c la` nhu ,ng nghim
. cua
phu,o,ng tr`nh

x3 x2 x 1 = 0.

Chu,ng minh rang


` s
b1982 c1982
c1982 a1982
a1982 b1982
+
+
bc
ca
ab

la` mt
s
nguyn.
.
,
,
.
Lo` i giai. Ta dat
n
cn an
an bn ,
b cn
, sn =
, tn =
vo i n = 1, 2, . . .
rn =
bc
ca
ab
,
`
Ta se chu ng minh rang
rn+3 = rn+2 + rn+1 + rn , n 1.
,
,
,
,
ng nghim
dang

V` nhu
thu c
. b va` c thoa man
b3 = b2 + b + 1, c3 = c2 + c + 1,
nn
b n +3 c n +3
b n ( b2 + b + 1) c n ( c2 + c + 1)
=
bc
bc
n
+
2
n
+
2
n
+
1
n
+
1
b
c
b
c
bn cn
=
+
+
bc
bc
bc
= r n +2 + r n +1 + r n .

r n +3 =

,,
Chuong 10. Mt
` thi v dich
.
. s d

220

,,
,,
`
nhu, vy
Theo cung
phuong phap
o. c
. ta nhn
. du
sn+3 = sn+2 + sn+1 + sn , tn+3 = tn+2 + tn+1 + tn , n 1.
,
,,
,
`
quy nap
hoc
Ta se chu ng minh bang
phuong phap
i vo i n,
. toan
. d
,
sao cho rn + sn + tn la` s nguyn vo i moi
. n 1. V`
r1 + s1 + t1 = 3,
r2 + s2 + t2 = 2( a + b + c) = 2,
r3 + s3 + t3 = 2( a + b + c)2 3(bc + ca + ab) = 5.
,
,
, ,,
,
`

Khang
dinh
dung
vo i n = 1, 2, 3. Ta gia su rang
khang
dinh
.
.
,

cung
dung
vo i n = 1, 2, . . . , k + 2, k 1. Khi d
o

r k +3 + s k +3 + t k +3 = ( r k +2 + s k +2 + t k +2 ) + ( r k +1 + s k +1 + t k +1 ) + ( r k + s k + t k ).
,
,
,
Theo gia thit quy nap
no la` tng cua ba s nguyn va` suy ra
.
, ,,
` la` s nguyn.
tng o d
` bai

,
,,
V du. 10.9. (CHLB u,c 1981). Day
a1 , a2 , a3 , . . . du
o. c cho nhu
sau: a1 la` s tu. , nhin, an+1 = [1, 5an ] + 1 vo,i moi
. n = 1, 2, . . .. Co
,
,
,
,
th chon
` d
d

cho 100 000 s hang


`u tin cua day
. a1 nhu th nao
.
,
,
,

trn la` nhu ng s chan,


c`on s hang
thu 100 001 la` mt
.
. s le ?
,
,
,
,,
, `
` dat
. ra la` a1 =
Lo` i giai. Co th chon
o. c mt
u bai
. du
. s nhu d
,
,
100001
`
2
2. Bang
quy nap
. theo n ta se chu ng minh vo i moi
. n =
,
,

thu c sau dung


1, 2, . . . , 100001 dang
an = 3n1 .2100001n .

(10.6)

,
11 100001n 2 = 2100001 2 = a . Nu
Tht
1
. ,vy,
. vo i n = 1 ta co 3 .2
,

` d
ta gia thit (10.6) dung
vo i n nao
o
n

100000,
th`

an+1 = [1, 5an ] + 1 = [1, 5(3n1 .2100001n 2)] + 1

= [3n .2100000n 3] + 1 = 3n 2100000n 2.

221
,

Khi d
o vo i n = 1, 2, . . . , 100000 s 2100001n la` s chan,
,
,ng s
c`on 2100001100001 = 1 la` mt
. s le. D, thy nhu
c`on a100001 la` s le.
a1 , a2 , . . . , a100000 la` s chan,

V du. 10.10. (Ao-Balan 1980). Chu,ng minh rang


`
1

i1 i2 . . . i k

= n,

,
,
,
,

o, dy
tng thu. ,c hin
rng

. theo tt ca tp
. ho. p con khac
,
{i1 , i2 , . . . , ik } cua {1, 2, . . . , n}
,
,
,,
,
dung
quy nap
Lo` i giai. Ta ap
phuong phap
.
. theo n. Vo i n = 1 ta
,
,
,
1
,

co = 1, di
Ta gia thit dang
thu c theo di
`u hin nhin dung.
`u
1
,
tp

` toan
dung

` d
rng
kin
bai
vo i s n 1 nao
o. Mi
. con khac
,.
,
,
ng dang
cua tp
sau dy:

. ho. p {1, 2, . . . , n, n + 1} la` mt


. trong nhu
.
,
,
,
a) Tp
. ho. p con cua tp
. ho. p {1, 2, . . . , n};
,
,
,
b) Tp
. ho. p con cua tp
. ho. p {1, 2, . . . , n} va` s n + 1;
,,
,
c) Tp
. ho. p mt
. ph`n tu {n + 1}.
,
, ,
` toan
trong tru,o`,ng ho.,p
Khi d
o vo i tng theo di
`u kin
. cua bai
,,
n + 1 ta nhn
o. c
. du
1
1
.n +
= n + 1.
n+1
n+1
,
,
,,
` toan
da du
Nhu vy
o. c
. chu ng minh theo quy nap
. da xong va` bai
,
giai.
n+1

V du. 10.11. (Balan 1981). Cho cac


day
s x1 , x2 , . . . ; y1 , y2 , . . .
,
,
3
3
thoa man
cac
di
`u kin
. xn+1 = xn 3xn ; yn+1 = yn 3yn vo i moi
.
n 1 va` x12 = y1 + 2. Chu,ng minh rang
`
xn2 = yn + 2 vo,i moi
.
n 1.

,,
Chuong 10. Mt
` thi v dich
.
. s d

222

,
,
,
,
`
quy nap
theo n. Vo i n = 1, cng
Lo` i giai. Ta chu ng minh bang
.
,
, ,,
,
,
,

thu c dung
theo gia thit. Gia su no dung
vo i n = k, tu c la` xk2 =
,
,
` toan
dung

yk + 2
(k 2). Ta phai chu,ng minh bai
vo i n =
,
k + 1, tu c la` xk2+1 = yk+1 + 1. Tht
. vy,
.
xk2+1 = ( xk3 3xk )2 = xk6 6xk4 + 9xk2

= ( y k + 2)3 6( y k + 2)2 + 9( y k + 2)
= y3k 3yk + 2 = yk+1 + 2
,
tu c la` xk2+1 = yk+1 + 2.

,
V du. 10.12. (Balan 1982). Cho q la` mt
thu. ,c su. ,
. s tu. nhin chan
s tu. , nhin n, s q(q+1)n + 1
lo,n ho,n 0. Chu,ng minh rang
` vo,i mi
chia ht cho (q + 1)n+1 va` khng chia ht cho (q + 1)n+2 .
,
,
,
,
`
Lo` i giai. Ta chu ng minh bang
quy nap
. theo theo n. Vo i n = 0,
0
` toan
dung

q(q+1) + 1 = q + 1 khng chia ht cho (q + 1)0+2 . Bai


,
voi n = 0.
, ,,
n
,
,
` toan
dung

vo i n(n > 0), tu c la` q(q+1) + 1 chia ht


Gia su bai

cho (q + 1)n+1 va` khng chia ht cho (q + 1)n+2 . Noi cach


khac
,
n
,
(
q
+
1
)
n
+
1

q
+ 1 = (q + 1) s chia ht cho q + 1. Ta phai chu ng minh
, ,
` toan
dung

bai
vo i ca n + 1. Tht
. vy
.
q ( q +1)

n +1

+ 1 = ((q(q+1) + 1) 1)q+1 + 1 = ((q + 1)n+1 s 1)q+1 + 1

q +1

Cq+1 (q + 1)(n+1) j .s j .(1)q+1 j + 1


j

j =0

q +1

Cq+1 (q + 1)(n+1) j .s j .(1)q+1 j = (q + 1)(q + 1)n+1 s


j

j =1

Cq2+1 (q + 1)2(n+1) s2 + + (q + 1)(q+1)(n+1) sq+1 =


= (q + 1)n+2 (s Cq2+1 (q + 1)n s2 + + (q + 1)qn+q1 sq+1 ).

223
n +1
`,a s thu, hai chia ht cho (q + 1). Do d
Thu
o q(q+1) + 1 chia ht
cho (q + 1)n+2 va` khng chia ht cho (q + 1)n+3 .

V du. 10.13. (Anh 1978). Chu,ng minh rang


` vo,i moi
. n 1, n N
,
,
,
th` 2 cos n la` mt
thu c bc
. da
. n cua 2 cos vo i h. s nguyn.
,
,
,
,
`
Lo` i giai. Ta se chu ng minh bang
quy nap
. theo n. vo i n = 1,
,

mnh
d
Vo i n = 2, 2 cos 2 = 2(2 cos2 1) = (2 cos )2
` dung.
.
, ,,
,
,

2, tu c la` mnh
Gia su mnh
d
vo i moi
` dung.
` dung
. , d
.
. n, n
,
,
`

k (k 2). Ta phai chu ng minh mnh


d
d
ung
v
o
i
n
=
k
+
1. Ta co

.
2 cos(k + 1) + 2 cos(k 1) = 4 cos k. cos = (2 cos k )(2 cos ).
,
Suy ra 2 cos(k + 1) = 2 cos (2 cos k ) 2 cos(k 1). V phai ro
,
,
,
`, d
`
rang
la` mt
thu c bc
o
. da
. k + 1 cua 2 cos vo i h. s nguyn. Tu
,
,

suy ra mnh
d
vo i n = k + 1 va` do d
vo i moi
` dung
o no dung
.
. n.

V du. 10.14. (` thi Olympic Toan


quc t l`n thu, 18, 1976).
5
, , dinh theo cach
Day
u0 , u1 , u2 , . . . du
sau u0 = 2; u1 = ;
o. c xac
.
2
un+1 = un (u2n1 2) u1 ,
Chu,ng minh rang
` vo,i n 1 [un ] = 2
nguyn lo,n nht khng lo,n ho,n x.

2n

( n 1).
(1)n
,
3
, o, dy
[ x ] la` s

,
2k (1)k
,
,
`
. k =
Lo` i giai. Ta dat
, k 0. Ta se chu ng minh rang
3
,
n+1 = 2n + (1)n vo i moi
. s nguyn n 0. Tht
. vy
.
2n+1 2(1)n + 3(1)n
2n (1)n
+ (1)n =
3
3
n
+
1
n
n
+
1
n
+
1
2
+ (1)
2
(1)
=
=
= n +1 .
3
3

2n + (1)n = 2

,,
Chuong 10. Mt
` thi v dich
.
. s d

224

,
,
,
`, cac
dang
` suy ra tt ca k la`

V` 0 = 0 va` 1 = 1 tu
thu c nay
,
,
`
nguyn. Ta se chu ng minh rang
un = 2n + 2n , n 0. Vo i k =
,
, ,
,
` d kim tra. Gia su, no dung

0, 1 di
vo i k = n 1 va` k = n,
`u nay
,,
,,
,a n va` n1 , ta se nhn
va` su dung
mi lin quan giu
o. c
.
. du
un+1 = un (u2n1 2)

5
2

= (2n + 2n ).[(2n1 + 2n1 )2 2]


= (2n + 2n ).(22n1 + 22n1 )2 )
= (2n + 2n ).(2n ()
n

= 22n +(1) + 2(1)


= 2 n +1 + 2 n +1 ,
k

n 1

n 1

+ 2(1)

+ 2(1)

5
2

n 1

n 1

5
2

)2 )

5
2
n

+ 22n (1)

5
2

,
5 ,
,,
k 1. Bai
` toan
da du
vo i mi
o. c giai v`
2
1

n
+ 2 ] = 2 n , do n < 1 vo,i n 1.
2
k

v` 2(1) + 2(1) =

[ u n ] = [ 2 n

V du. 10.15. (` thi Olympic Toan


quc t l`n thu, 22, 1981).
,
Bit rang
` ham
` s f ( x, y) thoa man
nhu ,ng di
`u kin:
.
a) f (0, y) = y + 1;
b) f ( x + 1, 0) = f ( x, 1);

,
c) f ( x + 1, y + 1) = f ( x, f ( x + 1, y)) vo,i tt ca nhu ,ng s nguyn
khng m x va` y. Hay
t`m f (4, 1981).
,
,
Lo` i giai. Ta co f (1, 0) = f (0, 1) = 2, f (1, 1) = f (0, f (1, 0)) =
,
`
`
f (0, 2) = 3. Ta se chu ng minh bang
quy nap
f (1, y) = y + 2.
. rang
,
, ,, ,
,
,

` d

Voi y = 1, khang
dinh
dung.
gia su vo i s tu. nhin k nao
o ta co
.

225
f (1, k ) = k + 2. Khi d
o
f (1, k + 1) = f (0, f (1, k )) = f (0, k + 2) = k + 3.
,,
`, b) ta nhn
`, c) f (2, 1) =
Tu
o. c f (2, 0) = f (1, 1) = 3, c`on tu
. du
f (1, f (2, 0)) = f (1, 3) = 5.
,
,
,
`
By gio` ta chu ng minh rang
f (2, k ) = 2k + 3. Tht
. vy,
. cho vo i
` d
k nao
o ta co f (2, k ) = 2k + 3. Khi d
o f (2, k + 1) = f (1, f (2, k )) =
f (1, 2k + 3) = 2k + 5.
,,
` ra f (3, y + 1) =
Ta nhn
o. c f (3, 0) = f (2, 1) = 5. Ngoai
. du
,
,
. la` vo i moi
f (2, f (3, y)) = 2 f (3, y) + 3, hoac
s tu. nhin y ta co
.
,
,
,,
dung

f (3, y + 1) = 2 f (3, y) + 3. Ta ap
dang
thu c k l`n, nhn
o. c
.
. du
f (3, y + 1) = 2k+1 . f (3, y k ) + 3(2k + 2k1 + + 2 + 1),
,,
,
`, dy
tu
vo i k = y ta nhn
o. c f (3, y + 1) = 52y+1 + 3.(2y+1 1).
. du
,
,
,
,
`, dang

Suy ra f (3, y) = 2y+3 3 vo i moi


thu c
. s tu. nhin y. Tu
,,
`
cui cung
ta nhn
o. c
. du
f (4, y) = f (3, f (4, y 1)) = 2 f (4,y1)+3 3

= 22

f (4,y2)+3 3+3
y 1

3 = 22

f (4,y2)+3

y +2

z}|{13+3
z}|{
..2
..2
.
2
2.
= 22
3 = 22 3
,
,,
,
dung

o dy
dang
thu c f (4, 0) = f (3, 1) = 24 3 = 13. Thay
ta ap
.
,
,
,,
` dang

gia tri. vao


thu c trn ta co du
o. c f (4, 1981).

, ,
CHUONG

11

,
,
` TP
BAI
. TU. GIAI
`
. 11.1. Chu,ng minh rang
a) 1.2.3 . . . p + 2.3 . . . p.( p + 1) + + n(n + 1) . . . (n + p 1) =

n ( n + 1) . . . ( n + p )
.
p+1

n(n + 1)(n + 2)(3n + 1)


.
12
1 1 1
1
1
1
1
c) 1 + + +

=
+
++
2 3 4
2n 1 2n
n+1 n+2
b) 2.12 + 3.22 + + (n + 1).n2 =

1
.
2n

d) 12 22 + 32 42 + + (1)n1 .n2 = (1)n1

n ( n + 1)
.
2

,
,
dang

. 11.2. Chu,ng minh cac


thu c sau
a+1 a+3 a+7
a + 2n 1
( a 1)(2n 1)
+
+
++
=
+ n.
2
4
8
2n
2n
,
,
`
. 11.3. Chu,ng minh rang
vo i moi
. s tu. nhin n
a) 62n 1 chia ht cho 35;
b) n6 3n5 + 6n4 7n3 + 5n2 2n chia ht cho 24;
c) 2n+2 .3n + 5n 4 chia ht cho 25;
d) 52n+1 + 2n+4 + 2n+1 chia ht cho 23.
`
. 11.4. Chu,ng minh rang
226

227

1
1
1
,
n < 1 + + + + < 2 n vo i n 2.
n
3
2
1
n ( n 1)
> n! vo,i n 3.
b) 2 2
,
,

. 11.5. Chu,ng minh bt dang


thu c
log1 + log2 + + logn
log(n + 1) >
.
n
,
,

. 11.6. Chu,ng minh dang


thu c
a)

(1 + x )(1 + x2 )(1 + x4 ) . . . (1 + x2

n 1

)=

= 1 + x + x + x3 + + x2
,
,

. 11.7. Chu,ng minh dang


thu c sau
1 a + 1 ( a + 1)(b + 1)
1+ +
+
+
a
ab
abc
( a + 1)(b + 1) . . . (s + 1)(l + 1)
+
=
abc . . . skl
( a + 1)(b + 1) . . . (k + 1)(l + 1)
=
.
abc . . . kl
`
. 11.8. Chu,ng minh rang

n 1

(1 an )(1 an1 ) . . . (1 ank+1 )


= n.
1 ak
k =1
,
tnh tng
. 11.9. Hay
a a( a 1) a( a 1)( a 2) a( a 1) . . . ( a n + 1)
Sn = +
+
+
.
b b(b 1) b(b 1)(b 2) b(b 1) . . . (b n + 1)
`
s 0, 1, 2, . . . , n 1)
( b khng bang
mt
. trong cac
n

. 11.10. Cho
Sn = a1 + ( a1 + 1) a2 +( a1 + 1)( a2 + 1) a3 +

+ ( a1 + 1)( a2 + 1) . . . ( an1 + 1) an .

,,
, ,
` tp
Chuong 11. Bai
. tu. giai

228
,
`
Chu ng minh rang

Sn = ( a1 + 1)( a2 + 1) . . . ( an + 1) 1.
`
Fibonacci, chu,ng minh rang
. 11.11. Cho {un } la` day
[

un =

n1
]
2

Cnk k1 .

k =0

,
,
,,
,,
t`m tt ca nghim
. 11.12. Hay
. nguyn duong cua phuong tr`nh
,,
x1 + x2 + + xn = m (m la` s nguyn duong).
,
,,
`
. 11.13. Chu,ng minh rang
s nghim
. chung nguyn duong cua
,ng phu,o,ng tr`nh x + 2y = n; 2x + 3y = n 1; . . . ; nx + (n +
nhu
`
1)y = 1; (n + 1) x + (n + 2)y = 0; bang
n + 1.
`
. 11.14. Chu,ng minh rang
s nghim
. chung nguyn khng m
,
,
,
,
ng phuong tr`nh sau x + 4y = 3n 1; 4x + 9y = 5n 4;
cua nhu
`
9x + 16y = 7n 9, . . . , n2 x + (n + 1)2 y = n(n + 1); bang
n.
,
`
s tuy` y
. 11.15. Chu,ng minh rang
vo i gia tri. tuy` y 1 va` cac
,
cac
di
x1 , . . . , xn thoa man
`u kin
. 1 x1 x2 . . . xn > 0, ta
,
,

co bt dang
thu c

(1 + x1 + x2 + + xn ) 1 + 11 x11 + 21 x2 + + n1 xn .
,
`

. 11.16. Chu,ng minh rang


vo i cac
gia tri. tuy` y m, n N va` cac
,
cac
di
s tuy` y x1 , x2 , . . . , xn , y1 . . . , yn [0, 1] thoa man
`u kin
.
,
,
,

thu c
xi + yi = 1 voi i = 1, 2, . . . , n, ta co bt dang

(1 x1 x2 . . . xn )m + (1 y1m ) . . . (1 ym
n ) 1.
,
gia tri. n N hay
t`m gia tri. lo,n nht k Z + d
. 11.17. Vo,i mi

s [(3 + 11)2n1 ] chia ht cho 2k .

229
,
,
l`i co s canh
` cac
dnh cua mt
. 11.18. Ngu,o`,i ta t mau
giac
. da
.
,
,
,
,
`

` khac
nhau.

le, sao cho bt ky` 2 dnh ln cn


2 mau
o. c t bang
. du
,
,
,
`
`
t mau
` thoa man
di

Chu ng minh rang


vo i moi
u kin
. cach
. trn
,
,,
,
,
`
co th chia thanh
`
tam giac
boi cac
du
da
cac
giac
ong cheo khng
,
,

`
`
` khac
nhau.
nhau, ma` hai d
cat
u mi du
ong cheo, co hai mau
,
,
,
s du,o,ng a, a2 , ..., an thoa man
bt dang

. 11.19. Day
thu c a2n
,
,
,
`
an an+1 vo i n N. Chu ng minh rang
vo i bt ky` gia tri. n N
1

co danh
gia an < .
n
,
,
,
s { an } thoa man
bt dang

. 11.20. Cho day


thu c { ak+m ak
,
,
,
`
am } 1, vo i k, m N. Chu ng minh rang
vo i bt ky` p, q N co
,
,

bt dang
thu c


ap

aq < 1 + 1 .
p
q
p q
,
,
`
s hang
s
. 11.21. Chu ng minh rang
mi
cua day
.
!
!
n
n
3 5
3+ 5
+
2, (n N )
2
2
,
,
du,o,i dang
. m2 (m N ) vo,i n
la` s tu. nhin va` biu din
5m2 hoac
.
,
,,
,
hoac
. le.
tuong u ng chan
`

s nguyn
. 11.22. Chu,ng minh rang
t`n tai
mt
. dung
. day
,
3
`
di
a1 , a2 , . . . thoa man
u kin
. a1 = 1, a2 > 1, an+1 + 1 = an an+2
,
voi n N.
,
`
vo i cac
s tuy` y m, n N, s
. 11.23. Chu,ng minh rang
m

Sm,n = 1 +

(1)k

k =1

( n + k + 1) !
n!(n + k )

,
,
chia ht cho m!, nhung vo i mt
. s gia tri. m, n N s Sm,n khng
chia ht cho m!(n + 1).

,,
, ,
` tp
Chuong 11. Bai
. tu. giai

230

,
,,
nhau co ban
knh
. phang

. 11.24. Trn mat


co n du
o` ng tr`on khac
,
,
,
,
`

`
`

nhau. Chu ng minh rang


1 du
xp khac
ch co mt
d
u bang
o. c sap
.
,
,
,

mt
trong s d
o chu a mt
o` ng tr`on trong s
. cung, khng cat
. du
,
2P
,,
,ng du
` khng nho ho,n
.
nhu
o` ng tr`on c`on lai
. va` co d. dai
n
,
. 11.25. Co th chia mt
d
bt ky` thanh
`

cac
giac
`u 2n giac
. da
h`nh thoi hay khng?
,
`

. 11.26. Chu,ng minh rang


vo i moi
a > 2 t`n tai
. a N,
. v s cac
,
,
,
n

s n N d
dinh
dung
vo i a = 2
s a 1 chia ht cho n. Khang
.
khng?
,,
,
`
. 11.27. Chu,ng minh rang
vo i s n N tuy` y phuong tr`nh
,
x12 + x22 + + xn2 = y2 co nghim
. s tu. nhin.
,,
,ng s tu., nhin du
a1 , a2 , . . . , an , . . . nhu
`
. 11.28. Day
o. c tao
. thanh
,
s bt ky` a1 , c`on a2 la`

theo cach
sau: Chon
. mt
. s, co ba chu
,
,
,
,
,
,
s a1 , a3 la` tng cac
b`nh phuong cua cac
chu
b`nh
tng cua cac
,
,,
,
,
,
s cua a2 va` tip tuc
chu
phuong cac
. qua tr`nh nhu vy.
. Chu ng
`
gap
a1 , a2 , a3 , . . . bat
. hoac
. la` s 1, hoac
. s 4.
minh rang
trong day
`
v han
s tu.,
. 11.29. Chu,ng minh rang
khng t`n tai
. day
. cac
,
hai di
nhin n1 , n2 , . . . thoa man
`u kin

. sau dy:
,
a) n < n
vo i k = 1, 2, . . .
k

k +1

,
b) nkl = nk + nl vo i moi
. k = 1, 2, . . . va` l = 1, 2, . . .
,
,
,
`

. 11.30. Chu,ng minh rang


vo i moi
. gia tri. s tu. nhin bt dang
,

thu c sau dung


2n > n 2 .

, ,
CHUONG

12

,
,
,
` go. i y
` tp
` i giai va
Lo
bai
.
,
,,
,
,
` go. i y
` tp
12.1. L`oi giai va
bai
. chuong 1 . . . . . . . . . . . . . . . . .
,
,,
,
,
` go. i y
` tp
12.2. L`oi giai va
bai
. chuong 2 . . . . . . . . . . . . . . . . .
,
,,
,
,
` go. i y
` tp
12.3. L`oi giai va
bai
. chuong 3 . . . . . . . . . . . . . . . . .
,
,,
,
,
` go. i y
` tp
12.4. L`oi giai va
bai
. chuong 4 . . . . . . . . . . . . . . . . .
,
,,
,
,
` go. i y
` tp
12.5. L`oi giai va
bai
. chuong 5 . . . . . . . . . . . . . . . . .
,
,,
,
,
` go. i y
` tp
12.6. L`oi giai va
bai
. chuong 6 . . . . . . . . . . . . . . . . .
,
,,
,
,
` go. i y
` tp
12.7. L`oi giai va
bai
. chuong 7 . . . . . . . . . . . . . . . . .
,
,,
,
,
` go. i y
` tp
12.8. L`oi giai va
bai
. chuong 8 . . . . . . . . . . . . . . . . .
,
,,
,
,
` go. i y
` tp
12.9. L`oi giai va
bai
. chuong 9 . . . . . . . . . . . . . . . . .
Muc
l
uc
.
.
.
.
.
.
.
.
.
.
.
.
.
.
.
.
.
.
.
.
..............................
.
.

231
233
236
236
237
240
240
244
247
250

,
,,
,
,
` go. i y
` tp
12.1. L`oi giai va
bai
. chuong 1

,
,
,
,
. 1.11. a) Lo` i giai: Ta thit lp
. bang cho mt
. s gia tri. cua n.

n=
Sn =

1
1

2
-3

3
6

4
-10

5
15

,
,,
,
, ,

` d
Ta so sanh
vo i bang s o bai
`u tin d du
a d
n giai thit
Sn = (1)n1 .
231

n ( n + 1)
.
2

,
,,
,
,
` tp
232
Chuong 12. Lo` i giai va` go. i y bai
.
,
, ,,
,
,
,

` d

vo i n nao
Gia su dung
thu c d
o, ta se chu ng minh dang
o cung

dung
cho n + 1. Ta co
n ( n + 1)
+ (1)n (n + 1)2
Sn+1 = Sn + (1)n (n + 1)2 = (1)n1
2
(n + 1)(n + 2)
= (1)n
.
2
, ,
n ( n + 1) 2
` i: Sn = [
] .
b) Tra lo
2
,
,
`,i giai: Ta lp
c) Lo
`u
. bang mt
. s gia tri. ban d
n= 1 2 3
4
Sn = 1 5 23 119
,
,
` giai
Tai
. v` 1! = 1, 2! = 2, 3! = 6, 4! = 24, 5! = 120, ta co th lam
,,
,
thit Sn = (n + 1)! 1. Vo i Sn+1 ta nhn
o. c
. du
Sn+1 = (1.1! + 2.2! + + n.n!) + (n + 1).(n + 1)!

= ( n + 1) ! 1 + ( n + 1).( n + 1) !
= (n + 1)!(1 + n + 1) 1 = (n + 2)! 1.
,
,
,
,
dung
. 1.12. a) Lo` i giai: Ta ap
tnh cht cua tng
.
n

n(4n2 1)
.
3
=1
=1
=1
=1
,
,
,,
,
`,a cua cac
dung
s tu.,
o dy
ta ap
cng thu c tnh tng luy thu

.
nhin.
,
Tu,o,ng tu., ph`n a).
b) Go. i y:
1
1
1
,
,
Ta ap
dung
c) Go. i y:
cng thu c
=
, ta t`m
.
x ( x + 1)
x
x+1
,,
du
o. c

(2 1)2 = 4 2 4 + 1 =

n
n
1
1
1
1
n
=
= 1
=
.
( + 1)
=1 + 1
n+1
n+1
=1
=1

,
,,
,
,
` tp
12.2. Lo` i giai va` go. i y bai
233
. chuong 2
,
,
`
hoc
quy nap
. 1.13. Lo` i giai: Chu,ng minh bang
. toan
. theo k
,
,

dang
thu c sau
1
,
vo i 0 k n 1.
nk
,
,
, ,,
,
,

Vo i k = 0, khang
dinh
dung
hin nhin. Gia su no dung
vo i moi
.
.
n k. Khi d
o
x0 + x1 + + x k =

1
+ x k +1
nk
1
1
( x0 + x1 + + x k ) +
=
nk1
nk
1
1
1
+
=
.
=
(n + k 1)(n k) n k
nk1
,,
,
,
,

Suy ra mnh
d
dung
vo i k + 1. Truo` ng ho. p ring, vo i k =
` cung
.
,,
n 1 ta nhn
o. c x0 + x1 + + xn1 = 1.
. du
x 0 + x 1 + + x k + x k +1 =

,
,,
,
,
` go. i y
` tp
12.2. L`oi giai va
bai
. chuong 2

,
,
,
,
thu,c d
. 2.31. Lo` i giai: Ta ky hiu
` ra la` (2.27). Chu ng
. bt dang
,
`, (2.27) ta co x15 + x17 2x16 , no
minh quy nap
. theo n. Vo i n = 1, tu
,,
,,
,
,,
,
,
` bt
tuong du
ong vo i (1 x1 )2 0, nhu vy
. trong truo` ng ho. p nay
,
,

dang
thu c dung.
,
, ,,
,
,
,
` d

Gia su bt dang
thu c dung
vo i s n 1 nao
o. Ta se chu ng
,
,

minh no cung
dung
vo i n + 1. Cho x1 < x2 < . . . < xn < xn+1 vo i
,
,ng s nguyn du,o,ng. Theo gia thit quy nap
nhu
. ta co (2.27) va`
,
,
,
`

trong no co dang
thu c khi va` ch khi xk = k, k = 1, 2, . . . , n. Ngoai
,
,
,

ra bt dang
thu c sau dung
(chu ng minh sau)
xn5 +1 + xn7 +1 2[2( x13 + x23 + + xn3 ) xn3 +1 + ( xn3 +1 )2 ]
(12.1)
,
,
,

va` dang
xay ra ch khi xk = k, k = 1, 2, . . . , n + 1 . Cng
theo v
.

,
,,
,
,
` tp
234
Chuong 12. Lo` i giai va` go. i y bai
.
,
,,
cua (2.27) va` (12.1) ta nhn
o. c
. du
x15 + x25 + + xn5 + xn5 +1 + x17 + x27 + + xn7 + xn7 +1

2( x13 + x23 + + xn3 + xn3 +1 )2


,
,
, ,
,
,
,

va` dang
thu c ch xy ra khi va` ch khi co dang
thu c trong (2.27)
,
,

va` (12.1), ngha la` xk = k, k = 1, 2, . . . , n, n + 1. Nhu vy


. khang
,

dinh
dung
vo i n + 1.
.
,
,
,
,
,,

Ta ch c`on chu ng minh (12.1). Bt dang


thu c (12.1) tuong
,,
,
du
ong vo i
xn5 +1 ( xn+1 1)2
x13 + x23 + + xn3 .
(12.2)
4
,
,
,
,,
,
`, nhu
,ng bt dang

Bt dang
thu c (12.2) du
thu c va`
o. c suy ra tu
,
,

dang
thu c sau
x13 + x23 + + xn3 13 + 23 + + xn3 ,
13 + 23 + + xn3 =

xn2 ( xn

xn xn+1 1,

+ 1)2

(12.3)

,
(12.4)

( xn+1 1)2 xn2 +1


xn2 ( xn + 1)2

.
4
4
,
,
,
,
,

ang
thu c trong (12.2) khi va` ch khi co dang
thu c trong (12.3)
va` (12.4), hay la` xk = k, k = 1, 2, . . . , n + 1.

,
,
,
,

thu,c da cho hin nhin dung


. 2.32. Lo` i giai: Vo,i k = 1 bt dang
,
, ,,
,,
,
,
`

va` tro thanh


dang
thu c. Gia su (2.28) dung
vo i s nguyn k 1.
.
Ta dat
N = ( a 1 + a 2 + + a k + a k +1 )2

= ( a1 + a2 + .. + ak )2 + a2k+1 + 2ak+1 ( a1 + a2 + + ak ).

,
,,
,
,
` tp
12.2. Lo` i giai va` go. i y bai
235
. chuong 2
,
,
,
`
`,a s thu, nht o, bn phai phu,o,ng tr`nh trn bang
Ta th thu
biu
, ,
,,
,
thu c lo n hon no trong (2.28), ta nhn
o. c
. du
N k ( a21 + a22 + + a2k ) + a2k+1 + 2ak+1 ( a1 + a2 + + ak )

= (k + 1)( a21 + a22 + + a2k + a2k+1 ) a21 a22 a2k ka2k+1 +


+ 2a1 ak+1 + 2a2 ak+1 + + 2ak ak+1 =
= (k + 1)( a21 + a22 + + a2k+1 ) ( a1 ak+1 )2 ( a2 ak+1 )2
( a k a k +1 )2 .
,
,
,
`, dy

Tu
thu c (2.28) vo i k + 1, hay la`
suy ra bt dang
N (k + 1)( a21 + a22 + + a2k+1 ).

,
,
f (k+1) ( x ) theo f (k) ( x ) bang
` mt
Ta biu din
. 2.33. Go. i y:
. s gia
tri.
1
x
, f 00 ( x ) = 2
f 0 (x) = 2
( x 1)1/2
( x 1)3/2
3x
12x2 + 1
(iv)
f 000 ( x ) = 2
,
f
(
x
)
=

( x 1)5/2
( x2 1)7/2
60x3 + 31x
522x4 + 266x2 + 31
(vi )
f (v) ( x ) = 2
,
f
(
x
)
=

.
( x 1)9/2
( x2 1)11/2
,
,
biu lai
` toan:
nu f ( x ) = ( x2 1)1/2 , x > 1,
By gio` ta phat
. bai
khi d
o
gn ( x )
f (n) ( x ) = 2
( x 1)(2n1)/2
,,
,

o dy
gn ( x ) la` da
thuc bc
. n 2, va`
(
,
,

` chan
h. s khng m,
vo i tt ca cac
ham
nu n chan
gn ( x ) la`
, ,
,
,
` le vo i tt ca cac
h. s khng du,o,ng,
ham
nu n le.
,
`
` co th chu,ng minh bang
Mnh
d
quy nap.
` nay
.
.
,
,
va` le.
Chia hai tru,o`,ng ho.,p n chan
. 2.34. Go. i y:

,
,,
,
,
` tp
Chuong 12. Lo` i giai va` go. i y bai
.

236

,
,,
,
,
` go. i y
` tp
12.3. L`oi giai va
bai
. chuong 3

,
. 3.21. Lo`,i giai: Ta co

n 1

Sn =

n 1

ak bk = a1 b1 + ak+1 bk+1 = a1 b1 + (ak + d)qbk

k =1

= a1 b1 + q

k =1

k =1

n 1

k =1

k =1

ak bk + dq bk = a1 b1 + q(Sn an bn ) + dqb1

q n 1 1
.
q1

, `
`, d

Tu
toan.
o suy ra cng thu c bai
, ,
1
. 3.22. Tra lo` i: bn = n2 + 1, Sn = n(2n2 + 3n + 7).
6
, ,
1
. 3.23. Tra lo` i: Sn = n(4n2 + 7n + 1).
2
, ,
. 3.24. Tra lo` i:
2n
2n
+ C2 sin
;
3
3
b) xn = (C1 + C2 .n)(1)n ;
a) xn = C1 cos

c) xn = C1 + C2 (1)n ;
d) xn = C1 + C2 n + C3 n2 .
,
. 3.25. Tra lo`,i:
a) xn = 7 + 3n ; b) xn = 2n + 3n 4n ; c) xn = 2(3 2n).3n1 1.

,
,,
,
,
` go. i y bai
` tp
12.4. L`oi giai va
. chuong 4

,
,

. 4.19. Lo` i giai: Khi n = 1, th` 112 + 12 = 133, mnh


d
` dung
.
, ,, k+1
,
,
2k

1
voi n = 1. Gia su 11
+ 12
chia ht cho 133. Ta se chu ng

,
,,
,
,
` tp
12.5. Lo` i giai va` go. i y bai
. chuong 5

237

chi ht cho 133. Tht


minh 11k+2 + 122k+1 cung
. vy,
.
11k+2 + 122k+1 = 11.11k+1 + 122 .122k1

= 11.(11k+1 + 122k1 ) + 133.122k1 .


,
,,
,

Tng thu du
d
vo i moi
o. c chia ht cho 133. Vy
` dung
. mnh
.
. n 1.

,
, `
Tin hanh
`
. 4.20. Go. i y:
nhu bai
4.2 ph`n 1).
,
, `
Tin hanh
`
. 4.21. Go. i y:
nhu bai
4.2 ph`n 2).
,
,
,
,
Bai
` toan
du
. 4.22. Go. i y:
a v` chu ng minh cng thu c 102n1 + 1
,
chia ht cho 11 vo i moi
. n 1.

,
,,
,
,
` go. i y
` tp
12.5. L`oi giai va
bai
. chuong 5

,
,
2n+1 + 22n + 1.
. 5.28. Lo` i giai: Ta ky hiu
. an = 2
1+1
1
,
1) Vo i n = 1, a = 22 + 22 + 1 = 21.
1

2+1
2
,
2) Vo i n = 2, a2 = 22 + 22 + 1 = 28 + 24 + 1 = 256 + 16 + 1 =
,
273 = 21.13 , nhu vy,
. a2 chia ht 21.
,
,
,
3) Ta se chu ng minh an chia ht cho 21 vo i moi
. s tu. nhin n.
,
,
Vo i n = 1 va` n = 2 ta da kim tra.
, ,,
k +1
k
k +2
k +1
Gia su ak = 22 + 22 + 1 = 21m. Ta xet ak+1 = 22 + 22 +
k +1
k
k
,
1. Nhung 22
= ak 22 1 = 21m 22 1, nhu, vy
. a k +1 =
k +2
k
k.4
k
k
k.3
2
2
2
2
2
2
+ (21m 2 1) + 1 = 2 2 + 21m = 2 (22 1) +
21m,
k

k 1

k 1

ak+1 = 22 [(26 )2 1] + 21m = 22 [(64)2 1] + 21m.


k 1
,
Nhung (64)2 1 chia ht cho 64 1 = 63 = 21.3 va` suy ra
,
,
ak+1 chia ht cho 21. Nhu vy
. an chia ht cho 21 vo i moi
. n va` v`

,
,,
,
,
` tp
Chuong 12. Lo` i giai va` go. i y bai
.
,
,
s, th` u,o,c s chung lo,n
a1 = 21 la` s nho nht trong tt ca cac
,
,
nht cua tt ca an , n = 1, 2, . . . la` 21.
238

,
,
,
lai
chu,ng minh cua Euclide v` t`n
. 5.29. Lo` i giai: Nhac
. cach
,ng s nguyn t
tai
. v han
. s nguyn t. Nu p1 p2 . . . pn + 1 la` nhu
` s P = p1 p2 . . . pn + 1 khac
1 va` suy ra no chia ht cho mt
bt ky,
.
,
, ,
` d
p1 , p2 , . . . , p n .
uoc s nguyn t nao
o, tt nhin no phai khac
,
,,
,
nay
` ta t`m du
Nhu vy
o. c mt
. theo cach
. s nguyn t mo i. Nu
,,
,
p1 , p2 , . . . , pn la` n s nguyn t d
`u tin. Trong truo` ng ho. p d
o
, ,
,
uoc s trn se la` s nguyn t thu n + 1 : pn+1 . Do d
o p n + 1
,
n
,
,
,
2

p1 p2 . . . pn + 1. By gio` bt dang
thu c pn < 2 chu ng minh theo
,
,,
,
quy nap
hoc.

phuong phap
toan
Vo i n = 1 ta co p1 = 2 bt dang
.
.
,
,
, ,,
,
,
,

thu c la` hin nhin. Gia su bt dang


thu c dung
vo i moi
. gia tri.
, ,
,

nho hon n. Ta se chu ng minh no dung


cho n + 1. Tht
. vy,
.
n

pn+1 p1 p2 . . . pn + 1 < 22 .22 . . . 22 + 1 = 22

n +12

+ 1 < 22

n +1

,
,
. 5.30. Lo` i giai: 1) Vo,i n = 1 ta co
1 1
1 + 1
cos
2
2
1 1
1 1
cos
= sin( 1 1 )
2 sin
2
2
1 1
+ 1

(v` 0
1
)
2
2
2
,
, ,,
,
,

thuc da cho dung


vo i n 1. Khi d
2) Gia su bt dang
o
sin 1 sin 1 = 2 sin

n 1

(sin i sin i ) = (sin i sin i ) + sin n sin n

i =1

i =1

n 1

(sin i sin i ) + 2 sin

i =1

n n
n + n
cos
2
2

,
,,
,
,
` tp
12.5. Lo` i giai va` go. i y bai
. chuong 5
n 1

n 1

i =1
n 1

i =1

sin( i

i ) + 2 sin

239

n 1
n n
n n
cos( ( i i ) +
)
2
2
i =1

n 1

i =1

i =1

= sin( ( i i )) + sin( ( i i )) + sin(


i =1
n

( i i ))

= sin( ( i i ).
i =1

bt phu,o,ng tr`nh 1 0, 2 1 , 3 2 , . . . , n
V` cng
cac
.
,,
`, d
n1 ta nhn
o. c 1 + 1 + + n 1 + 2 + + n1 tu
o
. du
suy ra
n 1
n + n

n n
0 ( i i ) +

.
2
2
2
i =1

,
,
. Sn =
. 5.31. Lo` i giai: Ta dat

n + i . Vo i

n = 2 ta co S2 =

i =1

,
1
14
13
13 ,
1
`
`
+
=
> . Ta gia thit rang
Sn >
, vo i s n nao
2+1 2+2
24
24
24
,
,
,

d
co bt dang
thu c cho Sn+1 >
y. Ta se chu ng minh khi d
o cung
,
,
13
,
,

. Ta se chu ng minh bt dang


thu c sau la` du Sn+1 Sn > 0.
24
Tht
. vy,
.
n +1

S n +1 S n =

i =1

n
1
1

n + i + 1 i =1 n + i

1
1
1
1
+

=
> 0.
2n + 1 2n + 2 n + 1
2(n + 1)(2n + 1)

,
,
,
da cho thoa man
phu,o,ng tr`nh truy h`i
. 5.32. Lo` i giai: Day
, ,,

,
an =
c + an1 . Gia su gio i han
an t`n tai.
o
. l = nlim
. Khi d

. la` l = c + l,
lim an = lim c + an1 =
c + lim an1 hoac
n
n
n

1
,,
,
`, dy
tu
vo i chu y l > 0, ta t`m du
o. c l = (1 + 4c + 1). Ta c`n
2

,
,,
,
,
` tp
240
Chuong 12. Lo` i giai va` go. i y bai
.
,
,
,
`
a1 , a2 , . . .
phai chu ng minh gio i han
t`n tai.
day
. Ta chu y rang
,
,.
ho,n ta se

. Chnh xac
tang,
suy ra ch c`on kim tra no la` bi. chan.
,
,
,

chu ng minh bt dang


thu c

an < 1 + c, (n = 1, 2, 3, . . .).
,
,
, ,

,
a1 c < 1 + c. Gia su, bt dang

Tht
. vy,
. vo,i n = 1 (1) thoa man
,
vo,i n nao
` d
thu c (1) thoa man
o. Khi d
o
q

an+1 = c + an < c + (1 + c) < 1 + c.

,
,,
,
,
` go. i y
` tp
12.6. L`oi giai va
bai
. chuong 6
,
,
`
D dang
`
. 6.17. Go. i y:
chu ng minh bang
quy nap
. theo n.
,
,
,
`, mt
Tu
`

. 6.18. Go. i y:
s doan
thang
co th tao
thanh
da
giac
.
.
,
, . , ,
,
,

khi va` ch khi do


phai nho hon tng cac
an
. lo n nht trong chung
do
an
. c`on lai.
.
,
,
,,
,,
ra da du
,ng manh do n du
Nhu
. 6.19. Go. i y:
o. c t
o` ng tr`on cat
,
,,
,
,
son theo gia thit quy nap.
o` ng tr`on thu n + 1
. , Ta ve ,thm mt
. du
,
` trong du,o`,ng tr`on thu,
` tt ca cac
manh nam
, khi d
o ta d
i mau
,
,
,
. phang

n + 1. Nhu vy
phai t`m.
. ta co mat
,
,
Tng cac
s la` 2.3n .
. 6.20. Go. i y:

,
,,
,
,
` go. i y bai
` tp
12.7. L`oi giai va
. chuong 7

,
,
,
`
. 7.17. Lo` i giai: Ta chu,ng minh bang
quy nap
. theo n. Vo i n = 1

mnh
d
v`
` dung,
.
a
a
a
a
a
=
= (
) ( ).
x ( x + 1)
x x+1
x+1
x

,
,,
,
,
` tp
12.7. Lo` i giai va` go. i y bai
. chuong 7
,
,
,,
da
Cho n > 1. Ta biu din
thu c P( x ) duo i dang
.

241

P( x ) = c + ( x + n) P1 ( x ),
,
, ,
,
,,
`
o dy
s, c`on bc
c la` hang
. cua P1 ( x ) nho hon n 1. Theo gia
thit quy nap
.
P1 ( x ) = x ( x + 1) . . . ( x + n 1)( R1 ( x + 1) R1 ( x )).
,
,,
,u ty nao
` hu
` d
o dy
R1 ( x ) la` mt
o va` khi d
o
. ham
P( x ) = x ( x + 1) . . . ( x + n)( R( x + 1) R( x )).
,,
o dy

c
1
R ( x ) = R1 ( x ) .
.
n x ( x + 1) . . .,( x + n 1)
,
y:
Nu bc

Chu
dinh
trn la` khng
`u khang
.
. cua, P( x ) la` n, di

xem phan v du. sau


dung,
ta hay
P ( x ) = x ( x + 1) . . . ( x + n 1).

,
,
`
. 7.18. Lo` i giai: Chu,ng minh bang
quy nap
Nu n = 0,
. theo n.
,
,
`

P( x ) = a mt
s va` khi d
o P( x ) = a.P0 ( x ). Gia thit khang
. hang
,
, ,
,
,
n

dinh
vo i tt ca da
thu c bc
. dung
. nho hon n va` P( x ) = an x + .
,
,
. bn = n!an . a thu,c P( x ) bn Pn ( x ) se co bc
Ta dat
. khng lo n hon
,
n 1 va` co ngha la` theo gia thit quy nap
. co s b0 , b1 , . . . , bn1
sao cho
P( x ) bn Pn ( x ) = bn1 Pn1 + + b0 P0 ( x ).
,
,
,

Suy ra khang
dinh
dung
cho ca da
thu c P( x ).
.

,
,
,
,

. 7.19. Lo` i giai: 1) Vo,i n = 1 dang


thu c co dang
u1 x =
.
3
2
3
2
,
x +x x
u1 x + u2 x x
, `

. la` x = 2

hoac
, dang
thu c nay
dung.
x2 + x 1
x +x1

,
,,
,
,
` tp
Chuong 12. Lo` i giai va` go. i y bai
.

242

,
,
,
,
`
` d

2) Chu ng minh rang


dang
thu c dung
vo i s n nao
o. Ta se
,
,
,
,

thu c cung
chu ng minh dang
dung
cho n + 1. Tht
i
. vy,
. ta bin d
n +1

Fn+1 ( x ) =

uin = Fn (x) + un+1 xn+1

i =1

u n x n +2 + u n +1 x n +1 x
+ u n +1 x n +1 .
x2 + x 1
, ,
,,
` cng thu,c un+2 = un + un+1 ta nhn
Bin d
i du. a vao
o. c
. du

Fn+1 ( x ) =

u n +1 x n +3 + u n +2 x n +2 x
, ( x 2 + x 1 6 = 0)
x2 + x 1

,
,
Tin hanh
`
. 7.20. Go. i y:
nhu 7.16
,
,
Tin hanh
`
. 7.21. Go. i y:
nhu 7.16
,
,
`
. 7.22. Lo` i giai: Chu,ng minh bang
quy nap
. theo n.
1) n = 0, Khi d
o P( x ) = a0 . Nu |1 a0 | < 1 va` | a a0 | < 1.
,
Khi d
o | a 1| = | a a0 + a0 1| | a a0 | + | a0 1| < 2, nhung
d

a 3 dn
n v ly!.
, ,,
,
,

2) Gia su mnh
d
vo i k n 1. Ta xet da
` dung
thu c
.
P ( x + 1) P ( x )
.
a1
,
,
`
`
D dang
thy rang
Q( x ) la` da
thu c bc
. n 1. Theo gia thit quy
nap
. t`n tai
. s i (0 i n 1) sao cho
Q( x ) =

P ( i + 1) P ( i )
|
a1
| ai+1 P(i + 1) ai + P(i )|
| ai+1 P(i + 1)| | ai P(i )|
=

+
.
a1
a1
a1

1 | ai Q(i )| = | ai

,
,,
,
,
` tp
12.7. Lo` i giai va` go. i y bai
. chuong 7

243

, , 1
, ,,
s hang
Co t nht mt
khng nho hon . Gia su d
o la`
. trong cac
.
2
,
a1
,
,
biu thu c thu nht, th` v`
1 do a 3, | ai+1 P(i + 1)|
2
a1
1.
2
,
, , 1
,,
,
,
,
Tuong tu. , nu biu thu c thu hai khng nho hon , th` | ai
2
P(i )| 1.

,
1
1
,
. 7.23. Lo` i giai: 1) Vo,i n = 1. Khi d
o P1 ( x ) + P1 ( ) = x + va`
x
x
,
,
,,

bt dang
thu c nhn
o. c co dang
. du
.
1
2, ( x > 0).
(12.5)
x
,
,
,,
,,
,,
,

V` x > 0, nn bt dang
thu c vua` nhn
o. c tuong du
ong vo i
. du

( x 1)2 0 va` suy ra mnh


d
` dung.
.
1
1
,
Vo i n = 2 ta co P2 ( x ) + P2 ( ) = ( x2 + 1) + ( 2 + 1) = x2 +
x
x
,
1
1
1
,
`
2 + 2 . C`n phai chu ng minh rang
P2 ( x ) + P2 ( ) 2 + 1 + (1 +
x
x
2
, ,
1
,
2
2
` (12.5). Gia su,
` suy ra tu
. la` x + 2 2; di
(1) ) = 4 hoac
`u nay
x
,
,
,
`

thu c sau dung


mnh
d
d
ung
v
o
i
n.
Ta
co bt dang

.
x+

1
1
Pn ( x ) + Pn ( ) n + 1 + (1 + (1)n ).
x
2
,
,
,
,
,

thu c dung
cho n + 2
Ta se chu ng minh truo c tin bt dang
1
1
Pn+2 ( x ) + Pn+2 ( ) n + 3 + (1 + (1)n+2 ).
x
2
,
,
,
dang

Chu y Pn thoa man


thu c Pn+2 ( x ) = x n+2 + Pn ( x ). Suy ra
1
1
1
Pn+2 ( x ) + Pn+2 ( ) = [ Pn ( x ) + Pn ( )] + [ x n+2 + n+2 ]
x
x
x

244

,
,,
,
,
` tp
Chuong 12. Lo` i giai va` go. i y bai
.

,
,
1
,
`, cac
dang

Theo (*) bt dang


thu c sau dung
x n+2 + n+2 2. Tu
x
,
,
,
,
`

thu c va` bt dang


thu c sau cung
va` gia thit quy nap
. ta co:
1
1
Pn+2 ( x ) + Pn+2 ( ) n + 1 + (1 + (1)n ) + 2.
x
2
1
= (n + 2) + 1 + (1 + (1)n+2 ).
2
n
n
+
2
`

Ta chu y rang
(1) = (1) . Theo nguyn ly quy nap
. mnh
.
,
,
,
,
`

d
voi moi
voi n = 1) va` dung
voi moi
dung
. s le n (v` no dung
.
,
,
,
,
,

(v` no dung

n chan
voi n = 2). Nhu vy
thuc dung
voi
. bt dang
moi
. n 1.

,
,,
,
,
` go. i y
` tp
12.8. L`oi giai va
bai
. chuong 8

,
,
,

. 8.19. Lo` i giai: 1) vo,i n = 1 khang


dinh
dung,
v`
.

2 cos x 2 cos2 x
cos x (1 cos x )
2 cos x cos 2x 1
=
=
= cos x.
x
x
x
4 sin2
4 sin2
2 sin2
2
2
2
2) Cho
(k + 1) cos kx k cos(k + 1) x 1
.
cos x + 2 cos 2x + + k cos kx =
x
4 sin2
2
Khi d
o
cos x + 2 cos 2x + + k cos kx + (k + 1) cos(k + 1) x =

(k + 1) cos kx k cos(k + 1) x 1
+ (k + 1) cos(k + 1) x
x
4 sin2
2
(k + 1) cos kx k cos(k + 1) x 1 2(1 cos x )(k + 1) cos(k + 1) x
=
+
x
x
4 sin2
4 sin2
2
2
(k + 2) cos(k + 1) x + (k + 1) cos kx 2(k + 1) cos x cos(k + 1) x + 1
=

x
x
4 sin2
4 sin2
2
2
=

,
,,
,
,
` tp
12.8. Lo` i giai va` go. i y bai
. chuong 8

245

(k + 2) cos(k + 1) x + (k + 1) cos kx

x
4 sin2
2
(k + 1)[cos(k + 2) x + cos kx ] + 1

x
4 sin2
2
(k + 2) cos(k + 1) x (k + 1) cos(k + 2) x 1
=
.
x
4 sin2
2
=

,
,
,

. 8.20. Lo` i giai: 1) Vo,i n = 1 khang


dinh
dung,
v`
.
1

1 + i = 2 2 (cos + i sin ).
4
4
, ,,
,
,

2) Gia su dung
vo i n = k, tu c la`
k

(1 + i )k = 2 2 (cos

k
k
+ i sin
).
4
4

Khi d
o
k
k 1

+ i sin
).2 2 (cos + i sin ).
4
4
4
4
k +1
(
k + 1)
(
k + 1)
.2 2 (cos
+ i sin
).
4
4
k

(1 + i )k+1 = 2 2 (cos

,
,
,
,

. 8.21. Lo` i giai: Vo,i n = 2 dang


thu c dung:
a1 b1 + a2 b2 =
,
a2 (b1 + b2 ) ( a2 a1 )b1 = a2 B2 ( a2 a1 ) B1 . Ta se chu ng minh
,
,
,

` d

nu khang
dinh
dung
vo i n nao
dung
vo i n + 1.
o, th` no cung
.
Ta co
n +1

=1

=1

a b = a b + a n + 1 bn + 1
n 1

= [ an Bn

(a+1 a ) B ] + an+1 bn+1 .

=1

(12.6)

,
,,
,
,
` tp
Chuong 12. Lo` i giai va` go. i y bai
.

246

,,
,
Nhung bn+1 = Bn+1 Bn o dy
an Bn + an+1 bn+1 = an Bn +

,
,
,,

` kt qua cui cung


`
an+1 ( Bn+1 Bn ). Tu
va` (12.6) ta nhn
o. c
. du
n 1

n +1

a b = an+1 bn+1 ( an+1 an ) Bn

=1

(a+1 a ) B

=1
n

= a n + 1 bn + 1

(a+1 a ) B .

=1

,
,
,
`
. 8.22. Lo` i giai: Ta chu,ng minh bang
quy nap
. theo k. Vo i k = 1
,
`
` bang
tng nay
m
1
=
.
(12.7)
1
m+1
m+1
,,
,
Voi k = 2 ta tnh du
o. c
2
2.1
m
1
+
=
.
(12.8)
m + 1 (m + 1)(m + 2)
m+2
,
,
,
,
,
`
`, dang

Tu
thu c (12.7) va` (12.8) du
a d
n giai thit tng (12.6) bang
m
,
`
. Ta se chu ng minh rang
m+k
( k + 1) k
(k + 1)k . . . 2.1
k+1
+
+ (1)k+1
1
n + 1 (m + 1)(m + 2)
(m + 1)(m + 2) . . . (m + k + 1)
m
=
.
(12.9)
m+k+1
, `
Ta du
dinh
ngha sau
a vao
.

. i < 0, hoac
. i > k;
nu hoac
0,
Qi ( k ) =
k
(
k

1
)
.
.
.
(
k

i
+
1)
(1)i .
, nu 0 i k
( m + 1) . . . ( m + i )
,
Khi d
o (12.9) co th vit
k +1

k +1

Qi (k + 1) = [Qi (k) m + i Qi1 (k)] =

i =0

i =0

k +1

k +1

i =0

i =0

Q i ( k ) m + i Q i 1 ( k ) =

,
,,
,
,
` tp
12.9. Lo` i giai va` go. i y bai
. chuong 9

i =0

i =0

i+1

Qi ( k ) m + i + 1 Qi ( k )
k

247

m + i + 1 Qi ( k )

i =0
k

m + i + 1.m + k + 1.

i =0

m+k+1
Qi ( k )
m

k
m
m+k+1
Qi ( k )

m + k + 1 i =0 m + i + 1

k
m
m+k+1+ii
Qi ( k )

m + k + 1 i =0
m+i+1

k
k
m
ki
[ Qi ( k ) +
Qi (k )]
m + k + 1 i =0
m+i+1
i =0

k
k
m
m
[ Qi (k) Qi+1 (k)] =
.
m + k + 1 i =0
m+k+1
i =0

,
,,
,
,
` go. i y
` tp
12.9. L`oi giai va
bai
. chuong 9

,
, ,, `
,,
. 9.11. Go.,i y:
a) ap
dung

thu c o bai
9.4 ta nhn
dang
o. c
. du
.
P2i
P
q2i
2i2 = (1)2i1 .
,
Q2i
Q2i2
Q2i Q2i2
P2i
P
`, d
tu
> 2i2 (i = 1, 2, . . .).
o co
Q2i
Q2i2
,,
,
,
b) tuong tu. nhu ph`n a).
,
,,
P1 P3
,

` tp
. 9.12. Lo` i giai: a) Day
,
, . . . hi
tu,
boi v` theo bai
.
.
.
Q1 Q3
,
,
,
,
giam, mat
s hang
. khac
.
trn th` day
cua no d
`u duong. Ta dat
.

248

,
,,
,
,
` tp
Chuong 12. Lo` i giai va` go. i y bai
.

,
,
P0 P2
P2i+1
`

day
,
, . . . cung
hi
tu;
d
0 = lim
. Ta se ch ra rang

.
.
Q0 Q2
i Q2i +1
,
,
,
,,
. phai, v` ta da bit
chu ng minh du
o. c di
`u d
o c`n ch ra no bi. chan
, `
`, (9.13) ta co
tang.

nhu bai
tp
Tht
. trn no da la` day
. vy,
. tu
P2i+1 Q2i P2i Q2i+1 = (1)2i = 1,
`, dy
tu

P2i+1
P
P
Q P2i Q2i+1
1
2i = 2i+1 2i
=
> 0. (12.10)
Q2i+1
Q2i
Q2i Q2i+1
Q2i Q2i+1
,
,
,
,
P
phn s xp x vo,i ch s le
. 00 = limi 2i . Bo,i v` day
Ta dat
Q2i
,
P2m
,
,
th` 00 0 . Mat

. khac
<
tri
. hon phn s vo i ch s chan,
Q2m
,
P2n+1

00 , (m = 0, 1, . . .) va` 0 <
, (n = 0, 1, . . .). Suy ra bt dang
Q2n+1
,
thu c
P
P2m
< 00 0 < 2n+1
Q2m
Q2n+1
,
,
`
tt ca s tu., nhin m, n. Ta se chu,ng minh rang
thoa man
0 =
,
,
, ,
P2i+1
P
` ta phai kt lun
00 . dat
2i co th tro,
ch nay
. muc
. d
. Q
Q2i
2i +1
,
, ,
,

`
` trn day
Q0 , Q1 , . . . do
thanh
rt nho khi i du lo n. Theo bai
n di
u
.
,
,
,
` (12.10) ta nhn

tang
suy ra Qn n, (n = 0, 1, . . .). Khi d
o tu
o. c
. du
P2i+1
P
1
1
0<
2i =

,
Q2i+1
Q2i
Q2i Q2i+1
2i (2i + 1)
`, dy
tu

P
P
P2i+1
P
lim (
2i ) = lim 2i+1 lim 2i = 0 00 = 0.
Q2i
i Q2i +1
i Q2i +1
i Q2i
,
,
Pn
0
00
Ta ky hiu
.
. gia tri. chung cua va` co th vit = nlim
Qn
,
,
` la` gia tri. cua lin phn s. Ta da
Theo dinh
ngha gio i han

.
. nay
,,
,
,
ph`n tu nguyn la` hi
chu ng minh xong moi
. lin phn s vo i cac
.
tu.
.

,
,,
,
,
` tp
12.9. Lo` i giai va` go. i y bai
. chuong 9

249

,
Theo bai
` trn lin phn s da cho la` hi
. 9.13. Go. i y:
. tu,
. ta co
,

th vit
1
= (1, 1, . . .) = (1, ) = 1 + ,

,
,,
2
`
Ta thy rang
la` nghi
. cua phuong tr`nh 1 = 0. V`
m
1+ 5
.
> 0, th` =
2
,
,
,
,ng tnh cht cua lin phn s ta nhn
. 9.14. Lo` i giai: Tu`, nhu
.
,,
du
o. c

Pi
P + Pi1
P
Qi Pi1 Qi1 Pi
= i i +1
i =
Qi
Q i i +1 + Q i 1
Qi
Q i ( Q i i +1 + Q i 1 )

va` theo (9.13) ta co


i = |

Pi
1
|=
.
Qi
Q i ( Q i i +1 + Q i 1 )

(12.11)

V`
i +1 > q i +1 ,
(12.12)
`, dy
th` Qi ( Qi i+1 + Qi1 ) > Qi ( Qi qi+1 + Qi1 ) = Qi Qi+1 tu

1
Q i ( Q i i +1 + Q i 1 ) = Q i ( Q i ( i +1 q i +1 ) +
. khac
i <
. mat
Q i Q i +1
,
,
,,
`

Qi qi+1 + Qi1 ) va` dung


bt dang
thu c (12.12 ) ta nhn
o. c
. du
Qi ( Qi i+1 + Qi1 ) < Qi ( Qi + ( Qi qi+1 + Qi1 )) = Qi ( Qi + Qi+1 ).
,,
`, dy
Tu
va` (12.11) ta t`m du
o. c
i >

1
.
Q i ( Q i + Q i +1 )

,
`
TAI LIU
. THAM KHAO
[1] Phu,o,ng phap
irichl va` u,ng dung,
.
,
,

u in, NXB KHKT, 1999.


Nguyn Hu
,
[2] Phu,o,ng phap
s phu,c va` h`nh hoc
phang,

.
,
Hu
,u in, NXB HQG, 2000.
Nguyn
[3] Metod matematiqesko
u indukcii,
I. S. Sominski
u, Moskva, 1961.
[4] Matematiqeska indukci,
L. Petruxev, Sofi, 1983.
[5] Problem-Solving through problems,
Loren C. Larson, Springer-Verlag, 1983.
[6] Cac
d
cac
nu,o,c,
` thi v dich
. toan
,
X.V. Cnhiagin, G.A. Tnian, I.F. Sarugin,... NXB GD,
1996.
[7] Phep quy nap
. trong h`nh hoc,
.
L.I. Golovina, I.M. Yaglom NXB GD, 1997.

250

NI
. DUNG
`,i noi
d
Lo
`u . . . . . . . . . . . . . . . . . . . . . . . . . . . . . . . . . . . . . . . . . . . . . . . . .
,,
hoc
Chuong 1. Nguyn ly quy nap
. toan
. .................
va` quy nap
1.1. Suy din
. ..................................

3
4
4

hoc
1.2. Nguyn ly quy nap
. toan
. .......................... 6
,
1.3. Giai do
an
. quy nap
. va` gia thit quy nap
. ............... 8
, , ,
hoc
1.4. Hai buoc cua nguyn ly quy nap
. toan
. . . . . . . . . . . . 14
,
,
` dung
`
quy nap
1.5. Khi nao
phuong phap
. . . . . . . . . . . . . . . . . . 19
` tp
1.6. Bai
. . . . . . . . . . . . . . . . . . . . . . . . . . . . . . . . . . . . . . . . . . . . . . . . 22
,,
,,
`
quy nap
hoc
Chuong 2. Ky thut
phuong phap
. dung
. toan
.
23
hoc
2.1. Mt
nguyn ly quy nap
. s dang
.
. toan
. ............

23

hoc
2.2. Mnh
d
` trong nguyn ly quy nap
.
. toan
. ..........
,,
,,
,
2.3. Buo c quy nap
o. c xy du. ng trn P(k) . . . . . . . . . . . . . .
. du
,,
,,
,
2.4. Buo c quy nap
o. c xy du. ng trn P(k + 1) . . . . . . . . . .
. du
hoc
2.5. Quy nap
. toan
. va` phep truy h`i . . . . . . . . . . . . . . . . . .
,
hoc
hoa . . . . . . . . . . . . . . . . . .
2.6. Quy nap
. toan
. va` tng quat

31

` tp
2.7. Bai
. ...............................................
,
,,
,c tng quat
...................
Chuong 3. T`m cng thu
3.1. Cp s cng
va` cp s nhn . . . . . . . . . . . . . . . . . . . . . . . . . .
.
,
,
......................
3.2. Tnh tng va` s hang
tng quat
.
,,
3.3. Phuong tr`nh truy h`i tuyn tnh . . . . . . . . . . . . . . . . . . .
251

36
40
43
51
55
57
57
66
71

252

NI
. DUNG

,
,
,ng luy
`,a cung
thu
`
s tu., nhin .
bc
3.4. Tng cua nhu
. cac
` tp
3.5. Bai
. ...............................................
,,
Chuong 4. S hoc
. ........................................
4.1. Phep chia ht . . . . . . . . . . . . . . . . . . . . . . . . . . . . . . . . . . . . . . . .

84
87
89
89

Euclide . . . . . . . . . . . . . . . . . . . . . . . . . . . . . . . . . . 94
4.2. Thut
. toan
,
4.3. S phu c . . . . . . . . . . . . . . . . . . . . . . . . . . . . . . . . . . . . . . . . . . . . . . 99
,ng v du. khac
. . . . . . . . . . . . . . . . . . . . . . . . . . . . . . . . . . 105
4.4. Nhu
` tp.
4.5. Bai
. .............................................
,,
s . . . . . . . . . . . . . . . . . . . . . . . . . . . . . . . . . . . . . . .
Chuong 5. Day
s tu., nhin . . . . . . . . . . . . . . . . . . . . . . . . . . . . . . . . . . . . .
5.1. Day
,
tri
5.2. Day
. hon . . . . . . . . . . . . . . . . . . . . . . . . . . . . . . . . . . . . . . . .
,
,
,
,ng bt dang

5.3. Nhu
thu c ni ting . . . . . . . . . . . . . . . . . . . . .
,
do
5.4. Day
n di
u
. .......................................
5.5. S e . . . . . . . . . . . . . . . . . . . . . . . . . . . . . . . . . . . . . . . . . . . . . . . . .

108
110
110
117
121
128
131

s Fibonacci. . . . . . . . . . . . . . . . . . . . . . . . . . . . . . . . . . . .
5.6. Day

134

` tp.
5.7. Bai
. .............................................
,,
Chuong 6. H`nh hoc
. ....................................

139
140

hoc
6.1. V du. quy nap
. toan
. cho h`nh hoc
. ...............

140

` tp.
6.2. Bai
. .............................................
,,
,c . . . . . . . . . . . . . . . . . . . . . . . . . . . . . . . . . . . . . .
Chuong 7. a thu
,
`,a s . . . . . . . . . . . . . . . . . . . . . . .
7.1. Phn tch da
thu c ra thu

154

h. s . . . . . . . . . . . . . . . . . . . . . . . .
7.2. Nguyn ly so sanh
cac
,
,
` cua da
7.3. ao
thu c . . . . . . . . . . . . . . . . . . . . . . . . . . . . . . .
. ham
,
7.4. a thu c Chebychev . . . . . . . . . . . . . . . . . . . . . . . . . . . . . . . . .

156
156
160
169
172

NI
. DUNG

253

` tp
7.5. Baii
. .............................................
,
, ,
,,
,c . . . . . . . . . . . . . . . . . . . . . . .

Chuong 8. T ho. p va` dang


thu
, , ,
8.1. Mt
. s cng thu c t ho. p . . . . . . . . . . . . . . . . . . . . . . . . . . . .
,
,

8.2. Mt
thu c . . . . . . . . . . . . . . . . . . . . . . . . . . . . . . . . . . .
. s dang

174

` tp.
8.3. Bai
. .............................................
,,
Chuong 9. Lin phn s . . . . . . . . . . . . . . . . . . . . . . . . . . . . . . . .

176
176
186
193
194

nim
9.1. Khai
. lin phn s . . . . . . . . . . . . . . . . . . . . . . . . . . . . .
,
,u ty thanh
`
9.2. Phn tch s hu
lin phn s . . . . . . . . . . . .
,
9.3. Phn s xp x . . . . . . . . . . . . . . . . . . . . . . . . . . . . . . . . . . . . . .

198

9.4. Lin phn s v han


. ................................

203

9.5. V du.
. ...............................................

204

` tp
9.6. Bai
. .............................................
,,
Chuong 10. Mt
.....................
` thi v dich
.
. s d
,
,,
,
` tp
Chuong 11. Bai
. tu. giai . . . . . . . . . . . . . . . . . . . . . . . . . . . . .
,
,,
`,i giai va` go.,i y bai
` tp
Chuong 12. Lo
. ..................
,
,
,
,
,
` tp
12.1. Lo` i giai va` go. i y bai
. chuong 1 . . . . . . . . . . . . . . . . .
,
,,
,
,
` tp
12.2. Lo` i giai va` go. i y bai
. chuong 2 . . . . . . . . . . . . . . . . .
,
,,
,
,
` tp
12.3. Lo` i giai va` go. i y bai
. chuong 3 . . . . . . . . . . . . . . . . .
,
,,
,
,
` tp
12.4. Lo` i giai va` go. i y bai
. chuong 4 . . . . . . . . . . . . . . . . .
,
,,
,
,
` tp
12.5. Lo` i giai va` go. i y bai
. chuong 5 . . . . . . . . . . . . . . . . .
,
,,
,
,
` tp
12.6. Lo` i giai va` go. i y bai
. chuong 6 . . . . . . . . . . . . . . . . .
,
,,
,
,
` tp
12.7. Lo` i giai va` go. i y bai
. chuong 7 . . . . . . . . . . . . . . . . .
,
,,
,
,
` tp
12.8. Lo` i giai va` go. i y bai
. chuong 8 . . . . . . . . . . . . . . . . .
,
,,
,
,
` tp
12.9. Lo` i giai va` go. i y bai
. chuong 9 . . . . . . . . . . . . . . . . .

210

194
196

212
226
231
231
233
236
236
237
240
240
244
247

254

, `

Thu. c hanh
tnh toan

Muc
. luc
. ................................................

250

, ,
QUY NAP
HOC
PHUONG PHAP
. TOAN
.
Ma s: 8H663M0
,
,
In 3.000 ban (21TK), kh 14, 3 20, 3 cm Tai
. Cng ty In Ba

`nh, Thanh Hoa S in: 127; S XB 05/796-00.


,
,

2000.
In xong va` np
8 nam
. luu chiu thang

You might also like